b3

अब Quizwiz के साथ अपने होमवर्क और परीक्षाओं को एस करें!

8. A 37-year-old small business manager receives a gunshot wound in the pelvic cavity, resulting in a lesion of the sacral splanchnic nerves. Which of the following nerve fibers would primarily be damaged? (A) Postganglionic parasympathetic fibers (B) Postganglionic sympathetic (C) Preganglionic sympathetic fibers (D) Preganglionic parasympathetic fibers (E) Postganglionic sympathetic and parasympathetic fibers

8. The answer is C. The sacral splanchnic nerves consist primarily of preganglionic sympathetic neurons and also contain GVA fibers. None of the other fibers listed are contained in these nerves.

An 87yo woman with a history of metastatic adenocarcinoma of the breast presents with difficulty in breathing. A plain chest film reveals massive left pleural effusion, as noted in the photo. The physician chooses to drain the fluid with a thoracentesis. What is the proper location for placement of the cannula (hollow needle) to drain the pleural effusion? A. midaxillary line - 6th intercostal space B. midaxillary line - 9th intercostal space C. parasternal line - 6th intercostal space D. parasternal line - 2nd intercostal space E. midclavicular line - 5th intercostal space

B. midaxillary line - 9th interostal space

66. Destruction of this area would most likely cause weakness of pronation of the forearm and flexion of the wrist joints.

E

73. Atrophy of this structure impairs extension of both the metacarpophalangeal and interphalangeal joints.

E

A 20 year-old athlete presents to your office after sustaining an injury to the right upper extremity. Physical examination reveals very weak supination of the forearm and absence of biceps jerk. Further examination most likely reveals loss of sensation in which of the following areas? a. lateral three and a half fingers b. posterior forearm c. lateral forearm d. medial forearm e. medial one and half fingers

c. Lateral forearm

A 6-year-old boy playing barefooted in his backyard steps on a piece of broken glass and suffers a large transverse cut on his sole, at the level of the midfoot. In the emergency room, the examining physician determines the cut is to the depth of the first layer of the plantar muscles. Which of the following structures is most likely damaged in this injury? (A) Plantar arterial arch (B) Tendons of the flexor digitorum longus muscle (C) Tendon of the fibularis (peroneus) longus muscle (D) Abductor hallucis muscle (E) Superficial fibular (peroneal) nerve

(D) Abductor hallucis muscle

52. Which muscle can both dorsiflex and invert the foot? (A) Peroneus longus (B) Peroneus brevis (C) Peroneus tertius (D) Extensor hallucis longus (E) Extensor digitorum longus

52. The Answer is D. The extensor hallucis longus can dorsiflex and invert the foot. The peroneus longus, peroneus tertius, and extensor digitorum longus can dorsiflex and evert foot. The peroneus brevis can plantar flex and evert the foot.

59. Which fractured structure is likely to cause paralysis of the adductor magnus?

59. The Answer is C. The ischiopubic ramus and ischial tuberosity provide attachment for the adductor magnus.

60. Which structure becomes necrotic after the medial femoral circumflex artery is severed?

60. The Answer is A. The distal part of the femoral head receives blood mainly from the medial femoral circumflex artery, whereas the proximal part is supplied by a branch from the posterior division of the obturator artery.

During week 2 of development, the embryoblast receives its nutrients via (A) diffusion (B) osmosis (C) reverse osmosis (D) fetal capillaries (E) yolk sac nourishment

A. During week 2 of development, the embryoblast receives its nutrients from endometrial blood vessels, endometrial glands, and decidual cells via diffusion. Diffusion of nutrients does not pose a problem given the small size of the blastocyst during week 2. *Although the beginnings of a uteroplacental circulation are established during week 2, no blood vessels have yet formed in the extraembryonic mesoderm to carry nutrients directly to the embryoblast (this occurs in week 3).*

Into which structure does the azygos vein drain venous blood?

A. SVC

13. A 24-year-old woman comes to a hospital to deliver her baby. Her obstetrician uses a caudal anesthesia during labor and childbirth to block the spinal nerves in the epidural space. Local anesthetic agents are most likely injected via which of the following openings? (A) Intervertebral foramen (B) Sacral hiatus (C) Vertebral canal (D) Dorsal sacral foramen (E) Ventral sacral foramen

B

In the production of female gametes, which of the following cells can remain dormant for 12 to 40 years? (A) Primordial germ cell (B) Primary oocyte (C) Secondary oocyte (D) First polar body (E) Second polar body

B. Primary oocytes are formed by month 5 of fetal life and remain dormant until puberty, when hormonal changes in the young woman stimulate the ovarian and menstrual cycles. From 5 to 15, oocytes then begin maturation with each ovarian cycle throughout the woman's reproductive life.

Which of the labeled structures in the given PA radiograph of the chest indicates the right atrium?

B. right atrium

Stenosis of which structure may produce left ventricular hypertrophy

C. Ascending aorta

Becomes hypertrophied as result of the pulmonary stenosis?

C. Right ventricle

Contains the septomarginal trabecula?

C. Right ventricle

An infant is born with an abnormally thin wall in the smooth part of the right atrium. This condition may be related to underdevelopment of which of the following embryonic structures? A. pulmonary trunk B. conus cordis C. sinus venosus D. truncus arteriosus E. ascending aorta

C. sinus venosus

A 40yo man goes to his family physician for an annual examination. Which of the following locations is ideal for placement of the stethoscope for auscultation of the tricuspid valve of the heart?

C. tricuspid valve 5th intercostal parasternal

17. A 44-year-old woman comes to her physician and complains of headache and backache. On examination, she is found to have fluid accumulated in the spinal epidural space because of damage to blood vessels or meninges. Which of the following structures is most likely ruptured? (A) Vertebral artery (B) Vertebral vein (C) External vertebral venous plexus (D) Internal vertebral venous plexus (E) Lumbar cistern

D

70. The patient is unable to abduct her middle finger because of paralysis of this structure.

D

Takes its origin from the left ventricle and ends at the sternal angle?

D. Ascending aorta

Which structure is crossed superiorly by the aortic arch and left pulmonary artery?

D. Left primary bronchus

15. A 26-year-old heavyweight boxer was punched on his mandible, resulting in a slight subluxation (dislocation) of the atlantoaxial joint. The consequence of the injury was decreased range of motion at that joint. What movement would be most affected? (A) Extension (B) Flexion (C) Abduction (D) Adduction (E) Rotation

E

26. When the internal vertebral venous plexus is ruptured, venous blood may spread into which tissue and space?

E

Which of the following components of the respiratory tract would be directly affected by a developmental failure of normal differentiation of the foregut endoderm? A. nasal epithelium B. intrinsic laryngeal muscles C. bronchial cartilages D. trachealis muscle E. alveoli

E. alveoli

72. The patient is unable to adduct her little finger because of paralysis of this structure.

G

A 24-year-old man comes to his family physician with an irregular scrotal swelling on his left. He reports aching pain within the scrotum and a feeling of heaviness in his left testis. During the physical examination, the physician reports that the scrotal mass feels like a bag of worms. What is the most likely diagnosis? (A) Hydrocele (B) Varicocele (C) Testicular cancer (D) Indirect inguinal hernia (E) Spermatocele

The answer is B: Varicocele. A varicocele is an abnormal dila- tion of the pampiniform plexus of veins, which travels within the spermatic cord. This scrotal mass often resembles and feels like a "bag of worms" on physical examination, as seen in the photo. Idiopathic varicocele is usually caused by defec- tive one-way valves within the pampiniform plexus, which cause dilatation of these veins near the testis. However, a sec- ondary varicocele will be seen following compression of the venous drainage of the testis due to the presence of a pelvic or abdominal malignancy, and this possibility must be ruled out by the physician. Varicoceles are found in the left side in approximately 98% of cases, can develop in 15% to 20% of all males, and are most frequently diagnosed between 15 and 25 years of age. Choice A (Hydrocele) is incorrect. A hydrocele testis is a pathological accumulation of serous fluids within the spermatic cord due to serous secretions from a remnant piece of peritoneum, termed the tunica vaginalis. Patients with a hydrocele testis present with a painless swollen testis that feels like a water balloon, which may be corrected via drain- age with a needle (aspiration) or surgery. To identify a hydro- cele, a diagnostic technique called transillumination (shining a light through the enlarged portion of the scrotum) can be implemented. If the scrotum is full of clear fluid, the scrotum will light up. A hydrocele testis is not likely in this patient due to the irregular nature of the scrotal mass seen within the photo. Choice C (Testicular cancer) is incorrect. Testicular cancer presents with a lump on or hardening of the testis lead- ing to unilateral enlargement of the affected testis, abnormal sensitivity within the testes, dull pain within the lower back, abdomen, or groin, loss of libido, and general weakness and tiredness. The extent of the scrotal mass and its irregular nature make the diagnosis of testicular cancer unlikely. If testicular cancer were present, it would not grow to this size without the patient noticing its presence. Moreover, the cardinal sign for testicular cancer is unilateral enlargement of the testis; how- ever, in a varicocele, the testis is usually smaller on the affected side. Choice D (Indirect inguinal hernia) is incorrect. An indi- rect inguinal hernia is usually a congenital hernia that results when abdominal cavity contents herniate through a patent processus vaginalis, or in an adult, travel through the inguinal canal. The hernia traverses the deep and superficial inguinal rings to descend into the scrotum in males. The hernia often presents as a visible or palpable lump underneath the skin in the groin region, which was seen in this patient. However, given the irregular nature of this scrotal mass and the physi- cian's observation that the mass felt like a "bag of worms," a varicocele is more likely. Choice E (Spermatocele) is incorrect. A spermatocele is a cyst-like mass within the scrotum that develops in a tubule of the rete testis or the head of the epididymis. It usually contains a milky fluid and spermatozoa. These retention cysts are usually not painful and can be con- firmed by an ultrasound of the scrotum. Given the irregular nature, size, and "bag of worms" description of the scrotal mass in this patient, a varicocele is the most likely diagnosis.

Identify the lunate a. 2 b. 9 c. 1 d. 5 e. 8 f. 7 g. 4 h. 3 i. 6

h. 3

1. A 27-year-old patient exhibits a loss of skin sensation and paralysis of muscles on the plantar aspect of the medial side of the foot. Which of the following nerves is most likely damaged? (A) Common peroneal (B) Tibial (C) Superficial peroneal (D) Deep peroneal (E) Sural

1. The Answer is B. The common peroneal nerve divides into the deep peroneal nerve, which innervates the anterior muscles of the leg and supplies the adjacent skin of the first and second toes, and the superficial peroneal nerve, which innervates the lateral muscles of the leg and supplies the skin on the side of the lower leg and the dorsum of the ankle and foot. The sural nerve supplies the lateral aspect of the foot and the little toe.

2. A 43-year-old female patient has been lying down on the hospital bed for more than 4 months. Her normal, quiet expiration is achieved by contraction of which of the following structures? (A) Elastic tissue in the lungs and thoracic wall (B) Serratus posterior superior muscles (C) Pectoralis minor muscles (D) Serratus anterior muscles (E) Diaphragm

2. The answer is A. Normal, quiet expiration is achieved by contraction of extensible tissue in the lungs and the thoracic wall. The serratus posterior superior muscles, diaphragm, pectoralis major, and serratus anterior are muscles of inspiration.

22. An 18-year-old girl is thrust into the steering wheel while driving and experiences difficulty in expiration. Which of the following muscles is most likely damaged? (A) Levator costarum (B) Innermost intercostal muscle (C) External intercostal muscle (D) Diaphragm (E) Muscles of the abdominal wall

22. The answer is E. The abdominal muscles are the major muscles of expiration, whereas the other distractors are muscles of inspiration.

3. A 46-year-old male patient with high blood pressure was examined in the emer- gency department, and his physician found a leakage of blood from the blood vessel that normally carries richly oxygenated blood. Which of the following vessels would most likely be damaged? (A) Superior vena cava (B) Pulmonary arteries (C) Pulmonary veins (D) Portal vein (E) Coronary sinus

3. The answer is C. Pulmonary veins return oxygenated blood to the heart from the lungs. Pulmonary arteries carry deoxygenated blood from the heart to the lungs for oxygen renewal. The portal vein carries deoxygenated blood with nutrients from the intestine to the liver. The superior vena cava and coronary sinus carry deoxygenated blood to the right atrium.

9. A young couple is having difficulty conceiving a child. Their physician at a reproduction and fertility clinic explains to them that (A) The ovary lies within the broad ligament (B) The glans clitoris is formed from the corpus spongiosum (C) Erection of the penis is a sympathetic response (D) Ejaculation follows parasympathetic stimulation (E) Fertilization occurs in the infundibulum or ampulla of the uterine tube

9. The answer is E. Fertilization takes place in the infundibulum or ampulla of the uterine tube. The glans clitoris is derived from the corpora cavernosa, whereas the glans penis is the expanded terminal part of the corpus spongiosum. Erection of the penis is caused by parasympathetic stimulation, whereas ejaculation is mediated via the sympathetic nerve. The ovaries are not enclosed in the broad ligament, but their anterior surface is attached to the posterior surface of the broad ligament.

During a newborn examination, a pediatrician is unable to detect a pulse in the groin or legs of an infant and notes the lower extremities are cold on the touch. The pulse and blood pressure of the upper limbs are significantly elevated from normal. A pediatric cardiologist performs an echocardiogram and other imaging. The given sagittal MRI verifies the presence of what specific anomaly, as indicated by the white arrowhead? A. coarctation of aorta B. tetralogy of Fallot C. ventricular septal defect (VSD) D. atrial septal defect (ASD) E. transposition of the great vessels

A. coarctation of the aorta

22. A 25-year-old man with congenital abnormalities at birth has a lesion of the dorsal scapular nerve, making him unable to adduct his scapula. Which of the following muscles is most likely paralyzed? (A) Semispinalis capitis (B) Rhomboid major (C) Multifidus (D) Rotator longus (E) Iliocostalis

B

6. After a 26-year-old man's car was broadsided by a large truck, he is brought to the emergency department with multiple fractures of the transverse processes of the cervical and upper thoracic vertebrae. Which of the following muscles might be affected? (A) Trapezius (B) Levator scapulae (C) Rhomboid major (D) Serratus posterior superior (E) Rectus capitis posterior major

B

12. A 27-year-old stuntman is thrown out of his vehicle prematurely when the car used for a particular scene speeds out of control. His spinal cord is crushed at the level of the fourth lumbar spinal segment. Which of the following structures would most likely be spared from destruction? (A) Dorsal horn (B) Ventral horn (C) Lateral horn (D) Gray matter (E) Pia mater

C

5. A 57-year-old woman comes into her physician's office complaining of fever, nausea, vomiting, and the worst headache of her life. Tests and physical examination suggest hydrocephalus (widening ventricles) resulting from a decrease in the absorption of cerebrospinal fluid (CSF). A decrease of flow in the CSF through which of the following structures would be responsible for these findings? (A) Choroid plexus (B) Vertebral venous plexus (C) Arachnoid villi (D) Internal jugular vein (E) Subarachnoid trabeculae

C

1. During an outbreak of meningitis at a local college, a 20-year-old student presents to a hospital emergency department complaining of headache, fever, chills, and stiff neck. On examination, it appears that he may have meningitis and needs a lumbar puncture or a spinal tap. Cerebrospinal fluid (CSF) is normally withdrawn from which of the following spaces? (A) Epidural space (B) Subdural space (C) Space between the spinal cord and the pia mater (D) Subarachnoid space (E) Space between the arachnoid and dura maters

D

A 28-year-old man is evaluated after sustaining and neck and shoulder injury. To localize the specific spinal root of the break you'll plexus involved and the possible joint movements that's affected, the physician ask the patient to flex his meal for fingers at the metacarpophalangeal joints. Which of the following specific nerve roots was most likely tested by the physician? A. C5 B. T1 C. C5 D. C8E. C6

D. C8

18. A 69-year-old man has an abnormally increased curvature of the thoracic vertebral column. Which of the following conditions is the most likely diagnosis? (A) Lordosis (B) Spina bifida occulta (C) Meningocele (D) Meningomyelocele (E) Kyphosis

E

A new born baby is diagnosed with spina bifida occulta. One of the factors responsible for this anomaly could be that the caudal neuropore did not close. The caudal neuropore normally closes by which of the following days of intrauterine life? Select one: a. 25 days b. 20 days c. 37 days d. 35 days e. 27 days

E. 27

Between which two layers is the extraembryonic mesoderm located? (A) Epiblast and hypoblast (B) Syncytiotrophoblast and cytotrophoblast (C) Syncytiotrophoblast and endometrium (D) Exocoelomic membrane and syncytiotrophoblast (E) Exocoelomic membrane and cytotrophoblast

E. The extraembryonic mesoderm is derived from the epiblast and is located between the exocoelomic membrane and the cytotrophoblast. The overall effect is to completely separate the embryoblast from the trophoblast with the extraembryonic mesoderm serving as a conduit (connection) between them.

A 4-month-old male infant presents with a "growing sore" located posterior to his left ear. This sore is diagnosed as a postauricular hemangioma, as seen in the given photo with the auricle pulled anterior. The cells forming the hemangioma are derived from which of the following cell layers? (A) Endoderm (B) Neural crest (C) Neuroectoderm (D) Ectoderm (E) Mesoderm

The answer is E: Mesoderm. The mesoderm forms during gastrulation when invaginating epiblast cells form an additional germ layer between the endoderm and the ectoderm. The mesoderm has many notable derivatives, including muscle, connective tissue, bone, cartilage, blood cells, dermis of the skin, and organs, such as the kidney, spleen, and gonads.

A 72-year-old woman, who lost 50 lb (22.7 kg) of weight in the past year, visited her physician complaining of abdominal cramps, pain, and vomiting. Physical examination detected a distended abdomen with no visible or palpable masses in the groin and upper thigh area. She also described a sharp cutting pain in the medial aspect of her left thigh, which was exacerbated by the physician extending and medially rotating her left thigh at the hip joint. An abdominal CT revealed small bowel obstruction. What is the most likely cause of the obstruction? (A) Direct inguinal hernia (B) Indirect inguinal hernia (C) Umbilical hernia (D) Femoral hernia (E) Obturator hernia

The answer is E: Obturator hernia. An obturator hernia can present with small bowel obstruction and lancinating (sharp cutting) pain in the medial thigh region. This type of hernia is most often seen in elderly females (over the age of 70), espe- cially women who have recently lost a large amount of weight. It occurs when a portion of the small bowel is entrapped within the confines of the obturator canal, which compresses the obturator nerve and leads to sharp cutting pain within its distribution pattern. When a physician is able to exacer- bate this pain by extension, medial rotation, and adduction of the thigh at the hip joint, it is called a positive Howship- Romberg sign, which is further suggestive of an obturator hernia. Remember that an obturator hernia does not present with visible lumps because the herniation is trapped under the muscles of the medial compartment of the thigh. Choice A (Direct inguinal hernia) is incorrect. A direct inguinal hernia is an acquired hernia that results when abdominal cavity con- tents herniate through a weakness in the anterior abdominal wall in the inguinal (Hesselbach) triangle. The hernia traverses the superficial inguinal ring and often presents as a visible or palpable lump underneath the skin overlying the superficial inguinal ring, which was not present in this patient. More- over, direct inguinal hernias are rare in women; in fact, some references cite groin hernias are 25 times more likely to occur in men than women. Choice B (Indirect inguinal hernia) is incorrect. An indirect inguinal hernia is usually a congenital hernia that results when abdominal cavity contents herniate through a patent processus vaginalis, or in an adult, the ingui- nal canal. The hernia exits through the deep and superficial inguinal rings and descends into the labium majus of females or the scrotum of males. An indirect inguinal hernia often pres- ents as a visible or palpable lump underneath the skin in the groin region, and this finding was not reported in this patient. Choice C (Umbilical hernia) is incorrect. Umbilical hernias are protrusions in the umbilical region that can be congenital (seen in newborns) or acquired (occurring later in life). These hernias present as a visible or palpable lump underneath the skin in the umbilical region, and this finding was not reported in this patient. Choice D (Femoral hernia) is incorrect. A femoral hernia is a protrusion through the femoral ring that extends into the femoral canal and compresses its contents (lymphatic vessels and connective tissue). Initially the hernia is contained in the femoral canal; however, it may extend infe- riorly to the saphenous hiatus where it creates a bulge within the femoral triangle. The femoral ring is a weakened aspect in the anterior abdominal wall through which femoral hernias enter the femoral canal. These hernias are more common in females due to their wider pelves and often contain abdominal viscera. Moreover, femoral hernias are more susceptible to strangulation, wherein the blood supply to the herniated vis- cera can be interrupted due to the sharp boundaries of the femoral ring, particularly the lacunar ligament. Strangulation can lead to ischemia, sharp pain, and necrosis of the impinged tissue. However, a patient with a femoral hernia would not present with sharp cutting pain in the medial aspect of the thigh or a positive Howship-Romberg sign (exacerbation of this pain after extending and medially rotating the thigh at the hip joint).

Medical images: Which of the following labeled structures (Fig. 1) drains into a remnant of the sinus venosus from the fetal heart? a. 1 b. 4 c. 5 d. 3 e. 2

a. 1

A 76-year-old female visits her physician with complaints of increased fatigue. She states that she tires easily even with very low levels of activity. The woman has a history of coronary artery disease and diabetes. On physical examination, her doctor notices a disappearing arterial pulse on inspiration. The doctor orders an echocardiogram to confirm the cardiac tamponade. What invasive procedure will be necessary to treat this patient? a. Surgical reduction of an aortic aneurysm b. Aortic valve replacement c. Angioplasty d. Pericardiocentesis e. Mitral valve replacement

d. Pericardiocentesis

After surgical repair of a direct inguinal hernia, a 64-year-old man complains to his surgeon of numbness in the anterior scrotal area and the medial area of the adjacent thigh. Which of the following nerves is most likely injured? a. Iliohypogastric b. Medial femoral cutaneous c. Pudendal d. Subcostal e. Ilioinguinal

e. Ilioinguinal

A roofi ng installer falls off a high ladder and lands with the sole of his right foot hitting the ground first. He suffers a fracture and inferior displacement of the sustentaculum tali of the calcaneus. Which of the following structures is most likely torn? (A) Tendon of the tibialis posterior muscle (B) Tendon of the fl exor hallucis longus (FHL) muscle (C) Tendons of the fl exor digitorum brevis muscle (D) Small saphenous vein (E) Plantar arterial arch

(B) Tendon of the flexor hallucis longus (FHL) muscle

After spending 2 days moving heavy furniture out of her house, a 56-year-old woman goes to an emergency room in acute pain. The patient reports nausea, vomiting, and severe abdominal pain. On examination, the doctor discovers a painful, globular mass located inferior and lateral to the pubic tubercle. Given her presentation and history, what is the most likely diagnosis? (A) Indirect inguinal hernia (B) Direct inguinal hernia (C) Femoral hernia (D) Lymphadenitis of superfi cial inguinal nodes (E) Fractured hip

(C) Femoral hernia

1. A 22-year-old man presented to his family physician with a laceration of the fibrous sheets or bands that cover his body under the skin and invest the muscles. Which of the following structures would most likely be injured? (A) Tendon (B) Fascia (C) Synovial tendon sheath (D) Aponeurosis (E) Ligament

1. The answer is B. The fascia is a fibrous sheet or band that covers the body under the skin and invests the muscles. Although they are fibrous, tendons connect muscles to bones or cartilage, aponeuroses serve as the means of origin or insertion of a flat muscle, and liga- ments connect bones to bones or cartilage. Synovial tendon sheets are tubular sacs filled with synovial fluid that wrap around the tendons.

1. A 32-year-old patient who weighs 275 lb comes to the doctor's office. On the surface of the chest, the physician is able to locate the apex of the heart: (A) At the level of the sternal angle (B) In the left fourth intercostal space (C) In the left fifth intercostal space (D) In the right fifth intercostal space (E) At the level of the xiphoid process of the sternum

1. The answer is C. On the surface of the chest, the apex of the heart can be located in the left fifth intercostal space slightly medial to the midclavicular (or nipple) line. The sternal angle is located at the level where the second ribs articulate with the sternum. The xiphoid process lies at the level of T10 vertebra.

14. A 52-year-old woman slipped and fell and now complains of being unable to extend her leg at the knee joint. Which of the following muscles was paralyzed as a result of this accident? (A) Semitendinosus (B) Sartorius (C) Gracilis (D) Quadriceps femoris (E) Biceps femoris

14. The Answer is D. The quadriceps femoris muscle includes the rectus femoris muscle and the vastus medialis, intermedialis, and lateralis muscles. They extend the leg at the knee joint. The semitendinosus, semimembranosus, and biceps femoris muscles (the hamstrings) extend the thigh and flex the leg. The sartorius and gracilis muscles can flex the thigh and the leg.

20. If all of the arteries that are part of the cruciate anastomosis of the upper thigh are ligated, which of the following arteries maintains blood flow? (A) Medial femoral circumflex (B) Lateral femoral circumflex (C) Superior gluteal (D) Inferior gluteal (E) First perforating

20. The Answer is C. The superior gluteal artery does not participate in the cruciate anasto- mosis of the thigh. The inferior gluteal artery, transverse branches of the medial and lat- eral femoral circumflex arteries, and an ascending branch of the first perforating artery form the cruciate anastomosis of the thigh.

21. A 21-year-old marine biologist asks about her first bimanual examination, and it is explained to her that the normal position of the uterus is (A) Anteflexed and anteverted (B) Retroflexed and anteverted (C) Anteflexed and retroverted (D) Retroverted and retroflexed (E) Anteverted and retroverted

21. The answer is A. The normal position of the uterus is anteverted (i.e., angle of 90 degrees at the junction of the vagina and cervical canal) and anteflexed (i.e., angle of 160 to 170 degrees at the junction of the cervix and body).

28. A 72-year-old woman complains of a cramp-like pain in her thigh and leg. She was diagnosed as having a severe intermit- tent claudication. Following surgery, an infection was found in the adductor canal, damaging the enclosed structures. Which of the following structures remains intact? (A) Femoral artery (B) Femoral vein (C) Saphenous nerve (D) Great saphenous vein (E) Nerve to the vastus medialis

28. The Answer is D. The great saphenous nerve remains intact because it is not in the adductor canal. The adductor canal contains the femoral vessels, the saphenous nerve, and the nerve to the vastus medialis.

28. A 16-year-old boy presents to the emergency department with rupture of the penile urethra. Extravasated urine from this injury can spread into which of the following structures? (A) Scrotum (B) Ischiorectal fossa (C) Pelvic cavity (D) Testis (E) Thigh

28. The answer is A. Extravasated urine from the penile urethra below the perineal membrane spreads into the superficial perineal space, scrotum, penis, and anterior abdominal wall. However, it does not spread into the testis, ischiorectal fossa, pelvic cavity, and thigh because Scarpa fascia ends by firm attachment to the fascia lata of the thigh.

29. A 23-year-old woman visits her obstetrician for an annual checkup. During vaginal examination, which of the following structures may be palpated? (A) Apex of the urinary bladder (B) Fundus of the uterus (C) Terminal part of the round ligament of the uterus (D) Body of the clitoris (E) Uterine cervix

29. The answer is E. In addition to the uterine cervix, the uterus, uterine tubes, ovaries, and ureters can be palpated. The apex of the urinary bladder is the anterior end of the bladder; thus, it cannot be palpated. The fundus of the uterus is the anterosuperior part of the uterus. The terminal part of the round ligament of the uterus emerges from the superficial inguinal ring and becomes lost in the subcutaneous tissue of the labium majus.

The attending faculty in the coronary intensive care unit demonstrates to his students a normal heart examination. The first heart sound is produced by near-simultaneous closure of which of the following valves? (A) Aortic and tricuspid (B) Aortic and pulmonary (C) Tricuspid and mitral (D) Mitral and pulmonary (E) Tricuspid and pulmonary

38. The answer is C. The first heart sound ("lub") is produced by the closure of the tricuspid and mitral valves, whereas the second heart sound ("dub") is produced by the closure of the aortic and pulmonary valves.

40. A 47-year-old man with a known atrial fibrillation returns to see his cardiologist for follow-up of his cardiac health. The right atrium is important in this case because it: (A) Receives blood from the oblique cardiac vein (B) Is associated with the apex of the heart (C) Contains the SA node (D) Receives the right pulmonary vein (E) Is hypertrophied by pulmonary stenosis

40. The answer is C. The SA and AV nodes are in the wall of the right atrium and are not associated with the apex of the heart. The oblique cardiac vein drains into the coronary sinus, and the pulmonary veins empty into the left atrium. The right ventricle is hypertrophied by the pulmonary stenosis.

48. A trauma surgeon in the emergency department at a local center examines a 14-year-old boy with extensive pelvic injuries after a hit and run accident. The surgeon inspects the ischiorectal fossa because it (A) Accumulates urine leaking from rupture of the bulb of the penis (B) Contains the inferior rectal vessels (C) Has a pudendal canal along its medial wall (D) Is bounded anteriorly by the sacrotuberous ligament (E) Contains a perineal branch of the fifth lumbar nerve

48. The answer is B. The ischiorectal fossa is bounded posteriorly by the gluteus maximus and the sacrotuberous ligament. It contains fat, the inferior rectal nerve and vessels, and perineal branches of the posterior femoral cutaneous nerve. The pudendal canal runs along its lateral wall. Urine leaking from a ruptured bulb of the penis does not spread into the ischiorectal fossa because Scarpa fascia ends by firm attachment to the fascia lata of the thigh.

49. A 21-year-old tennis player comes to an emergency room and complains of pain in the knee joint. On examination, he has an infection inside the knee joint capsule but outside the synovial cavity. Which of the following structures is preserved from this infection? (A) Anterior cruciate ligament (B) Posterior cruciate ligament (C) Lateral meniscus (D) Lateral collateral ligament (E) Medial meniscus

49. The Answer is D. The lateral (fibular) collateral ligament extends between the lateral femoral epicondyle and the head of the fibula and is not attached to the lateral meniscus. All other ligaments lie outside the synovial cavity but within the joint capsule.

50. A general surgeon is giving a lecture to a team of surgery residents. She describes characteristics of structures above the pectinate line of the anal canal, which include (A) Stratified squamous epithelium (B) Venous drainage into the caval system (C) Lymphatic drainage into the superficial inguinal nodes (D) Visceral sensory innervation (E) External hemorrhoids

50. The answer is D. The pectinate line is a point of demarcation between visceral and somatic portions of the anal canal. Characteristics above the pectinate line include columnar epithelium, venous drainage into the portal system, lymphatic drainage into the internal iliac nodes, visceral sensory innervation, and internal hemorrhoids.

A newborn female is diagnosed with having albinism, A congenital disorder that is characterized by faulty development of the pigment cells. Which of the other following defects is most likely associated with this order? A. Hirschberg's disease B. Renal agenesis C. Omphalocele D. Anencephaly E. Exstrophy of the urinary bladder

A. Hirschberg's disease

A 12-year-old girl is examined by a school nurse who notices the girl's right scapula is more prominent than the left, her head is not centered directly over the pelvis, and her right hip is raised and more prominent. When the girl is asked to bend forward at the waist, the nurse observes asymmetry of the trunk. Which of the following diagnoses is most likely? (A) Scoliosis (B) Lordosis (C) Kyphosis (D) Osteoporosis (E) Osteoarthritis

A: Scoliosis. Scoliosis (G: crookedness) is abnormal lateral and rotational curvature of the spine that may present with uneven hips, shoulders, and rib cage, a head that is not centered over the pelvis, the entire body leaning to one side, back pain, and/or fatigue. The given Anterior- Posterior (AP) X-ray shows an S-shaped curvature of the spine or vertebral column in this patient. In this X-ray, no apparent vertebral anomaly is apparent, so the diagnosis is most likely adolescent idiopathic scoliosis, which has an onset of 10 to 18 years of age and has no known cause. Most cases of adolescent idiopathic scoliosis (with curvatures of less than 20 degrees) require no treatment; however, if the curvature goes above 25 degrees, a back brace can be implemented to slow the pro- gression of scoliosis. Choice B (Lordosis) is incorrect. Lordosis (G: bending backward) is an anteriorly convex curvature of the vertebral column (spine). The cervical and lumbar vertebral curvatures are normally lordotic; however, excessive lordotic curvature (also called hollow back, swayback, and saddle- back) can be caused by tight lower back muscles, excessive abdominal fat, and pregnancy. Excessive lordosis can lead to lower back pain and can be treated with strengthening of the abdominal muscles and hamstrings. Choice C (Kyphosis) is incorrect. Kyphosis (G: hump-back) is an anteriorly concave curvature of the vertebral column (spine). The thoracic and sacral vertebral curvatures are normally kyphotic; however, deformities of the spine, due to degenerative arthritis, osteo- porosis with compression fractures of the vertebrae, trauma, and developmental problems, can lead to excessive kyphotic curvature (or hunchback). Excessive kyphosis can cause pain and breathing difficulties. The given lateral X-ray shows an example of abnormal kyphotic curvature of the lumbar spine due to benign compression fractures of the L1 and L3 verte- brae secondary to osteoporosis. This disease is characterized by compromised bone strength and decreased bone mass. The affected L1 and L3 vertebrae depict an anterior wedging deformity, which would cause the patient to appear shorter (lose height). Choice D (Osteoporosis) is incorrect. Osteopo- rosis is a disease characterized by compromised bone strength and decreased bone mass, which can lead to an increased rate of fracture in the vertebral column, the mid-forearm, and more frequently the proximal femur resulting in hip fractures. Women are four times more likely to receive the prognosis of osteoporosis, with approximately 25% of women between 65 and 85 years old being diagnosed with osteoporosis. Due to the age of this 14-year-old patient and the findings of the nurse, osteoporosis is not likely. Choice E (Osteoarthritis) is incorrect. Osteoarthritis (or degenerative arthritis) erodes the articular cartilage in primarily weight-bearing joints. Because this type of arthritis is found in older populations, this diag- nosis is not likely in this 14-year-old patient, especially due to the symptoms noted by the nurse.

A 16-year-old boy visits the urologist with a lump in his left testis. Diagnosis of testicular teratocarcinoma is made. This tumor can be loosely referred to as "male pregnancy" because at an early stage the carcinoma contains all three primary germ layers: ectoderm, mesoderm, and endoderm. In normal embryologic development, which of the following processes give rise to these three primary germ layers? A. Morulation B. Gastrulation C. Cranio-caudal folding D. Cleavage E. Induction

B. Gastrulation occurs in early embryogenesis and is the process by which the blastula is reorganized into a three-layered structure, that is, ectoderm, mesoderm, and endoderm. Morulation involves the cleavage or division of the fertilized ovum usually into a 16-cell structure that resembles a ball.

A 25-year-old woman in her 28th week of pregnancy visits her obstetrician for a routine check-up. Her doctor explains that her baby's lungs are starting to produce surfactant, a necessary component for respiration. Surfactant is produced during which of the following periods of lung development? A. Pseudoglandular period B. Canalicular period C. Terminal sac period D. Bronchiole period E. Alveolar period

C. The development of the lungs occurs in four stages: the pseudoglandular stage (6 to 16 weeks), where the lungs resemble exocrine lungs. The parts of the lung involved in gaseous exchange have not yet formed. The canalicular stage (16 to 26 weeks) involves maturation of the bronchioles and development of a few primordial alveoli. The terminal sac stage (26 weeks to birth) is when the alveoli begin maturing, the blood gas barrier is formed, and surfactant is secreted by type 2 pneumocytes. The final stage is the alveolar stage (32 weeks to 8 years); alveoli continue to develop as the lungs prepare for the transition to autonomous gas exchange (Fig. 8-7). 130-135

An early term embryo spontaneously aborts due to incomplete lateral body wall folding and failure of the primitive paired endocardial tubes to merge and form the primitive heart tube. The endocardial tubes are derived from which of the following embryonic sources? A. paraxial mesoderm B. intermediate mesoderm C. splanchnic mesoderm D. parietal mesoderm E. extraembryonic mesoderm

C. splanchnic mesoderm

After several months of engaging in a vigorous exercise program, a 28-year-old woman experiences periodic pain and muscle spasms in her left upper limb. Thorough physical and radiographic examinations by her primary care physician and a consulting surgeon determine that she suffers a posterolateral herniation of the C5-6 intervertebral disc. Which of the following structures is this herniation most likely impinging? (A) C5 spinal nerve roots(B) C5 anterior primary ramus(C) C6 spinal nerve roots(D) C6 posterior primary ramus(E) C7 posterior horn segment of the spinal cord

C: C6 spinal nerve roots. Classic posterolateral herniation of the nucleus pulposus of the intervertebral disc occurs most often within the C4-5 and L4-5 intervertebral discs. A herniated nucleus pulposus may displace or disrupt the posterior longitudinal ligament and extend into the verte- bral canal and impinge upon spinal nerve roots either within the vertebral canal or as they traverse the intervertebral fora- men (or both). When dealing with such cases in the cervical and lower lumbar regions, remember the formula "N+1." That is, N = the number of the intervertebral disc; +1 = the number of the spinal nerve roots primarily contacted by the herniation. Thus, the herniated C5-6 disc will most likely impinge upon the C6 spinal nerve roots. In the cervical region, this formula works because there are eight pairs of cervical spinal nerves but only seven cervical vertebrae. Each numbered spinal nerve exits above the matching numbered vertebra. Thus, the C6 cervical spinal nerve exits the intervertebral foramen formed by the C5 and C6 vertebrae, where it may be impinged. The formula also works in the low lumbar region because of the acute angles the nerve roots take in entering the intervertebral foramina. Here, the numbered spinal nerves exit below the matching vertebrae, so the L4 nerve roots enter the interverte- bral foramen between the L4 and L5 vertebrae. Because these lumbar nerve roots take a very acute turn to enter that open- ing, they are typically held against the upper pedicle and are superior to the level of the intervertebral disc. Thus, the L4 roots would not be impinged by a bulging L4-5 disc. There- fore, the L4-5 intervertebral disc would, instead, push primar- ily against the L5 roots as they align to enter the interverte- bral foramen formed by the L5 and S1 vertebrae. Choice A (C5 spinal nerve roots) is incorrect. Because of the "extra" pair of cervical spinal nerves (compared to the number of verte- brae), the cervical nerves exit the vertebral canal superior to their matching numbered vertebra. Thus, the C5 spinal nerve roots pass into the intervertebral foramen formed by the C4 and C5 vertebrae and would not be impinged by the bulging C5-6 intervertebral disc. Remember the N+1 rule. Choice B (C5 anterior primary ramus) is incorrect. The anterior (ven- tral) and posterior (dorsal) primary rami are the first branches of the spinal nerves, located outside the vertebral canal. Regardless of the spinal segmental level, these structures are not impinged by a herniation of the nucleus pulposus of the intervertebral disc. Choice D (C6 posterior primary ramus) is incorrect. The posterior (dorsal) and anterior (ventral) pri- mary rami are the first branches of the spinal nerves, located outside the vertebral canal. Regardless of the spinal segmen- tal level, these structures are not impinged by a herniation of the nucleus pulposus of the intervertebral disc. Choice E (C7 posterior horn segment of the spinal cord) is incorrect. The C7 posterior horn segment is inferior to the level of the herni- ated disc. Also, this structure is located on the opposite side of the spinal cord from the disc and would not be directly contacted by a herniation of the nucleus pulposus of the inter- vertebral disc.

19. During a snowstorm, a 52-year-old man is brought to the emergency department after a multiple car accident. Which of the following conditions is produced by a force that drives the trunk forward while the head lags behind in a rear-end automobile collision? (A) Scoliosis (B) Hangman's fracture (C) Meningomyelocele (D) Whiplash injury (E) Herniated disk

D

2. A 23-year-old jockey falls from her horse and complains of headache, backache, and weakness. Radiologic examination would reveal blood in which of the following spaces if the internal vertebral venous plexus was ruptured? (A) Space deep to the pia mater (B) Space between the arachnoid and dura maters (C) Subdural space (D) Epidural space (E) Subarachnoid space

D

20. A 37-year-old man is brought to the emergency department with a crushed second cervical vertebra (axis) that he suffered after a stack of pallets fell on him at work. Which of the following structures would be intact after the accident? (A) Alar ligament (B) Apical ligament (C) Semispinalis cervicis muscle (D) Rectus capitis posterior minor (E) Obliquus capitis inferior

D

7. A 27-year-old mountain climber falls from a steep rock wall and is brought to the emergency department. His physical examination and computed tomography (CT) scan reveal dislocation fracture of the upper thoracic vertebrae. The fractured body of the T4 vertebra articulates with which of the following parts of the ribs? (A) Head of the third rib (B) Neck of the fourth rib (C) Tubercle of the fourth rib (D) Head of the fifth rib (E) Tubercle of the fifth rib

D

During meiosis, pairing of homologous chromosomes occurs, which permits large segments of DNA to be exchanged. What is this process called? (A) Synapsis (B) Nondisjunction (C) Alignment (D) Crossing over (E) Disjunction

D. Synapsis (pairing of homologous chromosomes) is a unique event that occurs only during meiosis I in the production of gametes. Synapsis is necessary so that crossing over, in which large segments of DNA are exchanged, can occur.

A 55-year-old man with severe mitral regurgitation is scheduled for minimally invasive mitral valve repair. A percutaneous coronary sinus catheter is placed to deliver retrograde cardioplegia. During the procedure, the catheter stuck in an enlarged thebesian valve. Which of the following embryonic structures gives rise to the coronary sinus? A. Primitive ventricle B. Bulbus cordis C. Truncus arteriosus D. Left horn of the sinus venosus-left common cardinal vein E. Primitive atria

D. The left horn of the sinus venosus gives rise to the coronary sinus. The primitive atria and ventricles will give rise to the atria and ventricles, respectively. Bulbus cordis and truncus arteriosus gives rise to the smooth parts of the right and left ventricles and their corresponding arteries. 190-194

A newborn baby is diagnosed with tricuspid atresia. The given echocardiographic apical four-chamber view shows the abnormal valve plus its typical associated defects: a widely patent foramen ovale (double arrows), ventricular septal defect (single arrow), hypoplastic right ventricle, and hypertrophied left ventricle. The patent foramen ovale most likely reflects a developmental failure of which of the following structures A. endocardial cushions B. valve of the coronary sinus C. ostium primum D. septum secundum E. septum spurium

D. septum secundum

A pregnant woman is in a car accident and goes into premature labor. Her fetus is approximately 24wks in gestation. Her physicians administer her steroids and try to delay the birth of her baby. The survival rate of a premature baby rises significantly if the baby can reach 28wks gestation mainly due to the maturation of the lungs. What stage of lung maturation are the doctors hoping to reach, in which the blood-air barrier is beginning to be established? A. embryonic period B. pseudoglandular period C. canalicular period D. terminal sac period E. alveolar period

D. terminal sac period

A 54yo man presents w/ loss of sympathetic innervation to the left side of his head, characteristic of Horner syndrome. An MRI revealed a left-sided pulmonary sulcus tumor with a location approximated by the black dot to which the arrow is pointing in the given photo. What structure is most likely compromised by the tumor? A. cervical anterior roots B. thoracic posterior primary rami C. thoracic posterior roots D. thoracic anterior roots E. thoracic gray rami communicantes

D. thoracic anterior roots

A 60-year-oldmale with past medical history of diabetes, hypertension, and immune suppression is presented to the hospital with opportunistic mucomycosis fungal infection. Since the vascular invasion of the fungus is well known, a digital subtraction angiography was done. The DSA picture of the patient given below demonstrates a. profunda brachii and posterior circumflex humeral b. internal thoracic and subscapular c. suprascapular and transverse cervical d. transverse cervical and subscapular e. acromiothoracic and lateral thoracic

D. transverse cervical and subscapular

A 73-year-old man comes to his physician complaining of a tingling numbness within the fourth and fifth digits of his right hand. These symptoms are exacerbated when he looks up at the ceiling. The given sagittal T2-weighted MRI reveals degenerative changes in his cervical vertebrae. Which of the following diagnoses is confirmed by the MRI and would result in the symptoms of this patient? (A) Burst fracture of C3 vertebral body (B) Ruptured anterior longitudinal ligament (C) Transection of the cervical spinal cord (D) Cervical spinal stenosis (E) Traumatic spondylolisthesis of C5

D: Cervical Spinal Stenosis. Cervical spinal stenosis describes a narrowing of the vertebral canal that is typically seen in older individuals with degenerative changes in the cervical spine. In the given MRI, the vertebral canal, which houses the spinal cord, is impinged by intervertebral disc degeneration in the midcervical region, which results in no cerebral spinal fluid (CSF), which appears as a hyperin- tense (white) signal in this MRI, being visible anterior or pos- terior to the spinal cord. CSF can be seen surrounding the spinal cord above and below the midcervical region. In this patient, the tingling numbness of the fourth and fifth digits within his right hand is exacerbated by hyperextension of the cervical vertebrae (or looking up), which implies the spinal cord is further compressed by hyperextension of the neck leading to myelopathy (damage to the spinal cord itself). Due to the age of the patient, the degenerative nature of the cervi- cal vertebrae, and the absence of trauma, spinal cord stenosis is the most likely diagnosis, and this condition was confirmed with the given T2-weighted MRI. Choice A (Burst fracture of C3 vertebral body) is incorrect. A burst (crush or compres- sion) fracture of the vertebral body of the 3rd cervical vertebra (C3) would likely result from trauma, and no trauma has been reported in this patient. Moreover, the given MRI does not confirm a burst fracture of C3, though degenerative changes can be seen within this vertebra. A burst fracture of C3 could lead to compression of the spinal cord due to a displaced posterior fracture fragment; however, the given MRI does not confirm this type of injury. Choice B (Ruptured anterior longitudinal ligament) is incorrect. The anterior longitudinal ligament is a vertically running band that attaches along the anterior sides of the vertebral bodies, which resists hyper- extension of the vertebral column. Damage to the anterior longitudinal ligament can occur following traumatic hyper- extension of the head on the neck, which was not reported in this patient. Moreover, damage to (or interruption of) the anterior longitudinal ligament would appear as a hyperintense (white) signal located anterior to the vertebral column at the location of the damage to this ligament, and this evidence is not apparent on this T2-weighted MRI. Choice C (Transec- tion of the spinal cord) is incorrect. Transection of the spinal cord would result in the loss of all sensation and voluntary movements inferior to the site of the lesion. Visual evidence of a transection of the cervical spinal cord is not apparent on this MRI, and because the patient only reported a tingling numb- ness of the fourth and fifth digits within his right hand, this diagnosis is not possible. Choice E (Traumatic spondylolis- thesis of C5) is incorrect. Traumatic spondylolisthesis of C5 describes the anterior displacement of the C5 vertebral body in relation to the C6 vertebra located below it. In this patient, spondylolisthesis of C5 is apparent on the sagittal MRI. How- ever, due to the absence of trauma, the anterior displacement of C5 is probably due to degenerative changes (such as facet arthritis, disc degeneration, and the presence of osteophytes, or bone spurs) within the cervical vertebrae. This radiologic evidence would imply degenerative spondylolisthesis, not traumatic spondylolisthesis, of C5 as the reason for the ante- rior displacement of C5.

During a routine prenatal examination of a pregnant 16-year-old woman entering her second trimester (15 weeks of gestation), elevated maternal serum alpha-fetoprotein (AFP) levels were detected and later confirmed by amniocentesis. A follow-up diagnostic ultrasound examination reveals defects in the lower back of the fetus. In the given longitudinal ultrasound scan, open arrows indicate the bases of the vertebrae, and solid arrows point to a cyst-like sac containing protruding neural tissue. Which of the following terms best describes this neural tube defect? (A) Meningocele (B) Anencephaly (C) Spinabifidaocculta (D) Meningomyelocele (E) Rachischisis

D: Meningomyelocele. A meningomyelocele is the most common form of spina bifida cystica. In this develop- mental disorder, the unfused portion of the vertebral column allows the meninges and the spinal cord to protrude through the structural defect, as noted in this longitudinal ultrasound scan. The protruded portion of the spinal cord is damaged (or not completed developed), and this defect may result in paral- ysis and loss of sensation below the level of the spinal cord defect. This condition is termed spina bifida cystica because of the characteristic presence of a cyst-like sac, composed of meninges and very thin skin, protruding from the defective area. The inclusion of a displaced portion of the spinal cord and nerve roots in the malformation designate this defect as a meningomyelocele. That is, the defect involves the meninges (meningo-), the spinal cord (myelo-), and the membranous sac (-cele). If it reaches full-term, this fetus will likely be paralyzed below the level of the meningomyelocele. In pre- natal tests, elevated levels of AFP in the amniotic fluid and maternal blood are strongly associated with the more severe forms of spina bifida, and an ultrasound scan is ordered to confirm the presence of a neural tube defect. Choice A (Meningocele) is incorrect. The least common form of spina bifida is a posterior meningocele (or meningeal cyst). In this developmental disorder, the bilateral neural arch elements fail to fuse completely in the dorsal midline, but the menin- ges protrude through the defect into a sac or cyst. Therefore, the multiple vertebral defects are accompanied with the presence of a cyst, which contains cerebrospinal fluid (CSF). In a meningocele, the spinal cord and nerve roots are typically in normal position, not protruding into the cyst, and there are usually no long-term effects on the individuals. In this fetus, the ultrasound scan showed a protruding cyst-like sac con- taining neural tissue, which would rule out a meningocele. Choice B (Anencephaly) is incorrect. Anencephaly is a severe neural tube defect in which the cephalic (head) end of the neu- ral tube fails to close in the embryo. This birth defect is lethal with the baby being born without a forebrain, calvaria, and scalp, leaving the remaining portions of the brain exposed. If proper prenatal care is available, most cases of anenceph- aly are detected by elevated maternal serum AFP levels and ultrasound scans during prenatal examinations. Elevated AFP levels are often correlated with neural tube defects, and the addition of folic acid to the diet of women in their childbearing years has been shown to reduce the incidence of neural tube defects. Because the given ultrasound scan localized the neural tube defect in the lower back, anencephaly can be eliminated as a valid answer. Choice C (Spina bifida occulta) is incor- rect. Spina bifida occulta is the mildest form of spina bifida (L: split spine). In this developmental disorder, the bilateral neural arch elements fail to fuse completely in the dorsal mid- line; however, the split in the vertebrae is so small that the meninges and elements of the spinal cord do not protrude through the defect. Because spina bifida occulta is such a mild defect, maternal serum AFP levels are usually not elevated during pregnancy. In fact, full-term babies with spina bifida occulta may be asymptomatic, and most individuals with this condition are unaware of having this mild developmental birth defect. Individuals with spina bifida occulta may possess a tuft of hair, dimple, lipoma, or even a birthmark in the overly- ing skin of the affected region. Some research suggests that approximately 10% of the general population have this mild- est form of spina bifida. Because maternal serum AFP levels would not be elevated in the case of spina bifida occulta, this defect can be eliminated as a valid answer. Choice E (Rachis- chisis) is incorrect. Rachischisis, or spina bifida with myelo- schisis, is the most severe form of spina bifida. In rachischi- sis, the protruded portions of the spinal cord are not afforded the enveloping protection of the meninges, so no cyst-like sac would be present. A physician would see a flattened mass of nervous tissue with no associated membranes, which makes the patient more prone to life-threatening infections. Because the presence of a cyst-like sac was noted in the ultrasound scan, rachischisis can be eliminated as a valid answer.

A 77-year-old man presents with shingles on his anterolateral abdomen and umbilicus, as shown in the figure. Shingles (or herpes zoster) is caused by the varicella zoster virus, which resides latent in sensory ganglia in the body for many years. When a patient is immunocompromised, this virus can cause a painful skin rash called shingles, which usually presents unilaterally along the infected nerve's dermatome distribution. Given the location of the rash, what ganglion is most likely affected in this patient? (A) Paravertebral ganglion (B) Prevertebral ganglion (C) Lumbar ganglion (D) Spinal ganglion of T10 (E) Spinal ganglion of L2

D: Spinal ganglion of T10. In this patient, her- pes zoster, or shingles, is a painful skin rash affecting the der- matome distribution pattern of the left 10th thoracic (T10) nerve, as evidenced by the involvement of the umbilicus. Shingles is seen in patients who have had previous exposure to the varicella zoster virus, which causes chickenpox in chil- dren or young adults. After the initial exposure to chickenpox, this virus can reside latent in ganglia of an individual for years. If this individual becomes immunocompromised, the skin (or dermatomes supplied by the infected ganglia) can develop shingles, a painful skin rash, which blisters, breaks open, crusts over, and then disappears. In this patient, the herpetic lesions were found in the sensory distribution of the left T10 nerve, which means the virus resides in this nerve's sensory ganglion, or the spinal ganglion of T10. Remember that two types of ganglia exist: sensory (afferent) and autonomic. The sensory ganglia, which are most often affected by shingles, are equivalent to the spinal (dorsal root) ganglia of spinal nerves in that they house typical pseudounipolar cell bodies of affer- ent neurons and do not contain synapses. If a spinal ganglion were infected by the varicella zoster virus, shingles may pres- ent in its dermatome distribution patterns when the patient is immunocompromised. Choice A (Paravertebral ganglion) is incorrect. The paravertebral (sympathetic) ganglia are located within the sympathetic trunk. They receive efferent fibers from the presynaptic (preganglionic) sympathetic neurons originating in the intermediolateral (IML) cell column of the thoracic and upper lumbar segments (T1-L2). Due to their location, these sympathetic ganglia are called paravertebral ganglia. Before it causes shingles (or herpes zoster), the vari- cella zoster virus resides latent in sensory ganglia for many years. Because the paravertebral ganglia are autonomic gan- glia, it is unlikely they would be responsible for the skin rash in this patient. Choice B (Prevertebral ganglion) is incorrect. The prevertebral ganglia are sympathetic ganglia that receive their name as they are located anterior to the vertebral column, as distinguished from the ganglia of the sympathetic trunk (paravertebral ganglia). Prevertebral ganglia exist near major branches of the abdominal aorta in the abdominopelvic cavity. These ganglia send postsynaptic (postganglionic) sympathetic fibers to the abdominopelvic organs with periarterial plexuses. Before it causes shingles (or herpes zoster), the varicella zoster virus resides latent in sensory ganglia for many years. Because the prevertebral ganglia are autonomic ganglia, it is unlikely they would be responsible for the skin rash in this patient. Choice C (Lumbar ganglion) is incorrect. The lumbar ganglia are paravertebral ganglia located in the abdominopelvic part of the sympathetic trunk. Before it causes shingles (or herpes zoster), the varicella zoster virus resides latent in sensory gan- glia for many years. Because the lumbar ganglia are autonomic ganglia, it is unlikely they would be responsible for the skin rash in this patient. Choice E (Spinal ganglion of L2) is incor- rect. The spinal (posterior root) ganglion is a sensory ganglion, so it could be infected by the varicella zoster virus and present with a painful, skin rash along its dermatome distribution pat- tern. The sensory distribution of L2 would extend along the back, hip, and extend into the anterior and medial aspects of the thigh. Because this patient presents with a skin rash at the level of the umbilicus, the spinal ganglion of T10 would be the source of this painful skin rash.

A 16-year-old male soccer player was brought to the ER because of the acute lower back pain he experienced after performing a high-velocity kick of the ball. The given right posterior oblique X-ray reveals pathology within the L5 vertebra that has the appearance of a "headless Scottie Dog" when outlined. Based upon the X-ray results, which of the following diagnoses is the most likely cause of his lower back pain? (A) Ankylosing spondylitis (B) High-grade spondylolisthesis (C) Spondyloptosis (D) Spondylolysis (E) Spondylosis

D: Spondylolysis. Spondylolysis refers to a defect in the pars interarticularis of the affected vertebra (or L5 in this patient). Its meaning is derived from the Greek words "spondylos" (G: vertebra) and "lysis" (G: loosening). The given right posterior oblique X-ray reveals a fracture in the isthmus (or neck) of the pars interarticularis or the bony column formed by the superior and inferior articular processes of L5. This fracture has the appearance of a "headless Scottie Dog" in this image. To visualize the Scottie Dog, look at the intact L4 vertebra, and remember that the ear is formed by the superior articular process, the head is formed by the transverse process, its eye is the radiopaque pedicle, its foreleg is the inferior articular process, its body is represented by the lamina and spinous process, and the hind leg of the dog is the opposite inferior articular process. Because there is a fracture within the isthmus (or neck) of the pars interarticularis of L5, spondylolysis of L5 is the proper diagnosis. Choice A (Ankylosing spondylitis) is incorrect. Ankylosing spondylitis is arthritis of the spine, which presents with inflammation, stiffening of the joint, and potentially bony ankylosis (synostosis) due to ossification of the posterior and anterior longitudinal ligaments. Ankylosing spondylitis is derived from the Greek words "ankylosis" (G: stiffening of the joint), "spondylos" (G: vertebra), and "itis" (G: inflammation). Ankylosing spondylitis, a degenerative joint disease of the spine, with an autoimmune component. However, this diagnosis is not sup- ported by the given lumbar spine X-ray. Choice B (High-grade spondylolisthesis) is incorrect. Spondylolisthesis describes the anterior displacement of a vertebra in relation to the vertebra below it. Its meaning is derived from the Greek words "spondylos" (G: vertebra) and "olisthesis" (G: slipping and falling). A high-grade (III or IV) spondylolisthesis implies the anteriorly displaced posterior edge of the L5 vertebral body is located anterior to the center of the S1 vertebral body. A high-grade spondylolisthesis means the vertebral body of L5 is displaced anteriorly by the width of more than half of the vertebral body of S1. The given lumbar spine X-ray does not support this diagnosis. Choice C (Spondyloptosis) is incorrect. Spondyloptosis is the most extreme type of spondylolisthesis in which the posterior edge of the L5 vertebra is dis- located anterior to the entire S1 vertebral body, so it would fall off the vertebral column. "Spondyloptosis" is derived from the Greek words "spondylos" (G: vertebra) and "ptosis" (G: falling). This extreme form of spondylolisthesis was not supported by the given lumbar spine X-ray. Choice E (Spondylosis) is incorrect. Spondylosis is a term that implies stiffening of the spine (ankylosis) due to any lesion of the spine of a degenerative nature. Due to the young age of this male athlete, a degenerative joint disease of the spine described by the word "spondylosis" is highly unlikely. Furthermore, this diagnosis is not supported by the given lumbar spine X-ray.

A 65-year-old woman suffers chronic constipation. Which of the following events occurs when she strains heavily in attempting to pass her stool? (A) Relaxation of the pubococcygeus (B) Relaxation of the puborectalis (C) Relaxation of the coccygeus (D) Contraction of the external anal sphincter (E) Contraction of the internal anal sphincter

The answer is B: Relaxation of the puborectalis. The pub- orectalis is the component of the levator ani that joins with its opposite side mate to form a U-shaped sling around the anorectal junction, as shown in the given figure. Tension of this muscle produces the anorectal angle (perineal flexure), which aids significantly in resisting fecal movement. Relax- ation of the puborectalis sling causes the anorectal junction to straighten, thus allowing defecation to proceed. The other parts of the levator ani normally contract during defecation. This action raises the pelvic floor and assists the abdomi- nal wall muscles in increasing intra-abdominal pressure and compressing the abdominopelvic contents, thus assisting in expelling feces during defecation. Thus, the complementary actions of relaxation of the puborectalis plus contraction of the remainder of the levator ani are important in voluntary control of defecation. Choice A (Relaxation of the pubococcygeus) is incorrect. The pubococcygeus is the major component of the levator ani. It contracts to raise the pelvic floor during def- ecation and other actions requiring increased intra-abdominal pressure (e.g., forced expiration, sneezing, vomiting, and uri- nation). Choice C (Relaxation of the coccygeus) is incorrect. The coccygeus is the most posterior component of the pelvic diaphragm. However, it is not part of the levator ani portion of the pelvic diaphragm. Even so, it contracts in the same fashion as the levator ani during defecation to raise and tense the pel- vic floor. Choice D (Contraction of the external anal sphinc- ter) is incorrect. The external anal sphincter is the voluntary sphincter surrounding the lower anal canal. It must relax in order to allow dilation of the anal canal and passage of feces. Choice E (Contraction of the internal anal sphincter) is incor- rect. The internal anal sphincter is the involuntary sphincter in the lower rectal wall. It relaxes during defecation to allow passage of feces into the anal canal.

A pregnant woman in active labor receives an epidural anesthetic to relieve pain from her uterine contractions and cervical dilation in preparation for the birth of her child. Pain derived from the labor contractions of the upper uterus is referred to what spinal cord level? (A) T6 (B) T12 (C) L4 (D) S1 (E) S3

The answer is B: T12. Visceral sensory fibers relaying pain from the labor contractions of the upper uterus are referred to the T11-L2 spinal cord levels because these organs are located above the pelvic pain line, as defined by their being in con- tact with or above the peritoneum. Because visceral sensory fibers follow the visceral motor (autonomic) fibers retrograde, visceral pain from the contractions of the upper uterus fol- low the presynaptic sympathetic nerves of the abdominopel- vic splanchnic nerves retrograde to spinal cord levels T11-L2. Conversely, pelvic organs located below the pelvic pain line are, by definition, located below, and not in contact with, the peri- toneum. The visceral sensory fibers from these organs follow the presynaptic parasympathetic nerves of the pelvic splanch- nic nerves retrograde to spinal cord levels S2-4. Therefore, labor pains derived from the contractions of the upper uterus will be referred by visceral sensory fibers to the T12 spinal cord level. Choice A (T6) is incorrect. Visceral sensory fibers associated with the T6 vertebral level would be involved with conveying visceral pain associated with the heart and lungs, due to traveling retrograde with cardiopulmonary splanch- nic nerves derived from postsynaptic sympathetic fibers from T1-6. Choice C (L4) is incorrect. Visceral sensory fibers car- rying pain from visceral organs and involuntary (smooth or cardiac) muscle follow retrograde the pathway of the visceral motor (or autonomic) nerve fibers. There are no presynaptic autonomic nerve fibers that arise from the L4 vertebral level, so there are no visceral sensory nerves associated with this ver- tebral level. Choice D (S1) is incorrect. Visceral sensory fibers carrying pain from visceral organs and involuntary (smooth or cardiac) muscle follow retrograde the pathway of the visceral motor (or autonomic) nerve fibers. There are no presynaptic autonomic nerve fibers that arise from the S1 vertebral level, so there are no visceral sensory nerves associated with this ver- tebral level. Choice E (S3) is incorrect. Pain derived from labor contractions of the upper uterus would not refer to vertebral level S3, because the upper uterus is in contact with the peri- toneum, which means it is located above the pelvic pain line. However, labor pains from the dilation of the uterine cervix, an important stage of labor, will follow the presynaptic para- sympathetic nerves of the pelvic splanchnic nerves retrograde to the vertebral levels of S2-4. Therefore, the uterine cervix is a pelvic organ located below the pelvic pain line, which means it is located below, and not in contact with, the peritoneum. Therefore, the vertebral level of S3 would receive visceral sen- sory fibers conveying pain from labor due to the dilation of the uterine cervix; however, this question asked specifically about labor pains derived from the contractions of the upper uterus, which would be referred to the T11-L2 vertebral levels.

11 A 23-year-old man was injured in a motor vehicle accident and X-rays confirmed a displaced distal radius fracture in his left forearm. Upon examination, the patient exhibits weakened pronation, weakened flexion of the index and middle fingers at the distal interphalangeal joints, and weakened flexion of the interphalangeal joint of the thumb. When asked to make the "okay" sign (make a circle with the thumb and index finger), the patient is unable to make a round circle, producing a "collapsed circle" on the affected hand (see photo). No areas of sensory loss are detected. Which nerve is most likely damaged? (A) Deep branch of the radial nerve (B) Superficial branch of the radial nerve (C) Anterior interosseous nerve (D) Median nerve, proximal to the carpal tunnel (E) Recurrent branch of the median nerve

The answer is C: Anterior interosseous nerve. The anterior interosseous nerve is a branch of the median nerve in the distal part of the cubital fossa, and it courses distally on the interosseous membrane. It supplies the deep forearm fl exors, including the fl exor digitorum profundus of digits 2 and 3, the fl exor pollicis longus, and the pronator quadratus. Loss of this nerve would cause weakness in pronation due to denervation of the pronator quadratus. This injury would also result in inability to fl ex the distal interphalangeal joints of the index and middle fi ngers and the interphalangeal joint of the thumb due to denervation of the fl exor digitorum profundus and fl exor pollicis longus, respectively. This defi cit would lead to the collapsed "O.K. sign" indicated in the photo. Because compromising the anterior interosseous nerve would not result in any cutaneous sensory defi cits, it is this nerve that was most likely damaged by the displaced end of the left radius. Choice A (Deep branch of the radial nerve) is incorrect. The deep branch of the radial nerve arises from the radial nerve in the cubital fossa, dives deep to pierce the supinator muscle, and supplies all the muscles in the posterior compartment of the forearm. Damage to this nerve would lead to "wrist drop" in the patient due to the fl exors of the forearm being unopposed and an inability to extend at the wrist. Choice B (Superfi cial branch of the radial nerve) is incorrect. The superfi cial branch of the radial nerve arises from the radial nerve in the cubital fossa. This nerve is entirely a sensory nerve, supplying the dorsum of the hand and fi ngers. Because the patient has motor defi cits, this nerve was not damaged by the displaced radius. Choice D (Median nerve proximal to the carpal tunnel) is incorrect. The median nerve proximal to the carpal tunnel would not be injured because damaging this nerve would lead to a signifi cant sensory loss over the thumb and the adjacent two and a half fi ngers in addition to motor loss to the thenar eminence (opponens pollicis, abductor pollicis brevis, and fl exor pollicis brevis muscles) and the fi rst two lumbrical muscles. So, the median nerve was not injured proximal to the carpal tunnel. Choice E (Recurrent [thenar] branch of median nerve) is incorrect. The recurrent (thenar) branch of the median nerve innervates most of the thenar muscles, including the abductor pollicis brevis, opponens pollicis, and superfi cial head of the fl exor pollicis brevis. Cutting this nerve would lead to atrophy of the thenar muscular complex, a condition known as "ape hand." In this condition, the patient would not be able to oppose the thumb and second fi nger. Though this nerve has no sensory distribution, it is unlikely that it was involved due to the weakness when pronating the forearm.

8 An 18-year-old boy is cut severely on the lateral wall of his right chest during a knife fight. Following healing, the medial border of his scapula moves away from the thoracic wall when he leans on his right hand. Which of the following nerves is likely damaged? (A) Axillary nerve (B) Thoracodorsal nerve (C) Long thoracic nerve (D) Dorsal scapular nerve (E) Suprascapular nerve

The answer is C: Long thoracic nerve. The photo demonstrates a case of "winged scapula," indicative of a lesion of the long thoracic nerve and subsequent paralysis of the serratus anterior muscle. The nerve runs down the lateral thoracic wall, on the superfi cial aspect of the serratus anterior, where it is unusually exposed (for a motor nerve) and vulnerable to injury, especially when the limb is elevated. Lesion of the nerve denervates the serratus anterior. This results in the medial border and inferior angle of the scapula pulling away from the posterior chest wall, giving the scapula a wing-like appearance when the affected limb is protracted. Additionally, the affected arm cannot be abducted above the horizontal plane because the serratus anterior is not available to superiorly rotate the glenoid cavity of the scapula to allow full abduction. Choice A (Axillary nerve) is incorrect. This nerve passes deeply through the axilla, around the surgical neck of the humerus, to supply the teres minor and deltoid muscles. Lesion here would result in signifi cant weakness in abduction of the arm and wasting of the rounded contour of the shoulder. Choice B (Thoracodorsal nerve) is incorrect. This nerve runs inferior through the axilla to supply the latissimus dorsi muscle. Loss of the nerve would result in weakness in extension and medial rotation of the arm, plus wasting of the posterior axillary fold. Choice D (Dorsal scapular nerve) is incorrect. This nerve courses into the upper, medial part of the back and the lower neck to supply the levator scapulae and rhomboid muscles. Paralysis of these muscles would result in weakness in elevation and retraction of the scapula and perhaps wasting of the contour of the back under the trapezius muscle. Choice E (Suprascapular nerve) is incorrect. The suprascapular nerve runs through the suprascapular notch and into the supraspinous and infraspinous fossae to supply the supraspinatus and infraspinatus muscles. Lesion of this nerve would result in weakness in the rotator cuff affecting the initiation of abduction (supraspinatus) and external rotation (infraspinatus) of the shoulder and wasting of the muscular contour of the posterior aspect of the scapula.

A 26-year-old medical student noticed a lump on his left testis, which was later diagnosed as testicular cancer. To rule out metastases of the testicular cancer, what group of lymph nodes should be checked in the first instance? (A) Superficial inguinal (B) Deep inguinal (C) Lumbar (D) External iliac (E) Internal iliac

The answer is C: Lumbar. The lymphatic drainage of the testes goes directly to the lumbar lymph nodes due to their develop- mental origin in the superoposterior abdominal wall. Lumbar lymph nodes, which are located anterior to the lumbar vertebrae and surrounding the inferior vena cava and abdominal aorta, also receive lymphatic drainage directly from the posterior abdominal wall, kidneys, ovaries, ureters, uterus, and uterine tubes. Because most lymphatic vessels supplying pel- vic viscera follow arteries retrograde, lymph from the testes, which receive arterial blood from testicular artery arising off the abdominal aorta at vertebral level L1, terminates primarily in the lumbar lymph nodes. Therefore, the lumbar lymph nodes should be checked initially to rule out metastases in the case of testicular cancer. Remember, when considering spread of cancer from pelvic visceral organs, it is crucial to remember the arterial supply of the organ in question. Choice A (Super- ficial inguinal) is incorrect. Superficial inguinal lymph nodes, which are located in the subcutaneous tissue near the termina- tion of the great saphenous vein, receive lymphatic drainage from the lower abdominal wall, buttock, lower limb, and all of the perineum and external genitalia, except for the glans penis and its distal spongy urethra. Despite the fact that the scrotum drains to the superficial inguinal lymph nodes, the testes drain to the lumbar nodes due to their descent from the posterior abdominal wall in development. Choice B (Deep inguinal) is incorrect. Deep inguinal lymph nodes, which are located deep to the fascia lata and medial to the femoral vein, receive lymphatic drainage directly from the glans penis, the distal spongy urethra of the penis, deep structures of the lower limb, and indirect lymphatic drainage from the superficial inguinal lymph nodes. Due to their descent from the posterior abdomi- nal wall in development, the lymphatic drainage of the testes goes directly to the lumbar lymph nodes. Choice D (External iliac) is incorrect. The external iliac lymph nodes, which are associated with the external iliac vein, receive lymphatic drain- age from pelvic organs in direct contact with the peritoneum, including the superior bladder and superior pelvic ureters. Even though the tunica vaginalis is a serous covering of the testes derived from the peritoneum, the lymphatic drainage of the testes goes directly to the lumbar lymph nodes due to their descent from the posterior abdominal wall in develop- ment. Choice E (Internal iliac) is incorrect. The internal iliac lymph nodes, which are located along the internal iliac artery and its branches, receive lymphatic drainage from the pelvic viscera, gluteal region, and deep parts of the perineum. The testes drain to the lumbar nodes due to their descent from the posterior abdominal wall in development and their arterial supply from the testicular artery arising off the abdominal aorta.

A 58-year-old postmenopausal mother (of four children) goes to her gynecologist complaining of discomfort and problems during micturition (emptying of the bladder). Physical examination reveals a herniation in the anterior aspect of the vaginal orifice, as illustrated below. What is the most likely diagnosis? (A) Urinary tract infection (B) Hydrocele (C) Urethrocele (D) Cystocele (E) Rectocele

The answer is D: Cystocele. A cystocele is a herniation of the urinary bladder into the anterior aspect of the vagina through a tear in the pubocervical fascia, which separates the bladder from the vagina. This fascia is often torn during childbirth (parturition), creating a passage for the hernia- tion of the bladder. A cystocele often causes discomfort or problems during micturition due to the change in the nor- mal position of the bladder. Because estrogen strengthens the elastic tissues around the vagina, a cystocele may not occur until menopause, when levels of estrogen decrease. Therefore, a cystocele is the most likely diagnosis due to the patient being a postmenopausal mother (of four children) and the location of the herniated bladder in the anterior aspect of the vaginal orifice. Choice A (Urinary tract infec- tion) is incorrect. Patients with urinary tract infections often present with dysuria, pain during voiding of urine, which is reported in this patient. However, this diagnosis does not account for the herniation in the vaginal orifice. Choice B (Hydrocele) is incorrect. A hydrocele testis is a pathological accumulation of serous fluids within the spermatic cord of the testis due to serous secretions from a remnant piece of peritoneum, termed the tunica vaginalis. This diagnosis can be easily eliminated in this female patient. Choice C (Ureth- rocele) is incorrect. A urethrocele is a prolapse of the urethra into the vestibule of the vagina, in which the inner lining of the urethra sticks out through the external urethral orifice. A urethrocele may occur with a cystocele, but in these cases, a distinct groove is usually seen between the border of the prolapsed urethra and bladder, not seen in this illustration. Interestingly, a urethrocele may occur in prepubertal girls and presents as a small pink donut-shaped mass (with the external urethra orifice making the hole of the donut) in the vestibule of the vagina. This condition is often detected by a parent who finds a small amount of blood in the child's underwear, and it can be treated with an estrogen cream to strengthen the elastic fibers around the urethra. Anatomical knowledge of the location of the female urethra being ante- rior to the vagina rules out this diagnosis because the hernia- tion was located in the anterior aspect of the vaginal orifice. Choice E (Rectocele) is incorrect. A rectocele is a herniation of rectal tissue through a tear in the rectovaginal septum, which separates the rectum from the vagina. It is detected in the posterior aspect of the vaginal orifice. A rectocele may be present in women following childbirth or hysterectomy, but this diagnosis can be ruled out by the location of the hernia- tion in the anterior aspect of the vaginal orifice.

43 As part of a physical examination to evaluate intrinsic hand muscle function, a physician asks the patient to assume the Z-position (seen in photo) with his hand, which involves flexion of the metacarpophalangeal joints and extension of the interphalangeal joints of the fingers. Which of the following nerves is being tested in assuming this position? (A) Deep branch of radial nerve (B) Superficial branch of radial nerve (C) Recurrent branch of median nerve (D) Deep branch of ulnar nerve (E) Superficial branch of ulnar nerve

The answer is D: Deep branch of ulnar nerve. The deep branch of the ulnar nerve innervates most of the intrinsic muscles of the hand, including the hypothenar muscles, medial two lumbrical muscles, the palmar and dorsal interossei, adductor pollicis, and the deep head of the fl exor pollicis brevis. Specifi cally, the palmar interossei, dorsal interossei, and lumbrical muscles insert into the extensor digital expansion to provide, collectively, fl exion of the metacarpophalangeal joints and extension of the interphalangeal joints of the fi ngers. All of these muscles, with the exception of the fi rst and second lumbrical muscles, are innervated by the deep branch of the ulnar nerve, so this nerve is being tested by placing the hand in the Z-position. If the deep branch of the ulnar nerve were severed, the hand would assume the opposite of the Z-position, which is called "claw hand." Choice A (Deep branch of radial nerve) is incorrect. The radial nerve descends from the arm into the cubital fossa, where it divides into superfi cial and deep branches. The deep branch of the radial nerve pierces the supinator muscle, winds around the proximal end of the radius within the substance of that muscle, and passes into the deep posterior compartment of the forearm as the posterior interosseous nerve. The nerve may be entrapped within the supinator, resulting in supinator syndrome. Such a condition would affect the deeper, more distal extensor muscles arising in the forearm and some sensory areas in the wrist joints. It is not responsible for the Z position of the hand. Choice B (Superfi cial branch of radial nerve) is incorrect. The superfi - cial branch of the radial nerve arises from the radial nerve in the cubital fossa. This nerve is purely a sensory nerve, supplying the dorsum of the hand and fi ngers. Because it does not have a motor component, this nerve would not be responsible for assuming the Z position of the hand. Choice C (Recurrent branch of median nerve) is incorrect. The recurrent (thenar) branch of the median nerve innervates most of the thenar muscles, including the abductor pollicis brevis, opponens pollicis, and superfi cial head of the fl exor pollicis brevis. Cutting this nerve would lead to atrophy of the thenar muscular complex, a condition known as "ape hand." The recurrent branch of the median nerve does not supply muscles outside of the thumb, so it would not be responsible for assuming the hand's Z position. Choice E (Superfi cial branch of ulnar nerve) is incorrect. The superfi cial branch of the ulnar nerve arises from the ulnar nerve distal to the fl exor retinaculum. This nerve supplies cutaneous branches to the anterior surface of the medial one and a half fi ngers. The palmaris brevis is the only muscle supplied by this nerve, and this muscle tightens the skin of the medial surface of the palm. Therefore, this nerve would not be responsible for the Z position of the hand.

A 67-year-old man falls off his roof while cleaning his gutters. The impact of the landing causes a dislocation of his right hip joint and fractures his right superior and inferior pubic rami. Which of the following structures is most likely damaged in company with this extensive trauma to the right lateral wall of the true pelvis? (A) Pubic symphysis (B) Piriformis muscle (C) Sacral promontory (D) Obturator internus muscle (E) Ganglion impar

The answer is D: Obturator internus muscle. The lateral wall of the true (lesser) pelvis is composed of the iliopubic ramus, ischiopubic ramus, body of the ischium, and the obturator foramen (mostly closed by the obturator membrane). The obturator internus muscle lies on the lateral wall of the true pelvis, surrounding and covering the obturator foramen. Frac- tures of the pubic and obturator areas are relatively common. In the case of a lateral wall fracture, the obturator internus muscle can easily be torn in association with disruption of the bony elements and/or the obturator membrane. Choice A (Pubic symphysis) is incorrect. Many students believe that the pubic symphysis forms the anterior border of the pelvis. However, the pelvis is tilted in its normal anatomical posi- tion such that the anterior superior iliac spines and the pubic tubercles are aligned in the same vertical plane. This orien- tation causes the bodies of the pubic bones and the sym- physis to actually occupy the floor of the true pelvis, where they take on a weight-bearing function. Choice B (Piriformis muscle) is incorrect. The piriformis muscle originates from the pelvic surface of the superior part of the sacrum, lateral to the anterior sacral foramina. It exits the pelvis through the greater sciatic foramen to reach the greater trochanter of the femur. Because the piriformis fills a large extent of the greater sciatic foramen, it forms the posterolateral wall of the true pelvis. Choice C (Sacral promontory) is incorrect. The sacral promontory is the anterosuperior lip of the S1 element of the sacrum. It forms the posterior midline part of the bony edge that defines the pelvic inlet (pelvic brim; superior pel- vic aperture) and separates the false (greater) pelvis from the true (lesser) pelvis. Due to the orientation of the pelvis in the anatomical position, the sacrum and coccyx form the roof of the posterior wall of the true pelvis. Choice E (Ganglion impar) is incorrect. The ganglion impar is the single, midline coccygeal ganglion that forms the inferior end of the sympa- thetic chain. It lies on the pelvic surface of the sacrum. Thus, it may be damaged in fractures of the posterior wall of the true pelvis.

A 15-year-old girl delivering her first child at home with only her mother's help experiences a difficult birthing. As the baby's head passes through the birth canal, the perineum begins to tear. Which of the following muscles is the most likely to tear in this event? (A) Coccygeus (B) Ischiocavernosus (C) Obturator internus (D) Pubococcygeus (E) External anal sphincter

The answer is D: Pubococcygeus. The pubococcygeus, the major component of the levator ani, is the muscle most often torn during childbirth. The pelvic diaphragm supports the fetal head during delivery. The pubococcygeus plays a key mechanical role in that it encircles the urethra, vagina, and anal canal and provides the main support for these organs during birthing. However, because of its proximity to the vagina, it also is subject to tearing if the perineum ruptures. Subsequent weakening of the muscle and associated pelvic fascia may cause changes in the positioning of the bladder and urethra that may lead to urinary stress incontinence. Choice A (Coccygeus) is incorrect. The coccygeus is the most posterior component of the pelvic diaphragm. However, it is not part of the levator ani portion of the pelvic diaphragm. Because of its relative distance from the birth canal, the coccygeus is less subject to the stresses applied to the pubococcygeus during delivery, therefore less subject to tearing. Choice B (Ischio- cavernosus) is incorrect. The ischiocavernosus is the thin skeletal muscle layer surrounding the crus of the clitoris. The crura are attached to the ischiopubic rami, well lateral to the birth canal. Thus, the levator ani will normally tear before that opening reaches the crus of the clitoris and the ischiocaverno- sus muscle, which envelops it. Choice C (Obturator internus) is incorrect. The obturator internus is applied to the internal lateral wall of the true pelvis and sends its tendon of insertion out of the lesser sciatic foramen into the gluteal region. This muscle is far removed from the birth canal, and it is highly unlikely to be damaged during childbirth. Choice E (Exter- nal anal sphincter) is incorrect. The external anal sphincter surrounds the lower anal canal. It is not part of the pelvic diaphragm. Because of its proximity to the vagina, it may be damaged during extensive tearing of the perineum. However, the pubococcygeus normally tears before the external anal sphincter becomes involved.

30 A 74-year-old man complains of pain in his right hand and fingers when he works with his hands for a while. Thorough testing reveals insufficient blood flow into the deep palmar arch. Occlusion of which of the following arteries is the most likely cause of this condition? (A) Posterior interosseous artery (B) Ulnar artery (C) Anterior interosseous artery (D) Radial artery (E) Inferior ulnar collateral artery

The answer is D: Radial artery. The radial and ulnar arteries anastomose to form the superfi cial and deep palmar arches. However, the superfi cial palmar arch is formed mainly by the ulnar artery, whereas the deep palmar arch is formed primarily by the radial artery. Thus, the insuffi ciency in this case is most likely due to reduced fl ow in the radial artery. Choice A (Posterior interosseous artery) is incorrect. The posterior interosseous artery branches off the common interosseous artery, which is derived from the ulnar artery in the proximal forearm. The posterior interosseous artery supplies the posterior compartment of the forearm. This artery does not reach the wrist and does not contribute blood to the palmar arterial arches. Choice B (Ulnar artery) is incorrect. The ulnar artery provides the primary supply into the superfi cial palmar arterial arch. The deep palmar arch is formed primarily by the radial artery, and it is the reduced fl ow of the radial artery most likely present in this patient. Choice C (Anterior interosseous artery) is incorrect. The anterior interosseous artery branches off the common interosseous artery, which is derived from the ulnar artery in the proximal forearm. The anterior interosseous artery supplies the anterior compartment of the forearm as well as the distal posterior forearm via its posterior terminal branch. This artery does reach the wrist, but it does not contribute blood to the palmar arterial arches. Choice E (Inferior ulnar collateral artery) is incorrect. This artery branches off the brachial artery just above the elbow. It passes distally across the anterior side of the medial epicondyle of the humerus to form a collateral connection with the anterior ulnar recurrent branch of the ulnar artery. This vessel is far removed from the hand.h

In a 15-day embryo, the epiblast is capable of forming which of the following germ layers? (A) Ectoderm only (B) Ectoderm and mesoderm only (C) Ectoderm and endoderm only (D) Mesoderm and endoderm only (E) Ectoderm, mesoderm, and endoderm

The answer is E: Ectoderm, mesoderm, and endoderm. The epiblast is capable of forming all three germ layers (ectoderm, mesoderm, and endoderm) during gastrulation. Epiblast cells migrate to the primitive streak and invaginate into a space between the epiblast and the hypoblast

18 A physician tests the myotatic biceps reflex. A normal response of involuntary contraction of the biceps brachii muscle is noted. This reflex confirms the integrity of what nerve? (A) Axillary nerve (B) Median nerve (C) Ulnar nerve (D) Radial nerve (E) Musculocutaneous nerve

The answer is E: Muculocutaneous nerve. A positive response to the myotatic biceps refl ex confi rms the integrity of the musculocutaneous nerve and the C5 and C6 spinal segments, from which this nerve arises. The musculocutaneous nerve supplies motor innervation and proprioception to the muscles in the anterior compartment of the arm, including the coracobrachialis, biceps brachii (tested here), and brachialis. Lesioning the musculocutaneous nerve would lead to loss of proprioception and weakness in fl exing the elbow (via the biceps brachii and brachialis muscles) and supinating the forearm (via the biceps brachii), resulting in a negative myotatic biceps refl ex. Choice A (Axillary nerve) is incorrect. The axillary nerve innervates the deltoid and teres minor muscles as well as the skin overlying the deltoid in the superolateral aspect of the arm. Loss of innervation to the deltoid muscle would cause weakness in abduction of the upper limb. The teres minor and posterior head of the deltoid are responsible for external rotation of the shoulder. Choice B (Median nerve) is incorrect. The median nerve innervates 6.5 of the 8 anterior forearm muscles. This nerve would help with fl exion of the forearm at the elbow; however, the myotatic biceps refl ex test specifi cally tests the innervation of the biceps brachii muscle, which is innervated by the musculocutaneous nerve. Choice C (Ulnar nerve) is incorrect. The ulnar nerve innervates only 1.5 muscles of the forearm, specifi cally the fl exor carpi ulnaris and the ulnar side of the fl exor digitorum profundus. This nerve also innervates most of the intrinsic muscles of the hand. It is not involved with the myotatic biceps refl ex test. Choice D (Radial nerve) is incorrect. The radial nerve supplies motor innervation to the posterior compartments of the arm and forearm. Damage to this nerve would cause weakness in extension at the elbow and wrist joints. The radial nerve is responsible for the posterior cutaneous

A pregnant woman in active labor receives an epidural anesthetic to relieve pain from her uterine contractions and cervical dilation in preparation for the birth of her child. Visceral sensory fibers project the pain derived from the dilation of the uterine cervix to what spinal cord level? (A) T4 (B) L1 (C) L4 (D) S1 (E) S3

The answer is E: S3. Visceral sensory fibers relaying pain from the lower part of the uterus (cervix) and upper part of the vagina are referred to the S2-4 spinal cord levels, as these organs are located below the pelvic pain line, which generally corresponds to the lower limit of the peritoneum. Because visceral sensory fibers follow the visceral motor (autonomic) fibers retrograde, visceral pain from the uterine cervix will follow the presynaptic parasympathetic nerves of the pelvic splanchnic nerves retrograde to spinal cord levels S2-4. Conversely, pelvic and lower abdominal organs located above the pelvic pain line are, by definition, in contact with or located above the peritoneum, and their visceral sensory fibers follow the visceral motor fibers retrograde to refer pain to the lowest limits of the sympathetic (thoracolumbar) nervous system located at spinal cord levels T11-L2. Therefore, labor pains derived from the dilation of the uterine cervix will be referred by visceral sensory fibers to the S3 spinal cord level. Choice A (T4) is incorrect. Visceral sensory fibers associated with the T4 spinal cord level would be involved in convey- ing visceral pain associated with the heart and lungs, due to traveling retrograde with cardiopulmonary splanchnic nerves derived from postsynaptic sympathetic fibers from T1 to T6. Choice B (L1) is incorrect. The uterine cervix is located below the peritoneum, which implies it is below the pelvic pain line. Because visceral sensory fibers follow the visceral motor (autonomic) fibers retrograde, visceral pain from the uterine cervix will follow the presynaptic parasympathetic nerves of the pelvic splanchnic nerves retrograde to the spinal cord levels S2-4. However, pain derived from the labor contractions of the upper uterus, which is in contact with the peritoneum and thus located above the pelvic pain line, travel via visceral sensory fibers that follow the visceral motor fibers retrograde to refer pain to the lowest limits of the sympathetic (thoracolumbar) nervous system located at spinal cord levels T11-L2. Though pain derived from the labor contractions of the upper uterus refer to the L1 vertebral level, this question asked specifically about labor pains derived from the dilation of the uterine cervix, which would be referred to the S2-4 vertebral levels. Choice C (L4) is incorrect. Visceral sensory fibers carrying pain from visceral organs and involuntary (smooth or cardiac) muscle follow retrograde to the pathway of the visceral motor (autonomic) nerve fibers. There are no pre- synaptic autonomic nerve fibers that arise from L4, so there are no visceral sensory nerves associated with this spinal cord level. Choice D (S1) is incorrect. Visceral sensory fibers carrying pain from visceral organs and involuntary (smooth or cardiac) muscle follow retrograde to the pathway of the visceral motor (autonomic) nerve fibers. Because there are no presynaptic autonomic nerve fibers that arise from S1, there are no visceral sensory nerves associated with this spinal cord level.

A genetic coding error during the 5th week of development causes a unilateral failure of development of the ureteric bud. This condition would directly affect the formation of which of the following structures? (A) Proximal convoluted tubules (B) Renal glomeruli (C) Urinary bladder (D) Uterine tube (E) Ureter

The answer is E: Ureter. The permanent kidneys develop from two sources: (1) the ureteric bud and (2) the meta- nephric mesoderm. The ureteric bud is an outgrowth of the mesonephric duct near its junction with the cloaca. It forms the renal collecting ducts, including the ureter, renal pelvis, renal calyces, and collecting tubules. The ureteric bud invades the metanephric mesoderm, inducing it to form a meta- nephric cap over its distal end. The cap differentiates into the kidney parenchyma and the nephrons (i.e., the excretory and functional units in the kidney). Choice A (Proximal convo- luted tubules) is incorrect. The metanephric mesoderm forms the nephrons in the kidney, including Bowman capsule, the proximal convoluted tubule, loop of Henle, and distal con- voluted tubule. Nephrons are formed throughout prenatal development. Choice B (Renal glomeruli) is incorrect. Renal glomeruli are small capillary bundles that form in association with Bowman capsule. They are not derived from either the ureteric bud or the metanephric mesoderm. Choice C (Uri- nary bladder) is incorrect. The urinary bladder is derived from the cloaca. The cloaca divides into a urogenital sinus and an anal canal, which are located anterior and posterior, respec- tively. The urogenital sinus subsequently differentiates into the urinary bladder and urethra. Choice D (Uterine tube) is incor- rect. The uterine tube is formed from the paramesonephric duct. More caudally, the paired paramesonephric ducts join together to form the uterus, cervix, and cranial third of the vagina.

A normal shoulder X-ray is shown. An avulsion at which of the following numbered structures would most likely affect the function of the subscapularis muscle? a. 3 b. 5 c. 2 d. 4 e. 1 f. 6

f. 6

A 35-year-old diabetes mellitus woman delivered a term 2.3 kg baby with multiple anomalies (Fig. 12). The most likely cause is disturbance during the migration of which of the following embryonic structures? Select one: a. Mesoderm b. Ectoderm c. Hypoblast d. Endoderm e. Neural tube

A. mesoderm

27. Dorsal and ventral roots of the lower lumbar and sacral nerves are lacerated. Which structure is most likely damaged?

C

A newborn baby is diagnosed with anencephaly. Enter parents with which of the following embryological structures is responsible for this anomaly? A. Caudal neuropore B. Primitive streak C. Cloacal membrane D. Cranial neuropore E. Buccopharyngeal membrane

D. Cranial neuropore

10. A 20-year-old patient cannot flex and medially rotate the thigh while running and climbing. Which of the following muscles is most likely damaged? (A) Semimembranosus (B) Sartorius (C) Rectus femoris (D) Vastus intermedius (E) Tensor fasciae latae

10. The Answer is E. The tensor fasciae latae can flex and medially rotate the thigh, so this is the muscle most likely damaged. The hamstring muscles (semitendinosus, semimembran- osus, and biceps femoris) can extend the thigh and flex the leg. The sartorius can flex the thigh and leg. The rectus femoris can flex the thigh and extend the leg. The vastus intermedius can extend the leg.

12. A construction worker falls feet first from a roof. He sustains a fracture of the groove on the undersurface of the sustentaculum tali of the calcaneus bone. Which of the following muscle tendons is most likely torn? (A) Flexor digitorum brevis (B) Flexor digitorum longus (C) Flexor hallucis brevis (D) Flexor hallucis longus (E) Tibialis posterior

12. The Answer is D. The tendon of the flexor hallucis longus muscle occupies first the groove on the posterior surface of the talus and then the groove on the undersurface of the sustentaculum tali. None of the other tendons would have been affected in such an injury.

21. A middle-aged coal miner injures his back after an accidental explosion. His magnetic resonance imaging (MRI) scan reveals that his spinal cord has shifted to the right because the lateral extensions of the pia mater were torn. Function of which of the following structures is most likely impaired? (A) Filum terminale internum (B) Coccygeal ligament (C) Denticulate ligament (D) Choroid plexus (E) Tectorial membrane

C

4. A 39-year-old woman with headaches presents to her primary care physician with a possible herniated disk. Her magnetic resonance imaging (MRI) scan reveals that the posterolateral protrusion of the intervertebral disk between L4 and L5 vertebrae would most likely affect nerve roots of which of the following spinal nerves? (A) Third lumbar nerve (B) Fourth lumbar nerve (C) Fifth lumbar nerve (D) First sacral nerve (E) Second sacral nerve

C

8. A young toddler presents to her pediatrician with rather new onset of bowel and bladder dysfunction and loss of the lower limb function. Her mother had not taken enough folic acid (to the point of a defi- ciency) during her pregnancy. On examination, the child has protrusion of the spinal cord and meninges and is diagnosed with which of the following conditions? (A) Spina bifida occulta (B) Meningocele (C) Meningomyelocele (D) Myeloschisis (E) Syringomyelocele

C

Receives oxygenated blood via pulmonary veins?

E. Left atrium

A 32-year-old male farmer cuts the medial aspect of his midthigh when climbing over a barbwire fence. Though he bandages the wound, he reports to the ER 5 days later with an infected wound, high fever (102.7°), and lymphadenitis (swollen lymph nodes). Given the location of the injury, which groups of nodes would be the first to receive drainage from the infected wound? (A) Popliteal (B) External iliac (C) Deep inguinal (D) Horizontal group of superfi cial inguinal (E) Vertical group of superficial inguinal

(E) Vertical group of superficial inguinal

10. A thoracic surgeon removed the right middle lobar (secondary) bronchus along with lung tissue from a 57-year-old heavy smoker with lung cancer. Which of the following bronchopulmonary segments must contain cancerous tissues? (A) Medial and lateral (B) Anterior and posterior (C) Anterior basal and medial basal (D) Anterior basal and posterior basal (E) Lateral basal and posterior basal

10. The answer is A. The right middle lobar (secondary) bronchus leads to the medial and lateral bronchopulmonary segments. The right superior lobar bronchus divides into the superior, posterior, and anterior segmental (tertiary) bronchi. The right inferior lobar bronchus has the anterior, lateral, posterior, and anterior segmental bronchi.

52. A 78-year-old man has carcinoma of the rectum. The cancer is likely to metastasize via the veins into which of the following structures? (A) Spleen (B) Kidney (C) Liver (D) Duodenum (E) Suprarenal gland

52. The answer is C. Cancer cells from rectal cancer are likely to metastasize to the liver via the superior rectal, inferior mesenteric, splenic, and portal veins. Cancer cells are not directly spread to the other organs listed. The spleen and duodenum drain their venous blood to the portal venous system, and the kidney and suprarenal gland empty into the caval (inferior vena cava) system.

6. A 59-year-old woman comes to a local hospital for uterine cancer surgery. As the uterine artery passes from the internal iliac artery to the uterus, it crosses superior to which of the following structures that is sometimes mistakenly ligated during such surgery? (A) Ovarian artery (B) Ovarian ligament (C) Uterine tube (D) Ureter (E) Round ligament of the uterus

6. The answer is D. The ureter runs under the uterine artery near the cervix; thus, the ureter is sometimes mistakenly ligated during pelvic surgery. The other structures mentioned are not closely related to the uterine artery near the uterine cervix.

Below are embryonic changes that occur during development of the upper limbs: 1. Separate fingers 2. Limb bud development 3. Webbed fingers 4. Digital rays. Which of the following most likely represents the correct order of developmental changes that occur in the upper limbs between weeks 5 and 8? A. 2-4-3-1 B. 3-4-2-1 C. 2-3-4-1 D. 4-2-3-1 E. 1-4-3-2

A. Development of the upper limb begins at 26 to 27 days after fertilization with the appearance of the upper limb buds; at 33 to 36 days the hand plates are formed and the digital rays are present; at 44 to 46 days notches form between digital rays, and at 49 to 51 days webbed fingers are distinct. Separate fingers appear at 52 to 53 days. 58-60

28. The spinal cord is crushed at the level of the upper part of the first lumbar vertebra. Which structure is most likely damaged?

B

Which of the labeled structures in the given CT scan of the thorax indicates the left main bronchus?

C. left main bronchus

The structure indicated with the "X" on the given axial CT scan is which of the following structures? A. ventricular septum B. right ventricle C. right atrium D. left ventricle E. left atrium

D. left ventricle

Familial dysautonomia is a rare genetic disorder characterized by abnormal functioning of the autonomic nervous system. Problems include difficulty in feeding and respiration, vasomotor instability, insensitivity to pain, and ataxia (an unsteady gait). Patients with this syndrome have low numbers of autonomic (visceral motor) neurons, probably related to defects in production and/or survival of the neural crest precursors of these neurons. Such a disorder could directly affect the innervation of the muscle cells in which of the following structures? (A) Pelvic diaphragm (B) External anal sphincter (C) Urogenital diaphragm (D) Internal urethral sphincter (E) Levator ani

The answer is D: Internal urethral sphincter. Familial dysautonomia is an autonomic disorder that affects control of smooth muscle. The internal urethral sphincter is a thickening of the circular layer of smooth muscle at the internal urethral orifice. It is the involuntary sphincter that regulates urination (mictu- rition) and is the only smooth muscle structure of the choices provided. It is supplied by both sympathetic (for contraction) and parasympathetic (for relaxation) fibers through the hypo- gastric plexuses in the pelvis. Choice A (Pelvic diaphragm) is incorrect. The pelvic diaphragm is a musculofascial sheet that forms a large part of the pelvic floor. It is composed of two main parts: the levator ani and the coccygeus. Because it is composed of skeletal muscle, it is innervated by somatic motor fibers. These nerves branch directly off the sacral plexus and off the perineal branches of the pudendal nerve. Choice B (External anal sphincter) is incorrect. The exter- nal anal sphincter is the skeletal muscle complex surround- ing the lower anal canal that forms the voluntary sphincter, regulating defecation. It is innervated by somatic motor fibers of the inferior anal (rectal) nerves (branches of the pudendal nerves). Choice C (Urogenital diaphragm) is incorrect. Tra- ditionally, the urogenital diaphragm has been described as a three-layered, triangular structure that makes up the deep perineal pouch within the urogenital triangle. The middle layer is a skeletal muscle zone composed mainly of the exter- nal urethral sphincter (sphincter muscle of urethra) and the deep transverse perineal muscles. However, current concepts describe this area as a much more complex region, with the muscle and fascial components arranged in three-dimensional relations rather than in a flat sandwich-like manner. In either case, the muscles here receive somatic motor innervation from the perineal nerves, which are branches of the pudendal nerve. Readers are advised to reference the most current texts for a proper and better understanding of this region. Choice E (Levator ani) is incorrect. The levator ani is the larger of the two parts of the pelvic diaphragm. It is usually divided into three main subparts: puborectalis, pubococcygeus, and iliococcygeus. These muscles are supplied by somatic motor fibers of the perineal nerves.

Which structure is most likely to be removed by a pulmonary surgeon in a surgical resection of a lobe (lobectomy) to remove lung cancer in the apex of the right lung?

B. Right superior secondary (eparterial) bronchus

6. A 19-year-old college student came to his doctor's office for a neurologic examination. His physician told him that normally syn- apses are absent in or on which of the following structures? (A) Anterior horn of the spinal cord (B) Dorsal root ganglia (C) Sympathetic chain ganglia (D) Dendrites (E) Cell bodies

6. The answer is B. Dorsal root ganglia consist of cell bodies of the unipolar or pseudouni- polar neurons and have no synapses. Axosomatic and axodendritic synapses are the most common, but axoaxonal and dendrodendritic contacts are also found in many nerve tis- sues.

60. Which structure secretes fluid containing fructose, which allows for forensic determination of rape?

60. The answer is B. The seminal vesicle is a lobulated glandular structure and produces the alkaline constituent of the seminal fluid, which contains fructose and choline. Fructose, which is nutritive to spermatozoa, also allows forensic determination of rape, whereas choline crystals are the preferred basis for the determination of the presence of semen.

62. The tibialis anterior and peroneus longus muscles are weakened. Which bone in the radiograph is most likely fractured?

62. The Answer is E. The first or medial cuneiform bone provides insertions for the tibialis anterior, tibialis posterior, and peroneus longus muscles.

An ultrasound of a 35-year-old pregnant woman reveals that she is carrying conjoined twins, a condition caused by incomplete division of embryonic discs. The most common location where conjoined twins (1/200 monozygotic conceptions) connects is: A. Thoracopagus B. Dicephalus C. Craniopagus D. Omphalopagus E. Rachipagus

A. Failure of complete division or fusion of adjacent embryonic discs leads to conjoined twins. These are always monozygotic and share a single chorion, placenta, and amniotic sac. The most common site of attachment is an anterior union of the thoracic regions called thoracopagus, with the second most common being omphalopagus, where the twins are joined at the anterior abdominal wall. The original "Siamese twins" were of this latter type. 87-88

16. A crush injury of the vertebral column can cause the spinal cord to swell. Which structure would be trapped between the dura and the vertebral body by the swelling spinal cord? (A) Anterior longitudinal ligament (B) Alar ligament (C) Posterior longitudinal ligament (D) Cruciform ligament (E) Ligamentum nuchae

C

68. Destruction of the structure indicated by the letter E most likely causes weakness of which of the following muscles? (A) Flexor carpi radialis (B) Palmaris longus (C) Flexor carpi ulnaris (D) Brachioradialis (E) Flexor digitorum superficialis

C

The left coronary artery arises from which structure?

C. Ascending aorta

Intermediate mesoderm gives rise to the (A) neural tube (B) heart (C) kidneys and gonads (D) somites (E) notochord

C. Intermediate mesoderm is a subdivision of intraembryonic mesoderm that forms a longitudinal dorsal ridge called the urogenital ridge from which the kidneys and gonads develop.

3. A 42-year-old woman with metastatic breast cancer is known to have tumors in the intervertebral foramina between the fourth and fifth cervical vertebrae and between the fourth and fifth thoracic vertebrae. Which of the following spinal nerves may be damaged? (A) Fourth cervical and fourth thoracic nerves (B) Fifth cervical and fifth thoracic nerves (C) Fourth cervical and fifth thoracic nerves (D) Fifth cervical and fourth thoracic nerves (E) Third cervical and fourth thoracic nerves

D

During the physical examination of a newborn male, a pediatrician noted a painless, tense, fluctuant scrotal mass on the left side. Shining a light toward the enlarged scrotal mass caused the left side of the scrotum to light up, as depicted in the given figure. What is the most likely diagnosis? (A) Hydrocele (B) Varicocele (C) Indirect inguinal hernia (D) Direct inguinal hernia (E) Spermatocele

The answer is A: Hydrocele. A hydrocele testis is an accumulation of serous fluids within the spermatic cord of the testis. This pathology may be due to serous secretions from a remnant piece of peritoneum (tunica vaginalis), or due to communication with the abdomen via a persistent processus vaginalis. Patients with a hydrocele testis present with a pain- less swollen testis that feels like a water balloon, which may be corrected via drainage with a needle (aspiration) or sur- gery. Verification of a hydrocele can be accomplished by the diagnostic technique of transillumination, in which a light is placed near the enlarged portion of the scrotum. If the scro- tum is full of clear fluid and a hydrocele testis is present, the scrotum will light up on the affected side, which was seen in this newborn. Choice B (Varicocele) is incorrect. A varicocele is an abnormal dilation of the pampiniform plexus of veins within the spermatic cord. This scrotal mass often resembles and feels like a "bag of worms" on physical examination (see photo in question 6). Idiopathic varicocele is usually caused by defective one-way valves within the pampiniform plexus, which cause dilatation of these veins near the testis. However, a secondary varicocele will be seen following compression of the venous drainage of the testis due to the presence of a pelvic or abdominal malignancy, and this possibility must be ruled out by the physician. Varicoceles are found in the left side in approximately 98% of cases, can develop in 15% to 20% of all males, and are most frequently diagnosed in males between 15 and 25 years of age. Due to the nature (tense and fluctuant) of this scrotal mass, the fact that the scrotum is transilluminated, and the age of this newborn, a hydrocele is the most likely diagnosis. Choice C (Indirect inguinal hernia) is incorrect. An indirect inguinal hernia is usually a congenital hernia that results when abdominal cavity contents herniate through a patent processus vaginalis (open communication between the abdomen and scrotum). The hernia traverses the deep and superficial inguinal rings to descend into the scro- tum in males. An indirect inguinal hernia often presents as a soft, nontender, reducible bulge in the inguinal canal that can extend into the scrotum. It is located off the midline, especially at times of increased intra-abdominal pressure, and exists as a visible or palpable lump underneath the skin in the groin region, which was seen in this patient. Due to the nature (tense and fluctuant) of this scrotal mass and the fact that the scrotum is transilluminated by the light used by the physician, a hydrocele is the most likely diagnosis. It should be noted that indirect inguinal hernias may present with a hydrocele, so further diagnosis with an ultrasound can be utilized to rule out a herniation of abdominal contents into the scrotum. Choice D (Direct inguinal hernia) is incorrect. A direct inguinal hernia is an acquired hernia that results when abdominal cavity contents herniate through a weakness in the anterior abdominal wall in the inguinal (Hesselbach) triangle. The hernia exits through the superficial inguinal ring and often presents as a soft, nontender, reducible bulge underneath the skin overlying the superficial inguinal ring. Given the age of this patient and the presence of the mass that has descended into the scrotum, a direct inguinal hernia is not likely. Choice E (Spermatocele) is incorrect. A sperma- tocele is a cyst-like mass within the scrotum that develops in a tubule of the rete testis or the head of the epididymis. It usually contains a milky fluid and spermatozoa. These reten- tion cysts are usually not painful and can be confirmed by an ultrasound of the scrotum. Given the size of this scrotal mass, the age of the patient, and the fact that the entire scro- tum is transilluminated with the light used by the physician, a hydrocele is the most likely diagnosis.

A young couple hoping for a pregnancy buys an overthe-counter pregnancy kit. What substance will this test most likely detect? (A) Early pregnancy factor (EPF) (B) Human chorionic gonadotropin (C) Progesterone (D) Estrogen (E) Luteinizing hormone (LH)

The answer is B: Human chorionic gonadotropin. Early pregnancy detection kits most likely detect the level of human chorionic gonadotropin (hCG) in the female. hCG is a glycoprotein hormone produced by the syncytiotrophoblast to prevent the disintegration of the corpus luteum of the ovary. While hCG is a reliable marker of pregnancy, it cannot be detected until after implantation (6 to 12 days after fertilization), which results in false negatives if the test is performed during the very early stages of pregnancy.

20. A 56-year-old patient recently suffered a myocardial infarction in the area of the apex of the heart. The occlusion by atherosclerosis is in which of the following arteries? (A) Marginal artery (B) Right coronary artery at its origin (C) Anterior interventricular artery (D) Posterior interventricular artery (E) Circumflex branch of the left coronary artery

20. The answer is C. The apex of the heart typically receives blood from the anterior interventricular branch of the left coronary artery. The marginal artery supplies the right inferior margin of the right ventricle, the right coronary artery at its origin supplies the right atrium and ventricle, and the posterior interventricular artery and a circumflex branch of the left coronary artery supply the left ventricle.

A 46-year-old patient comes to his doctor's office and complains of chest pain and headache. His computed tomography (CT) scan reveals a tumor located just superior to the root of the right lung. Blood flow in which of the following veins is most likely blocked by this tumor? (A) Hemiazygos vein (B) Arch of the azygos vein (C) Right subclavian vein (D) Right brachiocephalic vein (E) Accessory hemiazygos vein

24. The answer is B. The azygos vein arches over the root of the right lung and empties into the SVC. Other veins do not pass over the root of the right lung.

36. A 45-year-old woman presents with a tumor confined to the posterior mediastinum. This could result in compression of which of the following structures? (A) Trachea (B) Descending aorta (C) Arch of the aorta (D) Arch of the azygos vein (E) Phrenic nerve

36. The answer is B. The descending aorta is found in the posterior mediastinum. The superior mediastinum contains the trachea and arch of the aorta, and the middle mediastinum contains the ascending aorta, arch of the azygos vein, and main bronchi. The phrenic nerve runs in the middle mediastinum.

37. A 62-year-old patient with pericardial effusion comes to a local hospital for aspiration of pericardial fluid by pericardiocentesis. The needle is inserted into the pericardial cavity through which of the following intercostal spaces adjacent to the sternum? (A) Right fourth intercostal space (B) Left fourth intercostal space (C) Right fifth intercostal space (D) Left fifth intercostal space (E) Right sixth intercostal space

37. The answer is D. To aspirate pericardial fluid, the needle should be inserted into the pericardial cavity through the fifth intercostals space just left to the sternum. Because of the cardiac notch, the needle misses the pleura and lungs, but it penetrates the pericardium. Lung tissues lie beneath the fourth and sixth intercostal spaces.

61. The flexor hallucis longus tendon is damaged in a groove on the posterior surface of a tarsal bone. Which bone in the radiograph is likely fractured?

61. The Answer is A. The body of the talus has a groove on its posterior surface for the flexor hallucis longus tendon. This tendon also occupies the groove on the undersurface of the sustentaculum tali.

An overweight woman participates in her fi rst rugby match without proper training and conditioning. Upon catching the opening kickoff, she awkwardly twists her right knee, screams in pain, and falls to the ground. The team manager notes her patella is dislocated, residing on the lateral side of her knee. After straightening the woman's knee, the patellar dislocation is reduced (goes back into place). To prevent future dislocation of the patella, what specific muscle should be targeted during rehabilitation? (A) Vastus intermedius (B) Vastus lateralis (C) Vastus medialis (D) Rectus femoris (E) Tibialis anterior

(C) Vastus medialis

14. An 8-year-old boy with ASD presents to a pediatrician. This congenital heart defect shunts blood from the left atrium to the right atrium and causes hypertrophy of the right atrium, right ventricle, and pulmonary trunk. Which of the following veins opens into the hypertrophied atrium? (A) Middle cardiac vein (B) Small cardiac vein (C) Oblique cardiac vein (D) Anterior cardiac vein (E) Right pulmonary vein

14. The answer is D. The anterior cardiac vein drains into the right atrium. The middle, small, and oblique cardiac veins drain into the coronary sinus. The right and left pulmonary veins drain into the left atrium.

31. A 72-year-old man comes to his physician for an annual checkup. Which of the following structures is most readily palpated during rectal examination? (A) Prostate gland (B) Epididymis (C) Ejaculatory duct (D) Ureter (E) Testis

31. The answer is A. The prostate gland may be palpated on rectal examination. The ejaculatory duct runs within the prostate gland and cannot be felt. In the male, the pelvic part of the ureter lies lateral to the ductus deferens and enters the posterosuperior angle of the bladder, where it is situated anterior to the upper end of the seminal vesicle, and thus, it cannot be palpated during rectal examination. However, in the female, the ureter can be palpated during vaginal examination because it runs near the uterine cervix and the lateral fornix of the vagina to enter the posterosuperior angle of the bladder. The testes are examined during a routine annual checkup but obviously not during a rectal examination.

32. A 48-year-old college football coach undergoes a radical prostatectomy for a malignant tumor in his prostate. Following surgery, he is incapable of achieving an erection. Which of the following nerves is most likely damaged during the surgery? (A) Sacral splanchnic nerve (B) Pelvic splanchnic nerve (C) Pudendal nerve (D) Dorsal nerve of the penis (E) Posterior scrotal nerve

32. The answer is B. Parasympathetic preganglionic fibers in the pelvic splanchnic nerve are responsible for erection of the penis. Sympathetic preganglionic fibers in the sacral splanchnic nerve are responsible for ejaculation. The pudendal nerve supplies the external anal sphincter and perineal muscles and supplies GSA fibers to the perineal region. The dorsal nerve of the penis is a terminal branch of the pudendal nerve and supplies sensation of the penis. The posterior scrotal nerves are superficial branches of the perineal nerve and supply sensory fibers to the scrotum.

7. A patient presents with sensory loss on adjacent sides of the great and second toes and impaired dorsiflexion of the foot. These signs probably indicate damage to which of the following nerves? (A) Superficial peroneal (B) Lateral plantar (C) Deep peroneal (D) Sural (E) Tibial

7. The Answer is C. The deep peroneal nerve supplies the anterior muscles of the leg, includ- ing the tibialis anterior, extensor hallucis longus, extensor digitorum longus, and peroneus tertius muscles, which dorsiflex the foot. The medial branch of the deep peroneal nerve supplies the skin of adjacent sides of the great and second toes, whereas the lateral branch supplies the extensor digitorum brevis and extensor hallucis brevis. The superficial pero- neal nerve innervates the peroneus longus and brevis, which plantar flexes the foot, and supplies the skin on the side of the lower leg and the dorsum of the ankle and foot. The tibial nerve innervates the muscles of the posterior compartment that plantar flexes and supplies the skin on the heel and plantar aspect of the foot. The lateral plantar nerve innervates muscles and skin of the lateral plantar aspect of the foot. The sural nerve sup- plies the skin on the posterolateral aspect of the leg and the lateral aspect of the foot and the little toe.

9. A patient presents with a loss of sensation to the skin over the shoulder. Injury to which of the following nerve cells would most likely affect the conduction of sensory information to the central nervous system? (A) Multipolar neurons (B) Bipolar neurons (C) Unipolar or pseudounipolar neurons (D) Neurons in the ventral horn (E) Neurons in sympathetic chain ganglia

9. The answer is C. Sensation from the skin is carried by GSA fibers, and their cells are uni- polar or pseudounipolar types located in the dorsal root ganglia. Multipolar neurons and neurons in the ventral horn and in sympathetic chain ganglia are motor neurons. Bipolar neurons are sensory neurons, but they are not somatic sensory neurons.

A 28-year-old woman visits an obstetrician for in vitro fertilization. FSH-analogs are injected to stimulate follicles. Then hCG is injected to induce final oocyte maturation. Lastly, the oocyte is retrieved via a procedure called transvaginal oocyte retrieval. The retrieved oocyte is arrested at what stage of development? A. Prophase of meiosis 1 B. Metaphase of meiosis 1 C. Prophase of meiosis 2 D. Metaphase of meiosis 2 E. Prophase of meiosis 2

A. Primary oocytes begin the first meiotic divisions before birth, but completion of prophase in meiosis 1 does not occur until adolescence. The follicular cells surrounding the primary oocytes secrete a substance, oocyte maturation inhibitor, which arrests the meiotic process of the oocyte.

A 65-year-old man presents with a swollen neck, marked edema in both upper limbs, and engorged and prominent intercostal veins. Subsequent examination reveals a tumor in his right lung. The tumor is likely compressing structures in which of the following regions? A. anterior mediastinum B. posterior mediastinum C. superior mediastinum D. hilium of the lung E. vena caval foramen of the diaphragm

C. superior mediastinum

29. A basketball player was hit in the thigh by an opponent's knee. Which of the following arteries is likely to compress and cause ischemia because of the bruise and damage to the extensor muscles of the leg? (A) Popliteal (B) Deep femoral (C) Anterior tibial (D) Posterior tibial (E) Peroneal

29. The Answer is C. A muscular spasm or hypertrophy of the extensor muscles of the leg may compress the anterior tibial artery, causing ischemia. The popliteal artery supplies mus- cles of the popliteal fossa. The deep femoral artery supplies deep muscles of the thigh. The posterior tibial and peroneal arteries supply muscles of the posterior and lateral compart- ments of the leg.

43. During early development of the respiratory system, the laryngotracheal tube maintains communication with the primitive foregut. Which of the following embryonic structures is most likely responsible for partitioning these two embryonic structures? (A) Tracheoesophageal folds (B) Tracheoesophageal fistula (C) Tracheoesophageal septum (D) Laryngotracheal diverticulum (E) Laryngotracheal septum

43. The answer is C. The tracheoesophageal septum is formed by the fusion of the tracheoesophageal folds in the midline. This septum divides the foregut into a ventral portion, the laryngotracheal tube (primordium of the larynx, trachea, bronchi, and lungs), and a dorsal portion (primordium of the oropharynx and esophagus).

57. Which structure has a Houston valve or fold, with its venous blood drained by the portal venous system?

57. The answer is C. The mucous membrane and the circular smooth-muscle layer of the rectum form three transverse folds; the middle one is called Houston valve. The venous blood returns to the portal venous system via the superior rectal vein.

29. Which structure may herniate through the annulus fibrosus, thereby impinging on the roots of the spinal nerve?

A

The picture shown below (Fig. 8) is a diagrammatic representation of the cross section of an embryonic germ disc where all three germ layers already formed and differentiation of the intraembryonic mesoderm. Which of the following numbers identify the part which contributes to the development of the urogenital structures? Select one: a. 5 b. 4 c. 6 d. 8 e. 7 f. 1

A. 5

The first indication of gastrulation in the embryo is (A) formation of the primitive streak (B) formation of the notochord (C) formation of the neural tube (D) formation of extraembryonic mesoderm (E) formation of tertiary chorionic villi

A. The formation of the primitive streak on the dorsal surface of the bilaminar embryonic disk is the first indication of gastrulation.

The given coronary artery angiogram is from a 68yo man w/ recurrent angina. It reveals 90% stenosis of the left anterior descending (LAD) artery, indicated by the white arrow. Based upon this finding, which portion of the heart is most likely susceptible to ischemic damage? A. anterior 2/3 of interventricular septum B. posterior 1/3 of interventricular septum C. atrioventricular (AV) node D. diaphragmatic surface of left ventricle E. right atrium

A. anterior two thirds of interventricular septum

A 24-year-old man who has been overweight and sedentary most of his life experiences shortness of breath and cyanosis on exertion when he attempts to start an exercise program for the fi rst time. A thorough physical examination by his physician reveals an aortic valve defect and an aortic murmur. This murmur is best detected with the stethoscope placed over which of the indicated sites?

A. right 2nd intercostal space at the right parasternal line

A 7-day-old newborn with a diagnosis of patent ductus arteriosus undergoes surgery to ligate the ductus arteriosus. During the repair, the surgeon takes special care to avoid injury to a closely related nerve. The surgeon is protecting which of the following nerves? A. Left vagus] B. Left phrenic C. Left recurrent laryngeal D. Right recurrent laryngeal E. Right phrenic

C. left recurrent laryngeal

A 46-year-old supervisor was reading a work order on a construction site when a 60 lb bag of concrete mix was accidentally dropped on the apex of his head. He was immobilized and brought to the ER where he presented with upper neck pain but no neurological signs. Based upon the given axial CT scan and the patient's presentation, which of the following diagnoses is most likely? (A) Damage to the cervical spinal cord (B) No pathology is apparent on the CT scan (C) Jefferson (burst) fracture of C1 (D) Fracture of the dens axis (odontoid process) (E) Atlanto-axialsubluxation

C: Jefferson (burst) fracture of C1. The C1 vertebra, or atlas, is normally a closed ring with no vertebral body. Excessive vertical, or downward, force on the top of the head can fracture the anterior and posterior arches of C1 in multiple places, leading to a Jefferson (burst) fracture of C1. Due to the vertical force of the concrete mix striking the top of the man's head, the lateral masses of C1 are driven laterally due to the oblique articulation between the occipital condyles and the superior articular processes of the lateral masses of C1. This vertical compression force fractures the anterior and posterior arches of C1 bilaterally, as confirmed by the axial CT scan. Jefferson fractures of C1 often occur with axial loading force when the top of the head is impacted by a hard or heavy object. Because the fractures within the bony ring of the atlas actually increase its dimension, this type of fracture does not usually result in spinal cord injury; however, upper neck pain would be present. The axial CT scan confirms the diagnosis of a Jefferson (burst) fracture of C1. Choice A (Damage to the cervical spinal cord) is incorrect. Damage of the cervical spinal cord is highly unlikely due to the lack of neurological signs reported in this patient. The axial CT scan depicts bilateral fractures within the anterior and posterior arches of C1, and none of these fracture fragments are in close approximation to the spinal cord. Choice B (No pathology is apparent on the CT scan) is incorrect. The axial CT scan depicts bilateral fractures within the anterior and posterior arches of C1. Knowledge of the anatomy of the atlas (C1) would eliminate this diagnosis as a likely option. Choice D (Fracture of the dens axis [odon- toid process]) is incorrect. In the given axial CT scan of the C1 arch, it would be hard to detect a fracture of the dens axis (odontoid process). Moreover, odontoid fractures have been reported to occur following a combination of excessive flex- ion, extension, and some rotation within the cervical verte- brae, so the nature of this traumatic injury would not sug- gest an odontoid fracture. A sagittal CT scan would give more information concerning the integrity of the odontoid process. Choice E (Atlanto-axial subluxation) is incorrect. Atlanto- axial subluxation, or the incomplete dislocation of the median atlanto-axial joint, occurs following the rupture of the trans- verse ligament of the atlas, which holds the dens axis (odon- toid process) in place. Losing the integrity of the transverse ligament of the atlas can result in compression of the upper cervical spinal cord, leading to quadriplegia (paralysis of all four limbs) and even death (if the medulla of the brainstem is compressed). In this axial CT scan, the odontoid process (dens axis) of C2 is in its normal position, which implies an intact transverse ligament of the atlas. Therefore, atlanto-axial subluxation did not occur in this patient.

An 18-year-old soldier presents with shrapnel wounds in the lateral wall of his right chest following an explosion of a landmine. After several months of recovery, his physical therapist observes that his scapula moves away from the thoracic wall when he leans on his right hand, as noted by the black arrow in the given photo. Which of the following nerves is likely damaged? (A) Axillary nerve (B) Thoracodorsal nerve (C) Long thoracic nerve (D) Dorsal scapular nerve (E) Suboccipital nerve

C: Long thoracic nerve. The photo demonstrates a case of "winged scapula," indicative of lesion of the long thoracic nerve and subsequent paralysis of the serratus anterior muscle. The nerve is located on the lateral thoracic wall, on the superficial aspect of the serratus anterior, where it is not afforded the protection of the muscle it innervates (like most motor nerves), especially when the limb is elevated. In this patient, shrapnel wounds to the lateral thoracic wall caused damage to the long thoracic nerve and subsequent loss of innervation to the serratus anterior muscle. When the affected limb is protracted, the medial border and inferior angle of the scapula pull away from the posterior chest wall, giving the scapula a wing-like appearance. Additionally, the affected arm cannot be abducted above the horizontal plane because the serratus anterior is not available to superiorly rotate the glenoid cavity of the scapula to allow full abduction. Following damage to the long thoracic nerve, it takes several weeks for a winged scapula to the develop because the trapezius muscle, which attaches to the spine of the scapula, must stretch before winging is apparent. Choice A (Axillary nerve) is incorrect. This nerve passes deeply through the axilla, around the surgical neck of the humerus, to supply the teres minor and deltoid muscles. A lesion of this nerve results in significant weakness in abduction of the arm and wasting of the rounded contour of the shoulder, which is not reported in this patient. Choice B (Thoracodorsal nerve) is incorrect. This nerve runs downward through the axilla to supply the latissimus dorsi muscle. Loss of the nerve would result in weakness in extension and medial rotation of the arm, plus wasting of the posterior axillary fold. These signs and symptoms were not reported in this patient. Choice D (Dorsal scapular nerve) is incorrect. This nerve courses into the upper, medial part of the back and the lower neck to supply the levator scapulae and rhomboid muscles. Paralysis of these muscles would result in weakness in elevation and retraction of the scapula, and perhaps wasting of the contour of the back under the trapezius muscle, which was not reported in this patient. Choice E (Suboccipital nerve) is incorrect. The suboccipital nerve is also the posterior (dorsal) ramus of the first cervical nerve (C1), which passes through the suboccipital triangle to innervate the rectus capitis posterior major and minor, obliquus capitis superior and inferior, rectus capitis lateralis, and semispinalis muscles. It is generally believed that the suboccipital nerve contains only motor fibers, but it may receive afferent (sensory) fibers related to proprioception. Damage to the suboccipital nerve would result in weakness extending and rotating the head on the C1 and C2 vertebrae, which was not reported in this patient.

11. An elderly man at a nursing home is known to have degenerative brain disease. When cerebrospinal fluid (CSF) is withdrawn by lumbar puncture for further examination, which of the following structures is most likely penetrated by the needle? (A) Pia mater (B) Filum terminale externum (C) Posterior longitudinal ligament (D) Ligamentum flavum (E) Annulus fibrosus

D

30. Cerebrospinal fluid (CSF) is produced by vascular choroid plexuses in the ventricles of the brain and accumulated in which space?

D

69. A lesion of the radial nerve would most likely cause paralysis of muscles that are attached to this area.

D

In the accompanying diagram of the blastocyst implantation, identify the part labeled "7". Select one: a. Yolk sac cavity b. Syncytial trophoblast c. Hypoblast d. Amniotic cavity e. Cytotrophoblast f. Epiblast

D. Amniotic cavity

A 25-year-old woman trying to have her first child has a spontaneous abortion due to failure of implantation. Under normal circumstances, at what stage of embryonic development will an embryo most likely implant into the endometrium of the uterus? A. Trilaminar embryo B. Zygote C. Morula D. Blastocyst E. Bilaminar embryo

D. Implantation of the blastocyst begins at the end of the first embryonic week and normally occurs in the endometrium of the uterus, usually superiorly in the body of the uterus and slightly more often on the posterior than on the anterior wall. The blastocyst begins to implant on approximately the 6th day of the luteal phase. 29

A 58-year-old woman presents with acute abdominal pain, which radiates superior to the right shoulder region. The given CT reveals a large hepatic abscess. Which of the following nerves conveys the visceral sensory fi bers involved with the pain specifi c to the right shoulder region? A. greater splanchnic nerve B. lesser splanchnic nerve C. least splanchnic nerve D. phrenic nerve E. vagus nerve

D. phrenic nerve

A 53-year-old woman has had difficulty swallowing solids for the past several months. She has a 7-year history of cardiovascular disease. X-ray films of the esophagus with barium contrast show indentation and posterior displacement of the esophagus. Enlargement of which of the following structures is the most likely cause of her problem? a. Left atrium b. Right atrium c. Right lung d. Superior vena cava e. Left lung

a. Left atrium

A 62-year-old woman has a neoplasm in the upper lateral portion of the mammary gland. To which of the following group of nodes are cells from this neoplasm most likely to initially metastasize? a. Pectoral group of axillary lymph nodes b. Deep nodes between the pectoralis major and minor muscles c. Clavicular lymph nodes d. Parasternal lymph nodes e. Abdominal lymph nodes

a. Pectoral group of axillary lymph nodes

The relative positions of blood vessels can sometimes be explained in terms of rotation of the limbs during development. For example, upon entering the femoral triangle the femoral artery resides ____ to the femoral vein, whereas in the popliteal fossa the popliteal artery lies ____ to the popliteal vein. (A) medial...posterior (B) lateral...deep (C) lateral...posterior (D) medial...superfi cial (E) anterior...lateral

(B) lateral...deep

55. Which structure in the female is much shorter than the corresponding structure in the male?

55. The answer is A. In females, the urethra extends from the bladder, runs above the anterior vaginal wall, and pierces the urogenital diaphragm to reach the urethral orifice in the vestibule behind the clitoris. It is approximately 4 cm long. In males, the urethra is approximately 20 cm long.

48. A patient presents with a thrombosis in the popliteal vein. This thrombosis most likely causes reduction of blood flow in which of the following veins? (A) Greater saphenous (B) Lesser saphenous (C) Femoral (D) Posterior tibial (E) Anterior tibial

48. The Answer is C. The popliteal vein drains blood into the femoral vein; thus, blood flow in the femoral vein is reduced. The great saphenous vein drains into the upper part of the femoral vein. Other veins empty into the popliteal vein.

A 35-year-old woman suffers intractable pain in her left forearm. Neurosurgical consultation leads to a decision to conduct a rhizotomy to relieve the condition. At which of the following locations is the rhizotomy best performed to relieve the patient's pain? (A) Posterior (dorsal) roots (B) Posterior (dorsal) primary rami (C) Spinal nerves (D) Anterior (ventral) roots (E) Anterior (ventral) primary rami

A: Posterior (dorsal) roots. A rhizotomy is a neurosurgical procedure that selectively severs problematic spinal nerve roots to relieve pain or spastic paralysis (e.g., as often seen in cerebral palsy patients). The posterior roots are the only site where afferent (sensory) fibers are segregated from efferent (motor) fibers. In selective dorsal rhizotomy (SDR), severing the left C6-8 posterior roots could relieve the pain symptoms in the left forearm of this patient because they carry only sensory information. Choice B (Posterior [dorsal] primary rami) is incorrect. Cutting the posterior primary rami of spinal nerves would cause degeneration of the distal axonal processes of the general sensory (afferent) fibers as well as fibers to the intrinsic back muscles. However, the general sensory fibers contained within the posterior primary rami only supply innervation to the skin over the back, so the patient's pain within the left forearm would remain. Because they do not convey pain fibers from the left forearm, selective rhizotomy of the posterior primary rami would not relieve this patient's pain. Choice C (Spinal nerves) is incorrect. Cutting the spinal nerves would cause loss of sensory, motor, and (depending on the vertebral level) autonomic deficits. Because spinal nerves convey both sensory and motor fibers, this location is inappropriate for selective rhizotomy to relieve pain in the left forearm. Choice D (Anterior [ventral] roots) is incorrect. The anterior roots of the spinal nerve are the only site where motor fibers are segregated from sensory fibers. So, selective rhizotomy at the anterior roots would cause motor deficits but would not relieve the patient's pain. Choice E (Anterior [ventral] primary rami) is incorrect. Cutting the appropriate anterior primary rami of spinal nerves, which supply the left forearm, would cause degeneration of the distal axonal processes of the general sensory (afferent) fibers and relieve pain symptoms in this patient. However, the anterior primary rami also contain somatic motor (general somatic efferent or GSE) fibers, so a selective rhizotomy at this location would also cause motor deficits. Because they convey both sensory and motor fibers, the anterior primary rami are inappropriate for selective rhizotomy to afford relief from pain.

A newborn infant has difficulty breathing and swallowing. Radiographic examination reveals constrictions of both the trachea and esophagus in the same plane. Which of the following congenital malformations is the most likely cause of this condition? A. double aortic arch B. persistent right 2nd aortic arch C. coarctation of the aorta D. patent ductus arteriosus E. persistent left 4th aortic arch

D. double aortic arch

A 25-year-old man is brought to the emergency room after suffering a deep stab wound directly through the right 5th intercostal space in the midclavicular line. Which of the following structures is most likely pierced? A. superior lobe of the lung B. lingula of the lung C. inferior lobe of the lung D. apex of the lung E. middle lobe of the lung

E. middle lobe of the lung

A 30-year-old construction worker is brought to the emergency department complaining of sudden onset of pain in his right lower back that radiates down his thigh. The pain is aggravated by coughing and other Valsalva maneuvers and by passive straight-leg raising. It is relieved by lying down. An MRI shows a herniated intervertebral disc at L4-L5. Which of the following nerve roots is most likely being compressed? a. L3 b. S1 c. L4 d. L5 e. S2

d. L5

A 23-year-old woman is stabbed with an ice pick in the right third intercostal space immediately lateral to the sternum. The ice pick passed in an anterior to posterior direction. The most likely structure penetrated by this injury was which of the following? a. Pulmonary trunk b. Aortic arch c. Inferior vena cava d. Right atrium e. Right brachiocephalic vein

d. Right atrium

A 15-year-old girl is struck by a car while crossing the street. She suffers numerous pelvic injuries, including tearing of the sacrotuberous ligament. The damage to this ligament will most likely cause direct trauma to which of the following muscles? (A) Gluteus maximus (B) Gluteus medius (C) Gluteus minimus (D) Gemelli (E) Obturator externus

(A) Gluteus maximus

Two 11-year-old boys sneak up on their friend from behind. In surprising their friend, they shove him suddenly, which forcefully pushes his hips forward while he is standing relaxed talking to other friends. Which of the following ligaments best resists anterior dislocation of the head of the femur? (A) Iliofemoral ligament (B) Pubofemoral ligament (C) Ischiofemoral ligament (D) Lacunar ligament (E) Ligament of the head of the femur

(A) Iliofemoral ligament

In the production of male gametes, which of the following cells remains dormant for 12 years? (A) Primordial germ cell (B) Primary spermatocyte (C) Secondary spermatocyte (D) Spermatid (E) Sperm

A. Primordial germ cells migrate from the wall of the yolk sac during week 4 of embryonic life and enter the gonad of a genetic male, where they remain dormant until puberty (about age 12) when hormonal changes in the young man stimulate the production of sperm

Lies on the right side of the aortic arch and ascending aorta?

A. SVC

In the process of meiosis, DNA replication of each chromosome occurs thereby forming a structure consisting of two sister chromatids attached to a single centromere. What is this structure? (A) A duplicated chromosome (B) Two chromosomes (C) A synapsed chromosome (D) A crossover chromosome (E) A homologous pair

A. The structure formed is a duplicated chromosome. DNA replication occurs so that the amount of DNA is doubled (2 x 2N = 4N). However, the chromatids remain attached to the centromere forming a duplicated chromosome.

A 47-year-old homeless man presents to the emergency department with a severe toothache. He has a large facial swelling that had previously been of painless until this admission. The physician diagnoses an ameloblastoma, a benign tumor of ameloblasts. During normal development, what develops from the ameloblasts? A. Enamel B. Alveolar bone C. Cementum D. Periodontal ligaments E. Pulp

A. Tooth development (odontogenesis) usually begins in the sixth to eighth week. This development occurs in three stages: bud stage, cap stage, and bell stage. The enamel usually develops in the advanced bell stage when cells of the inner enamel epithelium develop into ameloblasts, which produce and deposit enamel in the form of rods over the dentine. 288-289

A young medical student finds herself at a moment of great relaxation during her pre-exam meditation. Which of the following events is characteristic of the inspiratory phase of normal, quiet respiration? A. diaphragm flattens B. intercostal muscles relax C. ribs lower D. abdominal wall muscles contract E. horizontal dimension of the rib cage decreases

A. diaphragm flattens

A neonate with marked cyanosis in the lower extremities, cardiomegaly, harsh heart murmurs, and dyspnea is diagnosed with a patent ductus arteriosus. This condition is due to faulty migration of neural crest cells that leads to partial development of which of the following embryologic structures? A. Right subclavian artery B. Aorticopulmonary septum C. Tricuspid valve D. Inferior vena cava E. Left subclavian artery

B. Aorticopulmonary septum is the correct answer. The cardiac neural crest cells are involved in the development of the muscle and connective tissue walls of large arteries, parts of the cardiac septum, and parts of the thyroid, parathyroid, and thymus glands. The aorticopulmonary septum is derived specifically from the cardiac neural crest cells; once formed it separates the aorta and pulmonary arteries and fuses with the interventricular septum within the heart during development. The right subclavian artery takes its origin from the fourth pharyngeal arch. The tricuspid valve is formed from proliferation of tissue around the atrioventricular canal. The inferior vena cava takes its origin from the vitelline veins. 209, 211-212

Which of the following structures must degenerate for blastocyst implantation to occur? (A) Endometrium in progestational phase (B) Zona pellucida (C) Syncytiotrophoblast (D) Cytotrophoblast (E) Functional layer of the endometrium

B. The zona pellucida must degenerate for implantation to occur. Early cleavage states of the blastula are surrounded by a zona pellucida, which prevents implantation in the uterine tube.

A 4-year-old boy presents with hypertension in the upper extremities and a diminished femoral pulse and pressure. Radiologic imaging reveals a postductal coarctation of the aorta. Which of the following is the most characteristic feature of this condition? A. Patent ductus arteriosus B. Closed ductus arteriosus C. Patent ductus venosus D. Patent foramen ovale E. Stenotic aortic valve

B. closed ductus arteriosus

A 14yo girl is having difficulty swallowing. Her case history reveals she has mitral valve stenosis related to rheumatic fever. Which of the following structures is most likely compresisng the esophagus? A. right atrium B. left atrium C. arch of the aorta D. pulmonary trunk E. superior vena cava

B. left atrium

A genetic coding defect in an early embryo results in absence of the lower part of the vertical portion of the cross of the endocardial cushions in the developing heart. Which of the following malformations is most likely present? A. Atrial septal defect B. Membranous ventricular septal defect (VSD) C. Transposition of the great vessels D. Tricuspid stenosis E. Muscular ventricular septal defect

B. membranous ventricular septal defect (VSD)

A 12-year-old boy suffers an enlarging neurogenic tumor in the right posterior mediastinum, as shown in the given lateral view radiograph. This growth causes multiple lesions of the right thoracic sympathetic trunk below the T5 chain ganglion. Which of the following functions is most likely compromised? A. ability to produce cardiac deceleration B. secretion of epinephrine C. secretion of gastric juices D. ability of produce bronchoconstriction E. vasodilation of the coronary arteries

B. secretion of epinephrine

A chest X-ray reveals lobar pneumonia located over the horizontal fissure of the lung. Which of the following lobes of the lung would be inflamed and full of fluid? A. Superior lobe of the left lung B. Superior lobe of the right lung C. Middle lobe of the right lung D. Inferior lobe of the right lung E. Inferior lobe of the left lung

B. superior lobe of right lung

Which of the following congenital defects is the direct outcome of malformation of the spiral partitioning of the conus cordis and truncus arteriosus? A. double aortic arch B. Transposition of the great vessels C. Patent foramen ovale D. Ventricular septal defect (VSD) E. ectopia cordis

B. transposition of the great vessels

An infant born to a 31-year-old homeless woman shows a mass overlying the lower spine with a patch of hair in the area. The pregnancy was complicated by preeclampsia and urinary tract infection, which was treated with cefazolin. The abnormality described in this patient could have been prevented with: Select one: a. Alcohol cessation b. Vitamin supplementation c. Avoiding antibiotics d. Hypertension control e. Smoking cessation

B. vitamin supplementation. The neural system is formed from ectoderm. During the third week of fetal life the ectoderm on the dorsal surface of the embryo thickens and forms the neural plate. It deepens in the center to create the neuraf groove that is bound on both sides by neuraf folds. The folds fuse to create a neuraf tube which is connected to the amniotic cavity by openings at the ends called anterior and posterior neuropores. These openings close during the 4th week of fetal life. Failure of the neuropores to close leads to the formation of neural tube defects (NTDs). The anterior neuropore closes on the 25th day of fetal life. Its failure to close leads to an anterior NTD, which includes anencephaly (complete absence of the brain) and encephalocele (protrusion of neural tissue through the cranial defect). The posterior neuropore closes on the 27th day. Posterior neural tube defects include spina bifida occulta, meningocele, meningomyelocele and rachisis. The neonate described in the vignette has a posterior neural tube defect. Meningocele and meningomyelocele both present as a cystic mass at the lower spine region. The mass is covered with skin; a tuft of hair may be present in the area. In meningocele, the mass consists of spinal meninges that protrude through the defect in vertebral arches. In meningomyelocele, a portion of the spinal cord or cauda equina lies within the protruding meningeal sac. Folate supplementation during early pregnancy decreases the incidence of neural tube defects. High dose folate intake is recommended for all women of childbearing age because the neural tube develops very early in fetal life (when most women do not yet know that they are pregnant). (Choice A) Alcohol consumption during pregnancy may lead to fetal alcohol syndrome, the most common cause of mental retardation. This syndrome is associated with cardiac defects (VSD), brain defects (microcephaly, holoprosencephaly) and abnormal facies (short palpebral fissures and a smooth/long philtrum). (Choice C) Cefazolin is safe during pregnancy and is not associated with neural tube defects. (Choice D) Uncontrolled hypertension during pregnancy causes uteroplacental insufficiency, and may lead to fetal death or intrauterine fetal growth retardation. Hypertension also increases the risk of placental abruption. It is not associated with any congenital defects. (Choice E) Smoking during pregnancy may cause fetal hypoxia, and may result in intrauterine growth retardation, low birth weight and premature delivery. Smoking is not associated with the development of neural tube defects.

A 22-year-old man is brought to the ER with severe back pain and lower limb weakness after falling off the top of a 16-ft ladder and landing on his feet. The ER physician orders a CT scan, and the given midsagittal CT directs the physician toward making which of the following diagnoses? (A) Herniated intervertebral disc (B) Burst fracture of T12 vertebral body (C) Lumbar spinal stenosis (D) Spondylolisthesis of L5-S1 articulation (E) Excessive lumbar lordosis

B: Burst fracture of T12 vertebral body. The sagittal CT clearly shows a burst (crush or compression) fracture of the 12th thoracic vertebral body (T12). This T12 burst fracture also presents with a fracture fragment that is displaced posterior into the vertebral canal where it would compress the terminal end of the spinal cord, the medul- lary cone (conus medullaris), which typically lies within the T12-L3 vertebral levels. Compression of the spinal cord would lead to pain within the lower back that would likely extend into the lower limbs. This injury might also result in incon- tinence, loss of sensation, and paraplegia or paraparesis (loss of motor function). Remember that a CT scan is a valuable means of viewing "blood and bone," and this type of radiologi- cal imaging is the standard way to view injuries resulting from trauma, as seen in this patient. With this patient falling off the ladder and landing on his feet, the downward force from the high fall and the upward force from the impact on the ground have crushed the body of the T12 vertebra, resulting in the burst fracture of the T12 vertebral body seen on this sagittal CT. Choice A (Herniated intervertebral disc) is incorrect. The intervertebral discs in a young person are usually so strong that the vertebrae often fracture before the discs rupture, and in this 22-year-old man, this scenario is confirmed by the sag- ittal CT scan. A herniated nucleus pulposus of the interverte- bral disc can cause lower back pain that extends into the lower limbs, as seen in this patient. However, in this CT scan, the spaces occupied by the discs, located between the vertebral bodies, are relatively uniform in both the lumbar and lower thoracic regions, which would also rule out a herniated inter- vertebral disc. A CT scan is valuable in assessing bone injuries while a MRI would be the best means to view damage to the intervertebral discs, the spinal cord and its nerve roots, and the ligaments of the spine. Choice C (Lumbar spinal stenosis) is incorrect. Lumbar spinal stenosis describes a narrowing of the vertebral foramen or canal that is normally seen in older individuals with a genetic disposition toward this condition. In the given CT, the vertebral canal, which houses the spinal cord, is relatively uniform throughout the lumbar region. The fracture fragment from the T12 vertebral body does enter the spinal canal and impinges the spinal cord and its nerve roots. However, lumbar spinal stenosis is usually due to degenera- tion of the intervertebral discs, which narrows the vertebral canal and potentially impinges the spinal cord and its nerve roots. Due to the age of the patient and the traumatic nature of his injury, lumbar spinal stenosis is not likely, especially with the visual evidence of a wide vertebral canal on the given CT scan. Choice D (Spondylolisthesis of L5-S1 articulation) is incorrect. Spondylolisthesis of the L5-S1 articulation is defined by the forward displacement of the vertebral body of L5 rela- tive to the upper part of the sacrum (S1). The given CT does not show spondylolisthesis of the L5-S1 articulation, so this choice can be eliminated. Choice E (Excessive lumbar lordo- sis) is incorrect. The natural curvature of the lumbar portion of the vertebral column is concave posterior, or lordosis. The secondary curvature of the lumbar spine to lordosis occurs when an infant begins to assume the upright position during standing or walking. The given CT does not show excessive lumbar lordosis (sway back or hollow back), which is often associated with weakened anterolateral abdominal muscula- ture, which is unlikely in this 22-year-old man.

A 60-year-old man has been feeling "pins and needles" and some sharp pains over his right upper chest and back for several weeks. A rash of red erupted vesicles is seen at the right border of the sternum, a few centimeters above the nipple. Antiviral treatment is initiated to treat herpes zoster. The patient recovers, is free of pain, and his skin looks normal. Which of the following structures has the same embryologic origin as the location where the viral particles are still residing? A. Dorsal horn B. Ventral horn C. Dorsal root ganglion D. Conus medullaris E. Dura mater

C. The herpes zoster virus after initial infection can remain latent in the dorsal root ganglion until the body becomes immunocompromised or stressed, and then reappears as a vesicular rash along one of the dermatomes, in this condition called shingles. The dorsal root ganglion sensory neurons are derivative of neural crest cells. Other neural crest cell derivatives include the pia mater, chromaffin cells of the adrenal medulla, thyroid parafollicular cells, Schwann cells, melanocytes, cranial nerves, connective tissue, and some bones of the skull and face. There are usually 21 pairs of denticulate ligaments, which take their origin from the pia mater and attach to the dura mater. As the pia mater is derived from neural crest cells the denticulate ligaments take their origin from the pia mater, which dictates that they share the same embryologic origin. The dorsal horn, ventral horn, and conus medullaris are all part of the spinal cord and are notochord derivatives. The dura mater is of mesenchymal origin. 252-253, 266

How much DNA does a primary spermatocyte contain? (A) 1N (B) 2N (C) 4N (D) 6N (E) 8N

C. Type B spermatogonia give rise to primary spermatocytes by undergoing DNA replication, thereby doubling the amount of DNA (2 x 2N = 4N) within the cell.

64. If the structure indicated by the letter B is fractured, which of the following structures is most likely injured? (A) Musculocutaneous nerve (B) Radial nerve (C) Deep brachial artery (D) Posterior humeral circumflex artery (E) Scapular circumflex artery

D

A 42-year-old woman presents with complaints of severe headaches, blurred vision, slurred speech, and loss of muscle coordination. Her last pregnancy 5 years ago resulted in a hydatidiform mole. Laboratory results show a high hCG level. Which of the following conditions is a probable diagnosis? (A) Vasa previa (B) Placenta previa (C) Succenturiate placenta (D) Choriocarcinoma (E) Membranous placenta

D. After a hydatidiform mole, it is very important to ensure that all the invasive trophoblastic tissue is removed. High levels of hCG are a good indicator of retained trophoblastic tissue because such tissue produces this hormone. In this case, the trophoblastic tissue has developed into a malignant choriocarcinoma and metastasized to the brain, causing her symptoms of headache, blurred vision, and so on.

A 42-year-old woman gave birth to an 8 lb baby girl with two additional toes to the right of her left big toe. Which of the following best describes this limb anomaly? A. Amelia B. Cleft foot C. Club foot D. Polydactyly E. Syndactyly

D. Polydactyly describes the presence of supernumerary digits of the hands and feet. Cleft foot occurs when one or more digital rays fail to develop, causing absence of the central digits. Clubfoot is a malrotation of the foot around the axis, and amelia is complete absence of a limb. Syndactyly is absence of digits either due to failure of digital rays to form or incomplete apoptosis. 244

A 14-month-old boy is brought to the ophthalmologist's office. His parents noticed that when photos of their child were taken, his eyes appeared white instead of red from the camera flash. Eye exam revealed poor vision in the right eye and CT scan showed calcifications within the right intraorbital mass. The boy is diagnosed with retinoblastoma. Which of the following structures share the same embryologic origin as the retina? A. CN II B. Lens C. Sclera D. Sphincter pupillae muscle of the iris E. Vitreous body

D. Retinoblastoma is a malignant carcinoma of the retina. The retina develops from the inner and outer layers of the optic cup, which is an invagination of the neuroectoderm. The outer thin layer becomes the inner pigment epithelium of the retina, while the inner layer becomes the light sensitive neural retina. The sphincter pupillae muscle of the iris develops from the neuroectoderm of the optic cup. CNII develops from the optic stalk. The lens develops from surface ectoderm. The sclera and the vitreous body develop from the mesoderm. 272-273

During a surgery rotation, a 3rd year medical student is asked by a cardiovascular surgeon to explain what "left coronary dominance" means in relation to the left coronary artery. Which of the following explanations should be given by an astute student? A. it gives rise to the anterior interventricular (IV) artery B. it is derived from the left 6th aortic arch C. it supplies the right border of the heart D. it gives rise to the posterior interventricular artery E. it supplies the margin of the heart

D. it gives rise to the posterior interventricular (IV) artery

A child is born at home w/out difficulty. Two weeks later, the mother takes the infant to her doctor, reporting that he "turns blue" when he cries. Physical examination reveals the infant in cyanotic and has a distinct systolic heart murmur. The physician suspects the baby has a tetralogy of Fallot (TOF). Which of the following conditions is a component of this syndrome? A. transposition of the great vessels B. hypertrophy of the left ventricle C. interatrial septal defect D. pulmonary infundibular stenosis E. aortic valvular atresia

D. pulmonary infundibular stenosis

A 2-year-old boy comes to the ER due to accidental inhalation of a peanut. Which of the following sites is the most likely location for the aspirated peanut in the thorax? A. larynx B. carina of the trachea C. left main bronchus D. right main bronchus E. right upper lobar bronchus

D. right main bronchus

An 87-year-old woman complains of not feeling anything on a portion of her chest wall. An examination determines that she has complete loss of sensory and motor function in a small portion of her anterior thoracic wall. Which of the following nervous structures is most likely responsible for this patient's deficit? a. Anterior ramus of a spinal nerve b. Spinal ganglion c. White rami communicantes d. Anterior roots of a spinal nerve

a. Anterior ramus of a spinal nerve

Lab: A patient has a small solid tumor in the mediastinum, which is confined at the indicated by "X" level on the image below (Fig. 2). Which of the following structures would most likely be found at this level? a. Tracheal bifurcation b. Apex of heart c. Aortic arch d. Ascending aorta beginning e. Third rib articulation

a. Tracheal bifurcation

46-year-old man is undergoing surgical repair of a gastric ulcer. An incision is made 2.5 cm left of midline, extending from just below the level of the xiphoid process to the level of the umbilicus. The surgeon retracts the rectus abdominis muscle and incises the posterior rectus sheath. At the level of the incision, aponeuroses from which of the following muscles contribute to this portion of the sheath? a. Internal oblique b. Internal oblique and transversus abdominis c. External oblique and internal oblique d. External oblique, internal oblique, and transversus abdominis e. Transversus abdominis and the transversalis fascia

b. Internal oblique and transversus abdominis

A 74-year-old woman comes to the emergency department after suffering a myocardial infarction. A thrombus in the circumflex branch of the left coronary artery is visible on a coronary angiogram. This event has most likely damaged which area of her heart? a. Right ventricle and the interventricular septum b. Left atrium and left ventricle c. Apex d. Right and left ventricles e. Right atrium and right ventricle

b. Left atrium and left ventricle

A 49-year-old woman has a neoplasm in the hilus of the left lung, anterior to the primary bronchus. Which of the following structures in the surrounding area is most likely to become compressed if the neoplasm were to continue to grow in an anterior direction? a. Esophagus b. Phrenic nerve c. Azygos vein d. Vagus nerve e. Sympathetic trunk

b. Phrenic nerve

While repairing a heart defect in a 9-year-old boy, the surgeon must avoid suturing the blood vessel that usually supplies the sinoatrial (SA) node. Which of the following major arteries usually supplies the branch to this part of the conduction system? a. Anterior interventricular b. Right coronary c. Marginal d. Circumflex e. Left coronary

b. Right coronary

Respiratory sounds can be heard more clearly with a stethoscope over the triangle of auscultation than over other areas of the back, because there is less intervening tissue to muffle the sounds. The floor of the triangle of auscultation is formed in part by which of the following spaces? a. Third intercostal b. Second intercostal c. Sixth intercostal d. Eighth intercostal e. Fourth intercostal

c. Sixth intercostal

Which of the following is the most common site for herniation of a cervical intervertebral disc? a. C7-T1 b. C1-C2 c. C2-C3 d. C6-C7 e. C3-C4

d. C6-C7

A 52-year-old woman comes to a clinic to donate bone marrow. During the procedure, the needle is pushed too deeply through the midline of the lower half of the manubrium and blood is obtained. In this situation, the needle has most likely entered which of the following structures? a. Superior vena cava b. Left ventricle c. Right atrium d. Pulmonary trunk e. Aortic arch

e. Aortic Arch

A 64-year-old woman and a 66-year-old man both have pleurisy. One has left-sided lateral thoracic pain, and the other has left-sided neck and shoulder pain. What combination of nerves, responsible for innervating the parietal pleura, is responsible for the distribution of pain reported by these individuals? a. Vagus and intercostal b. Phrenic and vagus c. Parasympathetic and sympathetic d. Greater splanchnics e. Intercostal and phrenic

e. Intercostal and phrenic

Due to torsioning of the lower limb and merging of separate premuscle masses during development, certain adult lower limb muscles are supplied by two separate nerves. These interneural fusions are termed "hybrid muscles". Which of the following is a hybrid muscle? (A) Adductor magnus (B) Gastrocnemius (C) Rectus femoris (D) Semimembranosus (E) Tibialis anterior

(A) Adductor magnus

3. A patient is unable to prevent anterior displacement of the femur on the tibia when the knee is flexed. Which of the following ligaments is most likely damaged? (A) Anterior cruciate (B) Fibular collateral (C) Patellar (D) Posterior cruciate (E) Tibial collateral

3. The Answer is D. The posterior cruciate ligament is important because it prevents forward displacement of the femur on the tibia when the knee is flexed. The anterior cruciate liga- ment prevents backward displacement of the femur on the tibia.

4. A 22-year-old woman receives a deep cut in the inguinal canal lateral to the pubic tubercle. Which of the following ligaments is lacerated within the inguinal canal? (A) Suspensory ligament of the ovary (B) Ovarian ligament (C) Mesosalpinx (D) Round ligament of the uterus (E) Rectouterine ligament

4. The answer is D. The round ligament of the uterus is found in the inguinal canal along its course. The other ligaments do not pass through the inguinal canal.

61. In which structure would ligation of the external iliac artery reduce blood pressure?

61. The answer is A. The external iliac artery becomes the femoral artery immediately after passing the inguinal ligament. Therefore, ligation of the external iliac artery reduces blood pressure in the femoral artery.

71. A lesion of the median nerve causes paralysis of this structure.

A

10. During a domestic dispute, a 16-year-old boy receives a deep stab wound around the superior angle of the scapula near the medial border, which injures both the dorsal scapular and spinal accessory nerves. Such an injury could result in paralysis or weakness of which of the following muscles? (A) Trapezius and serratus posterior superior (B) Rhomboid major and trapezius (C) Rhomboid minor and latissimus dorsi (D) Splenius cervicis and sternocleidomastoid (E) Levator scapulae and erector spinae

B

65. Destruction of this area would most likely cause weakness of supination and flexion of the forearm.

B

69. If the floor of the anatomic snuffbox and origin of the abductor pollicis brevis are damaged, which of the following bones is most likely to be involved? (A) A (B) B (C) C (D) D (E) E

B

A young mother brings her 3-year-old daughter to the pediatrician with concerns of developmental delay. The pediatrician orders a CT scan, which shows partial atresia of the cerebellum. What is the embryologic origin of the cerebellum? A. Telencephalon B. Diencephalon C. Mesencephalon D. Metencephalon E. Myelencephalon

D. The brain develops from the neural tube. By the 28th week the caudal and rostral neuropores close, leading to the development of three primary brain vesicles: prosencephalon (forebrain), mesencephalon (midbrain), and rhombencephalon (hindbrain). By the fifth week, the forebrain subdivides into the telencephalon, which becomes the cerebral hemispheres, and a diencephalon, which becomes the thalamus, hypothalamus and epithalamus. The midbrain remains as the midbrain. The hindbrain subdivides into a metencephalon, which becomes the pons and cerebellum, and the myelencephalon, which becomes the medulla oblongata. 38, 254

Which of the following are components of the definitive chorion? (A) Extraembryonic somatic mesoderm and epiblast (B) Extraembryonic somatic mesoderm and cytotrophoblast (C) Extraembryonic somatic mesoderm and syncytiotrophoblast (D) Extraembryonic somatic mesoderm, cytotrophoblast, and syncytiotrophoblast (E) Extraembryonic visceral mesoderm, cytotrophoblast, and syncytiotrophoblast

D. The definitive chorion consists of three components: extraembryonic somatic mesoderm, cytotrophoblast, and syncytiotrophoblast. The chorion defines the chorionic cavity in which the embryoblast is suspended and is vital in the formation of the placenta.

A 51-year-old man visits the emergency department with high fever, left lower quadrant pain, and blood in his urine (hematuria) for the past 3 days. A CT scan reveals the presence of a thrombus in the left inferior vena cava (IVC). Which of the following structures is most likely responsible for the presence of a left IVC? A. Right anterior cardinal vein B. Right vitelline vein C. Left vitelline vein D. Left supracardinal vein E. Left anterior cardinal vein

D. The left inferior vena cava results from regression of the right supracardinal vein while the left supracardinal vein persists. The right anterior cardinal vein and left cardinal veins contribute to the formation of the internal jugular veins and superior vena cava (with the common cardinal veins). The vitelline veins, during development, drain the yolk sac; they contribute to the formation of the hepatic veins, portal vein, and distal part of the inferior vena cava. 147, 192

A 34-year-old pregnant woman of 29 weeks pregnancy visits her gynecologist for a regular check up. General examination of the abdomen shows underdevelopment of the fetus for its developmental stage. Ultrasound examination of the fetus shows oligohydraminos. Which of the following conditions is most likely cause for this? A. Renol (kidney) agenesis B. Esophageal atresia C. Maternal diabetes D. Tracheo-esophageal fistula E. Anencephaly

D. Tracheo-esophageal fistula*

9. A 34-year-old woman crashes into a tree during a skiing lesson and is brought to a hospital with multiple injuries that impinge the dorsal primary rami of several spinal nerves. Such lesions could affect which of the following muscles? (A) Rhomboid major (B) Levator scapulae (C) Serratus posterior superior (D) Iliocostalis (E) Latissimus dorsi

D

A 6-hour-old female newborn is assessed in the neonatal intensive care unit (NICU). Medical records show cyanosis, an APGAR score of 0, pulmonary edema on radiograph, and a tetralogy of Fallot confirmed by echocardiography. Which of the following conditions is most likely associated with this clinical picture? A. Pulmonary edema B. Hepatosplenomegaly C. Cardiomegaly D. Anal atresia E. Meckel's diverticulum

D. VACTERL syndrome is a cooccurrence of birth defects. It commonly occurs in mothers who are 13 to 19 years of age who have taken progesteroneestrogen birth control pills during the critical stage of development. The acronym stands for Vertebral, Anal, Cardiac, Tracheal, Esophageal, Renal, and Limb anomalies. Tetralogy of Fallot is a common heart defect seen with the VACTERL association, as in anal atresia. 307

Diabetes mellitus is a disease characterized by elevated blood glucose levels (hyperglycemia). Persistent hyperglycemia is associated with degeneration of Schwann cells, followed by axonal damage. In the autonomic nervous system, the structures most susceptible to damage are the relatively smaller, nonmyelinated, postsynaptic axons. Which of the following structures are most likely to contain autonomic axons damaged secondary to diabetes mellitus? A. greater splanchnic nerves B. lumbar splanchnic nerves C. pelvic splanchnic nerves D. cardiac nerve branches of the cervical sympathetic ganglia E. cardiac nerve branches of the vagus nerve

D. cardiac nerve branches of the cervical sympathetic ganglia

25. A 25-year-old soldier suffers a gunshot wound on the lower part of his back and is unable to move his legs. A neurologic examination and magnetic resonance imaging (MRI) scan reveal injury of the cauda equina. Which of the following is most likely damaged? (A) Dorsal primary rami (B) Ventral primary rami (C) Dorsal roots of the thoracic spinal nerves (D) Ventral roots of the sacral spinal nerves (E) Lumbar spinal nerves

D

63. If the structure indicated by the letter A is calcified, which of the following muscles is most likely paralyzed? (A) Deltoid (B) Teres major (C) Teres minor (D) Infraspinatus (E) Subscapularis

D

Which structure branches into the bronchial arteries?

E. Descending (thoracic) aorta

A patient has process of a muscle indicated by the arrow in the picture of the back of the shoulder. He will have weakness in performing which of the following movements of the shoulder joint? A. Medial abduction of the arm B. Abduction of the shoulder from 0-15 degrees C. Abduction of the shoulder from 15-90 degrees D. Flexion of the arm E. Lateral rotation of the arm

E. Lateral rotation of the arm

A 23yo man is stabbed in a bar fight. The blade of the knife enters his chest in the left 5th intercostal space, just lateral to the sternum, and pierces to a depth of approximately 4cm. What structure is most likely damaged? A. left lung B. pulmonary trunk C. left bronchus D. stomach E. pericardium

E. Pericardium

A child is crying uncontrollably holding its flexed elbow in pronated position, is brought to the E/R by its mother. She explained that this happened as she pulled the child by forearm and since then the child is in severe pain. The physician diagnosed it as subluxation of the head of the radius. The head of the radius is pulled from which of the following ligaments? a. glenoidal labrum b. oblique cord c. radial collateral d. ulnar collateral e. Annular

E. annular

Attempting to do a backflip with his bicycle off a high ramp at the finish line of a race, a 24-year-old professional BMX rider fell from a height of 20 ft and attempted to land on his feet. A sagittal CT reveals a burst fracture of the L1 vertebral body with a posterior displaced fracture fragment compressing the medullary cone (conus medullaris). Which of the following spinal cord segments would most likely be impinged by the bone fragment in this injury? (A) T7 (B) T9 (C) T11 (D) L2 (E) S2

E: S2. The medullary cone (conus medul- laris) is the tapered terminal end of the spinal cord, composed of the sacral and coccygeal segments. In adults, the conus medullaris typically lies within the T12-L3 vertebral levels, but it generally ends at approximately L2. Thus, displacement fractures of these vertebrae are likely to affect one or more of the sacral-coccygeal spinal cord segments. In this case, given the statistical range of variation for the medullary cone, the displaced fracture fragment of the L1 vertebral body is more likely to impinge the lower lumbar and upper sacral segments of the medullary cone. S2 is the only choice from this region. When thinking of the spinal cord, remember the "rule of 2," which states the conus medullaris usually ends at the second lumbar vertebra (L2); the subarachnoid space ends at the sec- ond sacral vertebra (S2); and the terminal filum (filum termi- nale), a long connective tissue (pia mater) strand extending from the end of the medullary cone that anchors the inferior aspect of the spinal cord, usually attaches to the second coc- cygeal vertebra (Co2). Due to the location of the displaced fracture fragment of the L2 vertebral body, the higher sacral segments of the spinal cord related to the medullary cone are more likely to be damaged. Choice A (T7) is incorrect. The normal range of variation for the location of the conus medullaris in adults is the T12-L3 vertebral levels. The sev- enth thoracic (T7) spinal cord segment lies posterior to the vertebral bodies of the T6 and T7 vertebrae. Therefore, the T7 spinal cord segment is located well above the displaced posterior fracture fragment of the L1 vertebral body and the medullary cone, and it would not be damaged in this patient. Choice B (T9) is incorrect. The ninth thoracic (T9) spinal cord segment lies posterior to the vertebral bodies of the T8 and T9 vertebrae. Therefore, the T9 spinal cord segment is located well above the displaced posterior fracture frag- ment of the L1 vertebral body and the medullary cone, and it would not be damaged in this patient. Choice C (T11) is incorrect. The eleventh thoracic (T11) spinal cord segment lies posterior to the bodies of the T10 and T11 vertebrae. Therefore, the T11 spinal cord segment is located well above the displaced posterior fracture fragment of the L1 vertebral body and the medullary cone, and it would not be damaged in this patient. Choice D (L2) is incorrect. The second lum- bar (L2) spinal cord segment lies posterior to the vertebral body of T12. Therefore, the L2 spinal cord segment is located well above the displaced posterior fracture fragment of the L1 vertebral body and the medullary cone, and it would not be damaged in this patient. The spinal nerve roots of L2 may be damaged by the displaced posterior fracture fragment; how- ever, this question asks specifically for the spinal cord segment that would be damaged.

While performing a newborn checkup on a baby boy, a pediatrician discovers the external urethral orifice (urethral opening) is located on the ventral (bottom) surface of the penis. What is the most likely diagnosis in this newborn? (A) Epispadias (B) Hypospadias (C) Micropenis (D) Exstrophy of the bladder (E) Cryptorchidism

The answer is B: Hypospadias. Hypospadias is a congenital defect of the external genitalia, in which the external urethral orifice is incompletely developed and located along the ventral (bottom) surface of the penis. This common anomaly of the external genitalia is seen in approximately 1:300 newborn males. It may be easy to detect due to the presence of a hooded foreskin, i.e., a foreskin that is developed normally on the dorsal (top) and sides of the glans penis but not the ventral (bottom) surface, not appreciated in the figure. Though a hooded foreskin is common in newborns with hypospadias, the foreskin can be completely formed in approximately 5% of the cases. The severity of hypospadias can also vary greatly. The mildest form, glanular (first degree) hypospadias, occurs in approximately 60% of the cases. Penile (second degree) hypospadias occurs in approximately 25% of cases and is described as the external urethral orifice being located along the anterior shaft of the penis from the midshaft of the penis to the subcoronal region (ventral to the glans penis). Penile hypospadias is illustrated in the given drawing. Finally, perineal (third degree) hypospadias results in approximately 15% of cases. In this condition, the external urethral ori- fice is located in the perineum, midscrotum, or base of the penis. During development in males, the phallus elongates, and paired urogenital (urethral) folds come together in the midline and fuse to enclose the penile urethra. Hypospa- dias results when the urogenital folds fail to fuse completely along the midline. Choice A (Epispadias) is incorrect. Epis- padias is a rare congenital defect of the external genitalia, seen in 1:120,000 newborn males. In this malformation, the external urethral orifice is located along the dorsal (top) sur- face of the penis between the pubic bones and the dorsal surface of the glans penis. Epispadias can result in a short, widened penis with an abnormal curvature, reflux nephrop- athy (backward flow of urine into the kidney), and urinary tract infections. Epispadias results when the primordium of the genital tubercle develops in the region of the urorectal septum, so when the urogenital membrane ruptures, clefts are formed on the dorsal aspect of the phallus. Epispadias is often associated with exstrophy of the bladder, in which the bladder is exposed due to a defect in the anterior abdomi- nal wall. The etiology of these defects may be due to insuf- ficient migration of mesodermal tissue to the area cranial to the genital tubercle. Due to the external urethral orifice being located on the ventral (bottom) surface of the penis in this patient, hypospadias is the correct diagnosis. Choice C (Micropenis) is incorrect. Micropenis is a penis with normal developmental features that is well below the normal size range for an infant. It is caused by lack of growth during development, often due to reduced androgen production. Treatment of micropenis in infancy can be achieved with injections of human chorionic gonadotropin or testosterone. Choice D (Exstrophy of the bladder) is incorrect. Exstro- phy of the bladder is a rare congenital abnormality in which the bladder is everted through a midline defect in the lower anterior abdominal wall. It is thought to be caused by insuf- ficient migration of mesodermal tissue to the lower portion of the abdominal wall, and it often presents with epispadias, widening of the pubic symphysis, and maldescent of the tes- tes. Choice E (Cryptorchidism) is incorrect. Cryptorchidism is the absence of one or both of the testes within the scrotum due to maldescent of the testes. This finding was not noted in this patient.

During dissection of deep pelvic lymph nodes in a 60-year-old man, the surgeon accidentally severs the internal pudendal artery at its origin. Which of the following structures will maintain its normal blood supply immediately following this lesion? (A) Penis (B) Urinary bladder (C) Analcanal (D) Scrotum (E) Urogenital diaphragm

The answer is B: Urinary bladder. In males, the urinary bladder is supplied by the superior and inferior vesicle arteries, both of which usually are branches of the anterior division of the internal iliac artery. The internal pudendal artery is usually a terminal branch of the anterior division of the internal iliac artery. However, rather than supplying pelvic viscera, it exits the pelvis, enters the posterolateral aspect of the perineum, and provides the main supply to that entire region. Severing the internal pudendal artery usually does not alter blood flow to the bladder. However, remember that the internal iliac arterial tract is the most variable branching network of all the arteries. In this variation pattern, the internal pudendal artery may be closely related to the inferior vesicle artery, or, may be well removed from it. Choice A (Penis) is incorrect. The penis is supplied by terminal branches of the internal pudendal artery within the perineum. Thus, it will be deprived of blood in case of lesion at the origin of the internal pudendal artery. Choice C (Anal canal) is incorrect. The anal canal is supplied by the inferior anal (rectal) branches of the internal pudendal artery within the perineum. It will be deprived of blood in case of lesion at the origin of the internal pudendal artery. Choice D (Scrotum) is incorrect. The scrotum is supplied by posterior scrotal branches of the internal pudendal artery and anterior scrotal branches of the external pudendal artery (a branch of the femoral artery). Thus, lesion of the internal pudendal artery will reduce blood flow to the scrotum. Choice E (Urogenital diaphragm) is incorrect. The structures of the anatomically questionable urogenital diaphragm are supplied by the perineal branches of the internal pudendal artery. Thus, this area will be deprived of blood following lesion of the internal pudendal artery.

A 34-year-old woman has a murmur with its maximal intensity located at the apex of the heart. Which of the following disorders is the most likely cause of this finding? a. Pulmonary valve stenosis b. Mitral valve stenosis c. Aortic valve stenosis d. Atrial septal defect e. Tricuspid valve stenosis

b. Mitral valve stenosis

Lab: The indicated by "X" on the image below (Fig. 3) structure is projected on which of the following costal cartilage level? a. Sixth b. Third c. Forth d. Fifth e. Second

c. Forth

A 33-year old female is evaluated after sustaining a neck and shoulder injury. The nerve indicated by a red arrow on the illustration below (fig. 5) is injured in this patient. Which of the following functions is most likely affected? a. Loss of sensation on medial forearm b. Extension of the distal phalanx of the thumb c. Loss of sensation on lateral forearm d. Flexion of the distal phalanx of the thumb e. Flexion of the distal phalanx of the index finger f. Adduction and abduction of the medial four fingers

f. Adduction and abduction of the medial four fingers

A 16-year-old boy was fishing barefoot in a muddy river when the plantar surface of his foot was cut by unseen debris. He suffers a large transverse cut, penetrating the first two layers of his plantar musculature, in the area of the first cuneiform bone. In the emergency room, his physician notes a complete inability to flex and abduct the big toe and numbness on the plantar aspect of the three medial toes. Which of the following nerves is most likely damaged? (A) Medial plantar nerve (B) Lateral plantar nerve (C) Sural nerve (D) Deep fi bular nerve (E) Superfi cial fi bular nerve

(A) Medial plantar nerve

As part of a physical examination to evaluate lower limb function, a physician asks a patient to abduct her second through fifth toes. What specific nerve is the doctor testing? (A) Sural nerve (B) Deep fibular nerve (C) Superficial fibular nerve (D) Medial plantar nerve (E) Lateral plantar nerve

(E) Lateral plantar nerve

A 15-year-old girl, unaware that she is pregnant, borrows her friend's Accutane (retinoic acid; vitamin A) to combat an acne problem. She uses the Accutane for about 2 months, which corresponds to weeks 7 to 15 of the embryonic development of her fetus. Which of the following skeletal elements is most likely to be absent in the newborn infant? (A) Ilium (B) Femur (C) Patella (D) Tibia (E) Phalanges

(E) Phalanges

A veteran infantry soldier develops painful fl at feet after several years of service including hundreds of miles of marches. The pain is particularly acute on the medial aspect of his sole. Which of the following structures is most likely strained in this condition? (A) Calcaneal (Achilles) tendon (B) Plantar calcaneocuboid (long plantar) ligament (C) Extensor retinaculum (D) Tendon of the fi bularis (peroneus) longus muscle (E) Plantar calcaneonavicular (spring) ligament

(E) Plantar calcaneonavicular (spring) ligament

A 35-year-old male prisoner received a right gluteal intramuscular (IM) injection during a visit to the infi rmary. Following the injection, the man experienced a painful, swollen right leg. Within a month, he complained that his right leg started to shrink. Examination revealed muscle wasting with fasciculations in the L4-S1 distribution and marked weakness in dorsiflexion, inversion, and eversion at the ankle joint. He also exhibited a typical high-steppage gait indicating right foot drop. What nerve was most likely damaged during the gluteal IM injection? (A) Superior gluteal nerve (B) Common fi bular nerve (C) Tibial nerve (D) Inferior gluteal nerve (E) Sciatic nerve

(E) Sciatic nerve

12. A 53-year-old man with a known history of emphysema is examined in the emergency department. Laboratory findings along with examination indicate that the patient is una- ble to exchange oxygen in the air and carbon dioxide in the blood. This exchange occurs in which portion of the respiratory system? (A) Bronchi (B) Alveolar (air) sac (C) Nasal cavity (D) Larynx (E) Trachea

12. The answer is B. The respiratory portion of the lung contains the alveolar (air) sacs or alveoli, which are surrounded by networks of pulmonary capillaries. Oxygen and carbon dioxide exchange occurs across the thin walls of the alveoli and blood capillaries with the aid of the diaphragm and thoracic cage. The nasal cavity, larynx, trachea, and bronchi are air-conducting portions.

1. A 68-year-old woman with uterine carcinoma undergoes surgical resection. This cancer can spread directly to the labia majora in lymphatics that follow which of the following structures? (A) Pubic arcuate ligament (B) Suspensory ligament of the ovary (C) Cardinal (transverse cervical) ligament (D) Suspensory ligament of the clitoris (E) Round ligament of the uterus

1. The answer is E. The round ligament of the uterus runs laterally from the uterus through the deep inguinal ring, inguinal canal, and superficial inguinal ring and becomes lost in the subcutaneous tissues of the labium majus. Thus, carcinoma of the uterus can spread directly to the labium majus by traveling in lymphatics that follow the ligament.

12. An 83-year-old man with a typical coronary circulation has been suffering from an embolism of the circumflex branch of the left coronary artery. This condition would result in ischemia of which of the following areas of the heart? (A) Anterior part of the left ventricle (B) Anterior interventricular region (C) Posterior interventricular region (D) Posterior part of the left ventricle (E) Anterior part of the right ventricle

12. The answer is D. The circumflex branch of the left coronary artery supplies the posterior portion of the left ventricle. The anterior interventricular artery supplies the anterior aspects of the right and left ventricles and the anterior interventricular septum.

10. A 7-year-old girl comes to the emergency department with severe diarrhea. Tests show that the diarrhea is due to decreased capac- ity of normal absorption in one of her organs. Which of the following organs is involved? (A) Stomach (B) Gallbladder (C) Large intestine (D) Liver (E) Pancreas

10. The answer is C. The large intestine absorbs water, salts, and electrolytes. Hence, the patient's diarrhea stems from an absorption problem. The stomach mixes food with mucus and gastric juice, which contains hydrochloric acid and enzymes, and forms chyme. The gallbladder receives bile, concentrates it, and stores it. The liver produces bile, whereas the pancreas secretes pancreatic juice, which contains digestive enzymes and which releases hormones, such as insulin and glucagon.

10. A 46-year-old woman has a history of infection in her perineal region. A comprehensive examination reveals a tear of the superior boundary of the superficial perineal space. Which of the following structures would most likely be injured? (A) Pelvic diaphragm (B) Colles fascia (C) Superficial perineal fascia (D) Deep perineal fascia (E) Perineal membrane

10. The answer is E. The superior (deep) boundary of the superficial perineal space is the perineal membrane (inferior fascia of the urogenital diaphragm). Colles fascia is the deep membranous layer of the superficial perineal fascia. The deep perineal fascia essentially divides the superficial perineal space into a superficial and deep compartment. The pelvic diaphragm consists of the levator ani and coccygeus muscles.

11. A 58-year-old man is diagnosed as having a slowly growing tumor in the deep perineal space. Which of the following structures would most likely be injured? (A) Bulbourethral glands (B) Crus of penis (C) Bulb of vestibule (D) Spongy urethra (E) Great vestibular gland

11. The answer is A. The deep perineal space contains the bulbourethral (Cowper) glands. The crus of the penis, bulb of the vestibule, spongy urethra, and great vestibular gland are found in the superficial perineal space.

13. A thoracic surgeon is going to collect a portion of the greater saphenous vein for coronary bypass surgery. He has observed that this vein runs: (A) Posterior to the medial malleolus (B) Into the popliteal vein (C) Anterior to the medial condyles of the tibia and femur (D) Superficial to the fascia lata of the thigh (E) Along with the femoral artery

13. The Answer is D. The greater saphenous vein ascends superficial to the fascia lata. It courses anterior to the medial malleolus and posterior to the medial condyles of the tibia and femur and terminates in the femoral vein by passing through the saphenous opening. The small saphenous vein drains into the popliteal vein. The greater saphenous vein does not run along with the femoral artery.

15. A 36-year-old woman received a first- degree burn on her neck, arm, and forearm from a house fire. Which of the following skin structures or functions is most likely damaged or impaired? (A) GSE nerves (B) Parasympathetic general visceral efferent nerves (C) Trophic hormone production (D) Exocrine gland secretion (E) Vitamin A production

15. The answer is D. Skin has sweat glands and sebaceous glands, which are exocrine glands. Skin produces vitamin D, but it does not produce a trophic hormone and does not pro- duce vitamin A. In addition, skin contains no GSE and parasympathetic GVE nerve fibers.

18. An obstetrician performs a median episiotomy on a woman before parturition to prevent uncontrolled tearing. If the perineal body is damaged, the function of which of the following muscles might be impaired? (A) Ischiocavernosus and sphincter urethrae (B) Deep transverse perineal and obturator internus (C) Bulbospongiosus and superficial transverse perineal (D) External anal sphincter and sphincter urethrae (E) Bulbospongiosus and ischiocavernosus

18. The answer is C. The perineal body (central tendon of the perineum) is a fibromuscular node at the center of the perineum. It provides attachment for the bulbospongiosus, the superficial and deep transverse perineal muscles, and the sphincter ani externus muscles. Other muscles (ischiocavernosus, sphincter urethrae, and obturator internus) are not attached to the perineal body.

19. A woman experiences weakness when abducting and medially rotating the thigh after this accident. Which of the following muscles is most likely damaged? (A) Piriformis (B) Obturator internus (C) Quadratus femoris (D) Gluteus maximus (E) Gluteus minimus

19. The Answer is E. The gluteus medius or minimus abducts and rotates the thigh medially. The piriformis, obturator internus, quadratus femoris, and gluteus maximus muscles can rotate the thigh laterally.

23. A 78-year-old patient presents with an advanced cancer in the posterior mediastinum. The surgeons are in a dilemma as to how to manage the condition. Which of the following structures is most likely damaged? (A) Brachiocephalic veins (B) Trachea (C) Arch of the azygos vein (D) Arch of the aorta (E) Hemiazygos vein

23. The answer is E. The hemiazygos vein is located in the posterior mediastinum. The brachiocephalic veins, trachea, and arch of the aorta are located in the superior mediastinum, whereas the arch of the azygos vein is found in the middle mediastinum.

2. A patient with a deep knife wound in the buttock walks with a waddling gait that is characterized by the pelvis falling toward one side at each step. Which of the following nerves is damaged? (A) Obturator nerve (B) Nerve to obturator internus (C) Superior gluteal nerve (D) Inferior gluteal nerve (E) Femoralnerve

2. The Answer is C. The superior gluteal nerve innervates the gluteus medius muscle. Paralysis of this muscle causes gluteal gait, a waddling gait characterized by a falling of the pelvis toward the unaffected side at each step. The gluteus medius muscle normally functions to stabilize the pelvis when the opposite foot is off the ground. The inferior gluteal nerve inner- vates the gluteus maximus, and the nerve to the obturator internus supplies the obturator internus and superior gemellus muscles. The obturator nerve innervates the adductor mus-cles of the thigh, and the femoral nerve supplies the flexors of the thigh.

2. A 17-year-old boy suffers a traumatic groin injury during a soccer match. The urologist notices tenderness and swelling of the boy's left testicle that may be produced by thrombosis in which of the following veins? (A) Left internal pudendal vein (B) Left renal vein (C) Inferior vena cava (D) Left inferior epigastric vein (E) Left external pudendal vein

2. The answer is B. A tender swollen left testis may be produced by thrombosis in the left renal vein because the left testicular vein drains into the left renal vein. The right testicular vein drains into the inferior vena cava. The left internal pudendal vein empties into the left internal iliac vein. The left inferior epigastric vein drains into the left external iliac vein, and the left external pudendal vein empties into the femoral vein.

23. A 62-year-old man is incapable of penile erection after rectal surgery with prostatectomy. The patient most likely has a lesion of which of the following nerves? (A) Dorsal nerve of the penis (B) Perineal nerve (C) Hypogastric nerve (D) Sacral splanchnic nerve (E) Pelvic splanchnic nerve

23. The answer is E. The pelvic splanchnic nerve contains preganglionic parasympathetic fibers, whereas the sacral splanchnic nerve contains preganglionic sympathetic fibers. Parasympathetic fibers are responsible for erection, whereas sympathetic fibers are involved with ejaculation. The right and left hypogastric nerves contain primarily sympathetic fibers and visceral sensory fibers. The dorsal nerve of the penis and the perineal nerve provide sensory nerve fibers.

20. A 39-year-old man is unable to expel the last drops of urine from the urethra at the end of micturition because of paralysis of the external urethral sphincter and bulbospongiosus muscles. This condition may occur as a result of injury to which of the following nervous structures? (A) Pelvic plexus (B) Prostatic plexus (C) Pudendal nerve (D) Pelvic splanchnic nerve (E) Sacral splanchnic nerve

20. The answer is C. The perineal branch of the pudendal nerve supplies the external urethral sphincter and bulbospongiosus muscles in the male. All other nervous structures do not supply skeletal muscles but supply smooth muscles in the perineal and pelvic organs. The pelvic and prostatic plexuses contain both sympathetic and parasympathetic nerve fibers. The pelvic splanchnic nerve carries preganglionic parasympathetic fibers, whereas the sacral splanchnic nerve transmits preganglionic sympathetic fibers.

21. A 34-year-old woman sustains a deep cut on the dorsum of the foot just distal to her ankle joint by a falling kitchen knife. A physician in the emergency department has ligated the dorsalis pedis artery proximal to the injured area. Which of the following con- ditions most likely occurs as a result of the injury? (A) Ischemia in the peroneus longus muscle (B) Aneurysm in the plantar arterial arch (C) Reduction of blood flow in the medial tarsal artery (D) Low blood pressure in the anterior tibial artery (E) High blood pressure in the arcuate artery

21. The Answer is C. Reduction of blood flow in the medial tarsal artery occurs because it is a branch of the dorsalis pedis artery, which begins at the ankle joint as the continuation of the anterior tibial artery. The anterior tibial and peroneal arteries supply the peroneus longus muscle. The deep plantar arterial arch is formed mainly by the lateral plantar artery. Blood pressure in the anterior tibial artery should be higher than normal. The arcuate artery should have a low blood pressure because it is a terminal branch of the dorsalis pedis artery.

24. A knife wound penetrates the superficial vein that terminates in the popliteal vein. Bleeding occurs from which of the following vessels? (A) Posterior tibial vein (B) Anterior tibial vein (C) Peroneal vein (D) Great saphenous vein (E) Lessersaphenousvein

24. The Answer is E. The lesser (small) saphenous vein ascends on the back of the leg in com- pany with the sural nerve and terminates in the popliteal vein. The peroneal vein empties into the posterior tibial vein. The anterior and posterior tibial veins are deep veins and join to form the popliteal vein. The great saphenous vein drains into the femoral vein.

27. A patient with hereditary blood clotting problems presents with pain in the back of her knee. An arteriogram reveals a blood clot in the popliteal artery at its proximal end. Which of the following arteries will allow blood to reach the foot? (A) Anterior tibial (B) Posterior tibial (C) Peroneal (D) Lateral circumflex femoral (E) Superior medial genicular

27. The Answer is D. If the proximal end of the popliteal artery is blocked, blood may reach the foot by way of the descending branch of the lateral circumflex femoral artery, which participates in the anastomosis around the knee joint. Other blood vessels are direct or indirect branches of the popliteal artery.

27. A 43-year-old woman presents with a prolapsed uterus. Repair of a prolapsed uterus requires knowledge of the supporting structures of the uterus. Which of the following structures plays the most important role in the support of the uterus? (A) Levator ani (B) Sphincter urethrae (C) Uterosacral ligament (D) Ovarian ligament (E) Arcuate pubic ligament

27. The answer is A. The pelvic diaphragm, particularly the levator ani, provides the most important support for the uterus, although the urogenital diaphragm and the uterosacral and ovarian ligaments support the uterus. The arcuate pubic ligament arches across the inferior aspect of the pubic symphysis.

28. A 31-year-old man was involved in a severe automobile accident and suffered laceration of the left primary bronchus. The damaged primary bronchus: (A) Has a larger diameter than the right primary bronchus (B) Often receives more foreign bodies than the right primary bronchus (C) Gives rise to the eparterial bronchus (D) Is longer than the right primary bronchus (E) Runs under the arch of the azygos vein

28. The answer is D. The right primary bronchus is shorter than the left one and has a larger diameter. More foreign bodies enter it via the trachea because it is more vertical than the left primary bronchus. The right primary bronchus runs under the arch of the azygos vein and gives rise to the eparterial bronchus.

31. Radiographic examination reveals a fracture of the head and neck of the fibula. Which of the following nerves is damaged? (A) Sciatic (B) Tibial (C) Common peroneal (D) Deep peroneal (E) Superficial peroneal

31. The Answer is C. The common peroneal nerve is vulnerable to injury as it passes behind the head of the fibula and then winds around the neck of the fibula and pierces the peroneus longus muscle, where it divides into the deep and superficial peroneal nerves. In addition, the deep and superficial peroneal nerves pass superficial to the neck of the fibula in the substance of the peroneus longus muscle and are less susceptible to injury than the common peroneal nerve. Other nerves are not closely associated with the head and neck of the fibula.

3. A 23-year-old man received a gunshot wound, and his greater splanchnic nerve was destroyed. Which of the following nerve fibers would be injured? (A) General somatic afferent (GSA) and preganglionic sympathetic fibers (B) General visceral afferent (GVA) and postganglionic sympathetic fibers (C) GVA and preganglionic sympathetic fibers (D) General somatic efferent (GSE) and postganglionic sympathetic fibers (E) GVA and GSE fibers

3. The answer is C. The greater splanchnic nerves contain general visceral afferent (GVA) and preganglionic sympathetic general visceral efferent (GVE) fibers.

3. On a busy Saturday night in Chicago, a 16-year-old boy presents to the emergency department with a stab wound from a knife that entered the pelvis above the piriformis muscle. Which of the following structures is most likely to be damaged? (A) Sciatic nerve (B) Internal pudendal artery (C) Superior gluteal nerve (D) Inferior gluteal artery (E) Posterior femoral cutaneous nerve

3. The answer is C. The superior gluteal nerve leaves the pelvis through the greater sciatic foramen, above the piriformis. The sciatic nerve, internal pudendal vessels, inferior gluteal vessels and nerve, and posterior femoral cutaneous nerve leave the pelvis below the piriformis.

31. A 37-year-old man is brought to the emergency department complaining of severe chest pain. His angiogram reveals thromboses of both brachiocephalic veins just before entering the superior vena cava. This condition would most likely cause a dilation of which of the following veins? (A) Azygos (B) Hemiazygos (C) Right superior intercostal (D) Left superior intercostal (E) Internal thoracic

31. The answer is D. The left superior intercostal vein is formed by the second, third, and fourth posterior intercostal veins and drains into the left brachiocephalic vein. The right superior intercostal vein drains into the azygos vein, which in turn drains into the SVC. The hemiazygos vein drains into the azygos vein, whereas the internal thoracic vein empties into the brachiocephalic vein.

32. After injury to the common peroneal nerve, which of the following muscles could be paralyzed? (A) Gastrocnemius (B) Popliteus (C) Extensor hallucis longus (D) Flexor digitorum longus (E) Tibialis posterior

32. The Answer is C. The extensor hallucis longus is innervated by the deep peroneal nerve, whereas other muscles are innervated by the posterior tibial nerve.

30. An elderly woman fell at home and fractured the greater trochanter of her femur. Which of the following muscles would continue to function normally? (A) Piriformis (B) Obturator internus (C) Gluteus medius (D) Gluteus maximus (E) Gluteus minimus

30. The Answer is D. The gluteus maximus is inserted into the gluteal tuberosity of the femur and the iliotibial tract. All of the other muscles insert on the greater trochanter of the femur, and their functions are impaired.

30. A radiologist examines posterior-anterior chest radiographs of a 27-year-old victim of a car accident. Which of the following structures forms the right border of the cardiovascular silhouette? (A) Arch of the aorta (B) Pulmonary trunk (C) SVC (D) Ascending aorta (E) Left ventricle

30. The answer is C. A cardiovascular silhouette or cardiac shadow is the contour of the heart and great vessels seen on posterior-anterior chest radiographs. Its right border is formed by the SVC, right atrium, and inferior vena cava; its left border is formed by the aortic arch (aortic knob), pulmonary trunk, left auricle, and left ventricle. The ascending aorta becomes the arch of the aorta and is found in the middle of the heart.

33. While performing a pelvic exenteration, the surgical oncologist notices a fractured or ruptured boundary of the pelvic inlet. Which of the following structures is most likely damaged? (A) Promontory of the sacrum (B) Anterior-inferior iliac spine (C) Inguinal ligament (D) Iliac crest (E) Arcuate pubic ligament

33. The answer is A. The pelvic inlet (pelvic brim) is bounded by the promontory and the anterior border of the ala of the sacrum, the arcuate line of the ilium, the pectineal line, the pubic crest, and the superior margin of the pubic symphysis.

37. The obturator nerve and the sciatic (tibial portion) nerve of a 15-year-old boy are transected as a result of a motorcycle accident. This injury would result in complete paralysis of which of the following muscles? (A) Rectus femoris (B) Biceps femoris, short head (C) Pectineus (D) Adductor magnus (E) Sartorius

37. The Answer is D. The adductor magnus is innervated by both the obturator and sciatic (tibial portion) nerves. Hence, a lesion here could cause paralysis. The rectus femoris and sartorius are innervated by the femoral nerve. The biceps femoris long head is innervated by the tibial portion of the sciatic nerve, whereas the short head is innervated by the com- mon peroneal portion of the sciatic nerve. The pectineus is innervated by both the femoral and obturator nerves.

43. A 35-year-old man has difficulty in dorsi- flexing the foot. Which of the following muscles is most likely damaged? (A) Tibialis posterior (B) Flexor digitorum longus (C) Tibialis anterior (D) Peroneus longus (E) Peroneus brevis

43. The Answer is C. The tibialis anterior muscle can dorsiflex the foot, whereas all other muscles are able to plantar flex the foot.

43. A 49-year-old woman has a large mass on the pelvic brim. Which of the following structures is most likely compressed by this mass when crossing the pelvic brim? (A) Deep dorsal vein of the penis (B) Uterine tube (C) Ovarian ligament (D) Uterine artery (E) Lumbosacral trunk

43. The answer is E. All of the listed structures do not cross the pelvic brim except the lumbosacral trunk, which arises from L4 and L5, enters the true pelvis by crossing the pelvic brim, and contributes to the formation of the sacral plexus. The deep dorsal vein of the penis enters the pelvic cavity by passing under the symphysis pubis between the arcuate and transverse perineal ligaments.

44. An injury to the leg of a golfer results in loss of the ability to invert the foot. Which of the following muscles is most likely paralyzed? (A) Tibialis posterior (B) Peroneus longus (C) Peroneus brevis (D) Peroneus tertius (E) Extensor digitorum longus

44. The Answer is A. The tibialis posterior inverts the foot. The peroneus longus, brevis, and tertius and extensor digitorum longus can evert the foot.

44. A 32-year-old patient has a tension pneumothorax that can be treated with needle aspiration. To avoid an injury of the intercostal neurovascular bundle, the needle may be inserted in which of the following locations? (A) Above the upper border of the ribs (B) Deep to the upper border of the ribs (C) Beneath the lower border of the ribs (D) Between the external and internal intercostals (E) Through the transversus thoracis muscle

44. The answer is A. The intercostal veins, arteries, and nerves run in the costal groove beneath the inferior border of the ribs between the internal and innermost layers of muscles. The transversus thoracis muscles are situated in the internal surface of the lower anterior thoracic wall.

45. An orthopedic surgeon ligates the posterior tibial artery at its origin. Which of the following arteries has no blood flow immediately after the ligation? (A) Peroneal (B) Dorsalis pedis (C) Superior medial genicular (D) Anterior tibial (E) Descending genicular

45. The Answer is A. The peroneal artery is a branch of the posterior tibial artery. The dorsalis pedis artery begins anterior to the ankle as the continuation of the anterior tibial artery. The superior medial genicular artery is a branch of the popliteal artery, and the descending genicular artery arises from the femoral artery.

4. A 16-year-old patient received a stab wound, and axons of the general somatic efferent (GSE) neurons to the shoulder mus- cles were severed. The damaged axons: (A) Would carry impulses toward the cell bodies (B) Would carry impulses away from the cell bodies (C) Would carry pain impulses (D) Are several in numbers for multipolar neurons (E) Are found primarily in the gray matter

4. The answer is B. The axons of the neurons carry impulses away from the cell bodies, and dendrites carry impulses to the cell bodies. The axons contain sensory or motor fibers. Multipolar neurons have several dendrites and one axon. The GSE neurons do not carry sensory impulses. The gray matter of the central nervous system consists largely of neuron cell bodies, dendrites, and neuroglia, whereas the white matter consists largely of axons and neuroglia

4. A 17-year-old boy was involved in a gang fight, and a stab wound severed the white rami communicantes at the level of his sixth thoracic vertebra. This injury would result in degeneration of nerve cell bodies in which of the following structures? (A) Dorsal root ganglion and anterior horn of the spinal cord (B) Sympathetic chain ganglion and dorsal root ganglion (C) Sympathetic chain ganglion and posterior horn of the spinal cord (D) Dorsal root ganglion and lateral horn of the spinal cord (E) Anterior and lateral horns of the spinal cord

4. The answer is D. The white rami communicantes contain preganglionic sympathetic GVE fibers and GVA fibers, whose cell bodies are located in the lateral horn of the spinal cord and the dorsal root ganglia. The sympathetic chain ganglion contains cell bodies of the postganglionic sympathetic nerve fibers. The anterior horn of the spinal cord contains cell bodies of the GSE fibers. The dorsal root ganglion contains cell bodies of GSA and GVA fibers.

44. A 26-year-old man comes to a hospital with fever, nausea, pain, and itching in the perineal region. On examination by a urologist, he is diagnosed as having infected bulbourethral (Cowper) glands. Which of the following structures is/are affected by this infection? (A) Superficial perineal space (B) Sphincter urethrae (C) Production of sperm (D) Testis (E) Seminal vesicles

44. The answer is B. The bulbourethral glands lie on either side of the membranous urethra, embedded in the sphincter urethrae. Their ducts open into the bulbous part of the penile urethra. Semen—a thick, yellowish-white, viscous, spermatozoa-containing fluid—is a mixture of the secretions of the testes, seminal vesicles, prostate, and bulbourethral glands. Sperm, or spermatozoa, are produced in the seminiferous tubules of the testis and mature in the head of the epididymis. The seminal vesicles are lobulated glandular structures, produce the alkaline constituent of the seminal fluid that contains fructose and choline, and lie inferior and lateral to the ampullae of the ductus deferens against the fundus (base) of the bladder.

40. A 32-year-old carpenter fell from the roof. The lateral longitudinal arch of his foot was flattened from fracture and displace- ment of the keystone for the arch. Which of the following bones is damaged? (A) Calcaneus (B) Cuboid bone (C) Head of the talus (D) Medial cuneiform (E) Navicular bone

40. The Answer is B. The keystone for the lateral longitudinal arch is the cuboid bone, whereas the keystone for the medial longitudinal arch is the head of the talus. The calcaneus, navic- ular, and medial cuneiform bones form a part of the medial longitudinal arch, but they are not keystones. The calcaneus also forms a part of the lateral longitudinal arch.

41. A 57-year-old patient has a heart murmur resulting from the inability to maintain constant tension on the cusps of the AV valve. Which of the following structures is most likely damaged? (A) Crista terminalis (B) Septomarginal trabecula (C) Chordae tendineae (D) Pectinate muscle (E) Anulus fibrosus

41. The answer is C. The chordae tendineae are tendinous strands that extend from the papillary muscles to the cusps of the valve. The papillary muscles and chordae tendineae prevent the cusps from being everted into the atrium during ventricular contraction.

5. A 47-year-old woman is unable to invert her foot after she stumbled on her driveway. Which of the following nerves are most likely injured? (A) Superficial and deep peroneal (B) Deep peroneal and tibial (C) Superficial peroneal and tibial (D) Medial and lateral plantar (E) Obturator and tibial

5. The Answer is B. The deep peroneal and tibial nerves innervate the chief evertors of the foot, which are the tibialis anterior, tibialis posterior, triceps surae, and extensor hallucis longus muscles. The tibialis anterior and extensor hallucis longus muscles are innervated by the deep peroneal nerve, and the tibialis posterior and triceps surae are innervated by the tibial nerve.

51. A 28-year-old basketball player falls while rebounding and is unable to run andjump. On physical examination, he has pain and weakness when extending his thigh and flexing his leg. Which muscle involved in both movements is most likely injured? (A) Short head of biceps femoris (B) Adductor magnus (C) Semitendinosus (D) Sartorius (E) Gracilis

51. The Answer is C. The semitendinosus extends the thigh and flexes the leg. The short head of the biceps femoris flexes the leg. The adductor magnus adducts, flexes, and extends the thigh. The sartorius and gracilis can flex the thigh and leg.

62. A knife wound to the obturator foramen might injure which structure?

62. The answer is D. The obturator foramen transmits the obturator nerve and vessels. Therefore, the knife wound in this foramen injures the obturator nerve and vessels.

64. The peroneus longus muscle tendon is damaged in a groove of a tarsal bone by fracture. Which bone in the radiograph is most likely fractured?

64. The Answer is B. The cuboid bone has a groove for the peroneus longus muscle tendon.

8. A 5-year-old girl is brought to the emergency department because of difficulty breathing (dyspnea), palpitations, and shortness of breath. Doppler study of the heart reveals an atrial septal defect (ASD). This malformation usually results from incomplete closure of which of the following embryonic structures? (A) Ductus arteriosus (B) Ductus venosus (C) Sinus venarum (D) Foramen ovale (E) Truncus arteriosus

8. The answer is D. An atrial septal defect (ASD) is a congenital defect in the interatrial septum. During partitioning of the two atria, the opening in the foramen secundum (the foramen ovale) usually closes at birth. If this foramen ovale is not closed completely, this would result in an ASD, shunting blood from the left atrium to the right atrium.

9. A 67-year-old patient has been given a course of antibiotics by gluteal intramuscu-lar injections after a major abdominal surgery. To avoid damaging the sciatic nerve during an injection, the needle should be inserted into which of the following areas? (A) Over the sacrospinous ligament (B) Midway between the ischial tuberosity and the lesser trochanter (C) Midpoint of the gemelli muscles (D) Upper lateral quadrant of the gluteal region (E) Lower medial quadrant of the gluteal region

9. The Answer is D. To avoid damaging the sciatic nerve during an intramuscular injection, the clinician should insert the needle in the upper lateral quadrant of the gluteal region. The inserted needle in the lower medial quadrant may damage the pudendal and sciatic nerves. The inserted needle midway between the ischial tuberosity and the lesser trochanter may damage the sciatic and posterior femoral cutaneous nerves on the quadratus femoris. The inserted needle over the sacrospinous ligament may damage the pudendal nerve and vessels.

The parents of a 2-week-old infant bring their child to the emergency department complaining of poor feeding, wheezing, coughing, and a hoarse cry in the child. Imaging confirms a double arched aorta resulting in a ring around the trachea and esophagus. What is the embryologic origin of this condition? A. Failure of the distal part of the right aorta to disappear B. Failure of the fifth aortic arch to disappear C. Failure of the first aortic arch to disappear D. Failure of the second aortic arch to disappear E. Failure of the sixth aortic arch to disappear

A. A double arch of the aorta is a rare cardiac anomaly. There is the formation of a right and left arch of the aorta, which occurs due to a persistence of the distal part of the right dorsal aorta. This creates a vascular ring around the trachea and esophagus with resulting compression of these structures. Babies present with wheezing aggravated by crying, feeding, and flexion of the neck. 211-214

A 20-year-old woman is trying to become pregnant. If she does, a blastocyst will form early in her pregnancy. Which of following statements accurately describes the formation of the blastocyst? A. Blastomere → morula → blastocyst B. Morula → zygote → blastocyst C. Blastomere → blastocyst D. Zygote → blastocyte → blastocyst E. Morula → blastocyte → blastocyst

A. After fertilization of the ovum, cleavage commences with the formation of blastomere and then a 16-cell morula. Fluid would then enter the cavity of the morula with the formation of a blastocyst. The blastocyst will then invade the endometrial wall.

18 A 20-year-old woman is trying to become pregnant. If she does, a blastocyst will form early in her pregnancy. Which of following statements accurately describes the formation of the blastocyst? A. Blastomere → morula → blastocyst B. Morula → zygote → blastocyst C. Blastomere → blastocyst D. Zygote → blastocyte → blastocyst E. Morula → blastocyte → blastocyst

A. After fertilization of the ovum, cleavage commences with the formation of blastomere and then a 16-cell morula. Fluid would then enter the cavity of the morula with the formation of a blastocyst. The blastocyst will then invade the endometrial wall. 21-24

Oxygenated blood that reaches the heart in fetal circulation from the IVC passes through which of the following valves formed by a portion of the septum secundum? A. Foramen ovale B. Ductus arteriosus C. Foramen primum D. Ductus venosus E. Truncus arteriosus

A. In the fetal heart the left and right atria communicate with each other by an opening in the septum secundum, referred to as the foramen ovale. Ductus arteriosus is a vessel that connects the pulmonary artery and the aortic arch in the fetus. Foramen primum is the perforation in the inferior septum primum, which is closed off as it fuses to the endocardial cushion. Ductus venosus is the vessel that connects the left umbilical vein to the inferior vena cava. Truncus arteriosus will give rise to the aorta and pulmonary trunk (Fig. 8-5). 196-202

During weeks 3 to 8, the embryo is most susceptible to teratogens because major organs develop during this time. Which of the following exposures during this period would most likely result in congenital deafness, low birth rate, inflammation of the retina, and jaundice? A. Toxoplasmosis B. Heroin C. Mercury poisoning D. Alcohol E. Tetracycline

A. Infectious teratogens include toxoplasmosis, which causes jaundice, intracranial calcifications and chorioretinitis, microcephaly, microphthalmia, and hydrocephalus. Congenital syphilis can lead to abnormal teeth and bones, congenital deafness, hydrocephalus, and mental retardation. Herpes simplex exposure can result in microcephaly, microphthalmia, spasticity, and mental retardation. Cytomegalovirus infection may result in cerebral palsy, mental retardation, intrauterine growth retardation, microphthalmia, blindness, deafness, and hepatosplenomegaly. Congenital rubella syndrome presents with cataracts, cardiac defects, and deafness. Alcohol exposure can result in fetal alcohol syndrome, which has a wide spectrum including mental and growth retardation plus morphogenetic disturbances. Cocaine exposure can cause microcephaly, neurobehavioral disturbances, spon taneous abortion, and urogenital disturbances. Streptomycin causes vestibulocochlear nerve defects. Methadone and heroin are behavioral teratogens and present with small birth weight, central nervous system dysfunction, and small head circumference. Tetracycline causes tooth and bone defects including yellow discoloration and hypoplasia. 304-305, 310

And emergency room physician evaluates a patient who has injured his hand in a knife fight. The physician notes that the ring and little fingers cannot be flexed at the metacarpophalangeal joint or extended at the interphalangeal joints. Weakness of which of the following muscles results in loss of the above movements? A. Lumbrical muscles B. Flexor digitorum superficialis C. Extensor digiti minimi D. Flexor digitorum profundus E. Extensor carpi ulnaris

A. Lumbrical muscles

A 20-year-old pregnant woman in her third trimester visits an obstetrician complaining of periodic spotting. Ultrasound reveals implantation of the embryo near the internal os of the cervix. Despite periodic spotting, she experiences no pain during pregnancy. Which of the following describes this clinical symptom? A. Placenta previa B. Abruptio placentae C. Preeclampsia D. Leiomyoma E. Pelvic inflammatory disease

A. Placenta previa is defined as a placenta that has implanted into the lower segment of the uterus. It is now classified as either major, in which the placenta is covering the internal cervical os, or minor, when the placenta is sited within the lower segment of the uterus, but does not cover the cervical os. The mother will present with painless bleeding, often recurrent in the third trimester, and ultrasound scans will demonstrate the abnormal location of the placenta. The bleeding occurs due to separation of the placenta as the lower segment develops in the third trimester. A placental abruption is separation of a normally positioned placenta from the uterine wall. Preeclampsia is a serious disorder that occurs during pregnancy, usually after the 20th week of gestation. Maternal hypertension, proteinuria, and edema are essential features of this condition. Leiomyoma is a benign tumor of smooth muscle (fibroid). Pelvic inflammatory disease is characterized by inflammation and infection arising from the endocervix leading to endometritis, salpingitis, oophoritis, pelvic peritonitis and subsequently, formation of tubo-ovarian and pelvic abscesses. 81-82

A 24-year-old overweight woman visits an obstetrician and complains of not being able to get pregnant for the last 4 years. History reveals irregular menstrual periods, decreased breast size, and hair growth on the chest and belly. Transvaginal ultrasound most likely reveals a thick band around which of the following reproductive organs? A. Ovary B. Posterior wall of the uterus C. Fimbriae of the uterine (fallopian) tube D. Isthmus of the uterine (fallopian) tube E. Ampulla of the uterine (fallopian) tube

A. Polycystic ovary syndrome (PCOS) is a syndrome of ovarian dysfunction along with the cardinal features of hyperandrogenism and polycystic ovary morphology. Its clinical manifestations include menstrual irregularities, signs of androgen excess (e.g., hirsutism), and obesity. The ultrasound criteria for the diagnosis of a polycystic ovary are eight or more subcapsular follicular cysts <10 mm in diameter and increased ovarian stroma. 14-18

A 28-year-old woman visits an obstetrician for in vitro fertilization. FSH-analogs are injected to stimulate follicles. Then hCG is injected to induce final oocyte maturation. Lastly, the oocyte is retrieved via a procedure called transvaginal oocyte retrieval. The retrieved oocyte is arrested at what stage of development? A. Prophase of meiosis 1 B. Metaphase of meiosis 1 C. Prophase of meiosis 2 D. Metaphase of meiosis 2 E. Prophase of meiosis 2

A. Primary oocytes begin the first meiotic divisions before birth, but completion of prophase in meiosis 1 does not occur until adolescence. The follicular cells surrounding the primary oocytes secrete a substance, oocyte maturation inhibitor, which arrests the meiotic process of the oocyte. 14-19

44 A 3-day-old male infant was brought into the emergency department. The parents complained that their son cried constantly, especially hours after feeding. History revealed chronic diarrhea, fatigue, foul smelling stools, and inability to gain weight. The infant had not passed meconium and was also small for his age. Abdominal CT revealed obstruction of a small segment of the bowel due to the superior mesentery artery wrapping itself around the segment. A GI specialist diagnosed this intestinal deformity as a type IIIb intestinal atresia (also known as Christmas tree or apple peel deformity). Which of the following arteries is also affected? A. Middle colic/right colic/ileocecal artery B. Splenic artery C. Inferior mesenteric artery D. Left renal artery E. Left gastric artery

A. The superior mesenteric artery is closely related to the duodenum and gives rise to the middle colic, right colic, and ileocecal arteries. Therefore, any of these vessels may be affected. The splenic and left gastric arteries are branches of the celiac artery and lie superior to the intestines. The inferior mesenteric artery branches of the abdominal aorta are located too inferiorly. The left and right renal arteries branch from the aorta retroperitoneally and are not related to the intestinal tract. 141-159

A radiologist is examining a series of contrast-enhanced CT scans of a patient's thorax in evaluating fi ndings of a hypertrophied right heart. Which of the following structures is located in the pathologically enlarged right atrium? A. opening of the coronary sinus B. openings of the pulmonary veins C. septomarginal trabecula D. openings of the coronary arteries E. trabeculae carneae

A. opening of the coronary sinus

Between 1966 and 1969, seven out of eight girls whose mothers had taken a certain agent during pregnancy were diagnosed with clear cell adenocarcinoma of the vagina. As a result, the researcher who identified the cause for the rise of this condition performed a case-controlled study. Which of the following teratogens is the most likely agent that pregnant mothers took to cause this condition in their daughters? A. Alcohol B. Diethylstilbestrol (DES) C. Lithium D. Nicotine E. Folic acid

B. Diethylstilbestrol (DES) is a synthetic nonsteroidal estrogen once commonly used in pregnant women to prevent breast engorgement, among many other uses. Prenatal exposure increases risk of multiple conditions including vaginal clear cell adenocarcinoma and reproductive tract malformations. It has since been proven to be toxic and teratogenic. Fetal alcohol syndrome from maternal alcoholism has a wide spectrum including mental and growth retardation plus morphogenetic disturbances. Lithium causes heart and great vessel abnormalities in utero. Nicotine causes premature delivery, conotruncal defects, and urinary tract abnormalities (Table 8-1). 304-305

A 16-year-old boy visits the urologist with a lump in his left testis. Diagnosis of testicular teratocarcinoma is made. This tumor can be loosely referred to as "male pregnancy" because at an early stage the carcinoma contains all three primary germ layers: ectoderm, mesoderm, and endoderm. In normal embryologic development, which of the following processes give rise to these three primary germ layers? A. Morulation B. Gastrulation C. Cranio-caudal folding D. Cleavage E. Induction

B. Gastrulation occurs in early embryogenesis and is the process by which the blastula is reorganized into a three-layered structure, that is, ectoderm, mesoderm, and endoderm. Morulation involves the cleavage or division of the fertilized ovum usually into a 16-cell structure that resembles a ball. Cranio-caudal folding is a process that occurs with lateral folding of the embryo that transforms it from a flat disc into a three-dimensional tube within the body. Cleavage is the process of division of the fertilized ovum. Induction is the physiologic and chemical signal to stimulate cells to differentiate. 35-38

A woman in her first trimester visits her obstetrician for a routine ultrasound examination. Imaging reveals increased amniotic fluid surrounding the fetus. Which of the following is most likely to increase amniotic pressure? A. Renal hypoplasia B. Esophageal atresia C. Ureter obstruction D. Potter's syndrome E. Down's syndrome

B. Increase in amniotic fluid volume and pressure is called polyhydramnios. Esophageal atresia prevents the passage of excessive amniotic fluid into the stomach and intestines where it is absorbed. Fluid is eventually passed through the umbilical arteries into the placenta for removal into the maternal blood stream. This leads to a buildup of amniotic fluid and polyhydramnios. Renal hypoplasia and ureteric obstruction lead to a decrease in the amniotic fluid (oligohydramnios). Potter's and Down's syndromes have other associated anomalies. 84, 130, 138

A 27-year-old woman in a fishing town gives birth to an infant with severe neurologic symptoms of cerebral palsy including ataxia, numbness of hand and feet, and weakness of muscles. An evaluation of the infant reveals that he has Minamata disease. Which of the following teratogens/toxins consumed by the mother is the most likely cause of this congenital disorder? A. Lead B. Mercury C. Alcohol D. Cocaine E. Streptomycin

B. Minamata disease is a severe neurologic syndrome caused by methylmercury poisoning. It can be acquired or congenital, following maternal mercury ingestion as in this case. Accumulation of methyl mercury from maternal ingestion of fish and shellfish or pork (due to certain pesticide contamination) is the primary cause. Lead poisoning can cause permanent learning and behavioral disorders, and alcohol exposure can result in fetal alcohol syndrome, which has a wide spectrum including mental and growth retardation plus morphogenetic disturbances. Cocaine exposure can cause microcephaly, neurobehavioral disturbances, spontaneous abortion, and urogenital disturbances. Streptomycin exposure can result in vestibulocochlear (CN VIII) nerve defects. 309

A 32-year-old man presents to the emergency department with severe abdominal pain. Physical exam reveals rebound tenderness localized below the edge of the liver. Lab tests reveal a high white blood count, and CT scan shows an abnormal mass anterior to the right kidney. The physician suspects appendicitis due to an inflamed subhepatic appendix. Which of the following will cause the appendix to develop subhepatically? A. Rubella infection B. Malrotation of the midgut C. Failure of migration of the neural crest cells D. Meckel's diverticulum E. Failure of lateral folds to close

B. Misplacement of a normally formed appendix is due to malrotation of the midgut. Rubella infection leads to a myriad of birth defects including cataracts and cardiac problems but not GI problems. Neural crest cells do not directly contribute to the formation of the GI tract and failure of the lateral folds to close will form neural tube defects. Meckel's diverticulum is an outpouching of the ileum and may become infected, mimicking the symptoms of appendicitis, but would not be found subhepatically. 149

20 An obese 40-year-old woman is 33 weeks pregnant when she pays a visit to her obstetrician for her third trimester checkup. Physical exam reveals swollen hands. Her blood pressure (BP) is highly elevated, even though her BP throughout pregnancy has been normal. Laboratory tests reveal proteinuria. Because preeclampsia can develop into eclampsia (seizures) and put the baby and mother at risk, the physician immediately induces labor as treatment for the preeclampsia. Pain fibers from the uterus would follow which of the nerves below? A. Sympathetics (sympathetic pain line touching the peritoneum) B. Parasympathetics C. Somatic D. Somatic and sympathetics E. Sympathetics and parasympathetics

B. Pain fibers from organs and structures above the pelvic pain line will follow the sympathetic nerve fibers. Recall that the uterine fundus and body are located above the most inferior part of the pelvic peritoneal reflection, which is known as the pelvic pain line. Pain fibers from structures below the pelvic pain line will follow the parasympathetic nerves: the uterus is above the pelvic pain line. The uterus is a visceral organ and as such will not have somatic innervations. Pain fibers from the uterus are visceral afferent fibers and will only run with sympathetic fibers. 9-10

Can be removed in a surgical resection of a lobe to remove lung cancer on the diaphragmatic surface?

B. Right inferior lobar bronchus

Where does the blastocyst normally implant? (A) Functional layer of the cervix (B) Functional layer of the endometrium (C) Basal layer of the endometrium (D) Myometrium (E) Perimetrium

B. The blastocyst implants in the functional layer of the uterine endometrium. The uterus is composed of the perimetrium, myometrium, and endometrium. Two layers are identified within the endometrium: (1) the functional layer, which is sloughed off at menstruation, and (2) the basal layer, which is retained at menstruation and serves as the source of regeneration of the functional layer. During the progestational phase of the menstrual cycle, the functional layer undergoes dramatic changes; uterine glands enlarge and vascularity increases in preparation for blastocyst implantation.

A 63-year-old woman is admitted to the emergency department with severe pleuritic chest pain radiating to the bottom portion of her scapula. The pain is relieved by bending forward and worsened by lying down or during inspiration. During physical examination the patient exhibits a friction rub sound auscultated at the lower left sternal border. Which of the following embryonic structures will most likely give rise to the affected structure? A. Splanchnopleuric mesoderm B. Somatopleuric mesoderm C. Septum transversum D. Oropharyngeal membrane E. Coelomic cleft

B. The structure involved is likely the parietal pleura, a derivative of the somatopleuric mesoderm. The lateral plate mesoderm gives rise to somatopleuric mesoderm and splanchnopleuric mesoderm. The somatopleuric mesoderm is the dorsal layer that is associated with ectoderm, which forms the body wall lining and dermis. The splanchnopleuric mesoderm, the ventral layer of the lateral mesoderm, is associated with endoderm; this forms the viscera and heart. The septum transversum forms the central tendon of the diaphragm. The oropharyngeal membrane forms a septum between the primitive mouth and pharynx. The coelomic cleft is formed during the division of the coelomic cavity and does not relate to the development of the pleura (Fig. 8-4). 39, 41, 53

Which of the following components plays the most active role in invading the endometrium during blastocyst implantation? (A) Epiblast (B) Syncytiotrophoblast (C) Hypoblast (D) Extraembryonic somatic mesoderm (E) Extraembryonic visceral mesoderm

B. The syncytiotrophoblast plays the most active role in invading the endometrium of the mother's uterus. During the invasion, endometrial blood vessels and endometrial glands are eroded, and a lacunar network is formed.

A 46-year-old postal worker was exposed unknowingly to the powdered form of bacteria Bacillus anthracis (anthrax). He comes to the ER with fl u-like symptoms, including fever, chills, fatigue, headache, chest pain, and shortness of breath. A chest X-ray shows an abnormally wide space between the lungs. Enlargement of which of the following structures is the most likely cause of the widened mediastinum in this patient? A. aortic aneurysm rupture B. lymph nodes C. thymus D. respiratory bronchioles E. heart

B. lymph nodes

14. In a freak hunting accident, a 17-yearold boy was shot with an arrow that penetrated into his suboccipital triangle, injuring the suboccipital nerve between the vertebral artery and the posterior arch of the atlas. Which of the following muscles would be unaffected by such a lesion? (A) Rectus capitis posterior major (B) Semispinalis capitis (C) Splenius capitis (D) Obliquus capitis superior (E) Obliquus capitis inferior

C

23. After an automobile accident, a back muscle that forms the boundaries of the triangle of auscultation and the lumbar triangle receives no blood. Which of the following muscles might be ischemic? (A) Levator scapulae (B) Rhomboid minor (C) Latissimus dorsi (D) Trapezius (E) Splenius capitis

C

24. A 38-year-old woman with a long history of shoulder pain is admitted to a hospital for surgery. Which of the following muscles becomes ischemic soon after ligation of the superficial or ascending branch of the transverse cervical artery? (A) Latissimus dorsi (B) Multifidus (C) Trapezius (D) Rhomboid major (E) Longissimus capitis

C

When does a secondary oocyte complete its second meiotic division to become a mature ovum? (A) At ovulation (B) Before ovulation (C) At fertilization (D) At puberty (E) Before birth

C. At ovulation, a secondary oocyte begins meiosis II, but this division is arrested at metaphase. The secondary oocyte remains arrested in metaphase until a sperm penetrates it at fertilization. Therefore, the term "mature ovum" is somewhat of a misnomer, because it is a secondary oocyte that is fertilized and, once fertilized, the new diploid cell is known as a zygote. If fertilization does not occur, the secondary oocyte degenerates.

A 16-year-old girl presents on May 10 in obvious emotional distress. On questioning, she relates that on May 1 she experienced sexual intercourse for the first time, without any means of birth control. Most of her anxiety stems from her fear of pregnancy. What should the physician do to alleviate her fear? (A) Prescribe diazepam and wait to see if she misses her next menstrual period (B) Use ultrasonography to document pregnancy (C) Order a laboratory assay for serum hCG (D) Order a laboratory assay for serum progesterone (E) Prescribe diethylstilbestrol ("morningafter pill")

C. Human chorionic gonadotropin (hCG) can be assayed in maternal serum at day 8 of development and in urine at day 10. If this teenager is pregnant, the blastocyst would be in week 2 of development (day 10). Laboratory assay of hCG in either the serum or urine can be completed; however, serum hCG might be more reliable. It is important to note that if she is pregnant, she will not miss a menstrual period until May 15 at which time the embryo will be entering week 3 of development.

During an eye examination of a 6-year-old boy, an ophthalmologist sees a wormlike structure projecting from the optic disc. The diagnosis is the persistence of the distal portion of the hyaloid artery. What does the proximal portion of the hyaloid vessels eventually become? A. Optic artery and vein of the retina B. Optic artery and vein of the lens C. Central artery and vein of the retina D. Central artery and vein of the lens E. Supra-orbital vessels

C. In the area of the embryo that becomes the eyes, the optic cup (an invagination of the neuroectoderm) is formed. This is connected to the developing brain by the optic stalk. Linear grooves called retinal fissures develop in the optic cups and along the optic stalks. These fissures contain vascular mesenchyme, which develop into the hyaloid artery and vein. The hyaloid vessels supply the optic cup and lens. As the retinal fissures fuse, these vessels are enclosed within the primordial optic nerve. The distal portion usually degenerates but the proximal portion becomes the central artery and vein of the retina. 271-277

A young woman enters puberty with approximately 40,000 primary oocytes in her ovary. About how many of these primary oocytes will be ovulated over the entire reproductive life of the woman? (A) 40,000 (B) 35,000 (C) 480 (D) 48 (E) 12

C. Over her reproductive life, a woman ovulates approximately 480 oocytes. A woman ovulates 12 primary oocytes per year provided that she is not using oral contraceptives, does not become pregnant, or does not have any anovulatory cycles. Assuming a 40-year reproductive period, 40 x 12 = 480.

Laparotomy performed on a 3-year-old Caucasian male reveals an abdominal cyst. The cyst is connected by a fibrous band to the ileum and the umbilicus. Which of the following is an abnormality most closely related to this patient's condition? Select one: a. Duodenal atresia b. Hirschsprung disease c. Meckel diverticulum d. Umbilical hernia e. Imperforate anus

C. Meckel's diverticulum. The omphalomesenteric (vitelline) duct connects the midgut lumen with the yolk sac cavity early in embryonic life. It normally obliterates during the 7th week of embryonic development. If its obliteration is incomplete or abnormal, a number of abnormalities can result. 1. A persistent vitelline duct, or vitelline fistula, occurs due to complete failure of the vitelline duetto close. A small connection between the intestinal lumen and the outside of the body exists at the umbilicus. Meconium discharge from the umbilicus is seen soon after birth if such a fistula is present. 2. Meckel diverticulum is the most common vitelline duct anomaly. It results from a partial closure of the vitelline duct, with the patent portion attached to the ileum. The fibrous band may connect the tip of Meckel diverticulum with the umbilicus. 3. Vitelline sinus results from a partial closure of the vitelline duct, with the patent portion open at the umbilicus. 4. Vitelline duct cyst (enterocyst) forms if peripheral portions of the vitelline duct (connected to the ileum and umbilicus) obliterate, but the central part is left. This cyst is connected with the ileum and abdominal wall by fibrous bands. (Choice A) Duodenal atresia results from failure of the duodenum to recanalize in early embryonic life. Although often associated with Trisomy 21, it is not associated with any vitelline duct anomalies. (Choice B) Hirschsprung disease results from the failure of migration of neural crest cells into the intestinal wall. (Choice D) Umbilical hernias occur due to weakness of the abdominal wall at the umbilicus. These hernias are common in children and usually close spontaneously by 2 years of age. (Choice E) Imperforate anus occurs due to abnormal development of anorectal structures. This condition manifests during the first days of life, when the newborn fails to pass meconium. (Choice F) Omphalocele is a defect in the abdominal wall due to incomplete closure during fetal life. Abdominal organs protrude through the defect, covered with a peritoneal sac. (Choice G) Malrotation is a result of failure of the midgut to rotate counterclockwise as it returns to the abdominal cavity early during embryonic development. Malrotations leads to abnormal positioning of the intestine in the abdominal cavity and sometimes twisting of an intestinal loop (volvulus).

A neonate is found to have a mass in the sacrococcygeal area. Pathologic examination reveals that the mass contains several different tissue types resulting from a persistence of the primitive streak. The primitive streak normally gives rise to which of the following structures? Select one: a. Dorsal root ganglia b. Lining of the gastrointestinal tract c. Notochord d. Spinal cord e. Thyroid gland

C. Notochord. Gastrulation is the process that establishes three primary germ layers that derive from epiblast: ectoderm, mesoderm, and endoderm. Gastrulation begins with the development of the primitive streak and node. The primitive streak is the region of the epiblast through which pass the cells that give rise to the notochord and the mesoderm of the embryo. The only adult derivative of the notochord is the nucleus pulposus of the intervertebral disk. The mesoderm gives rise to several tissues, including muscle and connective tissue, and blood. This accounts for the many different tissue types found in a sacrococcygeal teratoma. (Choice A) The dorsal root ganglia are derived from neural crest cells. The neural crest develops at the time of neurulation, which is a process of infolding of the neural ectoderm that is induced by the notochord. (Choice B) The lining of the gastrointestinal tract is derived from endoderm. The endoderm also gives rise to evaginations of the gastrointestinal tract, such as the liver and pancreas. The smooth muscle and connective tissue of the gastrointestinal tract are derived from mesoderm. (Choice D) The spinal cord is derived from neural ectoderm, which invaginates during neurulation to form the neural tube. The caudal part of the neural tube forms the spinal cord, and the rostral part of the neural tube forms the brain. (Choice E) The thyroid gland is derived from endoderm. The endoderm of the floor of the pharynx evaginates to form the thyroglossal duct, which descends to form the thyroid gland. The adult site of the evagination of the thyroglossal duct is marked by the foramen cecum on the tongue.

A 59 year old woman is brought to the ER in respiratory distress after her abdomen impacted the steering wheel in a motor vehicle accident. Auscultation of the left chest reveals the absence of audible breath sounds, but the presence of bowel sounds is noted. The X-ray reveals radio parity in her lower left hemithorax and a rightward mediastinal shift. Which of the following diagnoses would best explain the patient's presentation? A. Tension pneumothorax B. Hemopneumothroax C. Diaphragmatic hernia D. Emphysema E. Left lower lobe pneumonia

C. diaphragmatic hernia

A 28-year-old pregnant woman in her third trimester visits her obstetrician's office with her husband. The husband is short, with short limbs, bow legs, lumbar lordosis, and a large head with frontal bossing. Ultrasound revealed the baby to have a short humerus. As a precaution, genetic testing is ordered. Assuming the genetic test comes back positive for a mutation in the FGF3 gene, what other features are more than likely to be present in their child? A. Pigeon chest (Marfan's syndrome) B. Brittle bones (osteogenesis imperfecta) C. Trident hand (achondroplasia) D. Cardiac abnormalities E. Mental retardation

C. The FGFR3 gene provides instructions for making a protein called fibroblast growth factor receptor 3. These proteins play a role in several important cellular processes, including regulation of cell growth and division, determination of cell type, formation of blood vessels, wound healing, and embryo development. Mutations in the FGFR3 gene cause more than 99% of cases of achondroplasia. In achondroplasia (autosomal dominant disorder) the problem is not in forming cartilage but in converting it to bone (endochondral ossification at the epiphysial cartilage plates), particularly in the long bones of the limbs. Other features include an average-size trunk, short arms and legs with particularly short upper arms and thighs, and limited range of motion at the elbows. The head is enlarged (macrocephaly) with a prominent forehead and a "scooped out" nose (flat nasal bone). Fingers are typically short and the ring finger and middle fingers may diverge, giving the hand a three-pronged (trident) appearance. Also, lordosis and kyphosis of the vertebral column may be present. 235, 236, 302

A 56yo construction worker comes to his family physician complaining of pain in the beck and shoulder, numbness and tingling in the right fingers, and a weak grip in his right hand. His symptoms are exacerbated by carrying heavy objects on his right shoulder. The given plain x-ray film indicates the presence of a right cervical (C7) rib in close approximation to the proximal part of the first rib. Given the signs and symptoms of this patient, what structure is most likely impinged by the cervical rib? A. right common carotid artery B. brachiocephalic trunk C. right subclavian artery D. vertebral artery E. right brachiocephalic vein

C. right subclavian artery

In preparing for board certification in thoracic surgery, a surgeon reviews the geographic relations of critical structures in the chest. Which of the following is correct regarding the relationship between the vagus nerves in their passage through the thorax? A. Right vagus passes onto the anterior aspect of the esophagus to become the anterior vagus B. Left vagus passes through the anterior mediastinum ] C. Right vagus passes posterior to the root of the lung D. Left vagus passes across the posterior side of the aortic arch E. Right vagus passes through the middle mediastinum

C. right vagus passes posterior to the root of the lung

A 9-month-old male infant previously diagnosed with Down's syndrome is brought to the pediatrician by his parents because they observe a noticeable lump in the back of his neck. The physician learns that the infant has just recovered from a respiratory infection. Further analysis reveals the presence of fluid and white blood cells, an indication of embryonic lymphatic fluid. A differential diagnosis by the pediatrician is that there has been a failure of the jugular lymph sacs to join the lymphatic system, thereby preventing lymph drainage. Which of the following embryonic conditions could explain this case? A. Hemangioma B. Thyroglossal cyst C. Lingual cyst D. Cystic hygroma E. Squamous cell carcinoma of the larynx

D. Cystic hygromas are large swellings that usually appear in the inferolateral part of the neck and consist of large, single or multilocular, fluid-filled cavities. Hygromas may be present at birth, but they often enlarge and become evident during later infancy. Hygromas are believed to arise from parts of a jugular lymph sac that are pinched off, or from lymphatic spaces that do not establish connections with the main lymphatic channels, whereas a thyroglossal cyst is a fibrous cyst that forms from a persistent thyroglossal duct. Lingual cysts in the tongue may be derived from remnants of the thyroglossal duct. They may enlarge and produce pharyngeal pain, dysphagia (difficulty in swallowing), or both. 222, 285

Nuchal translucency is used to screen for congenital defects during the 11th through 14th weeks of pregnancy. This procedure is recommended by an obstetrician for a 35-year-old pregnant woman who previously had a child with Down's syndrome (trisomy 21). Which of the following would be common to see in addition to a larger than normal accumulation of fluid at the back of the baby's head, indicating the possibility of another baby with Down's syndrome? A. Meromelia B. Hydrocephalus C. Anencephaly D. Atrioventricular septal defect E. Plagiocephaly

D. Down's syndrome is a congenital abnormality involving a triplication of chromosome 21. It is more common in children of women who become pregnant at 35 years or older (advanced maternal age is a risk factor). Down's syndrome is associated with various abnormalities including congenital heart defects. The most common congenital heart defect in Down's syndrome is endocardiac cushion type of atrial septal defect (40%) followed by ventricular septal defects (35%). 68, 298, 329

Surgery is performed on a 4-day-old baby who has a posterior mass that was diagnosed in utero. A well-defined mass located at the bottom of the spinal cord is found and completely removed. On pathologic examination, the mass contains tissue from all three germ layers. This mass most likely arises from which of the following? Select one: a. Abnormal gastrulation b. Endoderm c. Mesoderm d. Remnants of the primitive streak e. The notochord

D. Remnants of the primitive streak. This baby had a sacrococcygeal teratoma. Teratomas occur in many locations and present as masses. The sacrococcygeal region is the most frequent tumor site. These tumors occur most commonly in infants and may be diagnosed in utero or at birth. The majority of affected infants are female. The frequency of malignancy increases with age at presentation. The tumor arises from remnants of the primitive streak, and often contains derivatives of all three germ layers: ectoderm, mesoderm, and endoderm. Primitive streak appears in the embryonic period as an ectodermal ridge at the caudal end of the embryonic disc, and normally degenerates. Gastrulation is a process that establishes the three primary germ layers. This starts with the formation of the primitive streak within the epiblast. (Choice A) Abnormal gastrulation is the etiology of caudal dysplasia or sirenomelia. Specifically, there is disturbed migration of mesoderm. This is a constellation of syndromes ranging from minor lesions of the lower vertebrae to complete fusion of the lower limbs. (Choice B) Endoderm is one of the three primary germ layers. Endoderm gives rise to structures such as the biliary tree, urinary bladder, vagina, middle ear cavity, and parenchyma of the liver and pancreas, as well as other structures. It does not give rise to a sacrococcygeal teratoma. (Choice C) Mesoderm is one of the three primary germ layers. Mesoderm gives rise to muscle, connective tissue, bone and cartilage, heart, spleen, kidney, and other structures. It does not give rise to a sacrococcygeal teratoma. (Choice E) The notochord forms the nucleus pulposus of the intervertebral discs.

A 7yo boy w/ Down syndrome underwent surgery to repair a congenital ventricular septal defect. During this procedure iatrogenic injury to the coronary artery supplying the conduction system of the heart, including the sinoatrial (SA) and atrioventricular (AV) nodes, occurred, resulting in a heart block. What artery was most likely damaged by the surgeon? A. anterior interventricular B. left marginal C. left coronary D. right coronary E. circumflex branch

D. right coronary

A medical resident is preparing to insert a dental venous line into the left brachiocephalic vein of her 10 year old patient. Which of the following is a correct relation of the left brachiocephalic vein? A. The vein lies posterior to the arch of the aorta B. The brachiocephalic artery lies superficial to the vein C. The vein crosses the anterior aspect of the manubrium of the sternum D. The vein passes deep to the thymus gland E. The apex of the left lung lies superficial to the vein

D. the vein passes deep to the thymus gland

Which of the following events is involved in cleavage of the zygote during week 1 of development? (A) A series of meiotic divisions forming blastomeres (B) Production of highly differentiated blastomeres (C) An increased cytoplasmic content of blastomeres (D) An increase in size of blastomeres (E) A decrease in size of blastomeres

E. Cleavage is a series of mitotic divisions whereby the large amount of zygote cytoplasm is successively partitioned among the newly formed blastomeres. Although the number of blastomeres increases during cleavage, the size of individual blastomeres decreases until they resemble adult cells in size.

An obstetrician has delivered a small-forgestational age baby boy who is born 4 weeks too early. His mother is a 20-year-old chronic smoker who smoked a pack of cigarettes every day during her pregnancy. What teratogen most likely contributed to the low birth rate and premature delivery? A. Heroin B. Warfarin C. Rubella D. Tetracycline E. Nicotine

E. Nicotine can cause premature delivery, low birth weight, and poor physical growth due to its constrictive effect on uterine blood vessels. It also causes conotruncal defects and urinary tract abnormalities. Tetracycline exposure can cause tooth and bone defects including yellow discoloration and hypoplasia. Heroin is a behavioral teratogen and presents with small birth weight, central nervous system dysfunction, and small head circumference. Congenital rubella syndrome presents with cataracts, cardiac defects, and deafness among other defects. 306

A 23-year-old man was brought to the ER barely conscious but in great discomfort after being wounded by an ice pick in the anterior chest. His doctor records the entry wound as the fi fth intercostal space, immediately left of the sternum. She also notes the veins of the face and neck are engorged with blood. The given chest X-ray showed massive enlargement of the patient's cardiac silhouette. Which of the following structures is most likely damaged due to the entry site of the ice pick? A. right atrium B. left atrium C. aortic arch D. left ventricle E. right ventricle

E. right ventricle

A young couple presents to your office for an infertility evaluation. They have been trying to conceive for one year. Initial evaluation of the female partner suggests normal ovulation occurring on the 14th day of the menstrual cycle. An oocyte isolated from the fallopian tube soon after ovulation would be in which of the following stages of gametogenesis? Select one: a. Prophase of meiosis I b. Metaphase of meiosis I c. Telophase of meiosis I d. Prophase of meiosis II e. Metaphase of meiosis II f. Telophase of meiosis II g. Prophase of mitosis

E. metaphase of meiosis ii. In the female, gametogenesis begins early in embryonic development (at approximately four weeks gestation) when the primordial germ cells migrate from the yolk sac region to the developing gonadal region. The primordial germ cells then differentiate into oogonia (46,2N), before beginning meiosis I. Now called primary oocytes (46,4N), these cells arrest in prophase of meiosis I and remain there until puberty. A woman's full complement of oocytes is developed by five months gestational age. Oocytes slowly degenerate from that time throughout life until they are completely depleted at menopause. At puberty, ovulatory cycles begin and the female becomes capable of reproduction. FSH stimulation during the ovarian cycle causes some oocytes to complete meiosis I, forming secondary oocytes and polar bodies (both 23, 2N). The secondary oocyte begins meiosis II (the polar body degenerates), but halts in metaphase II. At day 14 of the menstrual cycle, a secondary oocyte is released from the ovarian follicle in response to high estrogen concentrations and a paradoxical LH surge (estrogen typically causes feedback inhibition of LH and FSH release). The secondary oocyte remains frozen in metaphase II until fertilization occurs, at which point it divides into a mature oocyte and second polar body. (Choice G) After migrating to the gonadal region of the embryo, primordial germ cells undergo several rounds of mitosis before differentiating into oogonia. Mitosis does not play a role in female gametogenesis after this point.

14 A 19-year-old man arrives at his campus health clinic complaining of soreness in his right wrist. He explains he landed on an outstretched hand when he was tackled in a rugby match. He indicates that the pain worsens with movement and is minimized by stabilization of the wrist. There are no sensory defi cits in his hand nor does he have trouble grasping or holding objects. Pressure applied to the anatomic snuffbox between the extensor pollicis brevis and extensor pollicis longus tendons produces no pain. Radiographic studies show no fractures but reveal an anterior dislocation of a bone in the proximal row of carpal bones. What carpal bone is most likely dislocated in this patient? (A) Scaphoid (B) Lunate (C) Capitate (D) Triquetrum (E) Trapezium

The answer is B: Lunate. The lunate is shaped like a moon, thus its name. It is situated in the center of the proximal row of carpal bones where it articulates with the radius. This bone is the most commonly dislocated carpal bone, which leads to severe carpal instabilities. This dislocation often occurs in association with a trans-scaphoid fracture. It is important to note that scaphoid fractures are often diffi cult to see in radiographic imaging; however, these fractures can be detected frequently by applying direct pressure to the anatomical snuffbox. Choice A (Scaphoid) is incorrect. The scaphoid bone is located in the proximal row and is the most frequently fractured carpal bone. As the weight of the body is transmitted through the upper limb onto the outstretched hand and the impact of the fall exerts pressure back upon the limb, the scaphoid bone is crushed by these opposing forces and is subsequently fractured, usually along its narrowest part. The scaphoid bone is clinically relevant due to the frequency of fractures in younger patients and the poor vascularization of its proximal part. Avascular necrosis is often a postfracture complication that slows the healing of this bone. Choice C (Capitate) is incorrect. The capitate is located in the distal row of carpal bones, so this selection can be easily eliminated. Choice D (Triquetrum) is incorrect. The triquetrum is located in the proximal row of carpal bones and may be involved in a severe dislocation of the wrist. Specifi cally, the triquetrum can be displaced in Stage III wrist dislocations when the triquetrolunate interosseous ligament is damaged. However, tearing this interosseous intercarpal ligament would only occur following dislocation of the lunate bone, which is the most commonly islocated carpal bone. Choice E (Trapezium) is incorrect. The trapezium bone helps form the distal floor of the anatomic snuffbox and forces would be exerted on this bone during the fall. However, it is not located in the proximal row of carpal bones. Moreover, this four-sided bone is rarely fractured or dislocated due to its shape and construction.

An anteroposterior (AP) X-ray depicts a humeral shaft fracture in a 22-year-old man. Given the location of the fracture at the midshaft, which of the following structures is most likely damaged? (A) Posterior circumflex humeral artery (B) Ulnar nerve (C) Axillary nerve (D) Profunda brachii artery (E) Mediannerve

The answer is D: Profunda brachii artery. It is important to recognize where neurovascular structures have close positional relations to each other and to underlying bony structures in order to predict the likely second order functional consequences of damage to the bones. In the given AP X-ray, the midshaft of the humerus is fractured slightly distal to the radial groove. At this point, the profunda brachii vessels (deep vessels of the arm) and the radial nerve emerge from the radial groove in a bundle tightly wrapped against the body of the humerus. A fracture here may readily damage any of these neurovascular structures. Lesion of the vessels may produce Upper Limb and Mammary Gland 151 swelling in the posterior compartment of the arm and loss of supply to the muscles therein. Lesion of the nerve will result in major motor and sensory deficits in the posterior aspect of the forearm and hand. Choice A (Posterior circumflex humeral artery) is incorrect. This vessel travels in companionship with the axillary nerve around the surgical neck of the humerus. Fractures here are the most common injuries to the proximal end of the humerus, especially in the elderly. Choice B (Ulnar nerve) is incorrect. In the arm, the ulnar nerve travels with the superior ulnar collateral artery. Both lie in contact with the posterior side of the large, projecting medial epicondyle of the humerus as they cross the elbow. Choice C (Axillary nerve) is incorrect. The axillary nerve travels in companionship with the posterior circumflex humeral artery around the surgical neck of the humerus. Fractures here are the most common injuries to the proximal end of the humerus, especially in the elderly. Choice E (Median nerve) is incorrect. This large nerve travels with the brachial vessels down the anterior midline of the arm. The nerve lies close to the distal end of the humerus, where it may be damaged due to fractures of the condyle.

57. Which structure in this radiograph may be fractured, resulting in loss of the chief flexor of the thigh?

57. The Answer is D. The iliopsoas muscle is the chief flexor of the thigh and inserts on the lesser trochanter.

A 21-year-old female basketball player lands on her opponent's foot after jumping to rebound the basketball. Her foot is forcefully inverted, and when leaving the court, she tells her trainer that she twisted or sprained her ankle. After getting her ankle taped for support, she reenters the game. What ligament was most likely damaged? (A) Calcaneofibular ligament (B) Anterior talofibular ligament (C) Posterior talofibular ligament (D) Plantar calcaneonavicular ligament (E) Medial (deltoid) ligament

(B) Anterior talofibular ligament

Following surgery to repair a broken right tibia, a 22-year-old patient is placed in a short leg cast. Several hours later, she complains of extreme pain, numbness with a "pins and needles sensation," infl ammation, and abnormal pressure on the anterior and lateral aspects of the affected lower leg. The cast is removed, and the physician notes weakness in dorsifl exion of the foot and toes, a weak dorsalis pedis arterial pulse, and sensory loss between the first and second toes. What nerve is most likely damaged? (A) Tibial nerve (B) Deep fibular nerve (C) Superficial fi bular nerve (D) Medial plantar nerve (E) Lateral plantar nerve

(B) Deep fibular nerve

A patient presents with extreme pain due to arterial insufficiency in the posterior femoral compartment. This compartment of the thigh receives its blood supply mainly from the perforating arteries. An arteriogram confi rms partial occlusion of the artery that gives rise to these perforating arteries. What artery is occluded in the arteriogram? (A) Femoral artery (B) Profunda femoris artery (C) Obturator artery (D) Popliteal artery (E) Medial femoral circumfl ex artery

(B) Profunda femoris artery

A 22-year-old soldier is injured from shrapnel from an improvised explosive device in the right upper thigh, below the midpoint of the inguinal ligament. Though he received fi eld dressings from a medic, he arrives at the military hospital having lost copious amounts of blood. What sign and/or symptom would accompany this patient's presentation? (A) Increased pulse in the right dorsalis pedis artery (B) Tachycardia (C) Warm right foot (D) Increased hematocrit (E) Hypertension

(B) Tachycardia

A 65-year-old man with a history of heavy smoking visits his physician complaining of intermittent pain in his feet, accompanied by pallor and coldness of his feet. The physician suspects vascular insuffi ciency and takes a pulse of the dorsalis pedis artery. That pulse is best palpated at which of the following locations? (A) Immediately anterior to the medial malleolus (B) Immediately anterior to the lateral malleolus (C) Between the tendons of the extensor hallucis longus and extensor digitorum longus muscles (D) Between the tendons of the extensor digitorum longus and fibularis (peroneus) tertius muscles (E) Immediately lateral to the tendon of the fibularis (peroneus) tertius muscle

(C) Between the tendons of the extensor hallucis longus and extensor digitorum longus muscles

A 25-year-old woman is brought to the emergency room in severe pain due to an ankle injury. She tells the attending physician she was wearing high-heel shoes, stepped off the curb into the street, lost her balance and landed awkwardly, causing her foot to turn extremely outward (eversion). Which of the following ligaments is most likely damaged? (A) Plantar calcaneonavicular (spring) ligament (B) Calcaneofi bular ligament (C) Deltoid ligament (D) Anterior talofi bular ligament (E) Plantar calcaneocuboid (long plantar) ligament

(C) Deltoid ligament

A 72-year-old woman slips and falls on a wet floor, fracturing the neck of her right femur. Subsequent physical examination in the ER shows her right foot is laterally rotated, and the right lower extremity appears slightly shorter than the left. Which of the following muscles is mainly responsible for the rotated posture of the right limb? (A) Semitendinosus (B) Adductor longus (C) Gluteus maximus (D) Rectus femoris (E) Gracilis

(C) Gluteus maximus

A 23-year-old female medical student notices she was gaining weight due to a sedimentary lifestyle and compulsive studying. Therefore, she decides to run in a marathon and starts her training by running 6 miles each morning before class. After 2 weeks, she presents with right lateral knee pain and infl ammation, specifi cally in the area of the lateral femoral epicondyle. This pain intensifi es throughout her morning jogs, especially when her right foot strikes the ground or when she is running downhill. What is most likely diagnosis in this patient? (A) Sprained fibular (lateral) collateral ligament (FCL) (B) Patellofemoral pain syndrome (C) Iliotibial band syndrome (ITBS) (D) Torn lateral meniscus (E) Pes anserinus bursitis

(C) Iliotibial band syndrome (ITBS)

When standing upright, the femur moves into the locked position by slightly hyperextending, gliding posteriorly, and medially rotating on the tibial plateaus. Which of the following muscles acts to initiate the unlocking of the knee? (A) Biceps femoris (B) Gastrocnemius (C) Popliteus (D) Sartorius (E) Plantaris

(C) Popliteus

A man working in a junkyard trips and falls into a pile of scrap metal, suffering a deep cut immediately posterior to the lateral malleolus. Which of the following is most likely to be injured? (A) Saphenous nerve (B) Tendon of the fibularis (peroneus) tertius (C) Tendon of the fibularis (peroneus) longus (D) Posterior tibial artery (E) Great saphenous vein

(C) Tendon of the fibularis (peroneus) longus

Human feet are everted so that their soles lie fully on the ground during ambulation. What muscle is developmentally unique to humans, inserts into the base of the fi fth metatarsal, and assists in eversion (or pronation)? (A) Extensor digitorum longus (B) Extensor digitorum brevis (C) Fibularis longus (D) Fibularis tertius (E) Flexor digiti minimi brevis

(D) Fibularis tertius

A rising second-year medical student spends his summer hiking the Appalachian Trial with a 100-lb backpack in tow. After a month of hiking 30-mi/day (48-km/day), he notices numbness, tingling, and burning sensations in the lateral aspect of his right upper thigh. His symptoms are exacerbated by his backpack pressing on the area surrounding his right anterior superior iliac spine. What nerve is most likely affected? (A) Femoral branch of genitofemoral nerve (B) Saphenous nerve (C) Anterior cutaneous branches of femoral nerve (D) Lateral femoral cutaneous nerve (E) Iliohypogastric nerve

(D) Lateral femoral cutaneous nerve

A 36-year-old man arrives at the ER unconscious and with a suspected spinal cord injury after being involved in a motor vehicle accident. If the physician wants to localize the spinal cord lesion, what reflex would test the integrity of the L1-2 spinal cord segment? (A) Anocutaneous reflex (B) Patellar reflex (C) Plantar reflex (D) Calcaneal reflex (E) Cremasteric reflex

(E) Cremasteric reflex

A 25-year-old man suffers a gunshot wound to the calf that severs the posterior tibial artery at its origin. Which of the following vessels will not receive blood flow immediately following the injury? (A) Anterior tibial artery (B) Inferior medial genicular artery (C) Dorsalis pedis artery (D) Popliteal artery (E) Fibular (peroneal) artery

(E) Fibular (peroneal) artery

A 55-year-old woman recently had pelvic surgery during which cancerous lymph nodes were removed from the lateral wall of her pelvis. During a postoperative examination, she says she has been having painful muscle spasms in her thigh. Which of the following muscles is most likely involved? (A) Sartorius (B) Biceps femoris (C) Tensor muscle of fascia lata (D) Vastus medialis (E) Gracilis

(E) Gracilis

A 70-year-old man reports an inability to climb stairs and stand up from a sitting position. Further examination also shows weakness when laterally rotating the thigh against resistance. What nerve is most likely compromised in this patient? (A) Femoral nerve (B) Obturator nerve (C) Sciatic nerve (D) Superior gluteal nerve (E) Inferior gluteal nerve

(E) Inferior gluteal nerve

11. A 21-year-old man was involved in a motorcycle accident, resulting in destruction of the groove in the lower surface of the cuboid bone. Which of the following muscle tendons is most likely damaged? (A) Flexor hallucis longus (B) Peroneus brevis (C) Peroneus longus (D) Tibialis anterior (E) Tibialis posterior

11. The Answer is C. The groove in the lower surface of the cuboid bone is occupied by the tendon of the peroneus longus muscle. The flexor hallucis longus tendon occupies a groove on the posterior surface of the body of the talus and a groove on the inferior surface of the calcaneus during its course. The tibialis posterior muscle tendon occupies the medial malleolar groove of the tibia. Other muscle tendons are not in the groove of the tarsal bones.

11. The bronchogram of a 45-year-old female smoker shows the presence of a tumor in the eparterial bronchus. Which airway is most likely blocked? (A) Left superior bronchus (B) Left inferior bronchus (C) Right superior bronchus (D) Right middle bronchus (E) Right inferior bronchus

11. The answer is C. The eparterial bronchus is the right superior lobar (secondary) bronchus; all of the other bronchi are hyparterial bronchi.

11. A 16-year-old girl with urinary diseases comes to a local hospital. Her urologist's examination and laboratory test results reveal that she has difficulty in removing wastes from the blood and in producing urine. Which of the following organs may have abnormal functions? (A) Ureter (B) Spleen (C) Urethra (D) Bladder (E) Kidney

11. The answer is E. The urinary system includes the kidneys, which remove wastes from the blood and produce the urine; the ureters, which carry urine; the urinary bladder, which stores urine; and the urethra, which conveys urine from the bladder to the exterior of the body. The spleen filters blood to remove particulate matter and cellular residue, stores red blood cells, and produces lymphocytes. Because the patient is not producing urine properly, the malfunctioning organs are the kidneys.

12. An elderly man with a benign enlargement of his prostate experiences difficulty in urination, urinary frequency, and urgency. Which of the following lobes of the prostate gland is commonly involved in benign hypertrophy that obstructs the prostatic urethra? (A) Anterior lobe (B) Middle lobe (C) Right lateral lobe (D) Left lateral lobe (E) Posterior lobe

12. The answer is B. The middle lobe of the prostate gland is commonly involved in benign prostatic hypertrophy, resulting in obstruction of the prostatic urethra, whereas the posterior lobe is commonly involved in carcinomatous transformation. The anterior lobe contains little glandular tissue, and the two lateral lobes on either side of the urethra form the major part of the gland.

13. A 26-year-old woman has an amenorrhea, followed by uterine bleeding, pelvic pain, and pelvic mass. Her obstetrician performed a thorough examination, and the patient was diagnosed as having an ectopic pregnancy. Which of the following organs is most likely to provide a normal site of fertilization? (A) Fundus of the uterus (B) Ampulla of the uterine tube (C) Fimbriae (D) Infundibulum of the uterine tube (E) Body of the uterus

13. The answer is B. Fertilization occurs in the ampulla of the uterine tube, and a fertilized oocyte forms a blastocyst by day 7 after fertilization and becomes embedded or implanted in the wall of the uterus during the progestational (secretory) phase of the menstrual cycle. Fertilization is the process beginning with the penetration of the secondary oocyte by the sperm and completed by fusion of the male and female pronuclei.

13. A 44-year-old man with a stab wound was brought to the emergency department, and a physician found that the patient was suffering from a laceration of his right phrenic nerve. Which of the following conditions has likely occurred? (A) Injury to only GSE fibers (B) Difficulty in expiration (C) Loss of sensation in the fibrous pericardium and mediastinal pleura (D) Normal function of the diaphragm (E) Loss of sensation in the costal part of the diaphragm

13. The answer is C. The phrenic nerve supplies the pericardium and mediastinal and diaphragmatic (central part) pleura and the diaphragm, an important muscle of inspiration. It contains general somatic efferent (GSE), general somatic afferent (GSA), and GVE (postganglionic sympathetic) fibers. The costal part of the diaphragm receives GSA fibers from the intercostal nerves.

13. A 59-year-old man is diagnosed with prostate cancer following a digital rectal examination. For the resection of prostate cancer, it is important to know that the prostatic ducts open into or on which of the following structures: (A) Membranous part of the urethra (B) Seminal colliculus (C) Spongy urethra (D) Prostatic sinus (E) Prostatic utricle

13. The answer is D. Ducts from the prostate gland open into the prostatic sinus, which is a groove on either side of the urethral crest. The prostate gland receives the ejaculatory duct, which opens into the prostatic urethra on the seminal colliculus (a prominent elevation of the urethral crest) just lateral to the prostatic utricle, which is a small blind pouch. The bulbourethral gland lies on the lateral side of the membranous urethra within the deep perineal space, but its duct opens into the bulbous portion of the spongy (penile) urethra.

14. A 29-year-old woman with a ruptured ectopic pregnancy is admitted to a hospital for culdocentesis. A long needle on the syringe is most efficiently inserted through which of the following structures? (A) Anterior fornix of the vagina (B) Posterior fornix of the vagina (C) Anterior wall of the rectum (D) Posterior wall of the uterine body (E) Posterior wall of the bladder

14. The answer is B. A needle should be inserted through the posterior fornix just below the posterior lip of the cervix while the patient is in the supine position to aspirate abnormal fluid in the cul-de-sac of Douglas (rectouterine pouch). Rectouterine excavation is not most efficiently aspirated by puncture of other structures.

14. A 29-year-old woman with abdominal pain was admitted to a local hospital, and examination shows that a retroperitoneal infection is affecting a purely endocrine gland. Which of the following structures is infected? (A) Ovary (B) Suprarenal gland (C) Pancreas (D) Liver (E) Stomach

14. The answer is B. The suprarenal gland is a retroperitoneal organ and is a purely endocrine gland. The pancreas is a retroperitoneal organ and contains endocrine cells, but it is not a purely endocrine gland. The liver and stomach contain endocrine cells, but they are not purely endocrine glands and also are surrounded by peritoneum. The ovary contains endocrine cells and is located in the pelvic cavity.

15. A patient experiences weakness in dorsiflexing and inverting the foot. Which of the following muscles is damaged? (A) Peroneus longus (B) Peroneus brevis (C) Tibialis anterior (D) Extensor digitorum longus (E) Peroneus tertius

15. The Answer is C. The tibialis anterior can dorsiflex and invert the foot. The peroneus lon- gus and brevis muscles can plantar flex and evert the foot, the peroneus tertius can dorsiflex and evert the foot, and the extensor digitorum longus can dorsiflex the foot and extend the toes.

15. A 37-year-old patient with severe chest pain, shortness of breath, and congestive heart failure was admitted to a local hospital. His coronary angiograms reveal a thrombosis in the circumflex branch of the left coronary artery. Which of the following conditions could result from the blockage of blood flow in the circumflex branch? (A) Tricuspid valve insufficiency (B) Mitral valve insufficiency (C) Ischemia of AV node (D) Paralysis of pectinate muscle (E) Necrosis of septomarginal trabecula

15. The answer is B. The circumflex branch of the left coronary artery supplies the left ventricle, and thus its blockage of blood flow results in necrosis of myocardium in the left ventricle, producing mitral valve insufficiency. The tricuspid valve, AV node, pectinate muscles, and septomarginal trabecula are present in the right atrium and ventricle.

15. A 37-year-old man is suffering from carcinoma of the skin of the penis. Cancer cells are likely to metastasize directly to which of the following lymph nodes? (A) External iliac nodes (B) Internal iliac nodes (C) Superficial inguinal nodes (D) Aortic (lumbar) nodes (E) Deep inguinal nodes

15. The answer is C. The superficial inguinal nodes receive lymph from the penis, scrotum, buttocks, labium majus, and the lower parts of the vagina and anal canal. These nodes have efferent vessels that drain primarily into the external iliac and common iliac nodes and ultimately to the lumbar (aortic) nodes. The internal iliac nodes receive lymph from the upper part of the rectum, vagina, uterus, and other pelvic organs, and they drain into the common iliac nodes and then into the lumbar (aortic) nodes. Lymph vessels from the glans penis drain initially into the deep inguinal nodes and then into the external iliac nodes.

16. Rupture of the ligamentum teres capitis femoris may lead to damage to a branch of which of the following arteries? (A) Medial circumflex femoral (B) Lateral circumflex femoral (C) Obturator (D) Superior gluteal (E) Inferior gluteal

16. The Answer is C. The obturator artery gives rise to an acetabular branch that runs in the round ligament of the head of the femur.

16. A 42-year-old woman who has had six children develops a weakness of the urogenital diaphragm. Paralysis of which of the following muscles would cause such a symptom? (A) Sphincter urethrae (B) Coccygeus (C) Superficial transversus perinei (D) Levator ani (E) Obturator internus

16. The answer is A. The urogenital diaphragm consists of the sphincter urethrae and deep transverse perineal muscles. Weakness of the muscles, ligaments, and fasciae of the pelvic floor, such as the pelvic diaphragm, urogenital diaphragm, and cardinal (transverse cervical) ligaments, occurs as a result of multiple child delivery, advancing age, and menopause. The pelvic diaphragm is composed of the levator ani and coccygeus muscles. The superficial transversus perinei is one of the superficial perineal muscles, and the obturator internus forms the lateral wall of the ischiorectal fossa.

16. A 75-year-old patient has been suffering from lung cancer located near the cardiac notch, a deep indentation on the lung. Which of the following lobes is most likely to be excised? (A) Superior lobe of the right lung (B) Middle lobe of the right lung (C) Inferior lobe of the right lung (D) Superior lobe of the left lung (E) Inferior lobe of the left lung

16. The answer is D. The cardiac notch is a deep indentation of the anterior border of the superior lobe of the left lung. Therefore, the right lung is not involved.

17. Fracture of the neck of the femur results in avascular necrosis of the femoral head, probably resulting from lack of blood supply from which of the following arteries? (A) Obturator (B) Superior gluteal (C) Inferior gluteal (D) Medial femoral circumflex (E) Lateral femoral circumflex

17. The Answer is D. In adults, the chief arterial supply to the head of the femur is from the branches of the medial femoral circumflex artery. The lateral femoral circumflex artery may supply the femoral head by anastomosing with the medial femoral circumflex artery. The posterior branch of the obturator artery gives rise to the artery of the head of the femur, which runs in the round ligament of the femoral head and is usually insufficient to supply the head of the femur in adults but is an important source of blood to the femoral head in children. The superior and inferior gluteal arteries do not supply the head of the femur.

17. A 43-year-old man has a benign tumor located near a gap between the arcuate pubic ligament and the transverse perineal ligament. Which of the following structures is most likely compressed by this tumor? (A) Perineal nerve (B) Deep dorsal vein of the penis (C) Superficial dorsal vein (D) Posterior scrotal nerve (E) Deep artery of the penis

17. The answer is B. The deep dorsal vein, dorsal artery, and dorsal nerve of the penis pass through a gap between the arcuate pubic ligament and the transverse perineal ligament. The perineal nerve divides into a deep branch, which supplies all of the perineal muscles, and superficial branches as posterior scrotal nerves, which supply the scrotum. The superficial dorsal vein of the penis empties into the greater saphenous vein. The deep artery of the penis runs in the corpus cavernosum of the penis.

17. A thoracentesis is performed to aspirate an abnormal accumulation of fluid in a 37-yearold patient with pleural effusion. A needle should be inserted at the midaxillary line between which of the following two ribs so as to avoid puncturing the lung? (A) Ribs 1 and 3 (B) Ribs 3 and 5 (C) Ribs 5 and 7 (D) Ribs 7 and 9 (E) Ribs 9 and 11

17. The answer is D. A thoracentesis is performed for aspiration of fluid in the pleural cavity at or posterior to the midaxillary line, one or two intercostal spaces below the fluid level but not below the ninth intercostal space and, therefore, between ribs 7 and 9. Other intercostals spaces are not preferred.

18. If the acetabulum is fractured at its posterosuperior margin by dislocation of the hip joint, which of the following bones could be involved? (A) Pubis (B) Ischium (C) Ilium (D) Sacrum (E) Head of the femur

18. The Answer is C. The acetabulum is a cup-shaped cavity on the lateral side of the hip bone and is formed superiorly by the ilium, posteroinferiorly by the ischium, and anteromedially by the pubis. The sacrum and the head of the femur do not participate in the formation of the acetabulum.

18. A newborn baby is readmitted to the hospital with hypoxia and upon testing is found to have pulmonary stenosis, dextraposition of the aorta, interventricular septal defect, and hypertrophy of the right ventricle. Which of the following is best described by these symptoms? (A) ASD (B) Patent ductus arteriosus (C) Tetralogy of Fallot (D) Aortic stenosis (E) Coarctation of the aorta

18. The answer is C. Tetralogy of Fallot is a combination of congenital cardiac defects consisting of (a) pulmonary stenosis, (b) dextraposition of the aorta (so that it overrides the ventricular septum and receives blood from the right ventricle), (c) ventricular septal defect (VSD), and (d) right ventricular hypertrophy. ASD is a congenital defect in the atrial septum, resulting from a patent foramen ovale. Patent ductus arteriosus shunts blood from the pulmonary trunk to the aorta, bypassing the lungs. Aortic stenosis is an abnormal narrowing of the aortic valve orifice, impeding the blood flow. Coarctation of the aorta is a congenital constriction of the aorta, commonly occurs just distal to the left subclavian artery, causing upper limb hypertension and diminished blood flow to the lower limbs and abdominal viscera.

19. A 22-year-old man has a gonorrheal infection that has infiltrated the space between the inferior fascia of the urogenital diaphragm and the superficial perineal fascia. Which of the following structures might be inflamed? (A) Bulb of the penis (B) Bulbourethral gland (C) Membranous part of the male urethra (D) Deep transverse perineal muscle (E) Sphincter urethrae

19. The answer is A. The bulb of the penis is located in the superficial perineal space between the inferior fascia of the urogenital diaphragm and the membranous layer of the superficial perineal fascia (Colles fascia). All of the other structures are found in the deep perineal pouch.

19. A 33-year-old patient is suffering from a sudden occlusion at the origin of the descending (thoracic) aorta. This condition would most likely decrease blood flow in which of the following intercostal arteries? (A) Upper six anterior (B) All of the posterior (C) Upper two posterior (D) Lower anterior (E) Lower six posterior

19. The answer is E. The first two posterior intercostal arteries are branches of the highest (superior) intercostal artery of the costocervical trunk; the remaining nine branches are from the thoracic aorta. The internal thoracic artery gives off the upper six anterior intercostal arteries and is divided into the superior epigastric and musculophrenic arteries, which gives off anterior intercostal arteries in the 7th, 8th, and 9th intercostal spaces and ends in the 10th intercostal space where it anastomoses with the deep circumflex iliac artery.

26. A 24-year-old woman complains of weakness when she extends her thigh and rotates it laterally. Which of the following muscles is paralyzed? (A) Obturator externus (B) Sartorius (C) Tensor fasciae latae (D) Gluteus maximus (E) Semitendinosus

26. The Answer is D. The gluteus maximus can extend and rotate the thigh laterally. The obturator externus rotates the thigh laterally. The sartorius can flex both the hip and knee joints. The tensor fasciae latae can flex and medially rotate the thigh. The semitendinosus can extend the thigh and medially rotate the leg.

7. A 27-year-old woman involved in a car accident is brought into the emergency department. Her magnetic resonance imag- ing reveals that she has a laceration of the spinal cord at the L4 spinal cord level. Which of the following structures would you expect to be intact? (A) Dorsal horn (B) Lateral horn (C) Ventral horn (D) Gray matter (E) White matter

7. The answer is B. The lateral horns are found in the gray matter of the spinal cord between T1 and L2 and also between S2 and S4. Therefore, the lateral horns are absent at the L4 spinal cord level.

2. On the basis of the examination at her doctor's office, a patient is told that her parasympathetic nerves are damaged. Which of the following muscles would most likely be affected? (A) Muscles in the hair follicles (B) Muscles in blood vessels (C) Muscles that act at the elbow joint (D) Muscles in the gastrointestinal (GI) tract (E) Muscles enclosed by epimysium

2. The answer is D. Smooth muscles in the gastrointestinal tract are innervated by both par- asympathetic and sympathetic nerves. Smooth muscles in the wall of the blood vessels and arrector pili muscles in hair follicles are innervated only by sympathetic nerves. Mus- cles that act at the elbow joint and muscles enclosed by epimysium are skeletal muscles that are innervated by somatic motor (general somatic efferent [GSE]) nerves.

25. A 21-year-old patient with a stab wound reveals a laceration of the right vagus nerve proximal to the origin of the recurrent laryngeal nerve. Which of the following conditions would most likely result from this lesion? (A) Contraction of bronchial muscle (B) Stimulation of bronchial gland secretion (C) Dilation of the bronchial lumen (D) Decrease in cardiac rate (E) Constriction of coronary artery

25. The answer is C. The parasympathetic nerve fibers in the vagus nerve constrict the bronchial lumen, contract bronchial smooth muscle, stimulate bronchial gland secretion, decrease heart rate, and constrict the coronary artery. The vagus nerve also carries afferent fibers of pain, cough reflex, and stretch of the lung (during inspiration).

25. A sexually active adolescent presents with an infection within the ischiorectal fossa. Which of the following structures is most likely injured? (A) Vestibular bulb (B) Seminal vesicle (C) Greater vestibular gland (D) Inferior rectal nerve (E) Internal pudendal artery

25. The answer is D. The ischiorectal fossa contains the inferior rectal nerves and vessels and adipose tissue. The bulb of the vestibule and the great vestibular gland are located in the superficial perineal space, whereas the bulbourethral gland is found in the deep perineal space. The internal pudendal artery runs in the pudendal canal, but its branches pass through the superficial and deep perineal spaces.

21. A 75-year-old woman was admitted to a local hospital, and bronchograms and radiographs revealed a lung carcinoma in her left lung. Which of the following structures or characteristics does the cancerous lung contain? (A) Horizontal fissure (B) Groove for superior vena cava (SVC) (C) Middle lobe (D) Lingula (E) Larger capacity than the right

21. The answer is D. The lingula is the tongue-shaped portion of the upper lobe of the left lung. The right lung has a groove for the horizontal fissure, superior vena cava (SVC), and middle lobe and has a larger capacity than the left lung.

22. A patient experiences paralysis of the muscle that originates from the femur and contributes directly to the stability of the knee joint. Which of the following muscles is involved? (A) Vastus lateralis (B) Semimembranosus (C) Sartorius (D) Biceps femoris (long head) (E) Rectus femoris

22. The Answer is A. The vastus lateralis muscles arise from the femur and all the other muscles originate from the hip (coxal) bone. The biceps femoris inserts on the fibula, and other muscles insert on the tibia; thus, all of them contribute to the stability of the knee joint.

22. After his bath but before getting dressed, a 4-year-old boy was playing with his puppy. The boy's penis was bitten by the puppy, and the deep dorsal vein was injured. The damaged vein (A) Lies superficial to Buck fascia (B) Drains into the prostatic venous plexus (C) Lies lateral to the dorsal artery of the penis (D) Is found in the corpus spongiosum (E) Is dilated during erection

22. The answer is B. The deep dorsal vein of the penis lies medial to the dorsal artery of the penis on the dorsum of the penis and deep to Buck fascia, drains into the prostatic plexus of veins, and is compressed against the underlying deep fascia of the penis during erection.

23. A patient is involved in a motorcycle wreck that results in avulsion of the skin over the anterolateral leg and ankle. Which of the following structures is most likely destroyed with this type of injury? (A) Deep peroneal nerve (B) Extensor digitorum longus muscle tendon (C) Dorsalis pedis artery (D) Great saphenous vein (E) Superficialperonealnerve

23. The Answer is E. The superficial peroneal nerve emerges between the peroneus longus and peroneus brevis muscles and descends superficial to the extensor retinaculum of the ankle on the anterolateral side of the leg and ankle, innervating the skin of the lower leg and foot. The great saphenous vein begins at the medial end of the dorsal venous arch of the foot and ascends in front of the medial malleolus and along the medial side of the tibia along with the saphenous nerve. Other structures pass deep to the extensor retinaculum.

24. A 23-year-old massage therapist who specializes in women's health attends a lecture at an annual conference on techniques of massage. She asks, "What structure is drained by the lumbar (aortic) lymph nodes?" Which of the following structures is the correct answer to this question? (A) Perineum (B) Lower part of the vagina (C) External genitalia (D) Ovary (E) Lower part of the anterior abdominal wall

24. The answer is D. The lymphatic vessels from the ovary ascend with the ovarian vessels in the suspensory ligament and terminate in the lumbar (aortic) nodes. Lymphatic vessels from the perineum, external genitalia, and lower part of the anterior abdominal wall drain into the superficial inguinal nodes.

25. A 10-year-old boy falls from a tree house. The resultant heavy compression of the sole of his foot against the ground caused a fracture of the head of the talus. Which of the following structures is unable to function normally? (A) Transverse arch (B) Medial longitudinal arch (C) Lateral longitudinal arch (D) Tendon of the peroneus longus (E) Long plantar ligament

25. The Answer is B. The keystone of the medial longitudinal arch of the foot is the head of the talus, which is located at the summit between the sustentaculum tali and the navicular bone. The medial longitudinal arch is supported by the spring ligament and the tendon of the flexor hallucis longus muscle. The cuboid bone serves as the keystone of the lateral longitudinal arch, which is supported by the peroneus longus tendon and the long and short plantar ligaments. The transverse arch is formed by the navicular, three cuneiform, the cuboid, and five metatarsal bones and is supported by the peroneus longus tendon and the transverse head of the adductor hallucis.

26. A neonate appears severely cyanotic and breathing rapidly. Cardiac echocardiogram reveals that the aorta lies to the right of the pulmonary trunk. Which of the following is most likely occurred during development? (A) AP septum failed to develop in a spiral fashion (B) Excessive resorption of septum primum (C) Pulmonary valve atresia (D) Persistent truncus arteriosus (E) Coarctation of the aorta

26. The answer is A. Failure of the aorticopulmonary septum results in transposition of the great vessels, exhibiting that the aorta is to the right of the pulmonary trunk. Cyanosis is common in transposition of the great vessels. Excessive resorption of septum primum results in a secundum type of ASD. Pulmonary valve atresia may result in cyanosis, but it will not cause the aorta to be to the right of the pulmonary trunk. A persistent truncus arteriosus is caused by lack of development of the aorticopulmonary septum resulting in a single outflow track. Coarctation of the aorta is a severe narrowing of the aorta.

26. A first-year resident in the urology department reviews pelvic anatomy before seeing patients. Which of the following statements is correct? (A) The dorsal artery of the penis supplies the glans penis. (B) The seminal vesicles store spermatozoa. (C) The duct of the bulbourethral gland opens into the membranous urethra. (D) The duct of the greater vestibular gland opens into the vagina. (E) The anterior lobe of the prostate gland is prone to carcinomatous transformation.

26. The answer is A. The dorsal artery of the penis supplies the glans penis. The seminal vesicles store no spermatozoa. The duct of the bulbourethral gland opens into the bulbous portion of the spongy urethra, whereas the greater vestibular gland opens into the vestibule between the labium minora and the hymen. The anterior lobe of the prostate is devoid of glandular substance, the middle lobe is prone to benign hypertrophy, and the posterior lobe is prone to carcinomatous transformation.

27. A 12-year-old boy was admitted to a local hospital with a known history of heart problems. His left ventricular hypertrophy could result from which of the following conditions? (A) A constricted pulmonary trunk (B) An abnormally small left AV opening (C) Improper closing of the pulmonary valves (D) An abnormally large right AV opening (E) Stenosis of the aorta

27. The answer is E. Stenosis of the aorta can cause left ventricular hypertrophy. Right ventricular hypertrophy may occur as a result of pulmonary stenosis, pulmonary and tricuspid valve defects, or mitral valve stenosis.

29. A 62-year-old woman who is a heavy smoker has an advanced lung cancer that spread into her right third posterior intercostal space posterior to the midaxillary line. If cancer cells are carried in the venous drainage, they would travel first to which of the following veins? (A) SVC (B) Right superior intercostal vein (C) Right brachiocephalic vein (D) Azygos vein (E) Hemiazygos vein

29. The answer is B. The superior intercostal vein is formed by the union of the second, third, and fourth posterior intercostal veins and drains into the azygos vein on the right and the brachiocephalic vein on the left. The azygos vein drains into the SVC. The hemiazygos vein usually drains into the azygos vein.

30. A 53-year-old bank teller is admitted to a local hospital for surgical removal of a benign pelvic tumor confined within the broad ligament. There is a risk of injuring which of the following structures that lies in this ligament? (A) Ovary (B) Proximal part of the pelvic ureter (C) Terminal part of the round ligament of the uterus (D) Uterine tube (E) Suspensory ligament of the ovary

30. The answer is D. The uterine tubes lie in the broad ligament. The anterior surface of the ovary is attached to the posterior surface of the broad ligament of the uterus. The ureter descends retroperitoneally on the lateral pelvic wall but is crossed by the uterine artery in the base (in the inferomedial part) of the broad ligament. The terminal part of the round ligament of the uterus becomes lost in the subcutaneous tissue of the labium majus. The suspensory ligament of the ovary is a band of peritoneum that extends superiorly from the ovary to the pelvic wall.

32. A cardiologist is on clinical rounds with her medical students. She asks them, "During the cardiac cycle, which of the following events occurs?" (A) AV valves close during diastole (B) Aortic valve closes during systole (C) Pulmonary valve opens during diastole (D) Blood flow in coronary arteries is maximal during diastole (E) Aortic

32. The answer is D. During diastole, the AV valves open, and the aortic and pulmonary valves close; whereas during systole, the AV valves close, and the aortic and pulmonary valves open.

33. If the lateral (fibular) collateral ligament is torn by a fracture of the head and neck of the fibula, which of the following conditions may occur? (A) Abnormal passive abduction of the extended leg (B) Abnormal passive adduction of the extended leg (C) Anterior displacement of the femur on the tibia (D) Posterior displacement of the femur on the tibia (E) Maximal flexion of the leg

33. The Answer is B. The lateral (fibular) collateral ligament prevents adduction at the knee. Therefore, a torn lateral collateral ligament can be recognized by abnormal passive adduction of the extended leg. Abnormal passive abduction of the extended leg may occur when the medial (tibial) collateral ligament is torn. The anterior cruciate ligament prevents posterior displacement of the femur on the tibia; the posterior cruciate ligament prevents anterior displacement of the femur on the tibia. In addition, the posterior cruciate ligament is taut when the knee is fully flexed.

33. Coronary angiographs of a 44-year-old male patient reveal an occlusion of the circumflex branch of the left coronary artery. This patient has been suffering from myocardial infarction in which of the following areas? (A) Right and left ventricles (B) Right and left atria (C) Interventricular septum (D) Apex of the heart (E) Left atrium and ventricle

33. The answer is E. The left atrium and ventricle receive blood from the circumflex branch of the left coronary artery. The interventricular septum and the apex of the heart are supplied by the anterior interventricular branch of the left coronary artery. The right ventricle receives blood from the anterior interventricular artery and the marginal branch of the right coronary artery. The right atrium receives blood from the right coronary artery.

34. Which of the following arteries could be damaged by a fracture of the head and neck of the fibula? (A) Popliteal (B) Posterior tibial (C) Anterior tibial (D) Peroneal (E) Lateral inferior genicular

34. The Answer is C. The anterior tibial artery, which arises from the popliteal artery, enters the anterior compartment by passing through the gap between the fibula and tibia at the upper end of the interosseous membrane. The other arteries would not be affected because they are not closely associated with the head and neck of the fibula.

34. A patient has a small but solid tumor in the mediastinum, which is confined at the level of the sternal angle. Which of the following structures would most likely be found at this level? (A) Bifurcation of the trachea (B) Beginning of the ascending aorta (C) Middle of the aortic arch (D) Articulation of the third rib with the sternum (E) Superior border of the superior mediastinum

34. The answer is A. The sternal angle is the junction of the manubrium and the body of the sternum. It is located at the level where the second rib articulates with the sternum, the trachea bifurcates into the right and left bronchi, and the aortic arch begins and ends. It marks the end of the ascending aorta and the beginning of the descending aorta, and it forms the inferior border of the superior mediastinum.

34. A 32-year-old patient with multiple fractures of the pelvis has no cutaneous sensation in the urogenital triangle. The function of which of the following nerves is most likely to be spared? (A) Ilioinguinal nerve (B) Iliohypogastric nerve (C) Posterior cutaneous nerve of the thigh (D) Pudendal nerve (E) Genitofemoral nerve

34. The answer is B. The iliohypogastric nerve innervates the skin above the pubis. The skin of the urogenital triangle is innervated by the pudendal nerve, perineal branches of the posterior femoral cutaneous nerve, anterior scrotal or labial branches of the ilioinguinal nerve, and the genital branch of the genitofemoral nerve.

35. Which of the following conditions would occur from a fracture of the head and neck of the fibula? (A) Ischemia in the gastrocnemius (B) Loss of plantar flexion (C) Trendelenburg's sign (D) Anterior tibial compartment syndrome (E) Flat foot

35. The Answer is D. Anterior tibial compartment syndrome is characterized by ischemic necrosis of the muscles of the anterior tibial compartment of the leg resulting from dam- age to the anterior tibial artery. The gastrocnemius receives blood from sural branches of the popliteal artery. Loss of plantar flexion is due to necrosis of the posterior muscles of the leg, which are supplied by the posterior tibial and peroneal arteries. Trendelenburg's sign is caused by weakness or paralysis of the gluteus medius and minimus muscles. Flat foot results from the collapse of the medial longitudinal arch of the foot.

35. A 37-year-old house painter fell from a ladder and fractured his left third rib and the structures with which it articulated. Which of the following structures would most likely be damaged? (A) Manubrium of the sternum (B) Body of the second thoracic vertebra (C) Spinous process of the third thoracic vertebra (D) Body of the fourth thoracic vertebra (E) Transverse process of the second thoracic vertebra

35. The answer is B. The third rib articulates with the body of the sternum, bodies of the second and third thoracic vertebrae, and transverse process of the third thoracic vertebra.

35. A 22-year-old victim of an automobile accident has received destructive damage to structures that form the boundary of the perineum. Which of the following structures is spared? (A) Pubic arcuate ligament (B) Tip of the coccyx (C) Ischial tuberosities (D) Sacrospinous ligament (E) Sacrotuberous ligament

35. The answer is D. The sacrospinous ligament forms a boundary of the lesser sciatic foramen. The pubic arcuate ligament, tip of the coccyx, ischial tuberosities, and sacrotuberous ligament all form part of the boundary of the perineum.

36. A construction worker is hit on the leg with a concrete block and is subsequently unable to plantar flex and invert his foot. Which of the following muscles is most likely damaged? (A) Extensor digitorum longus (B) Tibialis anterior (C) Tibialis posterior (D) Peroneus longus (E) Peroneus brevis

36. The Answer is C. The tibialis posterior can plantar flex and invert the foot. The extensor digitorum longus can dorsiflex and evert the foot, the tibialis anterior can dorsiflex and invert the foot, and the peroneus longus and brevis can plantar flex and evert the foot.

36. A 32-year-old man undergoes vasectomy as a means of permanent birth control. A physician performing the vasectomy by making an incision on each side of the scrotum should remember which of the following statements most applicable to the scrotum? (A) It is innervated by the ilioinguinal and genitofemoral nerves. (B) It receives blood primarily from the testicular artery. (C) Its venous blood drains primarily into the renal vein on the left. (D) Its lymphatic drainage is primarily into upper lumbar nodes. (E) Its dartos tunic is continuous with the perineal membrane.

36. The answer is A. The scrotum is innervated by branches of the ilioinguinal, genitofemoral, pudendal, and posterior femoral cutaneous nerves. The scrotum receives blood from the posterior scrotal branches of the internal pudendal arteries and the anterior scrotal branches of the external pudendal arteries, but it does not receive blood from the testicular artery. Similarly, the scrotum is drained by the posterior scrotal veins into the internal pudendal vein. The lymph vessels from the scrotum drain into the superficial inguinal nodes, whereas the lymph vessels from the testis drain into the upper lumbar nodes. The dartos tunic is continuous with the membranous layer of the superficial perineal fascia (Colles fascia).

42. A patient has weakness when flexing both her thigh and leg. Which of the following muscles is most likely injured? (A) Rectus femoris (B) Semitendinosus (C) Biceps femoris (D) Sartorius (E) Adductor longus

42. The Answer is D. The sartorius can flex and rotate the thigh laterally, and flex and rotate the leg medially. The rectus femoris flexes the thigh and extends the leg. The semimembranosus extends the thigh and flexes and rotates the leg medially. The biceps femoris extends the thigh and flexes and rotates the leg laterally. The adductor longus adducts and flexes the thigh.

37. A 37-year-old woman complains of a bearing- down sensation in her womb and an increased frequency of and burning sensation on urination. On examination by her gynecologist, she is diagnosed with a uterine prolapse. Which of the following structures provides the primary support for the cervix of the uterus? (A) External anal sphincter (B) Broad ligament of the uterus (C) Cardinal (transverse cervical) ligament (D) Round ligament of the uterus (E) Suspensory ligament of the ovary

37. The answer is C. The cardinal (transverse cervical) ligament provides the major ligamentous support for the uterus. The sphincter ani externus does not support the uterus. The broad and round ligaments of the uterus provide minor supports for the uterus. The suspensory ligament of the ovary does not support the uterus.

38. A 24-year-old woman presents to her physician with weakness in flexing the hip joint and extending the knee joint. Which muscle is most likely involved in this scenario? (A) Sartorius (B) Gracilis (C) Rectus femoris (D) Vastus medialis (E) Semimembranosus

38. The Answer is C. The rectus femoris flexes the thigh and extends the leg. The sartorius can flex both the hip and knee joints. The gracilis adducts and flexes the thigh and flexes the leg, the vastus medialis extends the knee joint, and the semimembranosus extends the hip joint and flexes the knee joint.

38. A woman is delivering a breech baby. The obstetrician decides that it is best to perform a mediolateral episiotomy. Which of the following structures should the obstetrician avoid incising? (A) Vaginal wall (B) Superficial transverse perineal muscle (C) Bulbospongiosus (D) Levator ani (E) Perineal membrane

38. The answer is D. An obstetrician should avoid incising the levator ani and the external anal sphincter. The levator ani is the major part of the pelvic diaphragm, which forms the pelvic floor and supports all of the pelvic organs. None of the other choices applies here.

39. A 17-year-old boy was stabbed during a gang fight resulting in transection of the obturator nerve. Which of the following muscles is completely paralyzed? (A) Pectineus (B) Adductor magnus (C) Adductor longus (D) Biceps femoris (E) Semimembranosus

39. The Answer is C. The adductor longus is innervated by only the obturator nerve. Thus, injury here could completely paralyze the adductor longus. The pectineus is innervated by both the obturator and femoral nerves. The adductor magnus is innervated by both the obturator nerve and tibial part of the sciatic nerve. The biceps femoris is innervated by the tibial portion (long head) and common peroneal portion (short head) of the sciatic nerve. The semimembranosus is innervated by the tibial portion of the sciatic nerve.

39. During pelvic surgery, a surgeon notices severe bleeding from the artery that remains within the true pelvis. Which of the following arteries is most likely to be injured? (A) Iliolumbar artery (B) Obturator artery (C) Uterine artery (D) Internal pudendal artery (E) Inferior gluteal artery

39. The answer is C. Of all the arteries listed, the uterine artery remains within the pelvic cavity.

39. A 27-year-old patient with Marfan syndrome has an aneurysm of the aortic arch. This may compress which of the following structures? (A) Right vagus nerve (B) Left phrenic nerve (C) Right sympathetic trunk (D) Left recurrent laryngeal nerve (E) Left greater splanchnic nerve

39. The answer is D. The left recurrent laryngeal nerve loops around the arch of the aorta near the ligamentum arteriosum, whereas the right recurrent laryngeal nerve hooks around the right subclavian artery. All other nerves are not closely associated with the aortic arch.

42. A radiologist interprets a lymphangiogram for a 29-year-old patient with metastatic carcinoma. Upper lumbar nodes most likely receive lymph from which of the following structures? (A) Lower part of the anal canal (B) Labium majus (C) Clitoris (D) Testis (E) Scrotum

42. The answer is D. Lymphatic vessels from the testis and epididymis ascend along the testicular vessels in the spermatic cord through the inguinal canal and continue upward in the abdomen to drain into the upper lumbar nodes. The lymph from the other structures drains into the superficial inguinal lymph nodes.

4. A 41-year-old man was involved in a fight and felt weakness in extending the knee joint. On examination, he was diagnosed with a lesion of the femoral nerve. Which of the following symptoms would be a result of this nerve damage? (A) Paralysis of the psoas major muscle (B) Loss of skin sensation on the lateral side of the foot (C) Loss of skin sensation over the greater trochanter (D) Paralysis of the vastus lateralis muscle (E) Paralysis of the tensor fasciae latae

4. The answer is D. The femoral nerve innervates the quadratus femoris, sartorius, and vastus muscles. Therefore, damage to this nerve results in paralysis of these muscles. The second and third lumbar nerves innervate the psoas major muscle, the sural nerve innervates the skin on the lateral side of the foot, the iliohypogastric nerve and superior clunial nerves supply the skin over the greater trochanter, and the superior gluteal nerve innervates the tensor fasciae latae.

42. A mother with diabetes gives birth to a baby who is diagnosed as having dextroposition of the aorta and the pulmonary trunk with cyanosis and shortness of breath. Which of the following structures is required to remain patent until surgical correction of the deformity? (A) Umbilical arteries (B) Umbilical vein (C) Ductus arteriosus (D) Ductus venosus (E) Sinus venosus

42. The answer is C. A patent ductus arteriosus shunts blood from the pulmonary trunk to the aorta, partially bypassing the lungs, and thus allowing mixed blood to reach the body tissues and causing cyanosis. Dextroposition or transposition of the great arteries must be accompanied by a VSD or a patent ductus arteriosus for the infant to survive. The transposition causes oxygenated blood to pass from the left ventricle into the pulmonary trunk and then into the lungs, but deoxygenated blood travels from the right ventricle into the aorta and then into the systemic circulation.

40. A neurosurgeon performs a surgical resection of a rare meningeal tumor in the sacral region. He tries to avoid an injury of the nerve that arises from the lumbosacral plexus and remains within the abdominal or pelvic cavity. To which of the following nerves should he pay particular attention? (A) Ilioinguinal nerve (B) Genitofemoral nerve (C) Lumbosacral trunk (D) Femoral nerve (E) Lateral femoral cutaneous nerve

40. The answer is C. The lumbosacral trunk is formed by part of the ventral ramus of the fourth lumbar nerve and the ventral ramus of the fifth lumbar nerve. This trunk contributes to the formation of the sacral plexus by joining the ventral ramus of the first sacral nerve in the pelvic cavity and does not leave the pelvic cavity. All other nerves leave the abdominal and pelvic cavities.

41. While playing football, a 19-year-old college student receives a twisting injury to his knee when being tackled from the lateral side. Which of the following conditions most likely has occurred? (A) Tear of the medial meniscus (B) Ruptured fibular collateral ligament (C) Tenderness on pressure along the fibular collateral ligament (D) Injury of the posterior cruciate ligament (E) Swelling on the back of the knee joint

41. The Answer is A. The "unhappy triad" of the knee joint is characterized by tear of the medial meniscus, rupture of the tibial collateral ligament, and rupture of the anterior cruciate ligament. This injury may occur when a cleated shoe, as worn by football players, is planted firmly in the turf and the knee is struck from the lateral side. Tenderness along the medial collateral ligament and over the medial meniscus and swelling on the front of the joint are due to excessive production of synovial fluid, which fills the joint cavity and the suprapatellar bursa.

41. After repair of a ruptured diverticulum, a 31-year-old patient begins to spike with fever and complains of abdominal pain. An infection in the deep perineal space would most likely damage which of the following structures? (A) Ischiocavernosus muscles (B) Superficial transverse perineal muscles (C) Levator ani (D) Sphincter urethrae (E) Bulbospongiosus

41. The answer is D. The sphincter urethrae are found in the deep perineal space, whereas the other structures are located in the superficial perineal space.

45. A 21-year-old man is involved in a highspeed motor vehicle accident. As a result, he has extensive damage to his sphincter urethra. Which of the following best describes the injured sphincter urethra? (A) Smooth muscle (B) Innervated by the perineal nerve (C) Lying between the perineal membrane and Colles fascia (D) Enclosed in the pelvic fascia (E) Part of the pelvic diaphragm

45. The answer is B. The sphincter urethra is a striated muscle that lies in the deep perineal space and forms a part of the urogenital diaphragm but not the pelvic diaphragm. It is not enclosed in the pelvic fascia. It is innervated by a deep (muscular) branch of the perineal nerve.

45. A 9-month-old girl was admitted to the children's hospital with tachypnea (fast breathing) and shortness of breath. Physical examination further exhibits tachycardia (fast heart rate), a bounding peripheral pulse, and her angiographs reveal a patent ductus arteriosus. Which of the following embryonic arterial structures is most likely responsible for the origin of the patent ductus arteriosus? (A) Right fourth arch (B) Left fifth arch (C) Right fifth arch (D) Left sixth arch (E) Right sixth arch

45. The answer is D. The left sixth aortic arch is responsible for the development of both the ductus arteriosus and the pulmonary arteries. The ductus arteriosus closes functionally in an infant soon after birth, with anatomic closure requiring several weeks.

46. Before knee surgery, a surgeon ligates arteries participating in the anastomosis around the knee joint. Which of the following arteries is most likely spared? (A) Lateral superior genicular (B) Medial inferior genicular (C) Descending branch of the lateral femoral circumflex (D) Saphenous branch of the descending genicular (E) Anterior tibial recurrent

46. The Answer is D. The descending genicular artery gives off the articular branch, which enters the anastomosis around the knee joint, and the saphenous branch, which is not involved in the anastomosis but supplies the superficial tissue and skin on the medial side of the knee. Other arteries are involved in the anastomosis around the knee joint.

46. A 7-day-old baby is diagnosed as having congenital neonatal emphysema, which is caused by collapsed bronchi because of failure of bronchial cartilage development. Bronchial cartilages are derived from which of the following derivations? (A) Ectoderm (B) Mesoderm (C) Endoderm (D) Proctodeum (E) Neuroectoderm

46. The answer is B. Bronchial cartilages, smooth muscles, and connective tissue are derived from the mesoderm. The bronchial epithelium and glands are derived from the endoderm.

to the children's hospital because he has no testis in his scrotum. During physical examination, the pediatrician palpated the testis in the inguinal canal. What is the diagnosis of this condition? (A) Male pseudohermaphroditism (B) Hypospadias (C) Epispadias (D) Cryptorchid testis (E) Chordee

46. The answer is D. Cryptorchid testis is called an undescended testis, which is located in the inguinal region. Male pseudohermaphroditism is a condition in which the affected individual is a genetic and gonadal male with genital anomalies. Hypospadias occurs when the spongy urethra opens on the underside of the penis, frequently associated with the chordee, which is a ventral curvature of the penis. Epispadias occurs when the urethra opens on the dorsal surface of the penis.

47. A 25-year-old gladiator sustains a penetrating injury that severs the superficial peroneal nerve. This will most likely cause paralysis of which of the following muscles? (A) Peroneus tertius (B) Peroneus brevis (C) Flexor hallucis longus (D) Tibialis anterior (E) Tibialis posterior

47. The Answer is B. The peroneus brevis muscle is innervated by the superficial peroneal nerve. The peroneus tertius and tibialis anterior muscles are innervated by the deep peroneal nerve. The flexor hallucis longus and tibialis posterior muscles are innervated by the tibial nerve.

47. An obstetrician is about to perform a pudendal block so a woman can experience less pain when she delivers her child. He recalls what he learned in medical school about this nerve. Which of the following statements is correct? (A) It passes superficial to the sacrotuberous ligament (B) It innervates the testis and epididymis in a male (C) It provides motor fibers to the coccygeus (D) It can be blocked by injecting an anesthetic near the inferior margin of the ischial spine (E) It arises from the lumbar plexus

47. The answer is D. The pudendal nerve, which arises from the sacral plexus, provides sensory innervation to the labium majus (or scrotum in a male). It leaves the pelvis through the greater sciatic foramen and enters the perineum through the lesser sciatic foramen near the inferior margin of the ischial spine. Therefore, it can be blocked by injection of an anesthetic near the inferior margin of the ischial spine.

49 As part of a physical examination to evaluate muscle function in the hand, a physician holds the four fingers (digits 2 through 5) and asks the patient to spread their fingers. What muscle(s) is/are the doctor testing? (A) Lumbrical muscles (B) Palmar interosseous muscles (C) Dorsal interosseous muscles (D) Flexor digitorum superficialis (E) Flexor digitorum profundus

49 The answer is C: Dorsal interosseous muscles. The primary action of the dorsal interossei is to abduct the fingers, which is being tested in this photo. It is also important to remember the dorsal interossei insert into the extensor digital expansion, so these muscles work with the palmar interossei and the lumbrical muscles to extend the interphalangeal joints and fl ex the metacarpophalangeal joints. The nerve being tested in this patient, via abduction of the fi ngers, is the deep branch of the ulnar nerve. A mnemonic is "DAB", which stands for Dorsal interossei ABduct the fi ngers. Choice A (Lumbrical Muscles) is incorrect. The lumbrical muscles arise off the flexor digitorum profundus tendons and insert into the lateral (radial) aspect of the extensor digital expansion. Due to their course and insertion, the lumbrical muscles extend the interphalangeal joints and flex the metacarpophalangeal joints. These intrinsic hand muscles are not being tested when a patient abducts the fingers against resistance. Choice B (Palmar interosseous muscles) is incorrect. The primary action of the three palmar interossei is to adduct the index fi nger, ring fi nger, and little fi nger toward the axis of the hand, which is the middle finger. The deep branch of the ulnar nerve supplies the palmar interossei. A mnemonic is "PAD", which stands for Palmar interossei ADduct the fi ngers. It is also important to remember the palmar interossei insert into the extensor digital expansion, so these muscles work in concert with the dorsal interossei and the lumbrical muscles to extend the interphalangeal joints and flex the metacarpophalangeal joints of the fi ngers. Choice D (Flexor digitorum superficialis) is incorrect. The fl exor digitorum superficcialis muscle flexes the proximal interphalangeal joint in digits 2 to 5, and infl uences the distal interphalangeal joint by binding the tendons of the flexor digitorum profundus muscle. Flexion of the proximal interphalangeal joints of the fingers is not being tested in this patient. Choice E (Flexor digitorum profundus) is incorrect. The flexor digitorum profundus muscle is responsible for flexion of the distal interphalangeal joint in digits 2 to 5; however, this action is not being tested in this photo

49. An elderly man with prostatitis is seen at an internal medicine clinic. The seminal colliculus of his prostate gland is infected, and its fine openings are closed. Which of the following structures is/are most likely to be disturbed? (A) Ducts of the prostate gland (B) Prostatic utricle (C) Ducts of the bulbourethral glands (D) Ejaculatory ducts (E) Duct of the seminal vesicles

49. The answer is D. The ejaculatory ducts, which open onto the seminal colliculus, may be injured. The prostate ducts open into the urethral sinus, the bulbourethral ducts open into the bulbous part of the penile urethra, and the ducts of the seminal vesicle join the ampulla of the ductus deferens to form the ejaculatory duct. The prostatic utricle is a minute pouch on the summit of the seminal colliculus.

5. A 29-year-old carpenter sustains severe injuries of the pelvic splanchnic nerve by a deep puncture wound, which has become contaminated. The injured parasympathetic preganglionic fibers in the splanchnic nerve are most likely to synapse in which of the following ganglia? (A) Ganglia in or near the viscera or pelvic plexus (B) Sympathetic chain ganglia (C) Collateral ganglia (D) Dorsal root ganglia (E) Ganglion impar

5. The answer is A. The pelvic splanchnic nerves carry preganglionic parasympathetic general visceral efferent fibers that synapse in the ganglia of the inferior hypogastric plexus and in terminal ganglia in the muscular walls of the pelvic organs. The sympathetic preganglionic fibers synapse in the sympathetic chain (paravertebral) ganglia or in the collateral (prevertebral) ganglia. The dorsal root ganglia contain cell bodies of general somatic afferent (GSA) and general visceral afferent (GVA) fibers and have no synapsis. The two sympathetic trunks unite and terminate in the ganglion impar (coccygeal ganglion), which is the most inferior, unpaired ganglion located in front of the coccyx.

5. A 27-year-old cardiac patient with an irregular heartbeat visits her doctor's office for examination. Where should the physician place the stethoscope to listen to the sound of the mitral valve? (A) Over the medial end of the second left intercostal space (B) Over the medial end of the second right intercostal space (C) In the left fourth intercostal space at the midclavicular line (D) In the left fifth intercostal space at the midclavicular line (E) Over the right half of the lower end of the body of the sternum

5. The answer is D. The mitral valve (left atrioventricular [AV] valve) produces the apical beat (thrust) of the heart, which is most audible over the left fifth intercostal space at the midclavicular line. The pulmonary valve is most audible over the medial end of the second left intercostal space, the aortic valve is most audible over the medial end of the second right intercostal space, and the right AV valve is most audible over the right half of the lower end of the body of the sternum.

5. A 16-year-old patient received a lacera- tion of the posterior intercostal nerves by a penetrated knife blade. A pathologist obtained needle biopsy tissues and observed numerous degenerated cell bod- ies of the unipolar or pseudounipolar neu- rons. Which of the following structures would most likely provide the abnormal cell morphology? (A) Ventral horn of the spinal cord (B) Lateral horn of the spinal cord (C) Dorsal horn of the spinal cord (D) Dorsal root ganglion (E) Sympathetic chain ganglion

5. The answer is D. Ventral, lateral, and dorsal horns and sympathetic chain ganglia contain multipolar neurons, whereas the dorsal root ganglion contains unipolar or pseudounipo- lar neurons. A laceration of the intercostal nerve injures GSE, postganglionic sympathetic general visceral efferent (GVE), general visceral afferent, and general somatic afferent (GSA) fibers, whose cell bodies are located in the anterior horn, sympathetic chain ganglia, and dorsal root ganglia.

50 A 52-year-old retired professional cyclist, who still rides his bike 400 miles per week, comes to his physician complaining of hand problems. The physician notes hyperextension of the ring and little fingers at the metacarpophalangeal joints and flexion at the interphalangeal joints within the same fingers (see photo). During examination, the patient has no weakness in flexion or adduction of the wrist. What nerve is compressed at what location? (A) Ulnar nerve in the elbow (B) Ulnar nerve in the wrist (C) Median nerve in the wrist (D) Median nerve in the elbow (E) Median nerve in the axilla

50 The answer is B: Ulnar nerve in the wrist. The ulnar nerve can become compressed between the pisiform and hook of the hamate at the wrist in a condition termed "ulnar canal syndrome" or "Guyon tunnel syndrome." This entrapment syndrome is especially seen in professional cyclists who spend countless hours placing pressure on the hook of the hamate bone as they grasp their handlebars. This "handlebar neuropathy" presents with hyperextension of the metacarpophalangeal joints and flexion at the interphalangeal joints of the fourth and fifth fingers. The "clawing" of these two fingers is accompanied by sensory loss in the medial side of the hand. Choice A (Ulnar nerve in the elbow) is incorrect. The ulnar nerve crosses the elbow in a narrow space between the olecranon process and the medial epicondyle of the humerus, on the posteromedial (ulnar) side of the joint. Compression of the ulnar nerve at this location leads to "cubital tunnel syndrome" and presents with hand deficits similar to those seen in this patient. However, this patient does not exhibit weakness in fl exion or adduction of the wrist. Sparing of these actions indicates that damage to the ulnar nerve must be distal to the elbow. Choice C (Median nerve in the wrist) is incorrect. The median nerve

50. A 14-year-old gymnastic silver medalist falls from the parallel bar and complains of pains from the knee and ankle joints. On physical examination, her physician found that the muscle responsible for flexing the leg at the knee joint and plantar flexing the foot is severely weakened. Which of the following muscles involved in both movements was most likely damaged in this accident? (A) Tibialis posterior (B) Gastrocnemius (C) Soleus (D) Peroneus longus (E) Flexor digitorum longus

50. The Answer is B. The gastrocnemius can flex the knee joint and also plantar flex the foot. The tibialis posterior can plantar flex and invert the foot. The soleus can plantar flex the foot. The peroneus longus can plantar flex and evert the foot. The flexor digitorum longus can plantar flex the foot and flex the lateral four toes.

51. A 5-month-old boy is admitted to the children's hospital because of urine being expelled from the dorsal aspect of the penis. Which of the following embryologic structures failed to fuse in this patient? (A) Labioscrotal swellings or folds (B) Urogenital sinus (C) Spongy urethra (D) Phallus (E) Urethral folds

51. The answer is C. A developmental defect in the spongy urethra results in epispadias, causing the patient to pass urine through an opening on the dorsum of the penis. Labioscrotal swellings form the scrotum in males and the labia majora in females. Urogenital sinus forms the urinary bladder, urethra, prostate, and bulbourethral glands in males, and the bladder, urethra, lower vagina, and greater vestibular glands in females. The phallus (genital tubercle) forms the penis in males and the clitoris in females. Urethral (urogenital) folds form the spongy urethra and a portion of the shaft of the penis in males and the labia minora in females.

53. Rupture of the tendon superior to this structure would most likely cause an inability to extend the knee joint.

53. The Answer is C. The quadriceps tendon is ruptured. The quadriceps muscle is a powerful knee extensor used in climbing, running, jumping, and rising from a seated position.

53. During a partial hysterectomy leaving the ovaries in tact, the surgeon detaches the ovary from the uterus by transecting the ovarian ligament. This ligament developed from which embryonic structure? (A) Mesonephric duct (B) Urogenital folds (C) Gubernaculum (D) Processus vaginalis (E) Paramesonephric ducts

53. The answer is C. The ovarian ligament and the round ligament of the uterus are formed by the gubernaculum. The mesonephric duct gives rise only to the vestigial epoophoron in the female. The urogenital folds form the labia minora. The processus vaginalis forms no adult female structures, while the paramesonephric ducts form the uterine tubes, uterus, cervix, and upper vagina.

54. Fracture of this structure would most likely cause weakness in adduction, flexion, and extension of the thigh.

54. The Answer is A. The adductor tubercle is fractured. The adductor magnus inserts on the adductor tubercle on the femur and functions to adduct, flex, and extend the thigh.

Questions 54 to 58: Choose the appropriate lettered structure in this magnetic resonance image (see Figure below) of the female perineum and pelvis. 54. Which structure extends between the vestibule and the cervix of the uterus and serves as the excretory channel for the products of menstruation?

54. The answer is D. The vagina is the genital canal in the female, extending from the vestibule to the uterine cervix. The vagina transmits the products of menstruation and receives the penis in copulation.

55. A knife penetrating through this point would most likely cause muscle ischemia in the anterior compartment of the leg.

55. The Answer is B. The anterior tibial artery enters the anterior compartment by passing through a gap between the neck of the fibula and tibia. Therefore, a knife wound through the gap may cause an injury to the anterior tibial artery, resulting in muscle ischemia in the anterior compartment of the leg.

56. Fracture of this structure would most likely cause a lesion of the common peroneal nerve, resulting in paralysis of the muscles in the anterior and lateral compartments of the leg.

56. The Answer is E. The common peroneal nerve is vulnerable to injury as it passes behind the head of the fibula and then winds laterally around the neck of the fibula. Fracture of the fibular head causes a lesion of the common peroneal nerve, resulting in paralysis of the muscles in the anterior and lateral compartments of the leg.

56. Into which structure does hemorrhage occur after injury to the inferior rectal vessels?

56. The answer is E. The ischiorectal fossa lies in the anal triangle and is bound laterally by the obturator internus with its fascia and superomedially by the levator ani and external anal sphincter. It contains the inferior rectal vessels. Thus, hemorrhage occurs in the ischiorectal fossa when it is ruptured.

58. Fracture of which structure may destroy the site of insertion of the muscle that can rotate the thigh laterally and its tendon that passes through the lesser sciatic foramen?

58. The Answer is B. The greater trochanter is the site for insertion of the obturator internus muscle tendon, which leaves the pelvis through the lesser sciatic foramen.

58. Which structure is innervated by the nerve passing through both the greater and lesser sciatic foramina?

58. The answer is B. The obturator internus muscle and its fascia form the lateral wall of the ischiorectal fossa. This muscle is innervated by the nerve to the obturator internus, which passes through the greater and lesser sciatic foramen.

Questions 59 to 63: Choose the appropriate lettered structure in this computed tomography scan (see Figure below) of the male perineum and pelvis. 59. Which structure, when fractured, results in paralysis of the obturator internus muscles?

59. The answer is E. The greater trochanter provides an insertion site for the obturator internus muscle.

6. A 22-year-old patient is unable to "unlock" the knee joint to permit flexion of the leg. Which of the following muscles is most likely damaged? (A) Rectus femoris (B) Semimembranosus (C) Popliteus (D) Gastrocnemius (E) Biceps femoris

6. The Answer is C. The popliteus muscle rotates the femur laterally ("unlocks" the knee) or rotates the tibia medially, depending on which bone is fixed. This action results in unlock- ing of the knee joint to initiate flexion of the leg at the joint. The rectus femoris flexes the thigh and extends the knee. The gastrocnemius flexes the knee and plantar flexes the foot. The semimembranosus extends the thigh and flexes and rotates the leg medially. The biceps femoris extends the thigh and flexes and rotates the leg laterally.

6. A 19-year-old man came to the emergency department, and his angiogram exhibited that he was bleeding from the vein that is accompanied by the posterior interventricular artery. Which of the following veins is most likely to be ruptured? (A) Great cardiac vein (B) Middle cardiac vein (C) Anterior cardiac vein (D) Small cardiac vein (E) Oblique veins of the left atrium

6. The answer is B. The middle cardiac vein ascends in the posterior interventricular groove, accompanied by the posterior interventricular branch of the right coronary artery. The great cardiac vein is accompanied by the anterior interventricular artery, the anterior cardiac vein drains directly into the right atrium, and the small cardiac vein is accompanied by the marginal artery.

63. The medial longitudinal arch of the foot is flattened because the spring ligament is torn. Which bone in the radiograph is most likely fractured?

63. The Answer is D. The spring (plantar calcaneonavicular) ligament extends from the sustentaculum tali of the calcaneus to the navicular bone.

63. A stab wound immediately superior to the pubic symphysis on the anterior pelvic wall would most likely injure which visceral organ first?

63. The answer is C. The bladder is situated in the anterior part of the pelvic cavity. Thus, a stab wound superior to the pubic symphysis would injure the bladder.

7. A 29-year-old woman is admitted to a hospital because the birth of her child is several days overdue. Tearing of the pelvic diaphragm during childbirth leads to paralysis of which of the following muscles? (A) Piriformis (B) Sphincter urethrae (C) Obturator internus (D) Levator ani (E) Sphincter ani externus

7. The answer is D. The pelvic diaphragm is formed by the levator ani and coccygeus, whereas the urogenital diaphragm consists of the sphincter urethrae and deep transverse perinei muscles. The piriformis passes through the greater sciatic notch and inserts on the greater trochanter of the femur. The obturator internus forms the lateral wall of the ischiorectal fossa. The sphincter ani externus is composed of three layers, including the subcutaneous (corrugator cutis ani), superficial, and deep portions, and maintains a voluntary tonic contracture.

7. A 37-year-old patient with palpitation was examined by her physician, and one of the diagnostic records included a posterior-anterior chest radiograph. Which of the following comprises the largest portion of the sternocostal surface of the heart seen on the radiograph? (A) Left atrium (B) Right atrium (C) Left ventricle (D) Right ventricle (E) Base of the heart

7. The answer is D. The right ventricle forms a large part of the sternocostal surface of the heart. The left atrium occupies almost the entire posterior surface of the right atrium. The right atrium occupies the right aspect of the heart. The left ventricle lies at the back of the heart and bulges roundly to the left. The base of the heart is formed by the atria, which lie mainly behind the ventricles.

8. A motorcyclist falls from his bike in an accident and gets a deep gash that severs the superficial peroneal nerve near its origin. Which of the following muscles is paralyzed? (A) Peroneus longus (B) Extensor hallucis longus (C) Extensor digitorum longus (D) Peroneus tertius (E) Extensor digitorum brevis

8. The Answer is A. The superficial peroneal nerve supplies the peroneus longus and brevis muscles. Other muscles are innervated by the deep peroneal nerve.

8. A 33-year-old male patient complains of feeling severe pain when he tries to turn his neck. A physician realizes that the problem is in his pivot (trochoid) joint. Which of the following joints would most likely be examined? (A) Atlantooccipital joint (B) Atlantoaxial joint (C) Carpometacarpal joint (D) Proximal tibiofibular joint (E) Intervertebral disks

8. The answer is B. The atlantoaxial joint is the pivot or trochoid joint. The atlantooccipital joints are the condyloid (ellipsoidal) joints, the carpometacarpal joint of the thumb is the saddle (sellar) joint, and the proximal tibiofibular joint is the plane (gliding) joint. The intervertebral disk is the secondary cartilaginous (symphysis) joint.

9. A 54-year-old patient is implanted with an artificial cardiac pacemaker. Which of the following conductive tissues of the heart had a defective function that required the pacemaker? (A) Atrioventricular (AV) bundle (B) AV node (C) Sinoatrial (SA) node (D) Purkinje fiber (E) Moderator band

9. The answer is C. The sinoatrial (SA) node initiates the impulse of contraction and is known as the pacemaker of the heart. Impulses from the SA node travel through the atrial myocardium to the AV node and then race through the AV bundle (bundle of His), which divides into the right and left bundle branches. The bundle breaks up into terminal conducting fibers (Purkinje fibers) to spread out into the ventricular walls. The moderate band carries the right limb of the AV bundle from the septum to the sternocostal wall of the ventricle.

A 1-year-old female infant was brought to her pediatrician because of a small "strawberry" like swelling on her scalp that has been growing rapidly. A diagnosis of hemangioma is confirmed. Which of the following embryologic layers gives rise to this vascular tumor? A. Mesoderm B. Endoderm C. Ectoderm D. Trophoblast E. Syncytiotrophoblast

A. A hemangioma is a benign endothelial cell tumor. It is characterized by an increased number of normal or abnormal vessels filled with blood. Hemangiomas usually appear in the first weeks of life and grow most rapidly over the first 6 months. Hemangioblast is the common precursor for blood vessels and blood formation induced by vascular endothelial growth factor secreted by surrounding mesoderm. The endoderm forms the epithelial lining of the primitive gut, respiratory tract, tympanic cavity and auditory tube, and the allantois and vitelline duct. The ectoderm gives rise to the central nervous system, the peripheral nervous system, the sensory epithelium of the ear, nose, and eye, the epidermis, hair, and nails, the subcutaneous, mammary, and pituitary glands, and the enamel of the teeth. Trophoblast become cells that form the outer layer of a blastocyst and that provide nutrients to the embryo and later develop into a large part of the placenta. They are the first cells to differentiate from the fertilized egg. Syncytiotrophoblast is the epithelial lining of the placental villi, which will penetrate the walls of the uterus to establish maternofetal circulation. 285

Very often the first indication that a woman has that she is pregnant is a missed menstrual period. In which week of embryonic development does a woman experience her first missed menstrual period? (A) Start of week 3 (B) Start of week 4 (C) Start of week 5 (D) Start of week 8 (E) End of week 8

A. Assuming a regular 28-day menstrual cycle, a woman who starts menses on February 1, for example, will ovulate on February 14, and the secondary oocyte is fertilized within 24 hours. So, the zygote undergoes week 1 of development from February 15 to 21. Week 2 of development is from February 22 to 28. On March 1, the woman should enter her next menstrual cycle, but because she is pregnant, she will not menstruate. Therefore, this first missed menstrual period corresponds with the start of week 3 of embryonic development. The embryonic period (week 3-8) is a time of high susceptibility to teratogens.

24 A pregnant mother is due in 2 weeks. Her body mass index (BMI) is in the overweight category and she has developed gestational diabetes. Pregnant women with gestational diabetes are more likely to have babies weighing 9 lb or more. During delivery, the obstetrician would be most concerned about which of the following for this expecting mother? A. Interspinous distance B. Distance of the pelvic inlet C. AP diameter D. Placenta previa E. Distance of the pelvic outlet

A. During embryogenesis, poorly controlled diabetes mellitus in the mother is associated with a 2- to 3-fold increase in the incidence of birth defects such as macrosomia, which is diagnosed as birth weight greater than 8 pounds 13 ounces (4000 grams), regardless of gestational age. Fetal macrosomia makes vaginal delivery difficult and puts the baby at risk of injury during birth. Therefore, cesarean section is recommended, during which epidural anesthesia is given. Hence, the interspinous distance is of concern. Other common anomalies with maternal diabetes mellitus include holoprosencephaly (failure of the forebrain to divide into hemispheres), meroencephaly (partial absence of the brain), sacral agenesis, congenital heart defects, and limb anomalies. 311

A 28-year-old woman visits an obstetrician for in vitro fertilization. FSH-analogs are injected to stimulate follicles. Then hCG is injected to induce the final oocyte maturation. Lastly, the oocyte is retrieved via a procedure called transvaginal oocyte retrieval. Knowing the anatomic relationship between the ovary and the vagina, the physician will insert the needle through which space of the vaginal wall to reach the ovary? A. Posterior fornix B. Lateral fornix C. Pouch of Morison D. Retropubic space of Retzius E. Extraperitoneal space

A. Each ovary is oval shaped, and is attached to the back of the broad ligament by the mesovarium. The position of the ovary is, however, extremely variable, and it is often found hanging down in the rectouterine pouch (pouch of Douglas). The posterior wall of the vagina is longer than the anterior wall and the posterior fornix is deeper than the other fornices. The posterior fornix is covered by peritoneum of the front of the rectouterine pouch (of Douglas) (GAS Fig. 5-58). 14-18

A 32-year-old woman visits an obstetrician complaining of significant bleeding in her first trimester and pain in the lower back. Ultrasound confirms the diagnosis of an abdominal pregnancy. Where is the most common place for an ectopic abdominal pregnancy? A. Pouch of Douglas B. Pouch of Morison C. Retropubic space D. Extraperitoneal space E. Retroperitoneal space

A. Ectopic pregnancy is the existence of a pregnancy outside the normal confines of the uterus or abnormally in the uterus. Although the uterine (fallopian) tube is the most common site of an ectopic pregnancy, it can also occur in the abdominal cavity, and when that is the case, the pouch of Douglas in the most common site.

A 32-year-old woman visits an obstetrician complaining of significant bleeding in her first trimester and pain in the lower back. Ultrasound confirms the diagnosis of an abdominal pregnancy. Where is the most common place for an ectopic abdominal pregnancy? A. Pouch of Douglas B. Pouch of Morison C. Retropubic space D. Extraperitoneal space E. Retroperitoneal space

A. Ectopic pregnancy is the existence of a pregnancy outside the normal confines of the uterus or abnormally in the uterus. Although the uterine (fallopian) tube is the most common site of an ectopic pregnancy, it can also occur in the abdominal cavity, and when that is the case, the pouch of Douglas in the most common site. The pouch of Morison is the space between the liver and the right kidney and is not usually a site for ectopic pregnancy. The retropubic space is an extraperitoneal space between the pubic symphysis and the urinary bladder. It is a very unlikely site for ectopic pregnancy. Ectopic pregnancies rarely exist at the other sites listed (Fig. 8-1). 32-33

A cyanotic 9-year-old boy is brought to the emergency department, coughing up blood and complaining of severe chest pain. Examination by the physician reveals a heart arrhythmia and a heart murmur. Looking at the patient's history, the physician diagnoses the boy with Eisenmenger's syndrome, a left-to-right shunt converted into a right-to-left shunt secondary to elevated pulmonary artery pressure. Which of the following disorders could be the initial congenital defect (left-toright shunting) causing this syndrome? A. Ventricular septal defect B. Ebstein's anomaly C. Underdeveloped left ventricle (hypoplastic left heart syndrome) D. Common atrioventricular canal E. Large foramen secundum

A. Eisenmenger's syndrome or tardive cyanosis is the process in which a left-to-right shunt caused by a congenital heart defect causes increased flow through the pulmonary vasculature, causing pulmonary hypertension. This in turn causes increased pressure in the right side of the heart and reversal of the shunt into a right-to-left shunt. Eisenmenger's syndrome is a cyanotic heart defect characterized by a long-standing intracardiac shunt caused commonly by ventricular septal defects. In Ebstein's anomaly the septal leaflet of the tricuspid valve is displaced toward the apex of the right ventricle of the heart. There is subsequent atrialization of part of the right ventricle (which is now contiguous with the right atrium). This causes the right atrium to enlarge and the anatomic right ventricle to be small. Here there is an initial right-to-left shunt that results in a cyanotic baby. With the underdeveloped left ventricle, both the aorta and left ventricle are underdeveloped before birth, and the aortic and mitral valves are each too small to allow sufficient blood flow. As blood returns from the lungs to the left atrium, it must pass through an atrial septal defect to the right side of the heart. In this defect babies appear cyanotic. In the common atrioventricular canal defect the heart has one common chamber due to defects in the formation of its septae. Patients develop pulmonary hypertension by the second year of life. In a large foramen secundum (foramen ovale) the atrial septal defect (ASD) does lead to pulmonary hypertension but has a much slower progression. Symptoms usually appear after the third decade of life. 208, 211-212

A pediatrician examines a newborn baby and finds a smooth philtrum, thin vermilion border (upper lip), and small palpebral fissures. These craniofacial abnormalities are mostly associated with which teratogen that can cross the placenta in utero and mammary glands during breastfeeding? A. Alcohol B. Warfarin C. Heroin D. Rubella E. Tetracycline

A. Fetal alcohol syndrome (FAS) is associated with intrauterine growth restriction (IUGR), mental deficiency, microcephaly, ocular anomalies, joint abnormalities, and small palpebral fissures. Warfarin can be associated with nasal hypoplasia, stippled epiphyses, hypoplastic phalanges, eye anomalies, and mental deficiency. Tetracycline exposure can be associated with stained teeth and hypoplasia of enamel. Rubella virus is associated with various anomalies such as IUGR, postnatal growth retardation, cardiac and great vessel abnormalities, microcephaly, sensorineural deafness, cataract, microphthalmos (also referred to as microphthalmia), glaucoma, pigmented retinopathy, mental deficiency, neonate bleeding, hepatosplenomegaly, osteopathy, and tooth defects. Women who use heroin during pregnancy greatly increase their risk of serious pregnancy complications. These risks include poor fetal growth, premature rupture of membranes, premature birth, and stillbirth. 304-307

A couple hopes to start a family soon but the wife is currently taking medication for epilepsy. The obstetrician explains that exposure to this type of medication can cause serious birth defects, such as microcephaly, cleft palate, and congenital heart defects. Antiepileptic drug exposure during pregnancy will most likely cause which of the following? A. Fetal hydantoin syndrome B. Fetal alcohol syndrome C. Down's syndrome D. Treacher Collins syndrome E. DiGeorge's syndrome

A. Fetal hydantoin syndrome occurs in 5% to 10% of children born to mothers treated with phenytoin or hydantoin anticonvulsants. The usual pattern of defects consists of IUGR, microcephaly, mental deficiency, ridged frontal suture, inner epicanthal folds, eyelid ptosis, broad depressed nasal bridge, nail and/or distal phalangeal hypoplasia, and hernias. Fetal alcohol syndrome (FAS): intrauterine growth restriction (IUGR), mental deficiency, microcephaly, ocular anomalies, joint abnormalities, and short palpebral fissures. Trisomy 21 (Down's syndrome), the most common numerical abnormality resulting in birth defects (intellectual disability, abnormal facies, heart malformations), is usually caused by nondisjunction. The risk of meiotic nondisjunction increases with increasing maternal age. Treacher Collins syndrome is characterized by craniofacial deformities, such as absent cheekbones. Infants with DiGeorge syndrome are born without a thymus and parathyroid glands and have defects in their cardiac outflow tracts. 304-308

A 28-year-old woman who became pregnant 10 weeks ago has a house cat. She feeds the cat raw meat and handles the litter. Blood work reveals positive IgM for a zoonotic parasite. The fetus is susceptible to congenital deafness, low birth rate, inflammation of the retina, and jaundice. Which of the following teratogens can give rise to these findings? A. Toxoplasmosis B. Heroin C. Mercury poisoning D. Alcohol E. Tetracycline

A. Infectious teratogens include toxoplasmosis, which can result in jaundice, intracranial calcifications, chorioretinitis, microcephaly, microphthalmia, and hydrocephalus. Congenital syphilis can result in the formation of abnormal teeth and bones, congenital deafness, hydrocephalus, and mental retardation. Herpes simplex can also result in spasticity and mental retardation. Cytomegalovirus infection may result in cerebral palsy, mental retardation, intrauterine growth retardation, microphthalmia, blindness, deafness, and hepatosplenomegaly. Congenital rubella syndrome presents with cataracts, cardiac defects, and deafness among other defects. Alcohol exposure can result in fetal alcohol syndrome, which has a wide spectrum including mental and growth retardation plus morphogenetic disturbances. Cocaine use can cause microcephaly, neurobehavioral disturbances, spontaneous abortion, and urogenital disturbances. Streptomycin can cause vestibulocochlear (CN VIII) nerve defects. Methadone and heroin exposure can result in small birth weight, central nervous system dysfunction, and small head circumference. Tetracycline exposure can result in tooth and bone defects including yellow discoloration and hypoplasia. 304-305, 310

Genetic analysis of a 20-year-old, 6-foot-tall man reveals a mutation in the FBN1 gene. He has unusually long and thin limbs, a hollowed chest, and severe nearsightedness (myopia). He is diagnosed with Marfan's syndrome. A computed tomography (CT) scan of the thorax revealed an unusual bulge anterior to the spinal cord and to the left of midline. Which condition will he be most susceptible to? A. Ruptured aorta B. Double inferior vena cava C. Portal hypertension D. Hydronephrosis E. Pyloric stenosis

A. Marfan syndrome affects the connective tissue supporting the body's joints and organs (e.g., heart, aorta, and eyes). Abnormalities in the connective tissue are due to mutations in a single protein building block (amino acid) in the fibrillin-1 protein. This leads to a severe reduction in the amount of fibrillin-1 available to form microfibrils, and elasticity in many tissues is decreased. In Marfan's syndrome elastic fibers are significantly reduced, eventually leading to aortic aneurysm and rupture. The thoracic aorta is contained in the posterior mediastinal cavity. It begins at the lower border of the fourth thoracic vertebra, where it is continuous with the aortic arch, and ends in front of the lower border of the twelfth thoracic vertebra at the aortic hiatus, where it becomes the abdominal aorta. 225-229

Ultrasound of a 20-year-old pregnant woman in the 20th week of pregnancy revealed abnormal limb development of her fetus, showing one arm to be shorter than the other. Which of the following most accurately describes this condition? A. Meromelia B. Central digit ray deformity C. Talipes equinovarus D. Polydactyly E. Syndactyly

A. Meromelia is the partial absence of a limb (amelia is total absence). Syndactyly refers to fused digits, and polydactyly is an excess in the number of digits. Central digit ray formation is the underlying mechanism for syndactyly, and talipes equinovarus is a malrotation of the foot, more commonly referred to as clubfoot. 245, 304

A 22yo woman gives birth to an infant with an abnormally small coronary sinus that limits cardiac venous drainage. The coronary sinus is derived from which of the labeled structures in the given diagram of the primitive heart?

A. sinus venosus

Amyotrophic lateral sclerosis (ALS; Lou Gehrig's disease) is a progressive, fatal neurodegenerative disease caused by degeneration of the motor neurons controlling skeletal (voluntary) muscle movement. Postmortem analysis of which of the following structures would show the cell bodies of neurons affected by this disease? (A) Anterior gray horn of the spinal cord (B) Lateral gray horn of the spinal cord(C) Posterior gray horn of the spinal cord(D) Spinal ganglia(E) Lateral column of spinal cord white matter

A: Anterior gray horn of the spinal cord. Cell bodies of somatic motor neurons (a-motor neurons) innervating skeletal muscle are located within the anterior (ventral) gray horn of the spinal cord, at all segmental levels through- out the entire length of the spinal cord. The innervation of the skeletal muscles affected by ALS is through somatic motor (general somatic efferent or GSE) neurons and branchial motor (special visceral efferent or SVE) neurons (neurons that supply the embryonic pharyngeal arches). In ALS patients, postmortem analysis of the anterior gray horn of the spinal cord would show significant degeneration. In the given diagram, the anterior gray horn of the spinal cord is labeled as "A." The locations for all five possible choices for this question are also indicated in this figure. Choice B (Lateral gray horn of the spinal cord) is incorrect. The lateral gray horn of the spinal cord is the location of the intermediolateral cell column (IML), which contains the cell bodies of presynaptic (preganglionic) sympathetic neurons of the autonomic nervous system (ANS). The IML only exists within the lateral gray horn between spinal segmental levels T1-L2 (or L3), which is the reason the sympathetic division of the ANS is also called the thoracolumbar division of the ANS. Degeneration in the lateral gray horn of the spinal cord would cause autonomic deficits but would not include the somatic motor neurons involved in ALS. Choice C (Posterior gray horn of the spinal cord) is incorrect. The posterior (dorsal) gray horn, located along the entire length of the spinal cord, contains cell bodies of spinal interneurons. The central process of each pseudounipolar sensory neuron conveys general sensory (afferent) fibers through the posterior (dorsal) nerve root and synapses within the posterior horn of the spinal cord. Degeneration within the posterior gray horn of the spinal cord would cause sensory deficits but would not include the somatic motor neurons involved in ALS. Choice D (Spinal ganglia) is incorrect. The spinal (dorsal root) ganglia, located at the distal ends of the posterior (dorsal) roots of spinal nerves, contain cell bodies of the general sensory (afferent) neurons. Degeneration within the spinal ganglia would result in sensory deficits but would not include the somatic motor neurons involved in ALS. Choice E (Lateral column of spinal cord white matter) is incorrect. All columns within the spinal cord white matter contain neuronal processes and sup- porting glial cells. Degeneration within the lateral column of the spinal cord would not include the somatic motor neurons involved in ALS.

Idling at a stoplight in his vintage car without headrests, a 71-year-old-man's car is struck from behind by a truck going approximately 30 mph (48 kph). The man is brought to the ER suffering from a severe hyperextension neck injury due to the crash. The given T2-weighted MRI shows a rupture of the anterior anulus of the C4-5 intervertebral disc, inflammation of that disc (the white appearance), and a prevertebral hematoma, which compromised his airway and required intubation. Which of the following ligaments is disrupted in this injury? (A) Anterior longitudinal ligament (B) Posterior longitudinal ligament (C) Ligamentum flavum (D) Interspinous ligament (E) Intertransverse ligament

A: Anterior longitudinal ligament. The anterior longitudinal ligament is a vertical connective tissue band that attaches along the anterior aspects of the vertebral bodies. Its peripheral fibers have strong attachments to the intervertebral discs. The anterior longitudinal ligament resists hyperextension of the vertebral column. However, in this patient, the extreme forces involved with the hyperextension of the neck overpowers the resistance of this ligament, rupturing it as well as displacing part of the C4-5 intervertebral disc. In the given T2-weighted MRI, the anterior longitudinal ligament is represented by a hypointense (dark band) signal located anterior to the vertebral column. However, the locations where the anterior longitudinal ligament is interrupted appear as an abnormal hyperintense (white) signal, which is evident anterior to the C5 vertebral body. Choice B (Posterior longitudinal ligament) is incorrect. The posterior longitudinal ligament runs vertically along the posterior aspect of the vertebral column, mirroring the position of the anterior longitudinal ligament located along the anterior aspect of the vertebral column. The posterior longitudinal ligament resists flexion of the vertebral column. Posterolateral herniation of the gelatinous nucleus pulposus through the anulus fibrosus of an intervertebral disc most often projects lateral to the strong attachment sites of the posterior longitudinal ligament. If the herniated disc compresses spinal nerve roots, then neck, back, and/or limb pain may be present. The T2-weighted MRI clearly shows disruption of the anterior longitudinal ligament, evident by the abnormal hyperintense (white) signal located anterior to the C5 vertebral body. Choice C (Ligamentum flavum) is incorrect. The ligamenta flava (L: yellow ligament) are paired ligaments of yellow elastic fibrous tissue, which bind together the laminae of adjoining vertebrae and form the posterior wall of the vertebral canal. Because these ligaments resist flexion of the vertebral column, it is unlikely the ligamenta flava were damaged in this hyperextension injury of the neck. Choice D (Interspinous ligament) is incorrect. The interspinous ligament is composed of fibrous bands that connect the spinous processes of adjacent vertebrae. Because these ligaments resist flexion of the vertebral column, it is unlikely the interspinous ligaments were damaged in this hyperextension injury of the neck. Choice E (Intertransverse ligament) is incorrect. The intertransverse ligament is one ligament that connects the transverse processes of adjacent vertebrae. Because the intertransverse ligaments resist contralateral bend- ing (abduction; lateral flexion) of the vertebrae, it is unlikely the intertransverse ligaments were damaged in this hyperextension injury of the neck.

An anesthesiologist administers epidural anesthetic immediately lateral to the spinous processes of vertebrae L3 and L4 of a pregnant woman in labor. During this procedure, what would be the last ligament perforated by the needle in order to access the epidural space? (A) Ligamentum flavum (B) Anterior longitudinal ligament (C) Posterior longitudinal ligament (D) Interspinous ligament (E) Intertransverse ligament

A: Ligamentum flavum. The ligamentum fla- vum (L: yellow ligament) consists of yellow elastic fibrous tissue, which binds together the laminae of adjoining verte- brae and forms the posterior wall of the vertebral canal. Dur- ing proper administration of an epidural anesthetic, the needle will pass (in order) through the supraspinous ligament overly- ing the spinous processes of the lumbar vertebrae, the inters- pinous ligament connecting the spinous processes of adjacent vertebrae, and finally the ligamentum flavum, which stretches between the laminae of adjacent vertebrae. Due to its high elastic fiber content and the usual placement of the pregnant woman into the fetal position, the anesthesiologist will feel substantial resistance before the needle passes through the ligamentum flavum and potentially an audible "pop" when it is penetrated. In the given diagram, the epidural space resides between the ligamentum flavum and the posterior longitudi- nal ligament. Remember that the ligamentum flavum is the last ligament the needle penetrates during administration of an epidural anesthetic. Choice B (Anterior longitudinal liga- ment) is incorrect. The anterior longitudinal ligament is a vertical connective tissue band that attaches along the ante- rior sides of the vertebral bodies, as noted in the given figure. During proper administration of an epidural anesthetic, the needle will pass (in order) through the supraspinous ligament, the interspinous ligament, and finally the ligamentum flavum. Due to its position residing anterior to the vertebral bodies, the anterior longitudinal ligament will not be penetrated by the needle during proper administration of an epidural anes- thetic. Choice C (Posterior longitudinal ligament) is incorrect. The posterior longitudinal ligament runs vertically along the posterior aspect of the vertebral column, as noted in the given figure. Due to its position residing posterior to the vertebral bodies, the posterior longitudinal ligament will not be pene- trated during proper administration of an epidural anesthetic. Choice D (Interspinous ligament) is incorrect. The inters- pinous ligament is composed of fibrous bands that connect the spinous processes of adjacent vertebrae, as noted in the given figure. This ligament is penetrated by the needle dur- ing proper administration of an epidural anesthetic; however, the ligamentum flavum is the last ligament penetrated by the needle to reach the epidural space. Choice E (Intertransverse ligament) is incorrect. The intertransverse ligament is one ligament that connects the transverse processes of adjacent vertebrae. Because the intertransverse ligaments do not lie in the median sagittal plane, these ligaments would not be penetrated by the needle during administration of an epidural anesthetic and cannot be appreciated on the given figure.

An infant suffers a vertebral malformation in which the nucleus pulposus component of multiple intervertebral discs is hypertrophied. From what embryonic structure is the hypertrophied structure derived? (A) Notochord (B) Dermatome (C) Myotome (D) Sclerotome (E) Neural crest

A: Notochord. The nucleus pulposus is the sole remnant of the embryonic notochord, the initial longitudinal skeletal axis of the body. The developing bodies and intervertebral discs of the vertebral column replace the notochord, except for the nucleus pulposus. Remember that each intervertebral disc is composed of two parts: the central nucleus pulposus and the peripheral anulus fibrosus. Choice B (Dermatome) is incorrect. Each embryonic somite differentiates into three components: dermatome, myotome, and sclerotome. The dermatome contributes to the formation of the dermis, most of which is derived from the somatic layer of the lateral plate mesoderm. The nucleus pulposus component of an intervertebral disc is derived from the notochord, not the dermatome. Choice C (Myotome) is incorrect. Each embryonic somite differentiates into three components: myotome, dermatome, and sclerotome. The myotomes represent the muscle generating components of the somites. Remember, each myotome splits into two parts: epimere and hypomere. The epimere gives rise to the deep group of muscles in the back (intrinsic back muscles or epaxial muscles), which develop dorsal to the incipient vertebral column and are innervated by the posterior (dorsal) primary rami. The hypomere gives rise to muscles located lateral and anterior to the vertebral axis, which are innervated by the anterior (ventral) primary rami of spinal nerves. The nucleus pulposus component of an intervertebral disc is derived from the notochord, not the myotome. Choice D (Sclerotome) is incorrect. The sclerotomes give rise to most of the vertebral column, including the vertebrae, part of the occipital bone, and the anulus fibrosus portion of the intervertebral discs. The nucleus pulposus component of an intervertebral disc is derived from the notochord, not the sclerotome. Choice E (Neural crest) is incorrect. Most of the vertebral column and immediately surrounding tissues are derived from mesoderm. However, neural crest cells do migrate into this area, where they form spinal (dorsal root) ganglia and autonomic ganglia. The nucleus pulposus component of an intervertebral disc is derived from the notochord, not the neural crest cells.

After moving his parents' heavy furniture out of their house, a 38-year-old man experiences lower back pain and presents to his doctor with tingling numbness radiating down his right lower limb to the lateral part of his foot. A thorough physical examination reveals weakness when standing on the toes of his right foot. A T1-weighted MRI revealed an intervertebral disc herniation, as indicated by the white arrow. Which of the following structures is most likely impinged by this disc herniation? (A) S1 spinal nerve roots (B) S1 anterior primary ramus (C) L5 spinal nerve roots (D) L5 posterior primary ramus (E) L5 anterior primary ramus

A: S1 spinal nerve roots. Approximately 90% of intervertebral disc herniations occur toward the bottom of the spine at the L4-5 or L5-S1 segments. This T1-weighted MRI reveals posterolateral herniation of the nucleus pulposus of the L5-S1 intervertebral disc, as indicated by the white arrow. Remember that a herniated nucleus pulposus may displace or disrupt the posterior longitudinal ligament and extend into the vertebral canal and impinge upon spinal nerve roots either within the vertebral canal or as they traverse the interverte- bral foramen (or both). When dealing with such cases in the lower lumbar regions, remember the formula "N+1." That is, N = the number of the intervertebral disc; +1 = the number of the spinal nerve roots primarily contacted by the herniation. Thus, the herniated L5-S1 intervertebral disc would impinge the S1 nerve roots, which resulted in the lower back pain, tingling numbness radiating down his right lower limb to the lateral part of his foot, and weakness when standing on the toes of his right foot. Choice B (S1 anterior primary ramus) is incorrect. The anterior (ventral) and posterior (dorsal) pri- mary rami are the first branches of the spinal nerves, located outside the vertebral canal. Regardless of the spinal segmen- tal level, these structures are not impinged by a herniation of the nucleus pulposus of the intervertebral disc. Choice C (L5 spinal nerve roots) is incorrect. Because of the acute angles that the lower lumbar nerve roots take in exiting the vertebral canal via the intervertebral foramina, the L5 spinal nerve roots would typically be held against the upper pedicle of L5, above the level of the L5-S1 disc. Thus, the L5 roots would not be impinged by a bulging L5-S1 disc. Therefore, impingement of the S1 nerve roots is a more likely scenario. Choice D (L5 posterior primary ramus) is incorrect. The ante- rior (ventral) and posterior (dorsal) primary rami are the first branches of the spinal nerves, located outside the vertebral canal. Regardless of the spinal segmental level, these structures are not impinged by a herniation of the nucleus pulposus of the intervertebral disc. Choice E (L5 anterior primary ramus) is incorrect. The anterior (ventral) and posterior (dorsal) pri- mary rami are the first branches of the spinal nerves, located outside the vertebral canal. Regardless of the spinal segmental level, these structures are not impinged by a herniation of the nucleus pulposus of the intervertebral disc.

A physician orders a lumbar puncture (spinal tap) for his 43-year-old female patient in order to obtain a sample of cerebrospinal fluid (CSF). He explains to her that this procedure will be done in the lower back, between the spinous processes of the L3 and L4 vertebrae. What is the best reason for performing the lumbar puncture at this location? (A) The medullary cone ends at or above the L3 level (B) The subarachnoid space ends at the L3 level (C) The intervertebral foramina at L3-4 are large and easy to penetrate (D) No vertebral venous plexuses exist below the L3 level (E) The ligamenta flava are absent below the L3 level

A: The medullary cone ends at or above the L3 level. The objective of a lumbar puncture is to enter (tap) the subarachnoid space and access the CSF. This procedure is performed using a long spinal needle. For several reasons, this procedure is best performed in the low lumbar region, between the spinous processes of the L3 and L4 (sometimes L4 and L5) vertebrae. The medullary cone (or conus medullaris) is the tapered terminal end of the spinal cord. In adults, the med- ullary cone is normally located within the T12-L3 vertebral levels. Thus, penetrating the vertebral canal and subarachnoid space below L3 is the "safe" place to go, in that the spinal needle should not penetrate the spinal cord. Choice B (The subarach- noid space ends at the L3 level) is incorrect. The subarachnoid space is the CSF-filled space between the arachnoid and pia mater layers of the meninges. The dura-arachnoid layers line the vertebral canal and end at the S2 level. The pia mater lines the exterior surface of the neural tube and continues distally to envelope the spinal nerve rootlets and roots. Because the spinal cord ends at approximately the L1 level, there is a large separation between the dura and arachnoid mater layers of the meninges and the pia mater below L1. Thus, the subarachnoid space is quite large below the medullary cone (conus medul- laris), providing a significant pool (the lumbar cistern) of CSF. Choice C (The intervertebral foramina at L3-4 are large and easy to penetrate) is incorrect. The intervertebral foramina in the lumbar region are large openings; however, the lumbar puncture does not occur at these locations. During a spinal tap, the spinal needle is inserted through an interlaminar space, on the posterior side of the vertebral column. These spaces are largest in the low lumbar spine, thus affording a relatively open path to the vertebral canal. Before a spinal tap is administered, the patient is often bent forward into the fetal position by flexing the spine, which expands the interlaminar spaces during the lumbar puncture. Choice D (No vertebral venous plexuses exist below the L3 level) is incorrect. The ver- tebral venous plexuses are an extensive network of valveless, interconnecting vessels running the entire length of the verte- bral column. The external vertebral venous plexus lies on the anterior aspect of the vertebral column and the posterior side of the vertebral arch. The internal vertebral venous plexus is a major constituent of the epidural space within the vertebral canal. A lumbar puncture must traverse both the external and internal plexuses to reach the subarachnoid space. Choice E (The ligamenta flava are absent below the L3 level) is incor- rect. The ligamenta flava are short ligaments that connect the anterior sides of adjacent vertebral laminae along the entire length of the vertebral column. The spinal needle must pierce a ligamentum flavum in order to enter the vertebral canal.

A baby boy is born with a combination of congenital defects: atrial septal defect, patent ductus arteriosus, hypoplasia of the nasal bridge, and pectus carinatum (pigeon chest). The 35-year-old mother had not come in for any prenatal visits before parturition, and her medical history reveals that she had a heart surgery to repair a mitral valve defect secondary to rheumatic heart disease. She has been taking anticoagulant throughout her pregnancy. Which of the following teratogens will most likely explain the baby's congenital defects? A. Heroin B. Warfarin C. Rubella D. Tetracycline E. Nicotine

B. Fetal alcohol syndrome (FAS) is associated with intrauterine growth restriction (IUGR), mental deficiency, microcephaly, ocular anomalies, joint abnormalities, and short palpebral fissures. Warfarin is associated with nasal hypoplasia, stippled epiphyses, hypoplastic phalanges, eye anomalies, and mental deficiency. Tetracycline is associated with stained teeth and hypoplasia of enamel. Rubella virus is associated with various anomalies like IUGR, postnatal growth retardation, cardiac and great vessel abnormalities, microcephaly, sensorineural deafness, cataract, microphthalmos, glaucoma, pigmented retinopathy, mental deficiency, neonate bleeding, hepatosplenomegaly, osteopathy, and tooth defects. Women who use heroin during pregnancy greatly increase their risk of serious pregnancy complications. These risks include poor fetal growth, premature rupture of the membranes, premature birth, and stillbirth. 304-307

Atropine is a drug that acts to block stimulation of the receptors targeted by postsynaptic parasympathetic neurons. When it takes effect, atropine acts to do which of the following? A. paralyze the diaphragm B. stimulate bronchoconstriction C. increase sweat gland secretions D. paralyze the intercostal muscles E. increase heart rate

E. increase heart rate

A 29-year-old, woman in her 27th week of pregnancy visits her obstetrician for a follow-up appointment. Ultrasound from a previous visit revealed a 4-cm mass growing at the base of the spine. What is the most likely diagnosis? A. Incomplete closure of the embryonic neural tube (lumbosacral myelomeningocele) B. Remnants of the primitive streak (sacrococcygeal teratoma) C. Swelling or growth of the endothelial cells that line blood vessels (hemangioma) D. Neuroendocrine tumor arising from any neural crest element of the sympathetic nervous system (neuroblastoma) E. Benign nerve sheath tumor of the peripheral nervous system (neurofibroma)

B. Remnants of the primitive streak (a thick linear band of epiblast at the beginning of week 3) may persist and give rise to a large tumor known as a sacrococcygeal teratoma. Because it is derived from pluripotent primitive streak cells, the tumor contains tissues derived from all three germ layers in incomplete stages of differentiation. Sacrococcygeal teratomas are the most common tumors in newborn infants and have an incidence of approximately 1 in 27,000. These tumors are usually surgically excised promptly and the prognosis is good.

A 29-year-old, woman in her 27th week of pregnancy visits her obstetrician for a follow-up appointment. Ultrasound from a previous visit revealed a 4-cm mass growing at the base of the spine. What is the most likely diagnosis? A. Incomplete closure of the embryonic neural tube (lumbosacral myelomeningocele) B. Remnants of the primitive streak (sacrococcygeal teratoma) C. Swelling or growth of the endothelial cells that line blood vessels (hemangioma) D. Neuroendocrine tumor arising from any neural crest element of the sympathetic nervous system (neuroblastoma) E. Benign nerve sheath tumor of the peripheral nervous system (neurofibroma)

B. Remnants of the primitive streak (a thick linear band of epiblast at the beginning of week 3) may persist and give rise to a large tumor known as a sacrococcygeal teratoma. Because it is derived from pluripotent primitive streak cells, the tumor contains tissues derived from all three germ layers in incomplete stages of differentiation. Sacrococcygeal teratomas are the most common tumors in newborn infants and have an incidence of approximately 1 in 27,000. These tumors are usually surgically excised promptly and the prognosis is good (Fig. 8-2). 45, 47

A 3-year-old girl is brought to the emergency department for the fourth time in a year. The doctor suspects child abuse. Before social services is called, a genetics test is ordered and reveals that the patient has a mutation in the COL1A1 gene. In addition to bone fragility and short stature, physical exam may discover which of the following? A. Pigeon chest B. Short limbs C. Yellow sclera D. Ruptured aorta E. Stiff joints

B. The COL1A1 gene produces a component of type I collagen called the pro-α1(I) chain. Mutation of the gene leads to reduced production of pro-α1(I) chains and future reduced type I collagen. A shortage of this critical protein underlies the bone fragility and other characteristic features of osteogenesis imperfecta. Osteogenesis imperfecta has 4 types (I, II, III, and IV). In addition to more severe bone problems (fractures), features of these conditions can include blue sclerae, hearing loss, short stature, respiratory problems, and a disorder of tooth development. Discoloration of the sclera gives the appearance of a blue or blue gray color. This is due to the thinning of the sclera caused by a defective type I collagen; as a result the choroidal veins are evident, giving the appearance of a blue color. 225-229

A 55-year-old man visits his primary care provider complaining that he can no longer go bike riding without getting breathless and having chest pains. Upon examination, the physician detects a heart murmur. Echocardiography confirms aortic stenosis. What is the embryologic origin of the aorta? A. Bulbus cordis B. Truncus arteriosus C. Sinus venosus D. Ductus arteriosus E. Ductus venosus

B. The arch of the aorta develops from the truncus arteriosus, which is the cranial part of the primitive heart tube. It also gives rise to the pulmonary trunk. The bulbus cordis develops into the conus arteriosus of the right ventricle. The sinus venosus has two horns: a right horn that develops into the smooth sinus venarum of the right atrium and a left horn that develops into the coronary sinus. The ductus arteriosus connects the fetal pulmonary trunk to the aorta allowing for blood to bypass the pulmonary circulation. The ductus venosus connects the umbilical vein to the inferior vena cava allowing blood to bypass the hepatic circulation in the fetus. 204-205, 211-214

A female newborn was found to have a large midline tumor in the lower sacral area, which was diagnosed as a sacrococcygeal tumor. Which of the following courses of treatment is recommended for this child? (A) Immediate chemotherapy and radiation treatment (B) Surgical removal of the tumor by age 6 months (C) Surgical removal of the tumor at age 4-5 years (D) Surgical removal of the tumor at age 13-15 years (E) No treatment because this tumor normally regresses with age

B. The preponderance of sacrococcygeal tumors is found in female newborns. Since these tumors develop from pluripotent cells of primitive streak origin, malignancy is of great concern, and the tumor should be surgically removed by age 6 months. Occasionally, these tumors may recur after surgery, demonstrating malignant properties.

A 19-year-old girl gives birth to a baby girl diagnosed with a meningomyelocele, as seen in the given midsagittal MRI. The mother lived in a rural area and did not have prenatal visits with her obstetrician. Which of the following drugs, if taken in the proper doses during the periconceptional period, would have greatly reduced the chances of her fetus having a neural tube defect (NTD)? (A) Valproic acid (Depakote) (B) Folic acid (Folacin) (C) Cephalexin (Keflex) (D) Acetaminophen (Tylenol) (E) Famotidine (Pepcid)

B: Folic acid (Folacin). Supplementing a diet with folic acid (vitamin B9 or folacin) has been shown to reduce the incidence of NTDs up to 70%. Neural tube clo- sure occurs within the first 28 days after conception, and at this time in the pregnancy, many mothers do not realize they are pregnant. Women anticipating getting pregnant should take 400 mg of folic acid daily beginning 3 months prior to conception and continue the folic acid supplement throughout their pregnancy. NTD prevention is best achieved by adequate daily folic acid intake throughout the reproductive years. In this neonate, the sagittal MRI revealed a neural tube defect called a meningomyelocele, so the defect in the cervical region involves the meninges (meningo-), the spinal cord (myelo-) and the membranous sac (-cele). A meningo- myelocele is the most common form of spina bifida cystica. Choice A (Valproic acid [Depakote]) is incorrect. Valproic acid (brand name Depakote) is an anticonvulsant and mood-stabilizing drug used in the treatment of bipolar disorder, depres- sion, and epilepsy. Exposure to Valproic acid, during the first month of pregnancy, has been shown to have a teratogenic effect on the fetus, including increasing the incidence of neural tube defects. Choice C (Cephalexin [Keflex]) is incorrect. Cephalexin (brand name Keflex) is an antibiotic used to fight bacterial infections, including upper respiratory, ear, skin, and urinary tract infections. Exposure to Cephalexin, during the first month of pregnancy, has been shown to have a teratogenic effect on the fetus, including increasing the incidence of neural tube defects. Choice D (Acetaminophen [Tylenol]) is incorrect. Acetaminophen (brand name Tylenol) is an analgesic (or pain reliever) and antipyretic (or fever reducer). Exposure to Acetaminophen, during the first month of pregnancy, has been shown to have a teratogenic effect on the fetus, including increasing the incidence of neural tube defects. Choice E (Famotidine [Pepcid]) is incorrect. Famotidine (brand name Pepcid) is a histamine H2-receptor antagonist that inhibits stomach acid production, and it is often used to treat gastroesophageal reflux disease (GERD). Exposure to Famotidine, during the first month of pregnancy, has been shown to have a teratogenic effect on the fetus, including increasing the incidence of neural tube defects.

During a head-on motor vehicular accident (MVA), the upper cervical vertebrae of a 34-year-old woman are flexed violently forward until her head impacts the steering wheel and is thrown into hyperextension and slight rotation. She is immobilized and brought to an ER. The given sagittal CT of her upper cervical vertebrae is viewed by a radiologist and reveals which of the following diagnoses? (A) Traumatic spondylolisthesis of C2 (B) Fracture of dens axis (odontoid process) (C) Atlanto-axial subluxation (D) Ruptured intervertebral disc between C1 and C2 (E) No pathology is apparent on the CT scan

B: Fracture of the dens axis (odontoid process).The sagittal CT clearly shows a Type II dens axis (odontoid process) fracture, which is the most common type of fracture (approximately 60% of cases) involving the dens. This fracture occurs at the base of the dens where it extends superiorly from the axis (or second cervical vertebra). The precise mechanism for odontoid fractures is unknown, but it most likely includes a combination of flexion, extension, and rotation. In this case, the patient's cervical spine was forced into a flexed position by the impact of the crash and then a hyperextended position with slight rotation due to the impact of her head on the steering wheel. The forces involved fractured the odontoid process at its base, but luckily, the fractured dens axis does not extend poste- rior to impinge upon the spinal cord. Patients with this type of odontoid fracture are placed in halo immobilization or undergo internal fixation (or odontoid screw fixation) to reattach the fractured axis dens to the body of the axis. Remember Steele's Rule of Thirds, which states one third of the atlas (C1) ring is occupied by the dens axis, one third by the spinal cord, and one third by the fluid-filled space and surrounding tissues of the cord. This extra space within the atlas ring explains why people with odontoid fractures may not have spinal cord injuries, but they will often feel unstable when they move their head sud- denly. The incidence of odontoid fractures approaches 15% of all cervical spine fractures. Choice A (Traumatic spondylolis- thesis of C2) is incorrect. Spondylolisthesis describes the ante- rior displacement of a vertebra in relation to the vertebra below it. Its meaning is derived from the Greek words "spondyl" (G: vertebrae) and "olisthesis" (G: slip). One of the most com- mon injuries of the cervical vertebrae involves the fracture of the vertebral arch of the axis (or 2nd cervical vertebra). The fracture usually occurs in the pars interarticularis or the bony column formed by the superior and inferior articular pro- cesses of the axis, and it is called a traumatic spondylolisthesis (or defect in the pars interarticularis) of C2 or hangman frac- ture. The precise mechanism for traumatic spondylolisthesis of C2 is the hyperextension of the head on the neck, not the combination of hyperflexion, rotation, and hyperextension of the head and neck seen in this MVA. It receives its alternative name, hangman fracture, because it is often seen in criminals executed by hanging due to the hyperextension of the head due to the placement of the noose under their chin. A sag- ittal CT would not be the best means to view a hangman fracture because the vertebral arch of the axis is not seen. A lateral cervical X-ray would be a better imaging technique to visualize a traumatic spondylolisthesis of C2 (or hangman fracture). Choice C (Atlanto-axial subluxation) is incorrect. Atlanto-axial subluxation, or the incomplete dislocation of the median atlanto-axial joint, occurs following the rupture of the transverse ligament of the atlas, which holds the dens axis (odontoid process) in place. Losing the integrity of the transverse ligament of the atlas can result in compression of the upper cervical spinal cord, leading to quadriplegia (paralysis of all four limbs) and even death (if the medulla of the brainstem is compressed). Atlanto-axial subluxation is prevalent in patients with Down syndrome (trisomy 21) but it can occur with other connective tissue disorders. In this patient, a fracture at the base of the odontoid process of the axis was noted on the sagittal CT scan. This fracture fragment is held in place by an intact transverse ligament of the atlas, so atlanto-axial subluxation did not occur in this patient. Choice D (Ruptured intervertebral disc between C1 and C2) is incorrect. A ruptured intervertebral disc between C1 and C2 is not possible because no disc exists between these two vertebrae. The given sagittal CT clearly shows a dens axis (odontoid process) fracture. The space between the fractured odontoid process and the body of the axis could be misinterpreted as a translucent space for an interverte- bral disc. Knowledge of the proper anatomy of the atlanto- axial joint would eliminate this option. Remember that a CT scan is valuable in assessing bone injuries, so a herniated disc would be better seen with an MRI. An MRI would be the best means to view damage to the intervertebral discs, the spinal cord and its nerve roots, and the ligaments of the spine. Choice E (No pathology is apparent on the CT scan) is incorrect. The given sagittal CT clearly shows a dens axis (odontoid process) fracture. The space between the fractured odontoid process and the body of the axis could be misin- terpreted as a translucent space for an intervertebral disc; however, no disc exists in the atlanto-axial joint. Knowledge of the proper anatomy of the atlanto-axial joint would enableeasy elimination of this option."

A 34-year-old man fell 25 ft out of a barn loft, landing on his back. He was found unconscious and taken to the ER. The given sagittal CT reformat image reveals a T9 spinal fracture dislocation, as noted by the black arrow. The patient had unequivocal clinical findings indicating spinal cord transection. Given the results of this CT image and the clinical information, what deficits will the patient incur? (A) Quadriplegia and incontinence (B) Paraplegia and incontinence (C) Incontinenceonly (D) Loss of only sensory information below the lesion (E) Loss of only motor function below the lesion

B: Paraplegia and incontinence. Transection of the spinal cord results in loss of all motor and sensory functions movements that emerge from the cord inferior to the site of the lesion. General sensory, somatic motor, and visceral motor fibers for the pelvis and lower limbs would be lost inferior to the level of T9 due to the transection of the spinal cord. Also, parasympathetic outflow to the pelvis and perineum would be eliminated due to the origin of these nerve fibers from S2-4. The CT image reveals a T9 spinal fracture dislocation, and the clinical findings indicate a spinal cord transection. Therefore, this patient would present with paraplegia (or paralysis of both lower limbs), fecal and urinary incontinence, impotence, and loss of motor functions and sensation below the level of the T9 lesion, which is right above the level of the umbilicus (supplied by T10). Choice A (Quadriplegia and incontinence) is incorrect. A transection of the spinal cord at the level of T9 would spare the upper limbs from sensory and motor deficits because the upper limbs are supplied by the anterior rami of C5-T1. This patient would present with paraplegia (or paralysis of both lower limbs), fecal and urinary incontinence, impotence, and loss of motor functions and sensory input below the level of the T9 lesion. Choice C (Incontinence only) is incorrect. A transection of the spinal cord at the level of T9 would result in paraplegia (or paralysis of both lower limbs), fecal and urinary incontinence, impotence, and loss of motor functions and sensory input below the level of the T9 lesion. Though incontinence would result from this spinal cord transection at T9, this deficit would be accompanied by additional signs and symptoms due to the severity and location of this spinal cord injury. Choice D (Loss of only sensory information below the lesion) is incorrect. Transection of the spinal cord at T9 would result in the loss of all sensation and voluntary movements inferior to T9. Therefore, loss of only sensory information is not a viable option. Choice E (Loss of only motor function below the lesion) is incorrect. Transection of the spinal cord at T9 would result in the loss of all sensation and voluntary movements inferior to T9. Therefore, loss of only motor function is not a viable option in this patient.

An 81-year-old woman visits her family physician for an annual physical examination and complains of weakness and loss of sensation in her legs. Radiological studies show bony spurs (osteophytes) narrowing the intervertebral foramina at levels T12-L3. Which of the following structures is most likely to be impinged in this condition? (A) Posterior primary rami of spinal nerves (B) Spinal ganglia (C) Ganglia of the sympathetic trunk (D) Anterior primary rami of spinal nerves (E) Anterior rootlets of spinal nerves

B: Spinal ganglia. The posterior (dorsal) and anterior (ventral) roots of spinal nerves typically merge within the intervertebral foramina to form individual spinal nerves. Because the spinal (dorsal root) ganglia are located at the distal ends of the posterior roots, they are also normally found within the intervertebral foramina. Thus, narrowing (stenosis) or compression of the intervertebral foramina may cause impingement of the posterior and anterior roots, spinal nerves, and spinal ganglia due to the presence of bony spurs (osteophytes) in this patient. Choice A (Posterior primary rami of spinal nerves) is incorrect. Each spinal nerve divides into its first branches (primary rami) immediately after exiting the intervertebral foramen. Because the primary rami are located distal to the intervertebral foramina, the posterior primary rami would not be compressed by stenosis of the intervertebral foramina due to the presence of osteophytes in this patient. Remember that the posterior primary rami extend into the epaxial tissues to innervate the deep (intrinsic; epaxial) muscles of the back. Choice C (Ganglia of the sympathetic trunk) is incorrect. The sympathetic trunk lies along the lateral side of the vertebral column, outside the intervertebral foramina. Thus, the gan- glia of the sympathetic trunk would not be impinged by the osteophytes found in this patient. Choice D (Anterior primary rami of spinal nerves) is incorrect. Each spinal nerve divides into its first branches (primary rami) immediately after exiting the intervertebral foramen. Because the primary rami are located distal to the intervertebral foramina, the anterior primary rami would not be compressed by stenosis of the inter- vertebral foramina. Remember, the anterior (ventral) primary rami extend into the hypaxial tissues to innervate the superficial and intermediate (extrinsic; hypaxial) muscles of the back. Choice E (Anterior rootlets of spinal nerves) is incorrect. The anterior (ventral) rootlets of spinal nerves are located proximal to the intervertebral foramina, close to the spinal cord, and would not be impinged by a stenosis of the intervertebral foramina due to the presence of osteophytes.

A 25-year-old medical student feels a sharp pain in his lower back while helping his landlady move a piano. After 3 days, the pain does not subside, so he receives a comprehensive examination from his physician that reveals a compression of the left side L3 spinal nerve. Which of the following is the most likely cause of this condition? (A) The pedicles of L2 and L3 are compressed together (B) The pedicles of L3 and L4 are compressed together (C) The laminae of L2 and L3 are compressed together (D) The laminae of L3 and L4 are compressed together (E) The spinous processes of L3 and L4 are compressed together

B: The pedicles of L3 and L4 are compressed together. Spinal nerves exit the vertebral canal through the intervertebral foramina on the lateral sides of the vertebral column. Each intervertebral foramen is formed by the juxtaposing of the pedicles of successive articulated vertebrae. Thus, if the pedicles are compressed together, the interver- tebral foramen is narrowed and a spinal nerve may be com- pressed ("pinched") as it traverses the intervertebral foramen. In the thoracic, lumbar, and sacral regions, each numbered segmental nerve passes through the intervertebral foramen below the matched numbered vertebra. Thus, the L3 spinal nerve traverses the intervertebral foramen below the L3 ver- tebra, between the L3 and L4 vertebrae. Because there is one more cervical spinal nerve (N = 8) than cervical vertebrae (N = 7), the cervical nerves exit the intervertebral foramina above the matching numbered vertebrae. Thus, the C3 spinal nerve passes through the opening between vertebrae C2 and C3. In this patient, the damage to the L3 spinal nerve on the left side is due to narrowing the intervertebral foramen caused by the compression of the pedicles of L2 and L3. Choice A (The pedicles of L2 and L3 are compressed together) is incor- rect. When the pedicles of L2 and L3 are compressed together, the result is stenosis of the L2 intervertebral foramen. This nar- rowing would damage the ipsilateral L2 spinal nerve, result- ing in lower back pain and potentially lower limb deficits. In this patient, damage to the L3 spinal nerve on the left side was reported, so compression of the L2 and L3 pedicles is not likely. Choice C (The laminae of L2 and L3 are compressed together) is incorrect. Compression of the interlaminar spaces would not affect the spinal nerves, as they do not pass through these openings. In fact, such compression would cause a lever arm reaction that would expand the intervertebral foram- ina. Choice D (The laminae of L3 and L4 are compressed together) is incorrect. Compression of the interlaminar spaces would not affect the spinal nerves, as they do not pass through these openings. In fact, such compression would cause a lever arm reaction that would expand the intervertebral foramina. Choice E (The spinous processes of L3 and L4 are compressed together) is incorrect. Compression of the spinous processes does not affect the spinal nerves because they do not pass through these gaps. Again, a compression of the L3 and L4 spinous processes would expand the intervertebral foramina because of a lever arm reaction.

An internal medicine attending physician asks a medical student to place the bell of her stethoscope on the triangle of auscultation to hear a patient's breathing sounds. Which of the following structures make up the boundaries of this triangle? (A) Deltoid, latissimus dorsi, scapula (B) Trapezius, scapula, latissimus dorsi (C) Latissimus dorsi, ilium, external abdominal oblique (D) Rhomboids, levator scapulae, splenius capitis (E) Longissimus, rhomboids, vertebral spinous processes

B: Trapezius, scapula, latissimus dorsi. The triangle of auscultation is a space in the back bounded by the lateral edge of the trapezius muscle, medial border of the scapula, and upper edge of the latissimus dorsi muscle. Placing a stethoscope within the triangle of auscultation enables the medical student to hear breathing sounds because (1) this site offers a gap between layers of bone and muscle, and (2) the location lies directly over the midposterior chest wall and the lung, which is ideal for auscultation. Choice A (Deltoid, latissimus dorsi, scapula) is incorrect. A triangle marked by these boundaries is located over the posterior aspect of the axilla. A medical student would be unable to hear breathing sounds of the lungs at this location. Choice C (Latissimus dorsi, ilium, external abdominal oblique) is incorrect. The lumbar triangle is the space bounded by the lower edge of the latissimus dorsi muscle, the iliac crest of the coxal (hip) bone, and the posterior border of the external abdominal oblique muscle. An abdominal hernia may protrude through the lumbar triangle, a weak spot in the abdominal wall. The lumbar triangle also offers a surgical pathway through the back for access to the kidney. Choice D (Rhomboids, levator scapulae, splenius capitis) is incorrect. The intermuscular space marked by these muscles is located at the base of the neck. A medical student would be unable to hear breathing sounds of the lungs at this location. Choice E (Longissimus, rhomboids, vertebral spinous processes) is incorrect. These boundaries form a site just off the dorsal midline. Listening for breathing sounds of the lungs at this site is not optimal.

A researcher is studying the developmental origins of the arteries that supply blood to the brain. The investigation involves injecting contrast media into the hearts of fetal chicken eggs and studying the arteries that develop in the pharyngeal arches because blood vessels that develop bilaterally in several of these structures supply the cerebral arterial circle (of Willis). Which of the following pairs of pharyngeal arches most likely gives rise to arteries that contribute to the blood vessels in the circle of Willis? A. Pharyngeal arches 1 and 2 B. Pharyngeal arches 2 and 3 C. Pharyngeal arches 3 and 4 D. Pharyngeal arches 4 and 5 E. Pharyngeal arches 4 and 6

C. Pharyngeal arches 3 and 4 give rise to the common carotid artery, internal carotid artery, the aortic arch, and subclavian artery. The cerebral circle of Willis is formed by arteries that take their origin from these structures. Arches 1 and 2 give rise to the maxillary artery, external carotid artery, stapedial artery, and hyoid artery, which do not contribute to the cerebral circle of Willis. Although Arch 3 participates in the formation of the circle of Willis, second arch structures do not. Arch 5 is rudimentary (if present) and has no derivatives. Arch 6 gives rise to the aortic arches, pulmonary artery, and ductus arteriosus but does not contribute to the circle of Willis (Fig. 8-3). 104, 211-212

A 4-hour-old infant boy with respiratory distress is examined in the NICU. His symptoms are alleviated when placed in the prone position. Which of the following will most likely explain his symptoms? A. Down's syndrome B. Treacher Collins syndrome C. Pierre Robin sequence D. Patent ductus arteriosus E. Bronchopulmonary dysplasia

C. Pierre Robin sequence (also called Robin sequence) is an autosomal recessive congenital disorder due to failure of neural crest cells to migrate to the first arch. It is initiated by a hypoplastic mandible, which results in posterior displacement of the tongue and obstruction to the closure of the palatine plates leading to bilateral cleft palate. Respiratory symptoms are common in patients with Robin sequence particularly when the baby is placed on his/her back (supine). The small, recessed mandible (micrognathia) causes the tongue to fall backward and obstruct the airway when babies are placed in the prone position. Down's syndrome is a result of trisomy 21; patients present with slanted palpebral fissures, cardiac anomalies, and impaired cognitive function. Although Treacher Collins is also a first arch syndrome, it also includes other craniofacial anomalies, disfigured external ears, and impaired hearing. Patent ductus arteriosus is a cardiac anomaly due to failure of closure of the ductus arteriosus at birth that usually presents with a holosystolic murmur. Bronchopulmonary dysplasia is a respiratory syndrome seen in newborns and characterized by lung inflammation and scarring. Patients will have respiratory distress when lying in any position (Fig. 8-6). 108

A 6-year-old child was brought into the emergency department with vomiting and abdominal pain. He complains that pain is radiating down his back and is worsened when he lays flat on his back. Physical exam reveals abdominal tenderness. Lab results show elevated serum pancreatic enzymes. Abdominal CT scan shows a pancreas containing two separate duct systems emptying into the duodenum. The main pancreatic duct is very short and drains only a small portion of the head of the pancreas, while the remainder of the pancreas is drained via the other duct. The GI specialist diagnosed the patient with pancreas divisum. Which of the following accurately describes the embryologic anomaly? A. Formation of a bifid ventral bud B. Formation of a bifid dorsal bud C. Failure of the dorsal and ventral buds to fuse D. Fusion of the dorsal and bifid ventral bud E. Nonrotation of the midgut

C. The dorsal and ventral buds normally fuse to form the adult pancreas, the ventral duct although smaller becomes the terminal portion of the main duct. A bifid ventral bud, when fusing to the dorsal bud, typically leads to annular pancreas, which is a band of pancreatic tissue that wraps around the duodenum. Nonrotation of the midgut would result in displacement of the entire pancreas. 144

A 40-year-old alcoholic man visits his physician's office for the fourth time this year complaining of abdominal and back pain. Abdominal CT scan shows a pancreas containing two separate duct systems emptying into the duodenum. The main pancreatic duct is very short and drains only a small portion of the head of the pancreas, while the remainder of the pancreas is drained via the other duct. The gastrointestinal (GI) specialist diagnosed the patient with pancreas divisum. Which of the following accurately describes the embryologic abnormality? A. Formation of a bifid ventral bud B. Formation of a bifid dorsal bud C. Failure of the dorsal and ventral buds to fuse D. Fusion of the dorsal and bifid ventral bud E. Nonrotation of the midgut

C. The dorsal and ventral buds normally fuse to form the adult pancreas. The ventral duct, although smaller, becomes the terminal portion of the main duct. A bifid ventral bud, when fusing to the dorsal bud, typically leads to the formation of an annular pancreas, which is a band of pancreatic tissue that wraps around the duodenum. Nonrotation of the midgut would result in displacement of the entire pancreas. 144

Individual blastomeres were isolated from a blastula at the 4-cell stage. Each blastomere was cultured in vitro to the blastocyst stage and individually implanted into four pseudopregnant foster mothers. Which of the following would you expect to observe 9 months later? (A) Birth of one baby (B) Birth of four genetically different babies (C) Birth of four genetically identical babies (D) Birth of four grotesquely deformed babies (E) No births

C. This scenario would result in four genetically identical children. Blastomeres at the 4-cell to 8-cell stage are totipotent, that is, capable of forming an entire embryo. Since blastomeres arise by mitosis of the same cell (zygote), they are genetically identical. This phenomenon is important in explaining monozygotic (identical) twins. About 30% of monozygotic twins arise by early separation of blastomeres. The remaining 70% originate at the end of week 1 of development by a splitting of the inner cell mass.

One finding during autopsy of a newborn infant that died soon after birth is a significant ventricular septal defect at the upper end of the interventricular septum, as shown in the given photo. This malformation may have been related to maldevelopment of which of the following embryonic structures? A. septum secundum B. bulbus cordis C. endocardial cushions D. truncus arteriosus E. sinus venosus

C. endocardial cushions

A 27-year-old woman gives birth to a baby girl at term. The pathologist examines the placenta after the birth. Which of the following placental components is derived from the mother rather than the fetus? Select one: a. Chorion b. Cytotrophoblast c. Lacunar network d. Primary villus e. Syncytiotrophoblast

C. lacunar network. The correct answer is Lacunar network. The placenta is derived from both maternal and fetal tissues. The mother produces the capillary bed that forms the lacunar network of the placenta. The fetus produces both the cytotrophoblast and the syncytiotrophoblast, which together form the chorion. The earliest projections of this are called the primary villi. (Choice A). The fetus produces both the cytotrophoblast and the syncytiotrophoblast, which together form the chorion. (Choice B) The fetus produces the cytotrophoblast. (Choice D).The earliest projections of the chorion are called the primary villi. (Choice E) The fetus produces the syncytiotrophoblast.

A male baby is born with Down syndrome (trisomy 21) and associated cardiac defects. The given apical four-chamber view echocardiogram shows a complete atrioventricular canal defect that includes a primum atrial septal defect and a posterior inlet ventricular septal defect. What embryonic structures normally fuse to close the foramen primum? A. septum secundum and the septum primum B. septum secundum and the fused endocardial cushions C. septum primum and the fused endocardial cushions D. septum primum and the septum spurium E. left inferior and right superior truncus swellings

C. septum primum and the fused endocardial cushions

A 55-year-old man experiences severe lower back pain for 4 days after moving heavy furniture. After a thorough back evaluation and review of associated radiological imaging, a surgeon removes the herniated portion of the L3-4 intervertebral (IV) disc. What type of cartilage most likely gives the tensile strength of the disc extracted from this patient? (A) Hyaline (B) Elastic (C) Fibrous (D) Epiphysial (E) Anular

C: Fibrous. Fibrous cartilage, or fibrocar- tilage, is one of the three types of cartilage composed of chondrocytes that secrete, and are surrounded by, an exten- sive extracellular matrix composed of collagen fibers, elastin fibers, and a plethora of ground substance rich in proteo- glycan. Fibrocartilage is a very tough, white material that provides high tensile strength to the anulus fibrosus of the IV discs, which would have ruptured in this patient. The fibrocartilage of the anulus fibrosus is composed of a dense network of Type I collagen, but it can tear due to sudden hyperflexion in older individuals (such as when lowering heavy objects), which pushes the gelatinous nucleus pul- posus of the IV disc posterior toward the thinnest part of the anulus fibrosus. When the fibrocartilage of the anulus fibrosus is compromised, the nucleus pulposus may her- niate into the vertebral canal and compress spinal nerve roots or the spinal cord, leading to back pain. Fibrocar- tilage, or fibrous cartilage, provides the tensile strength of the IV discs. Choice A (Hyaline) is incorrect. Hyaline cartilage is one of the three types of cartilage composed of chondrocytes that secrete, and are surrounded by, an extensive extracellular matrix composed of collagen fibers, elastin fibers, and a plethora of ground substance rich in proteoglycan. Hyaline cartilage is a translucent, hard mate- rial rich in Type II collagen fibers and proteoglycan. Cov- ering the articular surfaces of bones, it is found in many joints. Hyaline cartilage is also found in the tracheal rings, larynx, and costal cartilages. In endochondral ossification, bone is formed from a hyaline cartilage intermediate. It is also found in maturing long bones within the epiphysial (or growth) plates. Hyaline cartilage does not provide the tensile strength of the IV discs. Choice B (Elastic) is incor- rect. Elastic cartilage is one of the three types of cartilage composed of chondrocytes that secrete, and are surrounded by, an extensive extracellular matrix composed of collagen fibers, elastin fibers, and a plethora of ground substance rich in proteoglycan. It is composed of large amounts of elas- tic fibers and Type II collagen, and it is found in the exter- nal ear (auricle), pharyngotympanic (auditory) tube, and epiglottis, where it provides a rigid but elastic framework. Elastic cartilage does not provide the tensile strength of the IV discs. Choice D (Epiphysial) is incorrect. In endochon- dral ossification, bone is formed from a hyaline cartilage intermediate. It is also found in maturing long bones and is called epiphysial cartilage, located within the epiphysial (growth) plates of bone. This new type of cartilage forms the resting zone of chondrocytes located on the epiphysial (distal) side of the zone of growing cartilage, which unites the epiphysis with the shaft of the long bone. Because the patient is 55 years old, the presence of epiphysial cartilage is unlikely. Moreover, fibrocartilage provides the tensile strength of the IV discs. Choice E (Anular) is incorrect. Anular cartilage is a synonym for the cricoid cartilage, the lowest portion of the laryngeal skeleton. Because the cricoid cartilage is shaped like a signet ring, with its arch located anterior and its widened lamina posterior, it is sometimes referred to as the anular cartilage. Because the anular cartilage is not a type of cartilage, it is unable to provide tensile strength to the IV disc and can easily be eliminated as a possible answer. Fibrocartilage provides the tensile strength of the IV discs.

A 45-year-old man goes to his family physician complaining of lower back pain after spending a weekend clearing trees off his property. During examination, the doctor notes a tuft of hair and a dimple on the skin of the patient's lower back. A plain film reveals a congenital defect in his L5 vertebra, indicated by the black arrow in the given X-ray. No other structural abnormalities are noted. Based on these findings, what is the most likely diagnosis for this patient? (A) Meningocele (B) Anencephaly (C) Spina bifida occulta (D) Spina bifida cystica (E) Spina bifida with myeloschisis

C: Spina bifida occulta. Spina bifida occulta is the mildest form of spina bifida (L: split spine). In this developmental disorder, the left and right neural arch elements fail to fuse completely in the dorsal midline. However, the split in the vertebra is so small that the meninges and elements of the spinal cord do not protrude through the defect. In the given plain film, the black arrow indicates a failure of fusion of the laminae of L5, producing a cleft. Individuals with spina bifida occulta may possess a tuft of hair and/or dimple in the skin overlying the affected vertebral levels, as noted in this patient. Other individuals with this condition may have no visible evidence, a lipoma, or even a birthmark in the overly- ing skin of the affected region. Some studies suggest approximately 10% of the general population have this mildest form of spina bifida. In this case, the X-ray verifies the diagnosis of spina bifida occulta. Because this condition is normally clinically asymptomatic and unnoticed, it is seemingly "hid- den" (occult). In fact, most research suggests no relationship between spina bifida occulta and back pain. Choice A (Meningocele) is incorrect. The least common form of spina bifida is a posterior meningocele (or meningeal cyst). In this develop- mental disorder, the bilateral neural arch elements fail to fuse completely in the dorsal midline but the meninges protrude through the defect into a sac or cyst. Therefore, multiple vertebral defects are present, accompanied with the presence of a cyst, which contains cerebrospinal fluid (CSF). In a meningocele, the spinal cord and nerve roots are typically in normal position, not protruding into the cyst, and there are usually no long-term effects on the individuals. Without the presence of a meningeal cyst in the lower back of this man, a meningocele can be easily eliminated. Choice B (Anencephaly) is incorrect. Anencephaly is a severe neural tube defect in which the cephalic (head) end of the neural tube fails to close in the embryo. This birth defect is lethal with the baby being born without a forebrain, skullcap (calvaria), and scalp, leaving the remaining portions of the brain exposed. If proper prenatal care is available, most cases of anencephaly are detected by elevated maternal serum alpha-fetoprotein (AFP) levels and ultrasound examinations during prenatal examinations. Elevated AFP levels are often correlated with neural tube defects. The addition of folic acid to the diet of women in their child- bearing years has been shown to reduce the incidence of neural tube anomalies. A patient with anencephaly would not reach 45 years of age. Choice D (Spina bifida cystica) is incorrect. Spina bifida cystica is a severe form of spina bifida and receives its name because of the characteristic presence of a cyst-like sac protruding from the defective area. The membranous walls of the sac are composed of very thin skin plus dura and arachnoid components. In these neural tube defects, the unfused neural arches of multiple vertebrae allow the meninges (and potentially the spinal cord) to protrude through the structural defect. The inclusion of a displaced portion of the spinal cord and nerve roots in the malformation designate this neural tube defect as a meningomyelocele, which is the most common form of spina bifida cystica. That is, the defect involves the meninges (meningo-), the spinal cord (myelo-) and the membranous sac (-cele). Because of the severity of this type of spina bifida, this 45-year-old man would not have this condition. Choice E (Spina bifida with myeloschisis) is incorrect. Spina bifida with myeloschisis, or rachischisis, involves the same signs and symptoms as the most common form of spina bifida cystica, a meningomyelocele; however, the protruded portions of the spinal cord are not afforded the enveloping protection of the meninges. Physical examination would reveal a flattened mass of nervous tissue with no associated membranes, which makes the patient more prone to life-threatening infections. Because of the severity of spina bifida with myeloschisis, this 45-year-old man would not have this condition.

A helmet-less 24-year-old man was riding an all-terrain vehicle (ATV or quad bike) when a thin, unseen, horizontal clothes line impacted the man's neck under his chin, knocking him backward off the ATV. Due to his forward momentum at the time of impact, his head was hyperextended on his neck. Emergency medical technicians (EMTs) established his airway, stabilized his neck, and brought him to the emergency room. A lateral cervical X-ray revealed pathology within the cervical vertebrae. Based upon the nature of the accident and the evi- dence of the given X-ray, which of the following diagnoses is most probable? (A) Ruptured anterior longitudinal ligament (B) Jefferson (burst) fracture of C1 (C) Traumatic spondylolysis of C2 (D) Fracture of the dens axis (odontoid process) (E) Atlanto-axial subluxation

C: Traumatic spondylolysis of C2. Spondylolysis means a defect in the pars interarticularis of the affected ver- tebra (or C2 in this patient). The lateral cervical X-ray of this patient reveals a bilateral fracture of the axis (or C2) in the pars interarticularis or the bony column formed by the supe- rior and inferior articular processes of C2. This type of cervical fracture is often seen after hyperextension of the head in rela- tionship to the neck, and it is called a traumatic spondylolysis of C2 or hangman fracture. It receives its alternative name, hangman fracture, because it is often seen in criminals exe- cuted by hanging due to the hyperextension of the head due to the placement of the noose under their chin. This lateral cervi- cal film also reveals a traumatic spondylolisthesis of C2, which is sometimes evident following an acute fracture of the pars interarticularis of C2. Spondylolisthesis describes the anterior displacement of a vertebra in relation to the vertebra below it, and its meaning is derived from the Greek words "spondyl" (G: vertebrae) and "olisthesis" (G: slip). In the given X-ray, the vertebral body of C2 is displaced anterior in relationship to C3, which confirms the traumatic spondylolisthesis of C2. Therefore, the X-ray clearly shows a bilateral fracture of the pars interarticularis of C2 (or traumatic spondylolysis of C2 [hangman fracture]) and spondylolisthesis, anterior displace- ment, of C2 in relationship to the vertebral body of C3. This specific fracture was classified in 1981 by Effendi into a sub- type called "forward dislocation, axial arch fracture, Effendi Type II." Choice A (Ruptured anterior longitudinal ligament) is incorrect. The anterior longitudinal ligament is a vertically running band that attaches along the anterior sides of the ver- tebral bodies. Its peripheral fibers have strong attachments to the intervertebral discs. This ligament resists hyperextension of the vertebral column, and in this patient who experienced hyperextension of the head on the neck, the extreme forces involved with the impact of the clothes line on the patient's neck may have damaged the anterior longitudinal ligament, particularly due to the spondylolisthesis of C2. However, a ruptured anterior longitudinal ligament is not apparent on this lateral cervical X-ray, so the integrity of this ligament is unknown. The anterior longitudinal ligament would be bet- ter viewed with a T2-weighted MRI, which would show this ligament as a hypointense (dark black) band located anterior to the vertebral column. If damaged, the hypointense anterior longitudinal ligament will be interrupted by abnormal hyper- intense (white) signal. Choice B (Jefferson [burst] fracture of C1) is incorrect. The C1 vertebra, or atlas, is a closed ring with no vertebral body. Excessive vertical, or downward, force on the top of the head can fracture the anterior and posterior arches of C1 in multiple places, leading to a Jefferson (or burst) fracture of C1. Jefferson fractures of C1 often occur with axial loading force when the top of the head is impacted by a hard or heavy object. The downward force of the impact drives the lateral masses of the atlas lateral and fractures the anterior and posterior arches of C1. Because of the nature of this hyperex- tension injury and lack of visual evidence on the lateral cervi- cal film of damage to C1, a Jefferson (or burst) fracture of C1 is not the diagnosis. Choice D (Fracture of the dens axis [odon- toid process]) is incorrect. In the given lateral cervical X-ray, there is no apparent fracture of the dens (odontoid process) of the axis, or second cervical vertebra (C2). Because odontoid fractures are likely due to a combination of excessive flexion, extension, and some rotation within the cervical vertebrae, this patient, whose injury was caused by hyperextension of the head on the neck, is unlikely to have an odontoid fracture. The lateral cervical film confirms the presence of traumatic spondylolysis of C2, or a hangman fracture, and traumatic spondylolisthesis of C2. Choice E (Atlanto-axial subluxation) is incorrect. Atlanto-axial subluxation, or the incomplete dis- location of the median atlanto-axial joint, occurs following the rupture of the transverse ligament of the atlas, which holds the dens axis (odontoid process) in place. Losing the integrity of the transverse ligament of the atlas can result in compres- sion of the upper cervical spinal cord, leading to quadriplegia (paralysis of all four limbs) and even death (if the medulla of the brainstem is compressed). Atlanto-axial subluxation is prevalent in patients with Down syndrome (trisomy 21), but it can also occur with other connective tissue disorders. In this lateral cervical film, a bilateral fracture of the pars interarticu- laris of C2 is apparent, and the odontoid process of C2 is in its normal position, which implies an intact transverse ligament of the atlas. Therefore, atlanto-axial subluxation did not occur in this patient.

Nuchal translucency is used to screen for congenital defects during the 11th through 14th weeks of pregnancy. This procedure is recommended by an obstetrician for a 35-year-old pregnant woman who previously had a child with Down's syndrome (trisomy 21). Which of the following would be common to see in addition to a larger than normal accumulation of fluid at the back of the baby's head, indicating the possibility of another baby with Down's syndrome? A. Meromelia B. Hydrocephalus C. Anencephaly D. Atrioventricular septal defect E. Plagiocephaly

D. Down's syndrome is a congenital abnormality involving a triplication of chromosome 21. It is more common in children of women who become pregnant at 35 years or older (advanced maternal age is a risk factor). Down's syndrome is associated with various abnormalities including congenital heart defects. The most common congenital heart defect in Down's syndrome is endocardiac cushion type of atrial septal defect (40%) followed by ventricular septal defects (35%).

A 25-year-old woman presents to her obstetrician after taking a home pregnancy test with a positive result. She states that twins run in her family and would like an ultrasound to determine if she has a twin pregnancy. Radiographic studies confirm that the embryo has split at the blastocyst stage. Splitting of the embryo at the blastocyst stage results in which of the following? Select one: a. Conjoined twins b. Dizygotic twins c. Fraternal twins d. Monozygotic twins e. Single gestation

D. Monozygotic twins. Monozygotic twins, or identical twins, develop from a single fertilized egg that subsequently splits during either the blastomere or blastocyst stage. It is more common in the blastocyst stage at the end of the first week. This results in two inner cell masses in the same blastocyst cavity. They usually develop a common placenta and chorionic cavities but separate amniotic cavities. Splitting in the second week usually results in shared amniotic cavities as well. (Choice A) Conjoined twins, or Siamese twins, result from incomplete splitting of the embryo. (Choice B) Dizygotic twins and fraternal twins (choice C) are the same, and are the most common type of twins. They share the same genetic relatedness as do siblings of separate pregnancies. This type of twinning occurs because of simultaneous double ovulation followed by fertilization by two sperm. They each develop their own placenta and membranes. (Choice C) Dizygotic twins and fraternal twins are the same, and are the most common type of twins. They share the same genetic relatedness as do siblings of separate pregnancies. This type of twinning occurs because of simultaneous double ovulation followed by fertilization by two sperm. They each develop their own placenta and membranes. (Choice E) A single gestation, and a single birth, is the result of a single fertilization without any splitting of the embryo.

A 34-year-old pregnant woman of 29 weeks pregnancy visits her gynecologist for a regular check up. General examination of the abdomen shows underdevelopment of the fetus for its developmental stage. Ultrasound examination of the fetus shows oligohydraminos. Which of the following conditions is most likely cause for this? A. Renol (kidney) agenesis B. Esophageal atresia C. Maternal diabetes D. Tracheo-esophageal fistula* E. Anencephaly

D. Tracheo-esophageal fistula

A 28-year-old woman who had no prenatal care gives birth at 42 weeks' gestation. During the pregnancy, an arnniocentesis revealed elevated levels of alpha-fetoprotein, but ultrasound imaging showed that there were no neural tube defects. The newborn has some of its abdominal viscera protruding through the abdominal wall adjacent to the midline that do not appear to be covered by amnion or peritoneum. Which of the following is the likely cause of this defect? Select one: a. Failure of the intestinal loop to retract from the umbilical cord b. Failure of the yolk stalk to degenerate c. Failure of peritoneal fusion d. Incomplete fusion of the lateral body folds e. Umbilical herniation

D. incomplete fusion of the lateral body folds. During the fourth week of development, the lateral body folds move ventrally and fuse in the midline to form the anterior body wall. Incomplete fusion results in a defect that allows abdominal viscera to protrude from the abdominal cavity, a condition known as gastroschisis. (Choice A) During development, the midgut normally herniates into the umbilical cord and then subsequently retracts into the abdominal cavity. Failure of the intestinal loop to retract from the umbilical cord results in omphalocele. (Choice B) Failure of the yolk stalk to degenerate results in an ileal (Meckel) diverticulurn or a vitelline fistula or cyst. In the early embryo, the gut tube is connected to the yolk sac by a narrow connection known as the yolk stalk. Normally, this connection degenerates. (Choice C) During development, certain peritoneal organs fuse with the posterior abdominal wall to become secondarily retroperitoneal. Failure of this peritoneal fusion will result in certain organs that are normally immobile being mobile (e.g., mobile cecum). (Choice E) Umbilical herniation results from abdominal viscera protruding through a weakness in the abdominal wall after development. Such protrusions are covered by subcutaneous fascia and skin, distinguishing them from gastroschisis.

Disruption of parasympathetic input to the thorax may be related to damage to which of the following structures? A. cervical paravertebral ganglia B. greater splanchnic nerves C. gray rami communicantes D. vagus nerves E. intercostal nerves

D. vagus nerves

A 35-year-old man presents with pain radiating from his lower back into his left lower limb, periodic sensory loss and paresthesia in that limb, and associated motor weakness. His physician suspects unilateral entrapment of lumbar nerve roots secondary to a herniated lumbar intervertebral disc. In examining the patient, the physician conducts a straight-leg raising test. Which of the following best describes a positive straight-leg raising sign (Lasèque's sign) for this condition in this patient? (A) Walking while keeping the left leg in a straight position (i.e., the knee is kept straight) relieves the pain in the back and limb (B) Flexing the hip by raising the straightened left leg (i.e., the knee is kept straight) with the patient lying in the supine position relieves the pain in the back and leg (C) Abducting the hip by raising the straightened left leg with the patient lying on his right side relieves the pain in the back and leg (D) Flexing the hip by raising the straightened left leg with the patient lying in the supine position increases pain in the back with radiation down the left leg (E) Extending the hip by raising the straightened left leg with the patient lying in the prone position increases pain in the back with radiation down the left leg

D: Flexing the hip by raising the straightened left leg with the patient lying in the supine position increases pain in the back with radiation down the left leg. The straight-leg raising test is designed to stretch the lower back and apply tension to the lumbar spinal nerves and roots. Thus, a posi- tive test will stress compromised nerves and elicit pain and/ or spasticity in the areas supplied by the affected nerves. If the diagnosis of left side lumbar nerve entrapment second- ary to a herniated lumbar disc is correct, raising the left leg (i.e., flexing the hip) with the knee straight (extended) from the supine position (lying on the back) will apply traction to the entrapped nerve roots and elicit pain in the affected areas. Conducting the same maneuver on the right side will not elicit a pain reaction. Lasèque sign is a medical maneuver that involves raising a straight leg to exacerbate pain and is useful in diagnosing lumbar disc disorders and tension of the sciatic nerve. Choice A (Walking while keeping the left leg in a straight position [i.e., the knee is kept straight] relieves the pain in the back and limb) is incorrect. The forward swing of the straight left leg during walking is essentially the same maneuver as raising the straight leg from the supine position. Traction is applied to the entrapped nerve roots during flexion of the hip, with pain stimulated rather than relieved. Choice B (Flexing the hip by raising the straightened left leg [i.e., the knee is kept straight] with the patient lying in the supine position relieves the pain in the back and leg) is incorrect. This maneuver is basically the same as the one explained in Choice A. Again, pain should be elicited rather than relieved in a positive test. Choice C (Abducting the hip by raising the straightened left leg with the patient lying on his right side relieves the pain in the back and leg) is incorrect. Raising (abducting) the limb toward the affected side will again apply traction to the affected nerve roots, resulting in stimulating pain rather than relieving pain. Choice E (Extending the hip by raising the straightened left leg with the patient lying in the prone position increases pain in the back with radiation down the left leg) is incorrect. From the prone position, rais- ing the left limb means extending the limb. This maneuver will not produce as great an excursion of the limb as flex- ing the straightened limb at the hip from the supine position. Therefore, this maneuver will reduce traction on the affected nerve roots and relieve pain rather than increase pain.

A 17-year-old gymnast grips a high bar with his arms out-stretched and begins to pull himself straight upward to the level of the bar, as in doing a chin-up. Which of the following muscles is the prime agonist in this action? (A) Serratus posterior superior (B) Rhomboid major (C) Levator scapulae (D) Latissimus dorsi (E) Longissimus

D: Latissimus dorsi. The latissimus dorsi (L: widest muscle of back) is a large, fan-shaped muscle and a member of the superficial extrinsic layer of back muscles, which connect the upper limbs to the trunk. The chin-up movement described includes lifting the body toward the upper limb, which is a powerful extension action. In con- junction with the pectoralis major muscle, the latissimus dorsi muscle raises the trunk to the arm, which is crucial in performing chin-ups or climbing a tree. Remember that the superficial layer of back muscles is composed of upper limb muscles that take large bases of origin in the back. Athletes who make heavy use of extension and medial rotation of the arm (as in chin-ups or climbing in conditional exercises) typi- cally have well-developed "lats" that give the classic "V" shape to the back. Choice A (Serratus posterior superior) is incor- rect. The serratus posterior superior muscle is a member of the intermediate layer of back muscles that lies deep to the rhom- boid muscles. Due to its insertion into the superior borders of the 2nd to 5th ribs, it is said to elevate the ribs to increase the diameter of the thorax, acting as an accessory muscle of respiration. Due to its small size and lack of attachment to the appendicular skeleton, it would not be the primary muscle involved with performing chin-ups. Choice B (Rhomboid major) is incorrect. The rhomboid major and minor muscles act to retract (adduct) the scapula. In the chin-up movement, the trunk of the body is lifting toward the upper limb, and the rhomboids are important in stabilization of the scapula to allow the latissimus dorsi muscle to raise the trunk toward the upper limb. However, these muscles are not the prime agonist in this climbing movement. Choice C (Levator scapulae) is incorrect. The levator scapulae muscle acts mainly to elevate the scapula. During the chin-up movement, the levator scap- ulae muscle also aids in downward rotation of the scapula, which would aid the latissimus dorsi muscle in raising the trunk toward the upper limb. However, the levator scapulae muscle is not the prime agonist in this climbing movement. Choice E (Longissimus) is incorrect. The longissimus muscle is one of the three components of the erector spinae group of deep back muscles. Its primary action is to extend the back. Because it does not attach or originate within the upper limb, the longissimus muscle is unable to lift the trunk toward the upper limb, which is what defines a chin-up.

A developmental biologist studying the derivatives of the somites and somitomeres treats the epimeres in a mouse embryo with a toxin that kills all of their cells. If the mouse develops to full term, which of the following muscles is most likely to be absent? (A) Trapezius (B) Latissimus dorsi (C) Rhomboid major (D) Longissimus (E) Levator scapulae

D: Longissimus. Destruction of the embryonic epimeres will result in loss of the deep group of back muscles, including the longissimus muscle, one of three erector spinae muscles (along with the iliocostalis and spinalis muscles). Muscles derived from the epimeres are the proper muscles of the back, which are innervated by the posterior (dorsal) primary rami of spinal nerves. These deep muscles of the back include the previously mentioned erector spinae group, splenius capitis and cervicis muscles, suboccipital muscles, transversospinales muscles (including the semispinalis, multifidus, and rotatores muscles), and other small muscles. Remember, each embryonic somite differentiates into three components: myotome, dermatome, and sclerotome. Each myotome splits into two parts: epimere and hypomere. The epimere develops dorsal to the incipient vertebral column, and the longissimus muscle is the only listed muscle which is derived from the embryonic epimere. The muscles derived from the epimeres form the deep group of muscles in the back and are often termed the intrinsic back muscles or epaxial muscles. Choice A (Trapezius) is incorrect. The trapezius is a member of the superficial extrinsic layer of back muscles, which connect the upper limbs to the trunk. When these muscles contract (or shorten), they produce movements of the upper limb. How- ever, the trapezius muscle does not originate from either the epimere or hypomere, as evidenced by its innervation via the spinal accessory nerve (CN XI). Instead, it is likely related to postbranchial origins, along with its mate, sternocleidomastoid. Thus, destruction of epimeres will not directly affect the formation of the trapezius. Choice B (Latissimus dorsi) is incorrect. The latissimus dorsi (L: widest muscle of back) is a member of the superficial extrinsic layer of back muscles, which connect the upper limbs to the trunk. When these muscles contract, they produce movements of the upper limb. The latissimus dorsi muscle is derived from hypomeres, which develop lateral and anterior to the vertebral axis, and it migrates secondarily into the back. All of the muscles derived from hypomeres are innervated by the anterior (ven- tral) primary rami of spinal nerves. The latissimus dorsi muscle is innervated by the thoracodorsal (middle subscapular) nerve and acts on the upper limb. Due to being derived from the embryonic hypomeres, the latissimus dorsi muscle would not be affected by the toxin injected into the epimeres within this mouse embryo. Choice C (Rhomboid major) is incorrect. The rhomboid major muscle is a member of the superficial extrinsic layer of back muscles, which connect the upper limbs to the trunk. When these muscles contract, they produce movements of the upper limb. The rhomboid major muscle is derived from hypomeres, which develop lateral and anterior to the vertebral axis. All of the muscles derived from hypomeres are innervated by the anterior (ventral) primary rami of spinal nerves. Due to being derived from the embryonic hypomeres, the rhomboid major muscle would not be affected by the toxin injected into the epimeres within this mouse embryo. Choice E (Levator scapulae) is incorrect. The levator scapulae muscle is a member of the superficial extrinsic layer of back muscles, which connect the upper limbs to the trunk. When these muscles contract, they produce movements of the upper limb. The levator scapulae muscle is derived from hypomeres, which develop lateral and anterior to the vertebral axis. All of the muscles derived from hypomeres are innervated by the anterior (ventral) primary rami of spinal nerves. Due to being derived from the embryonic hypomeres, the levator scapulae muscle would not be affected by the toxin injected into the epimeres within this mouse embryo.

A 32-year-old pregnant woman who is in the 30th week of her pregnancy visits her obstetrician for the first time (which is later in her pregnancy than usual). An ultrasound reveals that she is carrying twins. However, it is also revealed that one twin is surrounded by excessive amniotic fluid, while the other is surrounded by too little. A diagnosis of twin-to-twin transfusion syndrome is made. Which of the following types of pregnancy is most likely the cause of such a condition? A. Dichorionic, diamniotic twins B. Dichorionic, monoamniotic twins C. Monochorionic, monoamniotic twins D. Conjoined twins E. Monochorionic, diamniotic twins

E. In monochorionic diamniotic twins, the single placenta (due to as yet unknown factors) develops blood vessel connections between the umbilical vessels of the twins. These connections result in an unbalanced blood supply known as twin-twin transfusion syndrome (TTTS). In TTTS, the donor twin does not get enough blood and the recipient twin becomes volume overloaded. In an attempt to reduce its blood volume, the recipient twin increases urine production, eventually resulting in polyhydramnios. At the same time, the donor twin produces less than the usual amount of urine leading to oligohydramnios. As the disease progresses, the donor produces so little urine that its bladder may not be seen on ultrasound. The twin becomes wrapped up by its amniotic membrane (known as a "stuck" twin). Often the polyhydramnios of the recipient twin is the first thing noticed by the patient due to a sudden increase in the size of the uterus. This condition is only possible when twins share a single chorion but have separate amniotic sacks. See Moore KL, Persaud TVN, Torchia MG: Before We Are Born, ed 8, 2013, pp. 85-87*

A male infant is born with cleft hand, also known as "lobster-claw hand." This abnormality is caused by the apical ectodermal ridge (AER) failing to properly develop. Which of the following best describes the principal function of the AER? A. Establishes the anteroposterior axis of the limb bud B. Stimulates blood vessel growth into the limb bud C. Stimulates cartilage differentiation in the limb bud D. Stimulates nerve growth into the limb bud E. Stimulates outgrowth of the limb bud

E. The AER secretes growth factor, which initiates outgrowth of the limb mesenchyme that initiates formation of the limb bud. 235, 239

The developing embryo has a distinct human appearance by the end of (A) week 4 (B) week 5 (C) week 6 (D) week 7 (E) week 8

E. The embryo starts the embryonic period as a two-dimensional disk and ends as a three-dimensional cylinder. This dramatic change in geometry is caused by formation of all the major organ systems. As the organ systems gradually develop during the embryonic period, the embryo appears more and more human-like; it has a distinct human appearance at the end of week 8.

A 24-year-old woman visits an obstetrician and complains of not being able to get pregnant for the last 4 years. History reveals that she has been experiencing hot flashes, irregular periods, night sweats, and vaginal dryness. A diagnosis of premature ovarian failure is made. Estrogen replacement therapy is prescribed. What is the organ responsible for secreting estrogen? A. Anterior pituitary B. Posterior pituitary C. Hypothalamus D. Adrenal cortex E. Ovary

E. The ovaries are almond-shaped reproductive glands located close to the lateral pelvic walls on each side of the uterus that produce oocytes. The ovaries also produce estrogen and progesterone, the hormones responsible for the development of secondary sex characteristics and regulation of pregnancy. Follicle stimulating hormone (FSH) is produced by the anterior pituitary gland, which stimulates the development of ovarian follicles and the production of estrogen by the follicular cells. Luteinizing hormone (LH) serves as the "trigger" for ovulation (release of secondary oocyte) and stimulates the follicular cells and corpus luteum to produce progesterone. Gonadotropin releasing hormone (GnRH) is secreted from the hypothalamus and stimulates the production and secretion of FSH and LH from the anterior pituitary. 10, 14-17

A 5-year-old boy is brought to a plastic surgeon's office. He has a small jaw, large mouth, flat cheekbones, and malformed pinna. He had a cleft palate at birth, which was surgically corrected shortly afterward. The parents complain that sometimes he does not respond when they talk to him. What correctly describes this boy's embryologic defect? A. Trisomy 13 B. Defect in FBN1 gene C. Failure of fusion of medial nasal prominences D. Failure of rostral neuropore to close E. Failure of neural crest migration into the first arch

E. Treacher Collins syndrome (craniofacial dysostosis) is an autosomal disorder characterized by malar hypoplasia, downward-slanting palpebral fissures, defects of the lower eyelids and external ears, and sometimes abnormalities of the middle and internal ears. This is a first arch syndrome due to failure of neural crest cells to migrate to the first arch during the fourth week of development. Trisomy 13, also called Patau syndrome, is a chromosomal condition associated with severe intellectual disability and physical abnormalities in many parts of the body including microphthalmia, cleft lip and palate, and brain and spinal cord anomalies. Defect in the fibrillin 1 (FBN1) gene leads to Marfan's syndrome. Failure of fusion of the medial nasal prominences leads only to cleft lip and palate, while failure of closure of the rostral neuropore leads to meroencephaly or anencephaly. 108

A 29-year-old pregnant woman in her first trimester complains of preeclampsia and spotting. Ultrasound examination reveals an enlarged uterus and chorionic villi. Laboratory tests show elevated hCG levels. The physician suspects a hydatidiform mole, a condition characterized by gross edema of the chorionic villi surrounded by trophoblastic cells. The trophoblastic cell normally gives rise to which two cell layers? A. Ectoderm and endoderm B. Endoderm and mesoderm C. Hypoblast and epiblast D. Neural crest and neural tube E. Syncytiotrophoblast and cytotrophoblast

E. Upon trophoblastic invasion of the endometrium, the trophoblastic cells produce two cell layers: the syncytiotrophoblast is the outer layer that is related to the endometrial wall while the cytotrophoblast is the inner layer, which usually gives rise to the chorionic villi. The ectoderm, endoderm, and mesoderm are body germ layers produced during gastrulation. Hypoblast and epiblast cells are derived from the inner cell mass of the blastocyst, and the epiblast will ultimately, under gastrulation, form the germ layers. Neural crest cells do not give rise to the syncytiotrophoblast and cytotrophoblast; the neural tube forms the central nervous system. 24-25, 45

A preterm neonate chokes and coughs throughout her first feeding with her mother with the breast milk regurgitating from the infant's mouth and nose. Noticing the difficulty feeding, a pediatrician tries to pass a catheter into the baby's stomach and meets resistance. A chest film is taken of the baby while in an incubator. The given radiograph reveals the location of the radiopaque catheter, shows no air within the stomach of the infant, and verifies normal lungs. What is the most likely diagnosis based upon the results of the x-ray?

E. esophageal atresia

As a result of multiple vertebral fractures incurred in an automobile crash, an 8-year-old girl suffers a series of torn posterior primary rami of spinal nerves C1-6. Which of the following muscles will be paralyzed as a result? (A) Trapezius (B) Latissimus dorsi (C) Levator scapulae (D) Rhomboid major (E) Splenius capitus

E: Splenius capitis. The posterior (dorsal) pri- mary rami of spinal nerves innervate the embryonic epimere and all the skeletal muscles derived from it. The derivatives of the epimere constitute the deep (intrinsic) muscles of the back (or epaxial muscles), including the splenius capitis and cervicis muscles, suboccipital muscles, transversospinales muscles (including the semispinalis, multifidus, and rota- tores muscles), and several other small muscles. Thus, dam- age to the cervical posterior (dorsal) primary rami, specifically C2-6, would result in paralysis of the splenius capitis muscle. Choice A (Trapezius) is incorrect. The trapezius is a mem- ber of the superficial extrinsic layer of back muscles, which connect the upper limbs to the trunk. When these muscles contract, movements result in the upper limb. However, the trapezius muscle does not originate from either the epimere or hypomere, as evidenced by its innervation via the spi- nal accessory nerve (CN XI). Instead, it is likely related to postbranchial origins, along with the sternocleidomastoid. Thus, destruction of the cervical posterior (dorsal) primary rami will not directly affect the innervation of the trapezius muscle. Choice B (Latissimus dorsi) is incorrect. The latis- simus dorsi (L: widest muscle of back) is a member of the superficial extrinsic layer of back muscles, which connect the upper limbs to the trunk. When these muscles contract, movements result in the upper limb. The latissimus dorsi muscle is derived from hypomeres, which develop lateral and anterior to the vertebral axis. All of the muscles derived from hypomeres are innervated by the anterior (ventral) primary rami of spinal nerves. The latissimus dorsi muscle acts on the upper limb, and it migrates secondarily into the back. Due to being innervated by the anterior (ventral) primary rami, the latissimus dorsi muscle would not be affected by destruc- tion of the cervical posterior (dorsal) primary rami seen in this patient. Choice C (Levator scapulae) is incorrect. The levator scapulae muscle is a member of the superficial extrinsic layer of back muscles, which connect the upper limbs to the trunk. When these muscles contract, movements result in the upper limb. The levator scapulae muscle is derived from hypomeres, which develop lateral and anterior to the vertebral axis. All of the muscles derived from hypomeres are innervated by the anterior (ventral) primary rami of spinal nerves. Due to being innervated by the anterior (ventral) primary rami, the levator scapulae muscle would not be affected by destruction of the cervical posterior (dorsal) primary rami seen in this patient. Choice D (Rhomboid major) is incorrect. The rhomboid major muscle is a member of the superficial extrinsic layer of back muscles, which connect the upper limbs to the trunk. When these muscles contract, movements result in the upper limb. The rhomboid major muscle is derived from hypomeres, which develop lateral and anterior to the vertebral axis. All of the muscles derived from hypomeres are innervated by the anterior (ventral) primary rami of spinal nerves. Due to being innervated by the anterior (ventral) primary rami, the levator scapulae muscle would not be affected by destruction of the cervical posterior (dorsal) primary rami seen in this patient.

After a blood test showed an elevated prostate-specific antigen level in a 65-year-old man, the given axial T2-weighted MRI of the prostate was obtained via an endorectal coil. The image shows abnormally low signal intensity within the right peripheral zone of the prostate, which is characteristic of an adenocarcinoma and is identified by the white arrowheads. A radical prostatectomy was scheduled. Before surgery, his physician wants to rule out possible metastases of the cancer. Which group of lymph nodes should be checked in the first instance to rule out metastases? (A) Lumbar (B) Deep inguinal (C) Superficial inguinal (D) Internal iliac (E) External iliac

The answer id D: Internal Iliac Cancer cells from an adenocarcinoma within the posterior part of the prostate would most likely metastasize initially to the internal iliac lymph nodes, which lie along the internal iliac artery and receive lymph from the pelvic visceral and gluteal region. Because most lym- phatic vessels supplying pelvic viscera follow arteries retro- grade, lymph from the prostate gland, which receives arterial blood from the inferior vesical artery arising off the anterior division of the internal iliac artery, would terminate primarily in the internal iliac lymph nodes. Therefore, cancer cells from an adenocarcinoma within the posterior part of the prostate would most likely metastasize to the internal iliac lymph nodes. The sacral lymph nodes may also receive lymphatic drainage from the prostate, so these nodes should also be checked for metastases. Remember, when considering spread of cancer from pelvic organs, it is crucial to remember the arterial supply of the organ in question. Choice A (Lumbar) is incorrect. Lumbar lymph nodes, which are located anterior to the lumbar vertebrae and surrounding the inferior vena cava and abdominal aorta, receive lymphatic drainage directly from the posterior abdominal wall, gonads (ovaries and testes), kid- neys, ureters, uterus, and uterine tubes. Unless metastasis is in advanced stages by spreading through several sets of lymph nodes, the lumbar lymph nodes would not be affected because they do not directly receive lymphatic drainage from a prostatic adenocarcinoma. Choice B (Deep inguinal) is incorrect. Deep inguinal lymph nodes, which are located deep to the fascia lata and medial to the femoral vein, receive lymphatic drain- age directly from the glans penis, the distal spongy urethra of the penis, and deep structures of the lower limb and indirect lymphatic drainage from the superficial inguinal lymph nodes. Since the prostate is a pelvic organ, it does not have lymphatic drainage to the deep inguinal lymph nodes. Choice C (Super- ficial inguinal) is incorrect. Superficial inguinal lymph nodes, which are located in the subcutaneous tissue near the termina- tion of the great saphenous vein, receive lymphatic drainage from the lower abdominal wall, buttock, lower limb, and all of the perineum and external genitalia, except for the glans penis and its distal spongy urethra. Since the prostate is a pelvic organ, it does not have lymphatic drainage to the superficial inguinal lymph nodes. Choice E (External iliac) is incorrect. The external iliac lymph nodes, which are associated with the external iliac vein, receive lymphatic drainage from pelvic organs in direct contact with the peritoneum, including the superior bladder and superior pelvic ureters. Since the pros- tate is situated well below the peritoneum in the pelvis, it does not drain to the external iliac lymph nodes. 30

13 A dermatologist performed a biopsy on a suspicious mole on the right side of the posterior neck of a 57-year-old male construction worker. Pathology confirmed a malignant melanoma, so the physician excised a substantial amount of tissue surrounding the mole. After the procedure, the patient experienced difficulty elevating his right shoulder and lifting his right arm over his head. No sensory deficits were seen. What nerve was most likely damaged in this patient? (A) Accessory nerve (B) Axillary nerve (C) Dorsal scapular nerve (D) Long thoracic nerve (E) Thoracodorsal nerve

The answer is A: Accessory nerve. The accessory nerve (CN XI) traverses the posterior triangle of the neck to reach the deep surface of the trapezius muscle after it innervates the sternocleidomastoid muscle. Its position within the triangle is superfi cial, and it is at this location that this nerve is vulnerable to injury. Damage to the distal accessory nerve would inhibit elevation of the scapula and lateral rotation of the scapula during abduction greater than 90 degrees. Both of these actions were affected in this patient. Choice B (Axillary nerve) is incorrect. The axillary nerve travels through the quadrangular space to reach the deltoid and teres minor muscles. While a lesion of this nerve would affect abduction of the arm, the axillary nerve is not found in the neck, instead arising distally from the posterior cord of the brachial plexus. Moreover, damage to the axillary nerve would cause a sensory defi cit in the upper lateral arm, which was not reported. Choice C (Dorsal scapular nerve) is incorrect. This nerve courses into the upper, medial part of the back and the lower neck to supply the levator scapulae and rhomboid muscles. Paralysis of these muscles would result in weakness in elevation and retraction of the scapula and perhaps wasting of the contour of the back under the trapezius muscle. Choice D (Long thoracic nerve) is incorrect. The long thoracic nerve is a branch of the upper three roots (C5-C7) of the brachial plexus. It descends along the medial wall of the axilla and travels superficial to the serratus anterior muscle, which it innervates. This nerve is coalescing in the deep, most inferior region of the neck, so it would not be injured in the location of the incision. Damage to the long thoracic nerve would lead to "winging of the scapula," when the patient is asked to protract the affected shoulder. Choice E (Thoracodorsal nerve) is incorrect. This nerve runs inferiorly through the axilla to supply the latissimus dorsi muscle. Loss of the nerve would result in weakness in extension and medial rotation of the arm, plus wasting of the posterior axillary fold.

39 The traditional radical mastectomy includes removal of the pectoralis major muscle. Which of the following movements is most affected postoperatively by this surgical procedure? (A) Adduction of the arm (B) Abduction of the arm (C) Extension of the arm (D) Lateral rotation of the arm (E) Depression of the arm

The answer is A: Adduction of the arm. In a traditional (Halsted) radical mastectomy, both the pectoralis major and minor muscles are removed with the breast and associated axillary tissue. The primary actions of the pectoralis major are adduction, fl exion, and medial rotation of the arm. These actions are weakened but not lost postoperatively. Other neighboring muscles (e.g., subscapularis, latissimus dorsi, biceps brachii, anterior deltoid) also perform these actions and can compensate somewhat for loss of the pectoralis major. Choice B (Abduction of the arm) is incorrect. Abduction of the arm is performed by the supraspinatus, deltoid, and serratus anterior muscles. Removing the pectoralis major would not affect abduction of the arm. In fact, abduction, extension, and lateral rotation of the arm are the antagonistic actions to the pectoralis major. Choice C (Extension of the arm) is incorrect. The pectoralis major helps with fl exion of the arm, not extension (the antagonistic action). The latissimus dorsi and posterior part of the deltoid help with extension of the arm. Choice D (Lateral rotation of the arm) is incorrect. The pectoralis major helps with medial rotation of the arm, not lateral rotation ofthe arm (the antagonistic action). The infraspinatus, teres minor, and posterior part of the deltoid help with lateral rotation of the arm. Choice E (Depression of the arm) is incorrect. The pectoralis major muscle acts to adduct, fl ex, and medial rotate the arm. Therefore, a radical mastectomy would affect these actions.

A physician examining a 16-year-old girl for the first time notes that she is in good overall health and appears outwardly to have reached puberty. However, she explains that she has not yet had a menstrual period. Full physical and radiological examinations reveal a short, closed vaginal pouch, and no upper vagina, cervix, or uterus. Which of the following descriptions is the most likely cause of this condition? (A) Atresia of the paramesonephric ducts (B) Atresia of the mesonephric ducts (C) Atresia of the urogenital sinus (D) Imperforate hymen (E) Agenesis of the ureteric bud

The answer is A: Atresia of the paramesonephric ducts. In females, the paired embryonic paramesonephric ducts form the uterine tubes, uterus, cervix, and superior third of the vagina. Thus, if the paramesonephric ducts are absent (atretic), these organs do not form. The lower 2/3 of the vagina develops from the sinovaginal bulbs, which are paired evagi- nations of the wall of the urogenital sinus. The two parts of the vagina (paramesonephric duct part and sinovaginal bulb part) canalize to form a single, complete vaginal canal. How- ever, without the paramesonephric duct components, only a short vaginal pouch forms. Because the ovaries form from the indifferent embryonic gonads, they can still develop nor- mally, providing the hormonal stimuli to puberty. Choice B (Atresia of the mesonephric ducts) is incorrect. In females, the mesonephric ducts are normally suppressed. Only small vestigial structures form in association with the reproduc- tive tract. However, because the ureteric bud develops from the mesonephric duct, the renal collecting ducts would not form in case of atresia there. Choice C (Atresia of the uro- genital sinus) is incorrect. The urogenital sinus gives rise to the urinary bladder and the urethra. In addition, in females, the sinovaginal bulbs evaginate from the wall of the urogenital sinus and form the lower 2/3 of the vagina. Thus, with atresia of the urogenital sinus, the lower part of the vagina would be absent or imperforate, whereas the upper vagina, cervix, uterus, and uterine tubes would be intact. Further, one would expect malformation of the urinary bladder and/or urethra. Choice D (Imperforate hymen) is incorrect. The hymen is the thin plate of tissue that separates the vestibule from the lumen of the vagina. Remember, the sinovaginal bulbs origi- nate as solid tissue masses off the posterior wall of the uro- genital sinus. When these sinovaginal bulbs canalize to form the vaginal lumen, the hymen remains at the junction of the urogenital sinus and vagina. It usually develops one or more small openings during postnatal life. Choice E (Agenesis of the ureteric bud) is incorrect. The ureteric bud is an outgrowth from the wall of the mesonephric duct. It forms the renal col- lecting ducts, including the ureter, renal pelvis, renal calyces, and collecting tubules. Additionally, it induces the formation of the metanephric cap that forms the kidney parenchyma and the nephrons (i.e., the excretory units in the kidney).

17 A 17-year-old male football player suffers a shoulder injury and arrives at the ER 2 hours after the injury. The physician diagnoses a shoulder dislocation, and after administration of a local anesthetic solution, the doctor repositions the head of the humerus into the glenoid cavity of the scapula (reduction). No fractures are seen on X-rays. However, the patient displays weakness in abduction and external rotation at the shoulder. A loss of sensation is also noted at the superior and lateral aspects of the arm. What nerve was most likely damaged in this injury? (A) Axillary nerve (B) Median nerve (C) Ulnar nerve (D) Radial nerve (E) Musculocutaneous nerve

The answer is A: Axillary nerve. The axillary nerve may be damaged in approximately one of seven shoulder dislocations. This nerve innervates the deltoid and teres minor muscles as well as supplying innervation to the skin overlying the deltoid in the superolateral aspect of the arm. Loss of innervation to the deltoid muscle would explain the weakness in abduction of the upper limb. The teres minor assists the infraspinatus muscle in external rotation of the shoulder. Choice B (Median nerve) is incorrect. The median nerve does not branch proximal to the elbow, and its sensory distribution is limited to the hand. The median nerve could not be responsible for the patient's motor and sensory defi cits. Choice C (Ulnar nerve) is incorrect. The ulnar nerve can be damaged during shoulder dislocations; however, its sensory distribution is limited to distal to the wrist, and the fi rst muscle it innervates is in the forearm (fl exor carpi ulnaris). Therefore, the ulnar nerve could not be responsible for this patient's motor and sensory defi cits. Choice D (Radial nerve) is incorrect. The radial nerve supplies motor innervation to the posterior compartments of the arm and forearm. Damage to this nerve would cause weakness in extending at the elbow and wrist joints. The radial nerve gives rise to the posterior cutaneous nerves of the arm and forearm as well as the inferior lateral cutaneous nerve of the arm. However, it would not affect the superior lateral cutaneous nerve of the arm, arising from the axillary nerve, which was damaged in this patient. The radial nerve would also not affect abduction and external rotation of the shoulder. Choice E (Musculocutaneous nerve) is incorrect. The musculocutaneous nerve can be damaged during shoulder dislocations; however, this nerve supplies the motor innervation to the anterior compartment of the arm. Trauma to this nerve would lead to weakness in fl exing the elbow and supinating the forearm. Its sensory distribution is limited to the lateral aspect of the forearm, so it was not the nerve damaged in this patient.

An abscess is detected between the perineal membrane and the inferior fascia of the pelvic diaphragm in a 27-year-old man. Which of the following structures would most likely be affected by this abscess within the deep perineal space? (A) Bulbourethral glands (B) Perineal body (C) Superficial transverse perineal muscles (D) Bulbospongiosus muscles (E) Ischiocavernosus muscles

The answer is A: Bulbourethral glands. Bulbourethral (Cowper) glands secrete nonviscous material into the urethra before and during ejaculation to mix with sperm, seminal vesicle fluid, and prostatic fluid to form semen. The bulbourethral glands are found deep to the perineal membrane within the deep perineal compartment, so they are the most likely structures to be affected by the described abscess within the deep perineal compartment. Choice B (Perineal body) is incorrect. The perineal body is located at the central point of the perineum where the perineal muscles converge. This unique structure serves as an anchoring point for the perineum and contains collagen and elastic fibers as well as skeletal and smooth muscles. Due to its location between the anal and urogenital triangles of the perineum, it is generally not considered part of the superficial or deep perineal compartments. Therefore, it would not be the most likely structure on this list affected by the described abscess. Choice C (Superficial transverse perineal muscles) is incorrect. The superficial transverse perineal muscles attach to the ischial tuberosities and help to support and stabilize the superficial perineal region. The muscles are, however, located in the superficial perineal space, which is inferior to the perineal membrane, so they would most likely not be affected by this abscess in the deep perineal space. Choice D (Bulbospongiosus muscles) is incorrect. The bulbospongiosus muscles envelop and com- press the bulb of the penis and corpus spongiosum to aid in penile erection by increasing pressure on erectile tissue in the root of the penis. These muscles also support and fix the male pelvic floor and play an important role in emptying the penile (spongy) urethra of residual urine or semen. However, these muscles are located in the superficial perineal space, which is inferior to the perineal membrane, so the bulbospongiosus muscles would most likely not be affected by this abscess in the deep perineal space. Choice E (Ischiocavernosus muscles) is incorrect. The ischiocavernosus muscles are paired muscles covering the crura of the penis and the associated erectile tissue, the corpora cavernosa. These muscles force blood within the crura into the distal parts of the corpora cavernosa and compress venous outflow via the deep dorsal vein of the penis. Both of these actions increase the turgidity (firm dis- tension) of the penis during erection. However, these muscles are located in the superficial perineal space, which is inferior to the perineal membrane, so the ischiocavernosus muscles would most likely not be affected by this abscess in the deep perineal space.

A 52-year-old man riding his bicycle is struck by a passing car and thrown into a wall alongside the road. The damage this man sustained was to his pubic symphysis, anterior sacroiliac ligament, and sacrospinous ligament. Which of the following muscles is most likely damaged in company with the torn sacrospinous ligament? (A) Coccygeus (B) External anal sphincter (C) Iliococcygeus (D) Pubococcygeus (E) Puborectalis

The answer is A: Coccygeus. The sacrospinous ligament runs from the pelvic surface of the lower sacrum to the ischial spine. It aids in resisting loading-induced sacral rotation that could disarticulate the sacroiliac joint. The coccygeus muscle, the most posterior component of the pelvic diaphragm, attaches to the ischial spine and sacrospinous ligament. Often, the muscle is so well intertwined with the ligament that it is difficult to differentiate the two. Thus, damage to the ligament typically also includes damage to the muscle. Developmentally, the epimysium of the coccygeus muscle modifies and thickens to form the sacrospinous ligament, which underscores the inti- mate relation between these two structures. Choice B (Exter- nal anal sphincter) is incorrect. The external anal sphincter is a multilayered skeletal muscle bundle encircling the lower anal canal. It is entirely subcutaneous, having no bony or liga- mentous attachments. Choice C (Iliococcygeus) is incorrect. This component of the pelvic diaphragm originates from the tendinous arch of the obturator fascia on the lateral wall of the true pelvis. In the fetus, the muscle originally attaches to the pelvic brim along the arcuate line of the ilium, explaining the ilio-part of its name. During development, it shifts down- ward onto the thickened obturator fascia to reach its final position. Choice D (Pubococcygeus) is incorrect. This ante- rior component of the pelvic diaphragm originates from the pelvic side of the pubic bone. It is at the opposite end of the pelvic diaphragm from the sacrospinous ligament. Choice E (Puborectalis) is incorrect. This part of the pelvic diaphragm originates from the pubic bone and forms a loop around the anorectal junction with its opposite side mate. It is not related to the sacrospinous ligament.

6 As part of a physical examination to evaluate muscle function in the hand, a physician holds the proximal interphalangeal joint of his patient's index finger in the extended position and instructs him to try to flex the distal interphalangeal joint. Which of the following muscles is the doctor testing? (A) Flexor digitorum profundus (FDP) (B) Extensor indicis (C) First lumbrical (D) First dorsal interosseous (E) Flexor digitorum superfi cialis (FDS)

The answer is A: Flexor digitorum profundus (FDP). Flexion of the distal interphalangeal joint in digits 2 to 5 is produced by the FDP. The actions of this muscle are being tested in this illustration. Choice B (Extensor indicis) is incorrect. The extensor indicis extends the index fi nger (digit 2), which enables this fi nger to extend independent of the other fi ngers. Because the muscle arises from the distal third of the ulna and the interosseous membrane, it also acts to extend the hand at the wrist. The extensor indicis muscle is not involved in fl exion of the distal interphalangeal joint, which is being tested in this patient. Choice C (First lumbrical) is incorrect. The fi rst lumbrical muscle extends the interphalangeal joints of the index (second) fi nger and fl exes the metacarpophalangeal joint of the same fi nger. The fi rst lumbrical is an intrinsic hand muscle that arises off the tendon of the fl exor digitorum profundus and inserts into the extensor expansion of the index fi nger. This muscle is not involved with fl exion of the distal interphalangeal joint. Choice D (First dorsal interosseous) is incorrect. The primary movement of the fi rst dorsal interosseous is abduction of the index fi nger. However, because it inserts into the extensor expansion, it also extends the interphalangeal joints of the index (second) fi nger and fl exes the metacarpophalangeal joint of the same fi nger. This muscle is not involved with fl exion of the distal interphalangeal joint. Choice E (Flexor digitorum superfi cialis [FDS]) is incorrect. The FDS acts at the proximal interphalangeal joint in digits 2 to 5 and infl uences the distal interphalangeal joint by binding the tendons of the FDP. However, when the proximal interphalangeal joint is held in extension, the infl uence of the FDS is eliminated, allowing testing of only the FDP.

Which structure is derived from the same embryonic primordium as the kidney? (A) Gonad (B) Epidermis (C) Pineal gland (D) Liver (E) Adrenal medulla

The answer is A: Gonad. -Both the kidneys and the gonads are derived from the intermediate mesoderm. This longitudinal dorsal ridge of mesoderm forms the urogenital ridge, which is involved with the formation of the future kidneys and gonads. -The liver is derived from the lateral plate mesoderm (specifically the splanchnic mesoderm), not the intermediate mesoderm. -The lateral plate mesoderm is a thin plate of mesoderm in which large spaces (intraembryonic coelom) form. These spaces coalesce and divide the lateral plate mesoderm into the intraembryonic somatic mesoderm and the intraembryonic splanchnic (visceral) mesoderm.

The given axial T2-weighted MRI of the prostate of a 50-year- old man was obtained via an endorectal coil. An image shows prostate cancer with extracapsular spread, which damages the neighboring hypogastric plexuses. Lesions of these nerves are most likely to result in which of the following deficits? (A) Inability to control the detrusor muscle (B) Inability to control the external anal sphincter (C) Chronic increased motility in the sigmoid colon (D) Reduced sensation on the glans penis (E) Loss of vasomotor control in the lower limb

The answer is A: Inability to control the detrusor muscle. The hypogastric plexuses are an extensive network of visceral motor (autonomic—both sympathetic and parasympathetic) and visceral sensory fibers across the true pelvis. The pelvic splanchnic nerves send presynaptic parasympathetic fibers through the inferior hypogastric plexus to the urinary bladder, where they supply the detrusor muscle in the wall of the bladder and the internal urethral sphincter in the neck of the bladder. The aortic (intermesenteric) plexus sends sympathetic fibers descending into the hypogastric plexuses and on to the bladder and internal urethral sphincter. Thus, damage within the lower part of the hypogastric plexuses will compromise autonomic control of urination. The axial T2-weighted MRI demonstrates a low signal intensity adenocarcinoma (indicated by the black arrow) arising in the normally high intensity peripheral zone of the prostate. The prostatic adenocarcinoma damages the hypogastric plexus and leads to an inability to control the detrusor muscle of the bladder. Choice B (Inability to control the external anal sphincter) is incorrect. The external anal sphincter is a skeletal muscle complex innervated by the inferior anal (rectal) branches of the pudendal nerve. The pudendal nerve originates within the pelvis from anterior primary rami of spinal nerves S2-4, and immediately exits the pelvis through the greater sciatic foramen. It is not a component of the hypogastric plexuses. Choice C (Chronic increased motility in the sigmoid colon) is incorrect. Parasympathetic fibers from the pelvic splanchnic nerves course through the inferior hypogastric plexus and ascend posterior to the peritoneum to reach the descending colon and sigmoid colon. Their normal function is to stimulate peristalsis in these organs. Thus, loss of parasympathetic input results in decreased motility here. Choice D (Reduced sensation on the glans penis) is incorrect. Sensation on the glans penis (and glans clitoris) is conveyed by general sensory fibers in the dorsal nerve of the penis (or clitoris), a terminal branch of the pudendal nerve. As noted above, the pudendal nerve is not related to the hypogastric plexuses. Choice E (Loss of vasomotor control in the lower limb) is incorrect. Sympathetic fibers that govern vasomotor function in the vessels of the lower limb originate in the sympathetic chain, pass into the sacral plexus, and run into the lower limb via the lower limb branches of the sacral plexus (e.g., sciatic nerve). This pathway is essentially separate from the hypogastric plexuses.

A 25-year-old woman suffers a severe back injury in an automobile accident. Thorough examination in the ER reveals a fracture dislocation of T12 on L1 with a complete spinal cord transection that severes the sacral segments of the spinal cord. This patient will likely experience which of the following outcomes? (A) Loss of voluntary control of defecation (B) Intact ability to achieve erection of the clitoris (C) Intact voluntary control of micturition (D) Loss of ovulation (E) Loss of sensation in the skin overlying the urinary bladder

The answer is A: Loss of voluntary control of defecation. The sacral part of the spinal cord (segments S2-4) gives rise to the pudendal nerve. This nerve leaves the pelvic cavity and passes into the perineum, where it has a wide distribution. The first major branch of the pudendal nerve is the inferior anal (rec- tal) nerve to the inferior end of the anal canal, external anal sphincter, and perianal skin. The external anal sphincter is the voluntary anal sphincter, controlled by somatic motor fibers. It acts to constrict the end of the anal canal, resisting defecation. Thus, loss of the sacral segments of the spinal cord results in loss of voluntary control of defecation, as well as loss of sensation in the anal canal inferior to the pectinate line and in the perianal skin. Involuntary control of defecation is governed by the internal anal sphincter, a thickening of the circular smooth muscle layer in the wall of the upper part of the anal canal, which is controlled by autonomic (visceral motor) fibers. Choice B (Intact ability to achieve erection of the clitoris) is incorrect. Erection of the clitoris (and penis) is controlled by parasympathetic outflow from the sacral segments of the spinal cord (S2-4). Thus, destruction of the sacral cord results in loss of erection rather than intact control. Choice C (Intact voluntary control of micturition) is incor- rect. The pudendal nerve also controls the external sphincter muscle of the urethra, which is the voluntary sphincter that regulates urination (micturition), via its perineal branches. Thus, loss of the sacral spinal cord segments will cause loss of voluntary control of urination rather than intact voluntary control. Choice D (Loss of ovulation) is incorrect. Ovulation is not controlled by direct neural input. Thus, loss of the sacral spinal cord will not affect ovulation. Choice E (Loss of sensa- tion in the skin overlying the urinary bladder) is incorrect. The skin overlying the urinary bladder is that of the pubic (suprapubic; hypogastric) region of the lower abdominal wall. Cutaneous sensation here is conveyed by the ilioinguinal and iliohypogastric nerves, both branches of the first lumbar seg- ment of the spinal cord. Because only the sacral segments of the spinal cord were destroyed by the transection of the spinal cord, this sensation in the pubic region of the lower abdomi- nal wall would remain intact. Remember, the medullary cone (conus medullaris) of the spinal cord typically lies within the T12-L3 vertebral levels, so only the sacral segments of the spi- nal cord were destroyed in this injury.

1 Physical examination of a 40-year-old man injured in an automobile accident indicates that he has suffered nerve damage affecting his left upper limb. The patient exhibits significant weakness when pronating his left forearm and fl exing his left wrist. What nerve is most likely damaged? (A) Median nerve (B) Ulnar nerve (C) Superficial branch of the radial nerve (D) Deep branch of the radial nerve (E) Musculocutaneous nerve

The answer is A: Median nerve. The median nerve controls pronation through the actions of the pronator teres and pronator quadratus muscles in the anterior compartment of the forearm. It also controls much of fl exion of the wrist and lateral digits via the actions of most of the other muscles in that compartment. Choice B (Ulnar nerve) is incorrect. The ulnar nerve controls 1½ muscles in the anterior compartment of the forearm (fl exor carpi ulnaris and the ulnar half of the fl exor digitorum profundus) and most of the intrinsic muscles of the hand. However, neither of these 1½ forearm muscles produces pronation. Choice C (Superfi cial branch of the radial nerve) is incorrect. The superfi cial branch of the radial nerve is entirely cutaneous, carrying sensation from the dorsolateral part of the hand. So, cutting this nerve would not result in weakness in pronation of the forearm or fl exion of the wrist. Choice D (Deep branch of the radial nerve) is incorrect. The deep branch of the radial nerve supplies the posterior compartment of the forearm. It infl uences supination via motor control of the supinator muscle, but not pronation. Choice E (Musculocutaneous nerve) is incorrect. The musculocutaneous nerve supplies the anterior compartment of the arm, and then continues distally via its termination as the lateral cutaneous nerve of the forearm. It contributes signifi cantly to control of supination by its innervation of the biceps brachii muscle, but has no effect on wrist fl exion

47 Organization of the axillary lymph nodes into Levels I, II, and III for breast cancer treatment is based on the location of the nodes relative to which of the following muscles? (A) Pectoralis minor (B) Pectoralis major (C) Latissimus dorsi (D) Serratus anterior (E) Subscapularis

The answer is A: Pectoralis minor. The pectoralis minor muscle forms a portion of the anterior axillary wall and is used as a marker to organize the contained groups of axillary lymph nodes. In a clinical context, the axillary nodes are organized into three major groups (Level 1, 2, and 3) that denote the progressive lymph drainage of the breast. Level 1 (anterior; pectoral) nodes are located along the lateral border of the pectoralis minor. Level 2 (central; deep) nodes are situated in the center of the axilla, deep (posterior) to the pectoralis minor. Level 3 (apical; medial) nodes are positioned in the apex of the axilla, superior to the upper medial border of the pectoralis minor. Detection of cancer cells in each increasing number level of nodes indicates progressively greater metastasis of the disease out of the breast. Choice B (Pectoralis major) is incorrect. The pectoralis major and minor form the anterior axillary wall. However, the pectoralis major muscle is not used to classify the positions of the axillary lymph nodes or levels of metastasis. Choice C (Latissimus dorsi) is incorrect. The latissimus dorsi muscle helps to form the posterior wall of the axilla along with the subscapularis and teres major muscles. Due to its posterior location, it is not an important landmark for classifi cation of axillary lymph nodes in breast cancer. Choice D (Serratus anterior) is incorrect. This muscle contributes to the formation of the medial wall of the axilla, along with the thoracic cage. This muscle is not an important landmark for classifi cation of axillary lymph nodes in breast cancer. Choice E (Subscapularis) is incorrect. The subscapularis muscle helps to form the posterior wall of the axilla along with the latissimus dorsi and teres major muscles. Due to its posterior location, this muscle is not an important landmark for classifi cation of axillary lymph nodes in breast cancer.

32 In a fracture of the surgical neck of the humerus, what artery is most likely damaged? (A) Posterior circumflex humeral artery (B) Brachial artery (C) Deep brachial (profunda brachii) artery (D) Subscapular artery (E) Superior ulnar collateral artery

The answer is A: Posterior circumfl ex humeral artery. The radiograph shows a fracture of the surgical neck of the humerus. The posterior humeral circumfl ex artery, accompanied by the axillary nerve, lies against the posterior aspect of the surgical neck as it passes into the quadrangular space of the shoulder. This fracture places both of these structures in immediate danger. Choice B (Brachial artery) is incorrect. The brachial artery travels down the midline of the arm, close to the median nerve. It is not immediately endangered by the fracture of the surgical neck. Choice C (Deep brachial, or profunda brachii, artery) is incorrect. The deep brachial artery wraps tightly around the midshaft of the humerus, in the radial (spiral) groove. It, along with the radial nerve with which it travels, would be endangered by a fracture through the radial groove of the humerus. Choice D (Subscapular artery) is incorrect. The subscapular artery is the largest branch of the axillary artery. It descends along the axillary border of the scapula and is not in contact with the humerus. Choice E (Superior ulnar collateral artery) is incorrect. This artery is a branch of the brachial artery. It descends through the arm, moves into company with the ulnar nerve, and takes a close relation to the posterior aspect of the medial epicondyle of the humerus. This vessel would be endangered by a fracture of the medial epicondyle rather than a surgical neck fracture.

A 34-year-old woman presents with pain and discomfort related to an abscess in the posterior aspect of her left labium majus. The abscess is determined to be derived from a greater vestibular (Bartholin) gland cyst. What group of lymph nodes should be checked first for lymphadenitis? (A) Superficial inguinal (B) Deep inguinal (C) Lumbar (D) External iliac (E) Internal iliac

The answer is A: Superficial inguinal. Superficial inguinal lymph nodes, which are located in the subcutaneous tissue near the termination of the great saphenous vein, receive lymphatic drainage from the lower abdominal wall, buttock, lower limb, and all of the perineum and external genitalia, except for the glans clitoris. Therefore, the superficial inguinal lymph nodes should be checked initially for lymphadenitis in the case of a cyst located on the labium majus. Choice B (Deep inguinal) is incorrect. Deep inguinal lymph nodes, which are located deep to the fascia lata and medial to the femoral vein, receive lymphatic drainage directly from the glans clitoris, deep structures of the lower limb, and indirect lymphatic drainage from the superficial inguinal lymph nodes. Because the lymphatic drainage of the posterior labium majus drains to the superficial inguinal lymph nodes initially, these nodes should be checked for lymphadenitis before the deep inguinal nodes. Choice C (Lumbar) is incorrect. Lumbar lymph nodes, which are located anterior to the lumbar vertebrae and sur- rounding the inferior vena cava and abdominal aorta, receive lymphatic drainage directly from the posterior abdominal wall, gonads (ovaries and testes), kidneys, ureters, uterus, and uterine tubes. Because the lymphatic drainage of the posterior labium majus drains to the superficial inguinal lymph nodes initially, these nodes should be checked for lymphadenitis before the lumbar nodes. Choice D (External iliac) is incor- rect. The external iliac lymph nodes, which are associated with the external iliac vein, receive lymphatic drainage from pelvic organs in direct contact with the peritoneum, including the superior bladder and superior pelvic ureters. Because the lymphatic drainage of the posterior labium majus drains to the superficial inguinal lymph nodes initially, these nodes should be checked for lymphadenitis before the external iliac nodes. Choice E (Internal iliac) is incorrect. The internal iliac lymph nodes, which are located along the internal iliac artery and its branches, receive lymphatic drainage from the pelvic vis- cera, gluteal region, and deep parts of the perineum. Because the lymphatic drainage of the posterior labium majus drains to the superficial inguinal lymph nodes initially, these nodes should be checked for lymphadenitis before the internal iliac nodes.

4 As a result of chronic stress associated with an intense high school weight-lifting program, a 15-year-old boy suffers an avulsion fracture of the greater tubercle of the humerus. In the ER, he displays difficulty initiating abduction of the upper limb. Which of the following muscles was involved in this fracture? (A) Supraspinatus (B) Long head of biceps brachii (C) Long head of triceps (D) Subscapularis (E) Infraspinatus

The answer is A: Supraspinatus. The greater tubercle of the humerus is the insertion site of three (of the four) rotator cuff muscles: supraspinatus, infraspinatus, and teres minor. Avulsion of this structure could result in detachment of any of these rotator cuff muscles, depending upon the size and scope of the fracture. However, the wrestler is unable to initiate abduction of the upper limb, which implies damage to the supraspinatus muscle. Choice B (Long head of biceps brachii) is incorrect. This muscle originates from the supraglenoid tubercle of the scapula and passes between the greater and lesser tubercles of the humerus, in the intertubercular (bicipital) groove. Detachment of the tendon of this muscle causes the biceps brachii to bulge in the anterior arm. Avulsion of the biceps brachii muscle is not related to the greater tubercle of the humerus. Choice C (Long head of triceps) is incorrect. The long head of the triceps brachii muscle originates from the infraglenoid tubercle of the scapula and inserts on the olecranon process of the ulna. It would not be involved in avulsion of the greater tubercle of the humerus. Choice D (Subscapularis) is incorrect. The fourth rotator cuff muscle, subscapularis, inserts onto the lesser tubercle of the humerus, so it would not be directly involved with this avulsion injury. Choice E (Infraspinatus) is incorrect. The infraspinatus muscle does insert onto the middle aspect of the greater tubercle of the humerus; however, damage to this muscle would result in weakness in external rotation at the shoulder joint, not the problems with abduction seen in this patient.

36 A 56-year-old woman was stopped at a light when her car was rear-ended by another car. She had her right arm on the steering wheel, and the impact caused forced flexion at her elbow. Several months later, she comes to her physician complaining of numbness and a "pins and needles" sensation in her right little finger when she talks on the phone, rests her head on her right hand at work, or spends most of her day typing at work. She also notices the quality of her typing and her ability to play the violin have diminished. Which nerve is compressed at what location? (A) Ulnar nerve in the elbow (B) Ulnar nerve in the wrist (C) Median nerve in the wrist (D) Median nerve in the elbow (E) Median nerve in the axilla

The answer is A: Ulnar nerve in the elbow. Situations in which peripheral nerves are compressed or otherwise entrapped where they pass through narrow spaces ("tunnels") in muscles and/or osseo-fascial units are generally referred to as "tunnel syndromes." Such conditions may result in periodic or constant motor and/or sensory defi cits. The ulnar nerve crosses the elbow in a narrow space between the olecranon process and the medial epicondyle of the humerus, on the posteromedial (ulnar) side of the joint. The ulnar nerve can be compressed between these bony landmarks or between the humeral and ulnar heads of the attachment of the fl exor carpi ulnaris. Compression of the ulnar nerve within these areas leads to "cubital tunnel syndrome." The symptoms are exacerbated during events where fl exion of the elbow narrows these passageways and compresses the ulnar nerve at the elbow joint. The patient's cubital tunnel syndrome would explain the paresthesia and numbness on the medial aspect of the hand and the diminished fi ne motor control of the intrinsic hand muscles. Her condition originated due to the forced fl exion of the elbow in the motor vehicle accident, which compressed the ulnar nerve in the cubital tunnel. Choice B (Ulnar nerve in the wrist) is incorrect. The ulnar nerve can be compressed between the pisiform and hook of the hamate at the wrist in a condition termed "ulnar canal syndrome" or "Guyon tunnel syndrome." This entrapment syndrome presents with similar signs and symptoms as seen in this patient. However, the ability to fl ex the wrist would not be affected. In this patient, trauma to the wrist was not reported. Choice C (Median nerve in the wrist) is incorrect. The median nerve is often compressed at the wrist in "carpal tunnel syndrome" because this nerve travels deep to the transverse carpal ligament (fl exor retinaculum of the wrist) within the carpal tunnel. This entrapment syndrome presents with paresthesia of the lateral fi ngers as well as an inability to oppose the thumb and a wasting of the thenar eminence. These symptoms were not reported in this patient. Choice D (Median nerve in the elbow) is incorrect. The median nerve is not usually compressed by forced fl exion of the elbow because it lies loosely on the fl exor surface of the joint deep to the bicipital aponeurosis. The median nerve can be compressed in the proximal forearm as it passes between the two heads of the pronator teres. However, the symptoms in this patient were due to forced fl exion of the elbow and involve the ulnar nerve. Choice E (Median nerve in the axilla) is incorrect. The median nerve would not be damaged in the axilla without signifi cant trauma. Damage to the median nerve in the axilla would lead to weakness in fl exing the wrist, loss of pronation of the forearm, and wasting of the thenar muscles. However, these signs and symptoms were not reported.

2 A 50-year old man falls off a ladder while cleaning his windows, landing on the ground as seen in the given drawing. He does not seek medical aid, believing his general soreness will go away with time. However, after several months, he develops a postural deformity of his left upper limb that includes an adducted, medially rotated, and extended shoulder, extended elbow, and pronated forearm. The injury and subsequent condition reflect damage to what structure? (A) Upper trunk of the brachial plexus (B) Lower roots of the brachial plexus (C) Posterior divisions of the brachial plexus (D) Medial cord of the brachial plexus (E) Lateral root of the median nerve

The answer is A: Upper trunk of the brachial plexus. The illustration shows an injury in which the cervicobrachial angle (the angle between the neck and shoulder) is stretched widely. This abnormal impact eventually results in the postural presentation of a "waiter's tip" deformity (Erb-Duchenne palsy). This combination of injury and postural deformity is related to damage to both the C5 and C6 roots or upper trunk of the brachial plexus. The C5 and C6 roots converge to form the upper trunk of the brachial plexus and contribute heavily to the suprascapular, axillary, and musculocutaneous nerves. The suprascapular nerve supplies the supraspinatus and infraspinatus muscles. The axillary nerve controls the deltoid and teres minor muscles. The musculocutaneous nerve supplies the anterior compartment of the arm (coracobrachialis, biceps brachii, brachialis muscles). Therefore, a signifi cant weakness in abduction and lateral rotation of the shoulder, fl exion of the shoulder and elbow, and supination of the forearm would result from this brachial plexus injury. The ultimate postural deformity is a contracture effect in which the intact muscles act unopposed to draw the limb into a position that is the opposite of the actions of the affected muscles. Choice B (Lower roots of the brachial plexus) is incorrect. The lower roots (C8, T1) have a strong projection into the ulnar nerve. Trauma here would result in an ultimate postural deformity of "claw hand" due to the loss of fl exion of the medial digits. That injury does not match this patient's clinical presentation. Choice C (Posterior divisions of the brachial plexus) is incorrect. The posterior divisions supply the radial, axillary, upper and lower subscapular, and thoracodorsal nerves. The primary postural effect resulting from trauma here would be a case of "wrist drop" expressed due to loss of the extensor muscles innervated by the radial nerve. Choice D (Medial cord of the brachial plexus) is incorrect. The medial cord projects into the ulnar and median nerves. Damage to the ulnar nerve will result in "claw hand." The classic median nerve deformity of "ape hand" probably would not be realized because the median nerve also receives a strong input from the lateral cord. Choice E (Lateral root of the median nerve) is incorrect. The median nerve is formed by lateral and medial roots derived from the lateral and medial cords, respectively. Trauma to either root would weaken the territory of the median nerve but likely not result in the classic median nerve postural defect of "ape hand." The lateral root does not contribute to the suprascapular and axillary nerves, which have been affected in this patient.

The sagittal MRI of a 45-year-old woman revealed severe uterine fibroids (noncancerous tumors) within the walls of the uterus. She underwent a complete hysterectomy, and the uterine artery was ligated near the junction of the uterus and vagina during the procedure. What structure would most likely be at risk during the ligation of the uterine artery? (A) Ureter (B) Pudendal nerve (C) Round ligament of the uterus (D) Obturator artery (E) Ovarian artery

The answer is A: Ureter. As it descends into the pelvis, the ureter travels anterior to the external iliac artery just distal to the bifurcation of the common iliac vessels. It continues ante- rior and medial to pass inferior to the uterine artery, which is enveloped by the transverse cervical (cardinal) ligament, near the lateral aspect of the cervix. When ligating the uterine artery during a hysterectomy (surgical excision of the uterus), a surgeon must positively identify the ureter and exclude it from the ligature. To accomplish this task and avoid iatrogenic injury to the ureter, a surgeon will ligate the uterine artery as close to the cervix as possible. The relationship of the ureter to the uterine artery can be remembered with the mnemonic "The water runs under the bridge," in which the water equals urine traveling within the ureter and the bridge is the uter- ine artery as it travels horizontally toward the uterine cervix. Choice B (Pudendal nerve) is incorrect. The pudendal nerve is located inferior to the pelvic diaphragm where it inner- vates a majority of the perineum. It arises from the ventral primary rami of spinal levels S2-4 but quickly exits the true pelvic cavity via the greater sciatic foramen. It is not located in close proximity to the uterine artery, so it is unlikely the pudendal nerve would be damaged in this procedure. Choice C (Round ligament of the uterus) is incorrect. The round liga- ment of the uterus extends from the superior part of the body of the uterus (near the uterotubal junction) to the labium majus via the inguinal canal. It is not located in close proxim- ity to the uterine artery, so it is unlikely the round ligament of the uterus would be damaged in this procedure. Choice D (Obturator artery) is incorrect. The obturator artery emerges from the anterior division of the internal iliac artery near the uterine artery; however, the paths of these arteries diverge as the arteries travel distally. The uterine artery travels medially toward the uterine cervix, whereas the obturator artery trav- els along the lateral pelvic wall toward the obturator foramen. Because the ligation of the uterine artery was located near the junction of the uterus and vagina, it is unlikely the obturator artery would be damaged. Choice E (Ovarian artery) is incor- rect. The ovarian artery travels along the lateral wall of the pelvis within the infundibulopelvic (suspensory) ligament of the ovary. It splits into tubal and ovarian branches, which later coalesce and anastomose with the proper uterine artery at the junction of the uterine tube and the fundus of the uterus. This location is well above the ligated uterine artery, so it is unlikely the ovarian artery would be damaged.

The urinary system develops from which of the embryonic sources labeled in the given diagram?

The answer is B: Intermediate mesoderm. The intermediate mesoderm is a small bridge of mesoderm that connects the paraxial mesoderm with the lateral plate mesoderm. It differ- entiates into urogenital structures, including the kidney paren- chyma and nephrons, and the gonads. Choice A (Paraxial mesoderm) is incorrect. The paraxial mesoderm is the thick- ened portion of mesoderm closest to the midline and neural tube. It becomes segmented and forms somites, which further differentiate into myotome, dermatome, and sclerotome units that form skeletal muscles, dermis, and most of the vertebral column, respectively. Choice C (Parietal [somatic] meso- derm) is incorrect. This derivative of the lateral plate meso- derm becomes associated with the wall of the amniotic sac. Ultimately, it forms (along with the ectoderm) the lateral body wall folds and gives rise to the dermis plus the bones and con- nectives tissues in the body wall and limbs. Choice D (Visceral [splanchnic] mesoderm) is incorrect. The visceral (splanchnic) mesoderm is the other derivative of the splitting of the lateral plate mesoderm. It joins with the endodermal lining of the yolk sac and forms the wall of the gut tube. Choice E (Yolk sac endoderm) is incorrect. This germ layer becomes associated with the visceral mesoderm to form the wall of the gut tube. Specifically, it forms the lining of the gut tube.

45 A 48-year-old woman is diagnosed with a malignant tumor in the superolateral quadrant of the right breast, including the axillary tail. If it metastasizes, this cancer will most likely spread first to which of the following locations? (A) Lateral axillary lymph nodes (B) Anterior axillary lymph nodes (C) Deep cervical lymph nodes (D) Parasternal lymph nodes (E) Contralateral breast lymph nodes

The answer is B: Anterior axillary lymph nodes. The primary lymphatic drainage route for the lateral half of the breast (including the axillary tail) plus part of the medial half of the breast is fi rst into the anterior (pectoral; Level 1) axillary lymph nodes. These lymph nodes are located along the lateral border of the pectoralis minor muscle. Further metastasis would spread progressively to the central axillary (Level 2) nodes, then the apical axillary (Level 3) nodes. Choice A (Lateral axillary lymph nodes) is incorrect. These nodes, located along the distal segment of the axillary vein, collect the lymphatic drainage from most of the upper limb. They drain to the central axillary (Level 2) nodes. Choice C (Deep cervical lymph nodes) is incorrect. These nodes are located in the neck, along the internal jugular vein, and collect lymph from the head and neck. Choice D (Parasternal lymph nodes) is incorrect. The parasternal (internal thoracic) nodes lie along the internal thoracic vessels within the chest. Most of the medial half of the breast sends its lymphatic drainage primarily to these nodes. Choice E (Contralateral breast lymph nodes) is incorrect. Small areas of the medial aspect of the breast may send an accessory lymphatic drainage across the midline to the contralateral nodes of the opposite breast. Due to the site of the tumor, this lymphatic drainage pattern is unlikely.

27 The lateral cord of the brachial plexus is named because it lies immediately lateral to which of the following structures? (A) Long head of the biceps brachii muscle (B) Axillary artery (C) Subclavian vein (D) Surgical neck of the humerus (E) Pectoralis minor muscle

The answer is B: Axillary artery. The brachial plexus is divided into fi ve geographic parts: Roots (or ventral rami of C5-T1), Trunks, Divisions, Cords, terminal Branches (or Nerves). These sections of the brachial plexus can be remembered with the mnemonics, "Real Truckers Drink Cold Beer" or "Remember Those Darn Cervical Nerves". The cords are named according to their important positional relationship to the second part of the axillary artery, deep to the pectoralis minor muscle. Here, the nerves form a cradle-like bed for this segment of the vessel as it passes through the axilla. Thus, the lateral cord is located lateral, the medial cord is medial, and the posterior cord is posterior to the axillary artery. Choice A (Long head of the biceps brachii muscle) is incorrect. The long head of the biceps brachii muscle lies lateral to all three cords of the brachial plexus. Choice C (Subclavian vein) is incorrect. This vessel is located well proximal to the cords of the brachial plexus, medial to the fi rst rib. Choice D (Surgical neck of the humerus) is incorrect. This part of the humerus is located distal and lateral to all three cords of the brachial plexus. Choice E (Pectoralis minor muscle) is incorrect. The pectoralis minor lies superfi cial (anterior) to the cords of the brachial plexus. This muscle divides the axillary artery into its three parts. Thus, it defi nes the second part of the artery and creates the situation for naming the cords of the plexus.

During micturition (urination), excitation of parasympathetic nerve fibers causes contraction of which of the following muscles? (A) Pubococcygeus muscle (B) Detrusor muscle (C) Internal urethral sphincter (D) Bulbospongiosus muscle (E) External urethral sphincter

The answer is B: Detrusor muscle. The smooth muscle walls of the urinary bladder are composed mainly of the detrusor muscle. Parasympathetic input during urination stimulates the detrusor to contract, thus emptying the bladder into the urethra. Choice A (Pubococcygeus muscle) is incorrect. The pubococcygeus muscle is a skeletal muscle that forms part of the levator ani in the pelvic diaphragm. It acts to support and lift the pelvic floor. Its contraction is evoked by somatic motor branches of the sacral plexus, including branches of the pudendal nerve. Choice C (Internal urethral sphincter) is incorrect. The internal urethral sphincter is a smooth muscle bundle located at the internal urethral orifice in the neck of the urinary bladder in males. During urination (in males), para- sympathetic stimulation causes the internal urethral sphincter to relax, allowing urine to flow into the urethra. An organized internal urethral sphincter is not present in females. Choice D (Bulbospongiosus muscle) is incorrect. The bulbospongiosus is a skeletal muscle enveloping the bulb of the penis in males and the individual vestibular bulbs in females. During urina- tion in males, contraction of the muscle squeezes the bulb, assisting in draining the penile urethra. However, the innerva- tion of the bulbospongiosus muscle is via somatic motor fibers of the perineal nerves. Choice E (External urethral sphincter) is incorrect. The external urethral sphincter is composed of skeletal muscle and acts as the voluntary urethral sphincter to constrict the urethra and prevent urination. The innerva- tion of the external urethral sphincter is controlled by somatic motor fibers in the perineal nerves.

9 On his downswing, an amateur golfer strikes the hard earth with his club and feels pain in his right wrist. During a subsequent physical examination, he complains of wrist pain that is exacerbated by gripping, displays point tenderness in his medial wrist, and complains of numbness and weakness in his pinky finger (fifth digit). What carpal bone is most likely fractured in this patient? (A) Capitate (B) Hamate (C) Lunate (D) Pisiform (E) Scaphoid

The answer is B: Hamate. The hamate bone is identifi ed by the white arrow in the given X-ray. This bone resides in the medial (ulnar) aspect of the distal row of carpal (wrist) bones. Stress fractures can occur to this bone, particularly within its hook, which appears as a radiodense oval on this radiograph. This type of fracture is frequently seen in golfers due to the positioning of the proximal aspect of the golf club within their grip. A fracture of the hamate results in pain, which is exacerbated by gripping and point tenderness in the skin overlying the bone. The numbness and weakness in the medial aspect of the hand, seen in this patient, are due to impingement of the ulnar nerve. Remember, the eight carpal (wrist) bones form two rows that contain four bones each. The proximal row (from lateral to medial) is composed of the scaphoid, lunate, triquetrum, and pisiform. The distal row (from lateral to medial) is composed of the trapezium, trapezoid, capitate, and hamate. To remember the carpal bones as listed, some students use the mnemonic: "Some Lovers Try Positions That They Can't Handle". In a standard posteroanterior plane fi lm, the medial aspect of the wrist can be deceiving to the untrained eye. Here, the pisiform overlies the triquetrum, and the seemingly single radiopacity might not be distinguished as two separate bones. Instead, the distinctive hook of the hamate bone may be mistaken for the pisiform. Choice A (Capitate) is incorrect. The capitate (L: head) is a head-shaped bone that is the largest of the carpal bones. Its position is noted by its articulation with the third metacarpal distally. Choice C (Lunate) is incorrect. The lunate (L: moon) is a moon-shaped bone between the scaphoid and triquetral bones. It articulates with the radius proximally, and its position makes it the most commonly dislocated carpal bone. Choice D (Pisiform) is incorrect. The pisiform (L: pea) is a pea-shaped sesamoid bone that lies on the palmar aspect of the triquetrum bone. Choice E (Scaphoid) is incorrect. The scaphoid (G: boat) is a boat-shaped bone that is the largest of the carpal bones located in the proximal row. Its articulation with the radius proximally makes it the most commonly fractured carpal bone, especially when a person falls and impacts an abducted, outstretched hand.

A 70-year-old patient with a history of hypercholesterolemia complains of severe cramps in his gluteal region despite no history of physical exertion in the past few days. An arteriogram shows atherosclerotic blockage leading to insufficient perfusion to the gluteal region. What artery is most likely occluded leading to his complications? (A) External iliac (B) Internal iliac (C) Femoral (D) Internal pudendal (E) Abdominal aorta

The answer is B: Internal iliac. The blood supply to the gluteal region is supplied by the superior and inferior gluteal arteries, which arise off the internal iliac artery. The internal iliac artery is occluded in this patient, and his symptoms are due to lack of blood perfusion to the gluteal muscles and surrounding tissue, which causes ischemia and severe pain in the affected (gluteal) region. Remember that the internal iliac artery and its branches supply the gluteal region as well as contents of the pelvis and the medial thigh. Choice A (External iliac) is incorrect. The external iliac artery supplies blood to the iliac region and the anterior abdominal wall. This artery becomes the fem- oral artery, the major lower limb artery, after it passes under the inguinal ligament. Atherosclerotic blockage of the external iliac would affect arterial supply to the hip and lower limb, not the gluteal region. Choice C (Femoral artery) is incorrect. The femoral artery is the continuation of the external iliac artery, which changes its name to the femoral artery after passing under the inguinal ligament. Atherosclerotic blockage of the femoral artery would affect the blood supply to the lower limb, except for the gluteal region. Choice D (Internal pudendal) is incorrect. The internal pudendal artery is a branch of the anterior division of the internal iliac artery, which supplies the anal and urogenital triangles of the perineum with its three major branches: inferior anal, perineal, and dorsal artery of the penis. Blockage of this artery would not be responsible for poor blood perfusion to the gluteal region; however, block- age of the internal iliac artery would lead to similar signs and symptoms in the arterial distribution of the internal pudendal artery. Choice E (Abdominal aorta) is incorrect. Atherosclerotic blockage of the abdominal aorta would lead to more widespread complications than simply pain in the gluteal region.

A painful abscess is located in the ischioanal fossa of a 39-year-old man. A CT reveals the abscess contains air and displaces the rectum to the right. Which of the following locations should the surgeon avoid during drainage of the abscess to prevent damage to the pudendal nerve and internal pudendal vessels? (A) Roof of the ischioanal fossa (B) Lateral wall of the ischioanal fossa (C) Medial wall of the ischioanal fossa (D) Posterior recess of the ischioanal fossa (E) Anterior recess of the ischioanal fossa

The answer is B: Lateral wall of the ischioanal fossa. The lateral wall of the ischioanal fossa is formed by the ischium and the overlying inferior part of the obturator internus muscle. The pudendal (Alcock) canal is a shallow depression on the surface of the obturator internus muscle formed by the invest- ing fascia of this muscle. The pudendal canal contains the pudendal nerve and the internal pudendal vessels along their entry into the ischioanal fossa. When draining an ischioanal abscess, a surgeon must avoid damage to the pudendal canal and its contents by avoiding the lateral wall of the ischioanal fossa. Remember that the ischioanal fossa is a large wedge- shaped space on each side of the anal canal located between the skin of the anal region and the pelvic diaphragm. This fascia-filled fossa supports the anal canal while simultaneously providing a space that accommodates the expansion of the anal canal during defecation. Choice A (Roof of the ischioanal fossa) is incorrect. The roof of the ischioanal fossa is formed by the levator ani muscle, which ascends laterally to the point where this muscular component of the pelvic diaphragm arises from the investing fascia of the obturator internus. The surgeon draining the ischioanal abscess should avoid the lat- eral wall, not the roof, of the ischioanal fossa to avoid dam- age to the pudendal nerve and the internal pudendal vessels. Choice C (Medial wall of the ischioanal fossa) is incorrect. The medial wall of the ischioanal fossa is also formed by the levator ani muscle and the external anal sphincter, which merge to form this medial wall. An incision in the vicinity of the medial wall of the ischioanal wall would not damage the pudendal nerve and the internal pudendal vessels specifically. However, it may damage the inferior anal (rectal) nerve and inferior rectal artery, which branch off of this neurovascular bundle. Choice D (Posterior recess of the ischioanal fossa) is incorrect. The posterior recess of the ischioanal fossa extends posterior to the sacrotuberous ligament and gluteus maximus muscle. A surgeon can avoid damage to the pudendal nerve and the internal pudendal vessels by avoiding the medial wall of this posterior recess. Choice E (Anterior recess of the ischioanal fossa) is incorrect. The anterior recess of the ischioanal fossa extends into the urogenital triangle of the perineum superior to the perineal membrane. When draining an ischioanal recess, a surgeon can avoid damage to the pudendal nerve and the internal pudendal vessels by avoiding the medial wall of this anterior recess.

23 A 65-year-old man is brought to the emergency room after being attacked in his office by a disgruntled co-worker. The attacker reportedly used a long, narrow-bladed letter-opener to inflict multiple stab wounds to the man's back. Physical examination shows a puncture wound in the posterior axillary fold. The patient presents with weakness in extension, adduction, and medial rotation of his arm. Which of the following muscles is most likely cut in this injury? (A) Pectoralis minor (B) Latissimus dorsi (C) Levator scapulae (D) Serratus anterior (E) Teres minor

The answer is B: Latissimus dorsi. The axilla is a large, pyramidal space between the side of the chest and the upper part of the brachium. Its major importance is as a passageway from the root of the neck to the upper limb. The axilla is demarcated by four walls: anterior, posterior, medial, lateral. The posterior wall is composed of the latissimus dorsi, teres major, and subscapularis muscles. The posterior axillary fold forms the palpable lower margin of the wall and is composed of the latissimus dorsi and teres major. The subscapularis is not part of the posterior axillary fold. Damage to the latissimus dorsi would severely hinder adduction, extension, and medial rotation of the arm. Choice A (Pectoralis minor) is incorrect. The pectoralis minor contributes to the formation of the anterior wall of the axilla. The pectoralis major forms the bulk of the anterior wall and creates the noticeable anterior axillary fold. Choice C (Levator scapulae) is incorrect. The levator scapulae muscle passes out of the neck to attach onto the superior angle of the scapula. It is far removed from any of the walls of the axilla and does not contribute to rotation and adduction of the arm. Choice D (Serratus anterior) is incorrect. The serratus anterior lies against the thoracic wall and forms much of the medial wall of the axilla along with the thoracic wall. It fi xes the scapula to the thoracic wall and has no affect on rotation of the arm. Choice E (Teres minor) is incorrect. The teres minor is located immediately above the teres major. However, it is not a component of the posterior wall of the axilla and normally contributes to lateral rotation of the arm.

A pelvic CT scan of a 65-year-old man who has been suffering from colon cancer reveals recurrent perirectal masses, which compress the pelvic splanchnic nerves. Damage to these nerves could affect the organs supplied by which of the following arteries? (A) Right colic artery (B) Left colic artery (C) Renal arteries (D) Testicular arteries (E) Lumbar arteries

The answer is B: Left colic artery. The pelvic splanchnic nerves provide the sacral parasympathetic outflow that sup- plies the gut tube below the left colic flexure, the pelvic viscera, and the perineum. Part of the distribution route for the para- sympathetic nerves to the descending colon, sigmoid colon, and upper rectum (i.e., to most of the hindgut) is to hitch-hike along the arterial branches that supply those organs. Because the left colic artery is the parent vessel to those structures, damage to the pelvic splanchnic nerves would affect the organs supplied by the left colic artery. Choice A (Right colic artery) is incorrect. The right colic artery carries parasympathetic fibers derived from the vagus nerve to the ascending colon. This vessel/organ pairing is outside the pelvic splanchnic territory. Choice C (Renal arteries) is incorrect. These vessels supply the kidneys and suprarenal glands. Again, any parasympathetic supply to these organs is derived from vagal branches that travel with the renal arteries. Choice D (Testicular arteries) is incorrect. The gonads do not receive autonomic innervation. Thus, it is questionable if the gonadal vessels convey any para- sympathetic fibers. Choice E (Lumbar arteries) is incorrect. The lumbar arteries supply structures in the posterior abdomi- nal wall (e.g., psoas muscles, vertebral column). Parasympa- thetic nerves do not supply the body walls and limbs. Thus, the lumbar arteries do not convey parasympathetic fibers.

20 A 50-year female equestrian is thrown from a startled horse and dragged by the reins, which were wrapped around her left wrist, for some distance. At the ER, she is experiencing pain and paresthesia in the axilla and medial aspect of her upper limb. Despite being left-handed, she has marked weakness in the movements of her dominant hand, especially abduction and adduction of the fingers. What structure was most likely damaged in this woman? (A) Upper trunk of the brachial plexus (B) Lower trunk of the brachial plexus (C) Posterior cord of the brachial plexus (D) Lateral cord of the brachial plexus (E) Long thoracic nerve

The answer is B: Lower trunk of the brachial plexus. This woman has experienced a lower brachial plexus injury due to forced abduction of the upper limb during the accident. This injury presents with numbness and paresthesia in the C8 and T1 dermatomes, which supply the axilla and medial aspect of her upper limb. These nerve roots primarily supply the medial cord of the brachial plexus, which creates the ulnar nerve. Due to damage to the ulnar nerve, she is experiencing weakness in the movement of her left hand. The abduction and adduction of the fi ngers are controlled by the deep branch of the ulnar nerve by supplying the dorsal interosseous and palmar interosseous muscles, respectively. Choice A (Upper trunk of the brachial plexus) is incorrect. Damage to the upper trunk of the brachial plexus results in Erb-Duchenne palsy ("waiter's tip malformation"). In this injury, the patient presents with signifi cant weakness in abduction and lateral rotation of the shoulder, fl exion of the shoulder and elbow, and supination of the forearm. Choice C (Posterior cord of the brachial plexus) is incorrect. The posterior cord of the brachial plexus gives rise to the axillary and radial nerves. Damage to the axillary nerve causes weakness in abduction of the shoulder due to loss of the deltoid muscle. Damage to the radial nerve results in inability to extend at the elbow or wrist (leading to "wrist drop"). However, this patient had signifi cant weakness in the movements of the hand, which implies damage to the contributions of the ulnar nerve. Choice D (Lateral cord of the brachial plexus) is incorrect. Damage to the lateral cord of the brachial plexus results in signs and symptoms similar to those seen in Erb-Duchenne palsy, or the "waiter's tip malformation." In this injury, the patient presents with signifi cant weakness in abduction and lateral rotation of the shoulder, fl exion of the shoulder and elbow, and supination of the forearm. Choice E (Long thoracic nerve) is incorrect. A lesion of the long thoracic nerve leads to a "winged scapula" due to the subsequent paralysis of the serratus anterior muscle. Additionally, the affected arm cannot be abducted above the horizontal plane because the serratus anterior is not available to superiorly rotate the glenoid cavity of the scapula to allow full abduction. This defi cit was not seen in this patient.

28 "Pronator teres syndrome" is a condition in which one of the following nerves is excessively compressed where it passes between the two heads of the pronator teres muscle. Which of the following nerves is entrapped? (A) Deep branch of radial nerve (B) Median Nerve (C) Deep branch of ulnar nerve (D) Superfi cial branch of ulnar nerve (E) Musculocutaneous nerve

The answer is B: Median nerve. Each of the fi ve terminal branches of the brachial plexus (musculocutaneous, median, ulnar, radial, and axillary nerves) passes through a muscular or osseofascial tunnel at some point in its distribution, where it may be subject to entrapment in a tunnel syndrome. The pronator teres muscle arises via two heads, one from the medial epicondyle of the humerus and the other from the coronoid process of the ulna, with a tendinous arch connecting them. The median nerve exits the cubital fossa and enters the forearm by passing between these heads, where it may be unduly compressed in a pronator teres syndrome. This condition would infl uence much of the median nerve territory in the forearm plus the entire median nerve territory in the hand. Choice A (Deep branch of the radial nerve) is incorrect. The radial nerve descends from the arm into the cubital fossa, where it divides into superfi cial and deep branches. The deep branch of the radial nerve pierces the supinator muscle, winds around the proximal end of the radius within the substance of that muscle, and passes into the deep posterior compartment of the forearm as the posterior interosseous nerve. The nerve may be entrapped within the supinator, resulting in a supinator syndrome. Such a condition would affect the deeper, more distal extensor muscles arising in the forearm and some sensory areas in the wrist joints. Choice C (Deep branch of the ulnar nerve) is incorrect. The ulnar nerve enters the hand superfi cial to the fl exor retinaculum, runs through a groove between the pisiform and hook of the hamate (Guyon canal), and divides into superfi cial and deep branches at the base of the hypothenar eminence. The deep branch curls deeply there and enters the deep lying adductor/interosseous compartment in the palmar aspect of the hand. Compression of the ulnar nerve in Guyon canal may cause a Guyon tunnel syndrome, which affects the hypothenar muscles, medial two lumbricals, all interossei, adductor pollicis, and a large sensory area on both palmar and dorsal sides of the hand. Choice D (Superfi cial branch of the ulnar nerve) is incorrect. The superfi cial branch of the ulnar nerve does not enter a tunnel and is not subject to a tunnel syndrome. This nerve supplies the palmaris brevis muscle but is mostly cutaneous across the palmar and dorsal aspects of the medial third of the hand. Choice E (Musculocutaneous nerve) is incorrect. This nerve penetrates the coracobrachialis muscle, supplies the three fl exor muscles in the anterior compartment of the arm, and continues into the forearm as the lateral cutaneous nerve of the forearm. Entrapment of the nerve within the coracobrachialis is rare.

A medical resident is charged with catheterizing the urethra in a 68-year-old man in order to drain urine from the bladder. Which of the following sequence of structures correctly lists the order of the structures encountered when passing a catheter through the external urethral orifice to the urinary bladder? (A) Navicular fossa, spongy urethra, membranous urethra, ductus deferens, prostatic urethra (B) Navicular fossa, spongy urethra, membranous urethra, prostatic urethra, intramural urethra (C) Spongy urethra, membranous urethra, prostatic urethra, intramural urethra, ureter (D) Spongy urethra, membranous urethra, ejaculatory duct, ductus deferens, prostatic urethra (E) Intramural urethra, prostatic urethra, membranous urethra, spongy urethra, navicular fossa

The answer is B: Navicular fossa, spongy urethra, membranous urethra, prostatic urethra, intramural urethra. The urethra is a continuous tube that can be divided into four main parts: (1) spongy urethra, (2) membranous (intermediate) urethra, (3) prostatic urethra, (4) intramural urethra. The urethra terminates (or begins for catheter insertion) at the external urethral orifice. The spongy (penile) urethra is the longest part, running though the corpus spongio- sum in the penis. The distal end of the spongy urethra is the expanded navicular fossa within the glans penis. The membra- nous (intermediate) urethra passes through the deep perineal compartment and is the narrowest segment of the urethra. The prostatic urethra, which is the widest and most distensible part of the urethra, traverses the prostate gland. The final segment, the intramural (preprostatic) urethra, extends through the neck of the urinary bladder. Because the urethral wall is thin, it can be ruptured easily by inserted instruments. Thus, understand- ing and properly navigating the curved pathway of the ure- thra are critical. Choice A (Navicular fossa, spongy urethra, membranous urethra, ductus deferens, prostatic urethra) is incorrect. The ductus (vas) deferens is not part of the urethral pathway to the urinary bladder. The ductus deferens empties into the prostatic urethra after joining the duct of the seminal vesicle to form the ejaculatory duct. Choice C (Spongy urethra, membranous urethra, prostatic urethra, intramural urethra, ure- ter) is incorrect. The ureter is not part of the urethral pathway to the bladder. The paired ureters drain into the urinary bladder separately from the bladder drainage into the urethra. The two ureteric orifices plus the urethral orifice mark the angles of the trigone of the bladder. Choice D (Spongy urethra, membra- nous urethra, ejaculatory duct, ductus deferens, prostatic ure- thra) is incorrect. The ejaculatory duct and the ductus deferens are not parts of the urethral pathway to the urinary bladder. The ductus deferens joins with the duct of the seminal vesicle to form the ejaculatory duct, which empties into the prostatic urethra. Choice E (Intramural urethra, prostatic urethra, mem- branous urethra, spongy urethra, navicular fossa) is incorrect. This sequence is the correct order of urethral segments through which urine would leave the urinary bladder to exit the external urethral orifice, but the inverse order of structures encountered when inserting a catheter into the urinary bladder.

The notochord forms the initial axial skeletal element of the body and induces the formation of the neural plate. Which of the following structures is the sole postnatal remnant of the embryonic notochord? (A) Spinal cord (B) Nucleus pulposus (C) Rib cage (D) Anulus fibrosus (E) Spinal meninges

The answer is B: Nucleus pulposus. The only remnant of the notochord is the central core portion of each of the intervertebral discs, the nucleus pulposus. This hydrostatic structure is the component of the disc that herniates out of its normal position in cases of herniated ("slipped") discs.

29 The pulse of the radial artery is readily palpable where the vessel passes which of the following structures? (A) Across the anterior aspect of the lateral epicondyle of the humerus (B) Between the tendons of the palmaris longus and flexor carpi ulnaris (C) Lateral to the tendon of the fl exor carpi radialis (D) Superfi cial to the tendons of the extensor pollicis brevis and abductor pollicis longus (E) Superficial to the carpal tunnel

The answer is C: Lateral to the tendon of the fl exor carpi radialis. The most common location for measuring pulse rate is on the radial artery at the wrist. Here, the vessel lies on the anterior side of the distal end of the radius, lateral to the tendon of the fl exor carpi radialis. It is covered only by skin and a thin superfi cial fascia and can be palpated easily against the radius. Note the placement of the index and middle fi ngers of the physician in the given photo. Choice A (Across the anterior aspect of the lateral epicondyle of the humerus) is incorrect. The radial artery originates in the cubital fossa as one of the two terminal branches of the brachial artery (the ulnar artery is the other). The radial recurrent artery branches off the radial artery just below its origin and ascends across the anterior aspect of the lateral epicondyle of the humerus. However, the radial recurrent artery lies on muscle and is not normally palpable. Choice B (Between the tendons of the palmaris longus and fl exor carpi ulnaris) is incorrect. The ulnar artery enters the hand superfi cial to the fl exor retinaculum, lateral to the pisiform and medial to the hook of the hamate. The ulnar pulse may be palpable slightly lateral to the insertion of the fl exor carpi ulnaris onto the pisiform, between it and the tendon of the palmaris longus. Choice D (Superfi cial to the tendons of the extensor pollicis brevis and abductor pollicis longus) is incorrect. The radial pulse is also available in the anatomical snuffbox, where the radial artery crosses the fl oor of that space between the tendons of the extensor pollicis longus and brevis muscles. In entering the snuffbox, the artery passes deep (not superfi cial) to the tendons of the abductor pollicis longus and extensor pollicis brevis muscles. Choice E (Superfi cial to the carpal tunnel) is incorrect. As noted above, the ulnar artery enters the hand superfi cial to the fl exor retinaculum, thus, superfi cial to the carpal tunnel. The superfi cial palmar branch of the radial artery usually runs through the thenar muscles and is not palpable.

A 20-year-old woman delivers a stillborn infant with bilateral agenesis of the kidneys. During the later stages of pregnancy, the fetus likely also had which of the following conditions? (A) Polyhydramnios (B) Oligohydramnios (C) Renal hypoplasia (D) Pelvic kidneys (E) Polycystic kidneys

The answer is B: Oligohydramnios. Oligohydramnios is a diminution in the volume of amniotic fluid as a result of the fetus' failure to excrete urine into the amniotic sac, for example, as resulting from renal agenesis or urethral obstruction. Renal agenesis occurs when the ureteric bud fails to develop, thus preventing formation of the kidney. Normally, the fetus pro- duces urine and excretes it into the amniotic cavity, where it mixes with the amniotic fluid. The fetus drinks amniotic fluid, which is absorbed into the bloodstream, filtered at the placenta, and excreted by the fetus. Thus, the prenatal kidneys are not necessary for waste exchange but are important in regulating the volume of amniotic fluid. In renal agenesis (no kidneys), urine is not produced and not secreted, thus reducing the vol- ume of fluid in the amniotic sac (oligohydramnios). The given obstetric sonographic image, an ultrasound-based diagnostic imaging technique, depicts severe oligohydramnios associated with bilateral renal agenesis. In this image, the spine (Sp) of the infant is noted, and the shadow, which is highlighted by the black arrowheads, corresponds to the adrenal gland. The kidneys are absent. Bilateral renal agenesis is relatively rare, with a usual ultimate outcome of stillbirth or death soon after birth. Unilateral renal agenesis is relatively more common, often asymptomatic, and compatible with life, as the remain- ing kidney typically hypertrophies and produces an appropri- ate volume of urine. Choice A (Polyhydramnios) is incorrect. Polyhydramnios is an increased volume of amniotic fluid in the amniotic sac. In this case, the kidneys are normal, and urine is produced and excreted into the amniotic sac. However, obstruction (atresia) of the gut tube prevents the fetus from drinking and/or absorbing that fluid, resulting in accumula- tion of greater volume of fluid in the amniotic sac. Choice C (Renal hypoplasia) is incorrect. Renal hypoplasia is a condition of abnormally small kidneys. In renal agenesis, the kidneys are not formed. Choice D (Pelvic kidneys) is incorrect. In the case of pelvic kidneys, the kidneys are formed; however, one or both fail to ascend from their origination point in the pelvis. Thus, they are ectopic, located in the pelvis or lower lumbar region, and fully functional. Choice E (Polycystic kidneys) is incorrect. Here, the kidneys are formed. However, the collect- ing ducts and tubules are dilated, forming multiple, large fluid- filled cysts within the kidney parenchyma.

Multiple tumors in the pelvic cavity cause widespread destruction through the hypogastric plexuses in a 67-year-old man. The nerve fibers that degenerate as a result of these lesions are mainly those of which of the following? (A) Presynaptic sympathetic neurons (B) Presynaptic parasympathetic neurons (C) Postsynaptic parasympathetic neurons (D) Somatic motor neurons (E) General sensory neurons

The answer is B: Presynaptic parasympathetic neurons. The hypogastric plexuses are an extensive network of visceral motor (autonomic—both sympathetic and parasympa- thetic) and visceral sensory fibers within the pelvic cavity that supply the pelvic viscera and the perineum. The para- sympathetic components are mainly presynaptic neurons derived from the pelvic splanchnic nerves, which synapse in the intramural ganglia of the target organs. The sympa- thetic components are mainly postsynaptic neurons derived from the aortic (intermesenteric) plexuses in the abdomen, where they synapsed with presynaptic elements. Choice A (Presynaptic sympathetic neurons) is incorrect. The pre- synaptic sympathetic neurons that signal to pelvic viscera originate in the low thoracic and upper lumbar segments of the spinal cord and send axonal processes through the lumbar splanchnic nerves to the aortic (intermesenteric) plexuses, where they synapse with postsynaptic elements. Choice C (Postsynaptic parasympathetic neurons) is incor- rect. The pelvic splanchnic nerves send presynaptic fibers through the hypogastric plexuses to the hindgut, pelvic viscera, and perineum. These neurons synapse with post- synaptic cells in the intramural (terminal) ganglia in the walls of the target organs. The postsynaptic neurons make up a large component of the enteric nervous system in the walls of the gut tube. Choice D (Somatic motor neurons) is incorrect. Somatic motor neurons supply skeletal muscles in the pelvic walls and floor and perineum. They are con- veyed via the branches of the sacral plexus (e.g., pudendal nerve) which, strictly speaking, are not components of the hypogastric plexuses. Choice E (General sensory neurons) is incorrect. General sensory (general somatic afferent) neurons from the pelvic walls and floor as well as the perineum travel through the branches of the sacral plexus, in company with the somatic motor fibers to those areas. However, the visceral sensory fibers from pelvic viscera travel with visceral motor (autonomic) fibers through the hypogastric plexuses.

To reduce pain during her childbirth, an obstetrician performs a local nerve block on his 25-year-old patient while she is in the lithotomy position, as depicted in the given figure. Which of the following structures will remain fully sensitive following administration of this local anesthetic? (A) Lower anal canal (B) Rectum (C) Perinealbody (D) Urogenitaldiaphragm (E) Vulva

The answer is B: Rectum. The illustration shows a pudendal nerve block, which anesthetizes the area innervated by the pudendal nerve, that is, the majority of the perineum, includ- ing the lower vagina and lower anal canal. The rectum lies above the perineum and is supplied by visceral sensory fibers from the hypogastric plexuses rather than by the pudendal nerve. Thus, it retains visceral sensation following a pudendal block. The block is accomplished by injecting a local anesthetic into the tissues around the pudendal nerve where it crosses the sacrospinous ligament near the ischial spine. Achieving total anesthesia of the perineum additionally requires blocking the ilioinguinal nerve (covering the anterior perineum) and perhaps the posterior cutaneous nerve of the thigh (for the extreme posterior perineum). Choice A (Lower anal canal) is incorrect. The lower anal canal (i.e., the area below the pec- tinate line) receives general sensory innervation through the inferior anal (rectal) nerve, a branch of the pudendal nerve. Thus, this area is anesthetized by a pudendal block. Choice C (Perineal body) is incorrect. The perineal body is a dense con- nective tissue mass in the center of the perineum. This area is innervated by branches of the pudendal nerve. Choice D (Urogenital diaphragm) is incorrect. The urogenital diaphragm is a musculofascial complex within the urogenital triangle of the perineum. Its major component is the external urethral sphincter. This area is supplied by the perineal branches of the pudendal nerve. Remember, the anatomical concept of the urogenital diaphragm has changed considerably in recent years. It is important to reference the most current texts for a proper understanding of the morphology here so as not to be confused by the traditional, inaccurate description. Choice E (Vulva) is incorrect. The vulva is the part of the perineum in females containing the external genitalia, which includes the mons pubis, labia majora and minora, vestibule of the vagina, clitoris, and bulbs of the vestibule. The vulva is innervated largely by the dorsal nerve of the clitoris and the perineal nerves, all branches of the pudendal nerve. However, of note, the anterior aspect of the vulva (especially the mons pubis and anterior labia majora) is mainly supplied by the ilioinguinal nerve. Thus, a pudendal nerve block greatly reduces sensation in the vulva but likely does not provide total anesthesia.

As part of a general pelvic examination, an obstetrician measures the diagonal conjugate diameter of the true pelvis in his 26-year-old pregnant patient. This measurement is taken between which of the following points? (A) Ischial spine to the opposite ischial spine (B) Sacral promontory to the inferior edge of the pubic symphysis (C) Sacral promontory to the posterosuperior edge of the pubic symphysis (D) Sacroiliac joint to the posterior side of the opposite side body of the pubis (E) Tip of the coccyx to the pubic symphysis

The answer is B: Sacral promontory to the inferior edge of the pubic symphysis. The true (obstetrical) conjugate diam- eter of the true pelvis (see Discussion point C below) can- not be measured directly in a manual pelvic exam because of the presence of the urinary bladder. Therefore, the next best approximation of that, the diagonal conjugate, is mea- sured instead (see the given figure). This measurement is taken by first palpating the sacral promontory with the tip of the middle finger of the gloved examining hand. Next, the other hand marks the position of the inferior edge of the pubic symphysis on the examining hand. Finally, after withdrawing the examining hand, the distance from the tip of the index finger to the marked point for the symphysis is measured to obtain an estimate of the true conjugate. This distance should be 11.5 cm or greater. The diameters of the true pelvis are important measurements in determining the capacity of the pelvic canal for childbirth. Choice A (Ischial spine to the opposite ischial spine) is incorrect. This mea- surement is the interspinous distance in the pelvis. Because the ischial spines project into the true pelvis and toward one another, this distance is normally the narrowest part of the pelvic (birth) canal. However, the interspinous length is not a fixed distance because of normal relaxation of pel- vic ligaments and added joint mobility during pregnancy. Choice C (Sacral promontory to the posterosuperior edge of the pubic symphysis) is incorrect. This measurement is the true (obstetrical) conjugate of the true pelvis. It is the nar- rowest fixed distance (diameter) of the true pelvis and indi- cates the smallest anteroposterior space available for the fetal head to pass during vaginal delivery. Choice D (Sacroiliac joint to the posterior side of the opposite side body of the pubis) is incorrect. This distance is the oblique diameter of the true pelvis. It is a diameter that usually is noted via radio- graphic imaging rather than manual palpation. Choice E (Tip of the coccyx to the pubic symphysis) is incorrect. This distance indicates the anteroposterior diameter of the pelvic outlet. However, it is not a fixed distance because of enhanced pelvic mobility during pregnancy.

3 A 21-year-old man goes to his college campus health clinic complaining of soreness in his left wrist after falling on an outstretched hand during a basketball game the previous day. He is supporting his left wrist and indicates that the pain worsens with movement and is minimized with inactivity. There is no loss of feeling in his hand, nor does he have trouble grasping or holding objects. The physician exacerbates the wrist pain by applying pressure to the base of the thumb in the anatomical snuffbox Radiographic imaging will confirm a break of which carpal bone? (A) Capitate (B) Scaphoid (C) Hamate (D) Trapezium (E) Pisiform

The answer is B: Scaphoid. The most commonly fractured carpal bone is the scaphoid that forms the fl oor of the anatomical snuffbox. This area is a fossa located between the three long tendons of the thumb (tendons of abductor pollicis longus and extensor pollicis brevis laterally and tendon of the extensor pollicis longus medially). The scaphoid bone is frequently broken when an individual falls with an outstretched hand and lands on the palm with the hand abducted. A broken scaphoid bone is commonly seen in individuals under the age of 30. The scaphoid is broken due to its unfortunate position between the downward force transmitted by the weight of the upper limb and the upward force due to the impact of hitting the fl oor. The proximal aspect of a broken scaphoid bone can suffer from avascular necrosis due to its blood supply entering the bone distally. Choice A (Capitate) is incorrect. The capitate is located centrally in the distal row of carpal bones and articulates with most of the carpal bones (the triquetrum, pisiform, and trapezium being the exceptions). This bone is not located in the fl oor of the anatomical snuffbox, so it was not the most likely bone injured in this patient. Choice C (Hamate) is incorrect. The hamate is located in the distal row of carpal bones on the ulnar side. It is not related to the anatomical snuffbox, so it would not produce pain in the base of the thumb when broken. Choice D (Trapezium) is incorrect. The trapezium is located at the base of the thumb, but it is not commonly fractured when a person falls with an outstretched hand. A mnemonic for the position of the trapezium is "trapezium articulated with the thumb." Choice E (Pisiform) is incorrect. The pisiform is located in the proximal carpal row and is a sesamoid bone located within the tendon of the fl exor carpi ulnaris muscle. Its position on the ulnar side of the wrist would not cause pain in the anatomical snuffbox.

40 The lateral thoracic artery provides the main blood supply to the lateral side of the chest wall, including much of the breast. To deter excessive blood loss during a surgical procedure involving the breast, a surgeon can clamp the lateral thoracic artery near its origin. Which of the following arteries gives rise to this artery? A) First part of the axillary artery (B) Second part of the axillary artery (C) Third part of the axillary artery (D) Third part of the subclavian artery (E) First part of the brachial artery

The answer is B: Second part of the axillary artery. The lateral thoracic wall receives signifi cant arterial supply from the branches of the axillary artery. The axillary artery is divided into three parts by the overlying pectoralis minor muscle. The lateral thoracic artery, along with the thoracoacromial trunk, typically branches from the second part of the axillary artery, deep to the pectoralis minor. It descends along the lateral border of the pectoralis minor to supply the lateral aspect of the chest wall, including much of the breast and the serratus anterior muscle. However, this vessel is variable and may originate from other source points. Remember, arteries are often named for the regions they supply, not necessarily for their branching patterns. Choice A (First part of the axillary artery) is incorrect. This segment typically gives rise to one branch: the superior (or supreme) thoracic artery, which has a small distribution to the superolateral chest wall. Choice C (Third part of the axillary artery) is incorrect. This part typically gives rise to three branches: the subscapular artery and the anterior and posterior circumfl ex humeral arteries. The lateral thoracic and subscapular arteries may originate as a common trunk. Choice D (Third part of the subclavian artery) is incorrect. The subclavian artery is divided into three parts by the anterior scalene muscle. Usually, the third part of the subclavian artery has no branches; however, the dorsal scapular artery may occasionally arise from this location. More importantly, none of the branches of the subclavian artery supply the lateral chest wall. Choice E (First part of the brachial artery) is incorrect. The distribution of the brachial artery is limited to the upper limb; it does not supply the lateral chest wall.

35 A right-handed 21-year-old college student visits his physician because of pain in his right shoulder that developed after starting a summer job on a construction crew 2 weeks ago. He explains that on his job site he regularly lifts heavy construction materials over his head. During physical examination, the patient experiences sharp pain in the range of 80 to 150 degrees of abduction at the glenohumeral joint. What is the most likely diagnosis? (A) Infraspinatus tendonitis (B) Supraspinatus tendonitis (C) Acromioclavicular (AC) joint arthritis (D) Degenerative arthritis of the shoulder (E) Broken clavicle

The answer is B: Supraspinatus tendonitis. This patient is experiencing shoulder impingement syndrome, in which impingement of the supraspinatus tendon leads to supraspinatus tendonitis. Supraspinatus tendonitis is a result of stressful repetitive overhead motions, such as lifting heavy objects, which this patient noted in his summer job. This type of tendonitis presents with a painful arc of motion (between 80 and 150 degrees in this patient) during abduction of the upper limb. Choice A (Infraspinatus tendonitis) is incorrect. Infraspinatus tendonitis would lead to pain during external rotation at the glenohumeral joint against resistance, which is not noted in this patient. Choice C (Acromioclavicular (AC) joint arthritis) is incorrect. AC joint arthritis would result following repeated movements that wear away the cartilage surfaces in the AC joint. It can also be seen after repeated shoulder separations. However, due to the age of this patient and the fact that he was only participating in construction during the summer, this diagnosis is unlikely. Choice D (Degenerative arthritis of the shoulder) is incorrect. Given the young age of this patient, degenerative arthritis of the shoulder is extremely unlikely. Choice E (Broken clavicle) is incorrect. The patient is reporting pain only during abduction of the upper limb. A broken clavicle presents with constant pain and results from trauma; however, trauma was not reported in this patient.

42 An 80-year-old woman comes to the physician because of a lump in her right breast. Physical examination shows a 2-cm mass in the right breast with dimpling of the overlying skin and peau d'orange (edema of the breast with the skin assuming the appearance of an orange peel). Examination of a biopsy specimen confi rms a diagnosis of carcinoma. Involvement of what structure is the most likely cause of this patient's skin dimpling? (A) Clavipectoral fascia (B) Suspensory ligaments (C) Lactiferous ducts (D) Retromammary space (E) Pectoralis major

The answer is B: Suspensory Ligaments. Fibrous lobar septa are connective tissue partitions that separate the glandular lobes of the breast and compartmentalize breast tissue. These septa run through the depth of the breast from the dermis of the skin to the underlying pectoral fascia and are most pronounced in the superior aspect of the breast, where they are termed suspensory (Cooper's) ligaments of the breast. Edema and/or tumor growth within the breast can apply traction on the suspensory ligaments, and this tension causes dimpling of the skin that resembles an orange peel. Choice A (Clavipectoral fascia) is incorrect. This sleeve of connective tissue extends from the clavicle to the deep fascia of the axilla and envelopes the subclavius and pectoralis minor muscles. It does not enter the breast. Choice C (Lactiferous ducts) is incorrect. Each of the 15 to 20 glandular lobes of the breast is drained by a lactiferous duct that opens in the lactiferous sinus and then onto the nipple. Edema and/or tumor growth within the breast may apply traction to these ducts. However, the result of such traction is typically inversion of the nipple rather than dimpling of the skin. Choice D (Retromammary space) is incorrect. The retromammary space (or bursa) is a loose connective tissue plane between the breast and pectoral fascia, which contains a small amount of fat and allows the breast some degree of mobility on the pectoral fascia. It does not send connective tissue extensions into the substance of the breast and would not lead to dimpling of the skin due to the presence of an abnormal growth within the breast. Choice E (Pectoralis major muscle) is incorrect. The pectoralis major underlies the breast but does not send fi bers into the breast. It does not infl uence the appearance of the skin of the breast.

7 A 17-year-old boy comes to the emergency room after a hard fall onto the lateral aspect of his left shoulder during a high school basketball game. He complains of generalized pain during shoulder motion. On physical examination, the distal end of the clavicle is prominent and distinctly palpable. Radiological findings confirm the diagnosis of a severe (grade 3) shoulder separation. Which of the following features is a component of this condition? (A) Dislocated head of the humerus (B) Torn coracoclavicular ligament (C) Fractured clavicle (D) Dislocated sternal end of the clavicle (E) Torn anterior glenohumeral (GH) ligament

The answer is B: Torn coracoclavicular ligament. "Shoulder separation" describes a dislocation of the acromioclavicular joint. In its most severe form (grade 3), the condition includes a tearing of both the intrinsic acromioclavicular ligament and the extrinsic coracoclavicular ligament. As a result, the scapula separates from the clavicle and falls away due to the weight of the upper limb. Thus, the distal end of the clavicle is prominent. Choice A (Dislocated head of the humerus) is incorrect. Dislocations of the GH joint easily occur inferiorly due to its lack of muscular and ligamentous support. Thus, damage to the axillary nerve often occurs following inferior displacment of the head of humerus from the GH joint. However, the acromioclavicular joint, which is more proximal, was injured in this patient. Dislocations of the glenohumeral joint in other directions are more diffi cult (but not impossible) because of the support of the rotator cuff muscles (anteriorly and posteriorly) and the coracoacromial arch (superiorly). Choice C (Fractured clavicle) is incorrect. Radiological imaging would have detected a fractured clavicle, but these tests confi rmed a shoulder separation and not a fractured clavicle. Choice D (Dislocated sternal end of the clavicle) is incorrect. Due to its intrinsic strength, dislocation of the sternoclavicular (SC) joint is rare. Most dislocations of the SC joint occur in persons less than 25 years of age following a fracture of the epiphysial plate of the clavicle. The epiphysis at the proximal end of the clavicle does not close until approximately age 25. Though this patient was under the age of 25, his injury was localized to the acromioclavicular joint. Choice E (Torn anterior glenohumeral [GH] ligament) is incorrect. Three GH ligaments reinforce the anterior part of the joint capsule; however, the GH joint was not involved in this patient.

38 Because of repeated bad needle sticks, a heroin addict develops an infected abscess in the fl oor of the cubital fossa. Which of the following structures is the abscess most likely to invade first? (A) Brachioradialis muscle (B) Pronator teres muscle (C) Brachialis muscle (D) Head of the radius (E) Olecranon fossa of the humerus

The answer is C: Brachialis muscle. The cubital fossa is a triangular intermuscular space located anterior to the elbow, comparable to the popliteal fossa in the lower limb. It is an important transition zone between the arm and the forearm, containing major blood vessels and nerves. The roof of the cubital fossa is subcutaneous tissue carrying superfi cial veins, such as the median cubital vein, which is the most common site for venipuncture in the upper limb. The fl oor of the fossa is formed by two muscles: brachialis and supinator. Either of these muscles could house the deep abscess caused by repeated needle injections in the cubital fossa. Choice A (Brachioradialis muscle) is incorrect. The brachioradialis muscle forms the lateral wall of the cubital fossa. Choice B (Pronator teres muscle) is incorrect. The pronator teres muscle forms the medial wall of the cubital fossa. Choice D (Head of the radius) is incorrect. The head of the radius lies deep to the muscular fl oor of the cubital fossa, within the elbow joint complex. Choice E (Olecranon fossa of the humerus) is incorrect. The olecranon fossa is located on the posterior distal aspect of the humerus, well removed from the cubital fossa.

22 A 72-year-old man is brought to the ER after being struck by a car. Radiographic imaging reveals a rotationally unstable pubic ramus fracture. Which of the following structures is most likely damaged specifically by the fracture of the superior pubic ramus? (A) Sympathetic chain (B) Superior gluteal artery (C) Ductus deferens (D) Lumbosacral trunk (E) Piriformis muscle

The answer is C: Ductus deferens. The ductus (vas) deferens separates from the spermatic cord as the cord emerges from the deep inguinal ring. The ductus crosses over the external iliac vessels and enters the true pelvis over the pelvic brim at the superior pubic ramus. It then runs along the lateral pelvic wall and on to the posterolateral side of the urinary bladder. Thus, fractures of the lateral wall of the true pelvis, especially those at the superior pubic ramus, endanger the ductus def- erens. Choice A (Sympathetic chain) is incorrect. The sympa- thetic chain crosses the pelvic brim at the ala of the sacrum to enter the true pelvis. It descends along the pelvic surface of the sacrum and finally terminates as the ganglion impar. There- fore, the sympathetic chain is situated in relation to the roof of the posterior wall of the true pelvis and is not in position to be affected by a lateral wall fracture. Choice B (Superior glu- teal artery) is incorrect. The superior gluteal artery branches from the posterior division of the internal iliac artery, within the true pelvis. It immediately leaves the pelvis through the greater sciatic foramen at the upper margin of the piriformis muscle. This vessel does not cross the pelvic brim and is not in position to be affected by a fracture of the brim. Choice D (Lumbosacral trunk) is incorrect. The lumbosacral trunk is formed by the union of the L4 and L5 ventral primary rami as they descend into the true pelvis to merge with the sacral plexus. These nerves cross the ala of the sacrum and typically form the lumbosacral trunk below the pelvic brim, within the true pelvis. However, the junction may occur at a higher level. In either case, the lumbosacral trunk is related to the roof of the posterior wall of the true pelvis rather than to the lateral wall. Choice E (Piriformis muscle) is incorrect. The piriformis muscle originates from the pelvic surface of the superior part of the sacrum, lateral to the anterior sacral foramina, within the true pelvis. It exits the pelvis through the greater sciatic foramen to reach the greater trochanter of the femur. The piriformis does not cross the pelvic brim. It forms the postero- lateral wall of the true pelvis and is not affected by a fracture of the lateral wall.

A 72-year-old man comes to his urologist complaining of urinary frequency, urgency, weak urinary stream, hesitancy, and straining to void. A rectal examination and blood tests revealing normal prostate-specific antigen levels lead to a diagnosis of benign prostatic hyperplasia. The man undergoes a prostatectomy to remove the prostate gland. During the surgical procedure, the prostatic nerve plexus is damaged. What is the most likely complication of this surgery? (A) Loss of sensation from the anus (B) Fecalincontinence (C) Erectile dysfunction (D) Loss of sensation from the posterior scrotum (E) Vasomotor dysfunction in the rectum

The answer is C: Erectile dysfunction. The cavernous nerves of the penis supply parasympathetic (visceral motor; autonomic) fibers to erectile (cavernous) tissues of the penis, stimulating penile erection. These nerves arise from the pro- static portion of the pelvic nerve plexus, which surrounds the capsule of the prostate, and it is at this location that the cavernous nerves can be damaged during prostatectomy. After enveloping the prostatic capsule, the cavernous nerves descend to penetrate the perineal membrane to reach erectile tissues of the penis, providing parasympathetic stimulation. This innervation causes the helicine arteries of the penis, the coiled terminal branches of the deep and dorsal arteries of the penis, to uncoil, enabling blood at arterial pressure to fill the cavernous tissue causing penile erection. Thus, damage to these nerves would most likely result in erectile dysfunction. Choice A (Loss of sensation from the anus) is incorrect. The sensation from the skin surrounding the anus is conveyed by the inferior anal (rectal) nerves, which are branches of the pudendal nerve. The pudendal nerve exits the pelvis via the greater sciatic notch and enters the perineum through the lesser sciatic notch to supply the external anal sphincter and its surrounding skin, the superficial and deep perineal com- partments, and the external genitalia. Neither the inferior anal or pudendal nerves would be at risk during a prostatectomy due to their posterior locations. Choice B (Fecal incontinence) is incorrect. Fecal incontinence may be due to loss of inner- vation to the external anal sphincter, which is supplied by the inferior anal (rectal) nerves, which are branches from the pudendal nerve. Due to the posterior location of the pudendal nerve, it would not be at risk during a prostatectomy. Choice D (Loss of sensation from the posterior scrotum) is incorrect. The sensation from the posterior scrotum is supplied by the posterior scrotal nerve, which is a superficial perineal branch of the pudendal nerve. Due to the inferior and posterior loca- tions of the posterior scrotal and pudendal nerves, they would not be at risk during a prostatectomy. Choice E (Vasomotor dysfunction in the rectum) is incorrect. The vasomotor func- tions of the rectum are supplied by the rectal plexus, which is derived from the inferior hypogastric plexus. The rectal plexus contains visceral motor (parasympathetic and sympa- thetic) fibers, but due to its proximity to the lateral sides of the rectum, it would be unlikely for it to be at risk during a prostatectomy.

A 31-year-old woman delivers her first child via natural childbirth, without anesthetics. During the delivery, she experiences painful spasms in the muscles in the medial part of her right thigh and paresthesia in the skin in the medial thigh. Following delivery, these conditions gradually dissipate. Which of the following nerves may have been compressed during this childbirth? (A) Inferior gluteal nerve (B) Pudendal nerve (C) Obturator nerve (D) Sciatic nerve (E) Pelvic splanchnic nerves

The answer is C: Obturator nerve. The obturator nerve branches from the lumbar plexus in the false (greater) pelvis of the lower abdomen. It descends into the true pelvis, runs across the surface of the obturator internus muscle on the lateral pelvic wall, and passes through the obturator canal to enter the medial thigh. In the thigh, the obturator nerve supplies the adductor muscles in the medial compartment of the thigh and also a cutaneous area in the medial thigh. During childbirth, the nerve may be compressed against the lateral wall of the pelvis by the passing baby. Choice A (Inferior gluteal nerve) is incorrect. The inferior gluteal nerve arises from the sacral plexus and immediately exits the pelvis through the greater sciatic foramen to reach the gluteal region. In the buttock, it supplies the gluteus maximus muscle. Choice B (Pudendal nerve) is incorrect. This nerve arises from the sacral plexus, leaves the pelvis through the greater sciatic foramen, curls around the ischial spine, and enters the perineum. It is the pri- mary motor and sensory nerve to the perineum. Choice D (Sci- atic nerve) is incorrect. The sciatic nerve is the largest nerve in the body. It originates from the sacral plexus, leaves the pelvis through the greater sciatic foramen, runs through the buttock, and into the posterior thigh. The sciatic nerve supplies the hip joint, the muscles in the posterior compartment of the thigh, all muscles below the knee, and considerable cutaneous areas below the knee. Choice E (Pelvic splanchnic nerves) is incorrect. These nerves are presynaptic parasympathetic fibers arising off the sacral plexus. They distribute through the hypogastric plexuses to the hindgut organs and the pelvic and perineal viscera. Parasympathetic nerves do not supply the body wall or limbs.

A married woman who is having diffi culty getting pregnant undergoes an endometrial function test (EFT), which determines that the endometrium of her uterus is not capable of implantation. Which of the following substances would likely increase the chances for implantation of the fertilized oocyte (or preimplantation embryo) into the uterine mucosa? (A) Follicle-stimulating hormone (B) Testosterone (C) Progesterone (D) Estrogen (E) Luteinizing hormone

The answer is C: Progesterone. Progesterone is a steroid hormone produced by the ovaries, brain, and placenta of pregnant females. In the menstrual cycle, progesterone production remains low until after ovulation. The corpus luteum (the remnant of the collapsed ovarian follicle after ovulation) produces progesterone, which halts endometrial proliferation and builds the endometrial lining of the uterus in preparation for implantation of the fertilized oocyte (zygote). Therefore, progesterone supplementation would increase the chances for implantation of the fertilized oocyte into the uterine mucosa of this patient. Because increased serum levels of progesterone and estrogen suppress its release, FSH levels remain low during the buildup of the endometrial lining of the uterus, which is crucial for implantation. Also, the production of FSH peaks approximately 3 days after menstruation because serum levels of progesterone and estrogen are low. Therefore, FSH supplementation would decrease the chances for implantation of the fertilized oocyte into the uterine mucosa.

44 A 10-year-old boy was running across a parking lot when he tripped and received lacerations on the base of his thumb from a broken glass bottle. On examination, his thumb was unable to oppose to his fingers, and the thumb also showed weakness when abducting and flexing. No sensory deficits were reported. What nerve was most likely severed? (A) Deep branch of radial nerve (B) Superfi cial branch of radial nerve (C) Recurrent branch of median nerve (D) Deep branch of ulnar nerve (E) Superfi cial branch of ulnar nerve

The answer is C: Recurrent branch of median nerve. The recurrent (thenar) branch of the median nerve lies subcutaneously in the thenar eminence and can be damaged by lacerations in this area. This nerve innervates most of the thenar muscles, including the Opponens pollicis, Abductor pollicis brevis, and superfi cial head of the Flexor pollicis brevis (mnemonic = "OAF"). Opposition of the thumb would be lost by cutting the recurrent branch of the median nerve. Though the abductor pollicis brevis is denervated, the abductor pollicis longus, innervated by the deep branch of the radial nerve, is still intact. Also, the fl exor pollicis longus, innervated by the anterior interosseous nerve, would still allow fl exion at the interphalangeal joint of the thumb. Cutting the recurrent branch of the median nerve would lead to atrophy of the thenar muscular complex, a condition known as "ape hand." Choice A (Deep branch of the radial nerve) is incorrect. The deep branch of the radial nerve is also called the posterior interosseous nerve as it exits from the supinator muscle. It innervates the abductor pollicis longus and extensor pollicis longus and brevis muscles, which form the boundaries of the anatomical snuffbox. Cutting this nerve would cause loss of extension of the thumb and weakness in abduction. However, the puncture was on the thenar eminence, and the radial nerve does not travel there. Choice B (Superfi cial branch of the radial nerve) is incorrect. The superfi cial branch of the radial nerve arises from the radial nerve in the cubital fossa. This nerve is purely cutaneous, supplying sensation to the dorsum of the hand and fi ngers. Because it does not have a motor component, this nerve would not be responsible for the defi cit in thumb function. Choice D (Deep branch of ulnar nerve) is incorrect. The deep branch of the ulnar nerve innervates most of the intrinsic muscles of the hand, including the hypothenar muscles, medial two lumbrical muscles, the palmar and dorsal interossei, adductor pollicis, and the deep head of the fl exor pollicis brevis. With a small motor supply to the thenar compartment (specifi cally the deep head of the fl exor pollicis brevis), this nerve could not cause a substantial loss of function in the thenar eminence. Choice E (Superfi cial branch of the ulnar nerve) is incorrect. The superfi cial branch of the ulnar nerve arises from the ulnar nerve distal to the fl exor retinaculum. This nerve supplies cutaneous branches to the anterior surface of the medial one and a half fi ngers. The palmaris brevis is the only muscle supplied by this nerve, and this muscle tightens the skin of the medial surface of the palm. Therefore, this nerve would not be responsible for substantial loss of motor function in the thumb.

A young boy has a chronic problem with urine dripping from abnormal openings on the underside (anterior or ventral side) of the glans and body of the penis. Which of the following structures most likely did not fuse normally during development? (A) Mesonephric ducts (B) Paramesonephric ducts (C) Urethral folds (D) Scrotal swellings (E) Genital tubercle

The answer is C: Urethral folds. In male fetuses, the urethral (urogenital) folds close over the urethral groove and fuse in the midline to form the ventral (anterior) aspect of the penis. This enclosure of the urethral groove forms the penile ure- thra. Incomplete fusion of the urethral folds results in abnor- mal openings of the penile urethra along the underside of the penis. This condition is termed hypospadias and may include one or more openings anywhere from the glans to the base of the penis. In females, the urethral folds do not fuse but form the separate labia minora. The urethral groove remains open and forms the vestibule. Choice A (Mesonephric ducts) is incorrect. In males, the mesonephric ducts are stimulated but do not fuse with one another. They form the testicular drain- age ducts, including the efferent ductules, ductus epididymis, ductus deferens, and ejaculatory duct. The seminal vesicle forms from a secondary bud off the ductus deferens. In females, the mesonephric ducts are normally suppressed, leaving only small vestigial structures in association with the reproductive tract. Choice B (Paramesonephric ducts) is incorrect. These ducts are normally suppressed in male fetuses. The sole rem- nant is the small, vestigial appendix of the testis. In females, the paramesonephric ducts are stimulated and partly fused. They form the uterine tubes, uterus, cervix, and superior third of the vagina. Choice D (Scrotal swellings) is incorrect. The scrotal swellings arise from the genital swellings on the lateral sides of the urethral folds. They enlarge and fuse in the mid- line to form the scrotum. Rarely, hypospadias extends poste- rior into the scrotal raphe. In females, the genital swellings enlarge, but do not fuse, forming the labia majora. Choice E (Genital tubercle) is incorrect. The genital tubercle forms at the cranial junction of the cloacal folds and elongates to form the phallus, which in turn forms the glans penis, corpora cav- ernosa, and corpus spongiosum. In females, the phallus forms the glans clitoris, corpora cavernosa, and vestibular bulbs.

A 24-year-old woman is the driver of a car in a head-on collision. She suffers bilateral fractures of the pubic rami and dislocation of the pubic symphysis as a result of anteroposterior compression of the pelvis . Which of the following structures is most likely damaged in this patient? (A) Anal canal (B) Piriformis muscles (C) Urinary bladder (D) Uterus (E) Vagina

The answer is C: Urinary bladder. When empty (i.e., in the "resting" anatomical position), the adult urinary bladder is located in the true (lesser) pelvis, lying partly on the pubic bones and pubic symphysis. It is separated from these bones by only the thin, potential retropubic space. The bladder may be ruptured by fractures of the pubic bones or, especially when distended, by injuries to the anteroinferior abdominal wall. Rupture of the bladder wall may allow extravasation (passing out) of urine into extraperitoneal spaces or into the peritoneal cavity. Choice A (Anal canal) is incorrect. The anal canal is far posterior of the pubic bones, with other organs intervening. It is not normally directly affected by fracture of those bones. Choice B (Piriformis muscles) is incorrect. The piriformis muscles run from the pelvic surface of the sacrum through the greater sciatic foramina to the femur. They form part of the posterolateral wall of the true pelvis and so are on the oppo- site side of the true pelvis from the pubic bones. Choice D (Uterus) is incorrect. The uterus normally lays anteverted and anteflexed in the reproductive tract. Its anterior surface lies on the posterosuperior aspect of the urinary bladder. Thus, the bladder intervenes between the pubic bones and the uterus. Choice E (Vagina) is incorrect. The vagina lies posterior to the base of the urinary bladder and the urethra. Therefore, those organs would be directly affected in pubic fractures before the vagina.

12 A 23-year-old medical student complains of loss of sensation in the skin on the medial edge of her left hand, including the entire fifth digit. The associated motor deficit probably involves weakness in which of the following? (A) Pronation (B) Abduction of the wrist (C) Extension of the wrist (D) Abduction of the index finger (E) Flexion of the interphalangeal joints of the index fi nger

The answer is D: Abduction of the index fi nger. The sensory defi cit occurs in the cutaneous territory of the ulnar nerve. This nerve controls most of the intrinsic muscles of the hand, including the fi rst dorsal interosseous muscle, which controls abduction of the index fi nger. The four dorsal interosseous muscles of the hand are innervated by the deep branch of the ulnar nerve and function to abduct digits 2 to 4. Remember that two fi ngers (the thumb and little fi nger) have their own muscles dedicated to abduction. Also, remember the mnemonic "DAB," which stands for Dorsal interossei ABduct the fi ngers. Thus, the associated motor defi cit for the ulnar nerve on this list would be loss of abduction of the index fi nger. Choice A (Pronation) is incorrect. Pronation is governed by the median nerve, which supplies the pronator teres and pronator quadratus muscles. Choice B (Abduction of the wrist) is incorrect. Abduction of the wrist is controlled by the median nerve (supplying the fl exor carpi radialis) and the radial nerve (supplying the extensor carpi radialis longus and brevis). Choice C (Extension of the wrist) is incorrect. Extension of the wrist is produced by the radial nerve acting mainly on the extensor carpi radialis longus and brevis and the extensor carpi ulnaris muscles. Choice E (Flexion of the interphalangeal joints of the index fi nger) is incorrect. Flexion of the interphalangeal joints of the index fi nger is controlled by branches of the median nerve, which supply the fl exor digitorum superfi cialis muscle (to fl ex the proximal interphalangeal joint of the second fi nger) and the fl exor digitorum profundus muscle (to fl ex the distal interphalangeal joint of the index fi nger).

24 Physical examination of a 45-year-old man who had been stabbed in the back of the shoulder shows a deep wound penetrating into the quadrangular space of the shoulder, causing bleeding from the severed blood vessels there. Which of the following neural structures is most likely damaged as well? (A) Musculocutaneous nerve (B) Lateral cord of the brachial plexus (C) Radial nerve (D) Axillary nerve (E) Medial cutaneous nerve of the arm

The answer is D: Axillary nerve. The quadrangular space of the shoulder is an important passageway allowing the posterior humeral circumfl ex vessels and their companion axillary nerve to pass from the axilla to the posterior aspect of the shoulder. The neurovascular bundle runs across the surgical neck of the humerus to enter the quadrangular space. The space itself (sometimes termed the lateral axillary hiatus) is formed by four structures: teres major, teres minor, long head of the triceps, surgical neck of the humerus. The vessels contribute to the collateral network around the shoulder. The axillary nerve supplies the teres minor and deltoid muscles and a cutaneous area on the superolateral aspect of the arm (i.e., the skin overlying the lower aspect of the deltoid muscle). Choice A (Musculocutaneous nerve) is incorrect. The musculocutaneous nerve is a terminal branch of the lateral cord of the brachial plexus within the axilla. It supplies the anterior compartment of the arm and is not related to the quadrangular space. Choice B (Lateral cord of the brachial plexus) is incorrect. The lateral cord occupies a relatively lateral position in the axilla but is not related to the quadrangular space. The axillary nerve originates from the posterior cord of the brachial plexus. Choice C (Radial nerve) is incorrect. The radial and axillary nerves are the terminal branches of the posterior cord of the brachial plexus. The radial nerve runs through the radial (spiral) groove in the midshaft of the humerus to emerge through the lower triangular space, just below the quadrangular space, and enter the posterior compartment of the arm. Choice E (Medial cutaneous nerve of the arm) is incorrect. The medial cutaneous nerve of the arm is a branch of the medial cord of the brachial plexus. It is far removed from the quadrangular space.

22 A 23-year-old competitive weight lifter goes to his physician complaining of pain in his proximal forearm. During his examination, the pain is exacerbated by flexion of the elbow and supination of the forearm against resistance. A lateral radiograph shows chronic microtrauma to the proximal radius at the point of the radial tuberosity. Which of the following muscles attaches to, and most likely damaged, this osteological process? (A) Supinator (B) Brachialis (C) Pronator teres (D) Biceps brachii (E) Brachioradialis

The answer is D: Biceps brachii. The lateral X-ray reveals chronic microtrauma to the radial tuberosity, which is the insertion site for the biceps brachii muscle. In this weightlifter, the damage to the radial tuberosity is most likely due to overuse of the biceps brachii and the concomitant stress placed upon the radial tuberosity by lifting signifi cant weight loads. The biceps brachii muscle produces fl exion of the elbow (and shoulder) and is the powerful supinator of the forearm, explaining why these actions exacerbated the pain in this patient. Choice A (Supinator) is incorrect. The supinator muscle attaches onto the proximal shaft of the radius rather than the radial tuberosity, so it would not cause damage to the radial tuberosity. Moreover, the supinator muscle acts to produce lower resistance supination and does not fl ex the elbow. Choice B (Brachialis) is incorrect. The brachialis muscle is a powerful fl exor of the elbow. However, it is not related to the radial tuberosity as it attaches onto the coronoid process and tuberosity of the ulna. Furthermore, the brachialis muscle does not act in supination. Choice C (Pronator teres) is incorrect. The pronator teres muscle contributes somewhat to fl exion of the elbow. However, it inserts onto the midshaft portion of the radius, and its main action is pronation. In this patient, pain was exacerbated by supination against resistance. Choice E (Brachioradialis) is incorrect. The brachioradials muscle is a notable fl exor of the elbow. However, it attaches to the styloid process at the distal end of the radius and is not related to the radial tuberosity. Also, the brachioradialis muscle does not act in supination.

A 28-year-old woman who does not know that she is pregnant undergoes a chemotherapy treatment at the end of her 1st week of pregnancy. Chemotherapy is associated with slowing the rate of mitosis in exposed cells, which is good in cancer treatment. However, this treatment may also have a negative influence on the implantation and growth of an embryo. In which of the following layers would a lowered rate of cell division be most likely to hinder implantation of the blastocyst? (A) Amnioblast (B) Epiblast (C) Hypoblast (D) Cytotrophoblast (E) Syncytiotrophoblast

The answer is D: Cytotrophoblast. The trophoblast forms the fetal part of the placenta, so it is concerned with implantation. The cytotrophoblast is the mitotically active inner part of the trophoblast, and it forms the primary chorionic villi that extend into the syncytiotrophoblast. The cytotrophoblast also provides the cells that migrate into the syncytiotrophoblast and allow it to expand. Thus, chemotherapy may directly affect mitotic activity in the cytotrophoblast, causing stunted growth of both it and the syncytiotrophoblast and possibly hindering implantation. Choice A (Amnioblast) is incorrect. Amnioblasts are the epiblast cells that line the amniotic cavity adjacent to the cytotrophoblast. They are mitotically active in the growth of the amniotic membrane; however, they are not involved in implantation. Choice B (Epiblast) is incorrect. The epiblast is the dorsal cell layer of the bilaminar germ disc. It contributes to the formation of the embryo proper. While certainly mitotically active, it is not involved in implantation. Choice C (Hypoblast) is incorrect. The hypoblast is the ventral cell layer of the bilaminar germ disc. It also contributes to the formation of the embryo proper. Choice E (Syncytiotrophoblast) is incorrect. The syncytiotrophoblast is the outer, multinucleated part of the trophoblast that is mitotically inactive. The syncytiotrophoblast erodes the maternal endometrium and contributes to the formation of the primitive uteroplacental circulation. However, its growth depends on incorporation of new cells from the active cytotrophoblast. Thus, chemotherapy in this case would not affect cell division within the syncytiotrophoblast.

A 25-year-old man is admitted to the emergency room after a fall, complaining of lower back pain and paresthesia into the lower extremities. The CT reveals a L2 burst fracture with a fracture fragment that is displaced posterior into the vertebral canal. A comprehensive neurologic examination indicates destruction of the sacral segments of the spinal cord. Which of the following functional outcomes is expected? (A) Lowered sperm count (B) Paralysis of the arrector pili muscles in the lower limbs (C) Reduced sweat gland secretion in the lower limbs (D) Inability to achieve erection (E) Increased motility in the descending colon and rectum

The answer is D: Inability to achieve erection. Destruction of the sacral segments of the spinal cord by a posterior frac- ture fragment of L2 would lead to pain within the lower back and paresthesia in the lower limbs. It would also eliminate the sacral parasympathetic outflow due to damage to the pre- synaptic parasympathetic cell bodies, which are located in the grey matter of spinal cord segments S2-4. The sacral parasym- pathetic outflow affects the gut tube below the left (splenic) flexure of the colon, the pelvic viscera, and the external geni- talia. Parasympathetic stimulation produces genital erection, whereas sympathetic stimulation governs ejaculation. This pattern of innervation can be remembered by the mnemonic "Point" and "Shoot." Thus, this injury will result in inability to achieve erection. However, ejaculation can still occur, and such patients can impregnate a woman. Choice A (Lowered sperm count) is incorrect. Sperm production is not governed by the autonomic nervous system. This patient will continue to produce sperm and, as noted above, can still ejaculate and father a child. Choice B (Paralysis of the arrector pili muscles in the lower limbs) is incorrect. Parasympathetic fibers distrib- ute to the body cavities and the external genitalia but not to the body wall and limbs. The innervation of arrector pili mus- cles in the lower limbs is governed by the sympathetic nervous system, which was not damaged in this patient. Choice C (Reduced sweat gland secretion in the lower limbs) is incor- rect. Parasympathetic fibers distribute to the body cavities and the external genitalia but not to the body wall and limbs. The innervation of sweat glands in the lower limbs is governed by the sympathetic nervous system, which was not damaged in this patient. Choice E (Increased motility in the descend- ing colon and rectum) is incorrect. The sacral parasympathetic outflow does govern the descending colon and rectum, with one normal effect being increased motility (peristalsis) in these regions. Therefore, loss of parasympathetic supply, combined with unopposed sympathetic input, would result in decreased motility here.

16 An anesthesiologist administers an anesthetic solution into the axillary sheath of a 19-year-old college baseball player in preparation for repair of the ulnar collateral ligament of the elbow. After 5 minutes, the patient experiences numbness and paresthesia distal to the middle aspect of the arm; however, the medial aspect of the arm and elbow remain sensitive to pain. What nerve provides sensory innervation to the sensitive area and was not blocked by the anesthetic solution? (A) Long thoracic nerve (B) Median nerve (C) Medial cutaneous nerve of the arm (D) Intercostobrachial nerve (E) Ulnar nerve

The answer is D: Intercostobrachial nerve. The intercostobrachial nerve is the lateral cutaneous branch of the second intercostal nerve. As its name implies, it leaves the thorax by exiting between the second and third ribs (between the ribs = intercosto-) to supply cutaneous innervation to the axilla and medial aspect of the arm (brachium). In some instances, it may also supply skin distal to the elbow. The anesthetic solution would block all of the distal branches of the brachial plexus residing within the axillary sheath, thus sparing the intercostobrachial nerve. Choice A (Long thoracic nerve) is incorrect. The long thoracic nerve does not have a cutaneous distribution and provides only motor innervation to the serratus anterior. Because the long thoracic nerve arises from the ventral rami of C5-C7, it would not be affected by the anesthetic solution, especially if an occlusive tourniquet was utilized to retain the solution. Such a procedure is used to prevent spread of the anesthetic into the neck where it could affect the phrenic nerve and negatively affect respiration. Choice B (Median nerve) is incorrect. The median nerve does not branch proximal to the elbow, and its sensory distribution is limited to distal to the wrist. Furthermore, this nerve would be blocked by effective administration of the anesthetic reagent. Choice C (Medial cutaneous nerve of the arm) is incorrect. The medial cutaneous nerve of the arm arises from the medial cord of the brachial plexus, and it would be blocked by effective administration of the anesthetic solution into the axillary sheath. This nerve would also provide sensation to the area (medial aspect of the arm) that remains sensitive to pain, but it should be blocked by the drug. Choice E (Ulnar nerve) is incorrect. The ulnar nerve does not branch proximal to the elbow, and its sensory distribution is limited to distal to the wrist. Furthermore, this nerve would be blocked by effective administration of the anesthetic solution.

21 A child is born to a young woman who had utilized thalidomideto help relieve her morning sickness early in her pregnancy. The infant is missing the proximal segments of both upper and lower limbs. The hands and feet that are present are attached to the trunk of the body and resemble small seal's flippers. Which of the following is the correct term for this malformation? (A) A dysplasia (B) A duplication defect (C) Micromelia (D) Meromelia (E) Amelia

The answer is D: Meromelia. Thalidomide was once used widely to help combat morning sickness during pregnancy. Following a wave of children born with limb malformations, the medication was discovered to be a signifi cant teratogen and was taken off the market. Currently, thalidomide is being used to treat AIDS and cancer patients, raising concerns of a new rash of children born with limb defects. The specifi c malformation in this case is termed meromelia, a partial absence of limb segments. Meromelia is a type of reduction defect, in which whole or partial limb components are absent. Because of the resemblance of the limbs to a seal's fl ippers, this defect is also termed phocomelia ("seal limb"). Choice A (A dysplasia) is incorrect. Dysplasias are malformations in which elements are malformed. In this case, limb components are missing rather than malformed. Choice B (A duplication defect) is incorrect. Duplication defects are characterized by the presence of supernumerary (extra) elements. An example is polydactyly, in which extra digits are present. Choice C (Micromelia) is incorrect. Micromelia ("small limb") is a type of dysplasia. In this, the limb segments are present but are abnormally short. Choice E (Amelia) is incorrect. Amelia ("without limb") is a type of reduction defect in which an entire extremity is absent

A married couple having diffi culty with conception keeps a daily diary of the female's basal body temperature (BBT) throughout the month. On Friday, the woman noted a slight elevation in her BBT of approximately one-half to one degree Fahrenheit (one-quarter to one-half degree Celsius), which may indicate that she is ovulating. If her ovum, depicted on the right side in the given figure, is expelled into the peritoneal cavity from the ovary (ovulation), the secondary oocyte resides at what specific stage of meiosis? (A) Prophase of meiosis I (B) Prophase of meiosis II (C) Metaphase of meiosis I (D) Metaphase of meiosis II (E) Meiosis is completed at the time of ovulation

The answer is D: Metaphase of meiosis II. The secondary is arrested in metaphase of meiosis II about 3 hours before ovulation, and it will remain in this meiotic stage until fertilization occurs. Oogonia are formed in month 5 of a woman's fetal life. Of these 7 million oogonia, 5 million degenerate or become atretic before birth, leaving 2 million oogonia to differentiate into primary oocytes before birth. No oogonia are present at birth. *The primary oocytes are dormant in prophase of meiosis I until puberty because they are surrounded by follicular cells, which secrete oocyte maturation inhibitor (OMI) that causes the arrest of meiosis I.*

A forensic anthropologist is called to the basement of an abandoned house, where an intact human skeleton has been discovered in a shallow grave. Without the benefit of soft tissue, the gender of the body can still be established based upon the characteristics of the two unearthed coxal (hip) bones. Which of the following characteristics is indicative of the female pelvis? (A) Subpubic angle less than 70 degrees (B) Heart-shaped (android) pelvic inlet (C) More inverted ischial tuberosities (D) More everted alae of the ilia (E) Ischial spines close together in the midline

The answer is D: More everted alae of the ilia. The greater pelvis (pelvis major; false pelvis) of a female is relatively shallow due to the everted wings (alae) of the iliac portions of the coxal (hip) bones. This shallow greater pelvis shortens the distance the head of a fetus needs to descend through the pelvic aper- ture during parturition. If the wings of the iliac portions of the coxal bones are more everted, then the unearthed coxal bones are most likely female. The rest of the characteristics listed within this question are more characteristic of a male pelvis. Choice A (Subpubic angle less than 70 degrees) is incorrect. A subpubic angle of less than 70 degrees is a characteristic of a male pelvis. This angle can be approximated by spreading the index and middle finger. In the female pelvis, this sub- pubic angle is wider (greater) than 80 degrees, which enables the head of a fetus more space to descend through the pelvic outlet (inferior pelvic aperture). The size of the female subpu- bic angle can be approximated by spreading the thumb and index finger. Choice B (Heart-shaped [android] pelvic inlet) is incorrect. A heart-shaped (android) pelvic inlet (superior pelvic aperture) is a characteristic of a male pelvis. In contrast, the female pelvis has an oval and rounded pelvic inlet, which is better adapted for parturition. Choice C (More inverted ischial tuberosities) is incorrect. Inverted ischial tuberosities are often seen in the male pelvis. This feature decreases the diameter of the pelvic outlet (inferior pelvic aperture), which is not conducive to parturition. In contrast, the ischial tuber- osities in the female pelvis are more everted, which increases the diameter of the pelvic outlet. Choice E (Ischial spines close together in the midline) is incorrect. A small interspinous dis- tance, due to inverted ischial spines, is often seen in the male pelvis. This characteristic decreases the diameter of the pelvic outlet (or inferior pelvic aperture), which is not conducive to parturition. In contrast, the distance between the ischial spines in the female pelvis is wider, which increases the diam- eter of the pelvic outlet.

As part of a complete physical examination, an obstetrician conducts a digital vaginal examination of her 25-year-old patient. Which of the following structures is normally palpable through the posterior fornix? (A) Urethra (B) Ovaries (C) Ischial spines (D) Perineal body (E) Ureters

The answer is D: Perineal body. The vaginal fornix is the recess that surrounds the cervix. It has anterior, lateral, and posterior parts. Because of the postural angles between the vagina, cer- vix, and uterine body, the posterior fornix is the deepest part, affording a wide palpation area plus direct access to the rec- touterine pouch. The perineal body lies in the midline directly posterior to the vagina and can be readily palpated. Addition- ally, the rectum, anal canal, sacrum, and sacral promontory are available to palpation. Choice A (Urethra) is incorrect. The urethra and the base of the bladder lie against the anterior wall and fornix of the vagina. They can be palpated in that direction. Choice B (Ovaries) is incorrect. The ovaries, ischial spines, ureters, uterine tubes, and the uterine arteries can be palpated through the lateral fornices of the vagina. Choice C (Ischial spines) is incorrect. The ischial spines, ovaries, ure- ters, suterine tubes, and the uterine arteries can be palpated through the lateral fornices of the vagina. Choice E (Ureters) is incorrect. The ureters, ovaries, ischial spines, uterine tubes, and the uterine arteries can be palpated through the lateral fornices of the vagina.

19 A 21-year-old male college student reports to the student health clinic on Monday morning, the day after the Super Bowl. He explains that he was intoxicated and lost consciousness with his upper limbs draped over the back of a couch. He complains of numbness and paresthesia over the dorsum of his hand on the radial side and is unable to support the weight of his left hand when the hand is placed in a pronated position (see photo). What nerve was most likely damaged in this individual? (A) Axillary nerve (B) Median nerve (C) Ulnar nerve (D) Radial nerve (E) Musculocutaneous nerve

The answer is D: Radial nerve. This case represents a classic presentation of "Saturday Night Palsy," where the radial nerve is compressed against the humerus in the arm. Remember, the radial nerve supplies motor innervation to the posterior compartments of the arm and forearm, so damage to this nerve would cause weakness in extending the elbow and wrist. This patient is unable to extend the wrist when the hand is placed in a pronated position ("wrist drop"), implying damage to the radial nerve. Moreover, the superfi cial branch of the radial nerve is responsible for cutaneous innervation over much of the dorsum of the hand, which explains the numbness and paresthesia in his hand. Choice A (Axillary nerve) is incorrect. The axillary nerve innervates the deltoid and teres minor muscles as well as the skin overlying the deltoid in the superolateral aspect of the arm. Compressing the axillary nerve would not affect wrist function or cause paresthesia distal to the wrist, which was seen in this student. Choice B (Median nerve) is incorrect. The median nerve is responsible for fl exion at the wrist; however, this patient is having trouble with wrist extension. The sensory distribution for the median nerve is primarily on the palmar side of the hand rather than the dorsum of the hand, which is seen in this patient. Choice C (Ulnar nerve) is incorrect. The ulnar nerve is responsible for sensory innervation on the dorsum of the hand that is limited to the ulnar side (medial to the midline of the fourth fi nger). Damage to the ulnar nerve would not result in "wrist drop," which is an indication of a radial nerve injury. The ulnar nerve does supply motor innervation to most of the intrinsic muscles of the hand. Choice E (Musculocutaneous nerve) is incorrect. The musculocutaneous nerve supplies motor innervation to the anterior compartment of the arm and cutaneous innervation to the lateral aspect of the forearm. Therefore, damage to this nerve would not affect wrist function

A male infant is born with undescended testes that remain intra-abdominal 5 months later (cryptorchidism). Each testis possesses an elongate, distinctive gubernaculum testis running into the scrotum. Which of the following structures in female anatomy is homologous to the gubernaculum testis? (A) Uterine tube (B) Ureter (C) Labiumminus (D) Round ligament of the uterus (E) Vestibular bulb

The answer is D: Round ligament of the uterus. Both the round ligament of the uterus and the ovarian ligament are the homologues of the gubernaculum testis. In males, the testes descend along a path formed by the caudal genital ligament and gubernaculum testis. In females, the same structures guide the ovaries in their descent. However, the upward growth of the reproductive tract and different hormonal influences disrupt full ovarian descent. When the ovaries settle into their final position in the lateral aspect of the true pelvis, the remnants of the caudal genital ligament and gubernaculum are retained as the ovarian ligament and round ligament of the uterus. The round ligament extends through the inguinal canal and into the labium majus, marking what would be the full descent pathway in males. Choice A (Uterine tube) is incorrect. The uterine tubes (and uterus, cervix, and superior third of the vagina) are derived from the embryonic paramesonephric ducts. In males, the paramesonephric ducts are suppressed, and the only remnant is the vestigial appendix of the testis. Choice B (Ureter) is incorrect. The ureter is formed from the ureteric bud, an outgrowth of the mesonephric duct. Over- all, the ureteric bud gives rise to the renal collecting ducts, including the ureter, renal pelvis, renal calyces, and collect- ing tubules. Choice C (Labium minus) is incorrect. The labia minora are formed from the embryonic urethral (urogenital) folds. In males, these folds fuse in the midline to form the ventral (anterior) aspect of the penis enclosing the penile ure- thra. In females, the folds do not fuse, forming the separate labia minora. Choice E (Vestibular bulb) is incorrect. The vestibular bulbs are formed from the more posterior part of the embryonic phallus, which is continuous with the ure- thral folds. In males, this area fuses in the midline to form the corpus spongiosum and bulb of the penis. In females, the structures remain separate and form the bulbs of the vestibule, underlying the labia minora

41 Following a radical mastectomy procedure, a surgeon plans to conduct a breast reconstruction utilizing a latissimus dorsi muscle flap. What nerve will the surgeon need to keep intact during the surgical dissection of the chest wall to prevent atrophy of the muscle flap? (A) Long thoracic nerve (B) Intercostobrachial nerve (C) Medial pectoral nerve (D) Thoracodorsal nerve (E) Axillary nerve

The answer is D: Thoracodorsal nerve. Successful transposition of muscle fl aps in reconstructive surgery hinges largely on maintaining the neurovascular pedicles that supply the muscle. The latissimus dorsi muscle is innervated by the thoracodorsal (middle subscapular) nerve, a branch of the posterior cord of the brachial plexus. Lesion of this nerve during dissection of the chest wall will cause loss of the muscle fl ap. Choice A (Long thoracic nerve) is incorrect. The long thoracic nerve supplies the serratus anterior muscle. Damage to this nerve would cause a "winged scapula." Choice B (Intercostobrachial nerve) is incorrect. The intercostobrachial nerve is actually the lateral cutaneous branch of the second thoracic nerve and is entirely sensory in innervation. This nerve arises from the chest wall underneath the second rib and travels distally to give sensory innervation to the skin of the medial arm. Choice C (Medial pectoral nerve) is incorrect. The medial pectoral nerve innervates the pectoralis minor and major muscles. It would have been cut during the removal of these muscles during the radical mastectomy. Choice E (Axillary nerve) is incorrect. The axillary nerve traverses the quadrangular space (accompanying the posterior circumfl ex humeral artery) to supply the teres minor and deltoid muscles as well as give cutaneous innervation to the upper lateral aspect of the arm

A 48-year-old woman falls on an icy sidewalk and lands on her right elbow. She suffers a midshaft humeral fracture, as seen on the given X-ray. The attending physician wants to assess whether the nerve residing in the spiral groove of the humerus is damaged. What sign or symptom would confirm damage to this nerve? (A) Numbness on the lateral (radial) aspect of the forearm (B) Numbness of the medial aspect of the upper arm (C) Numbness over the superolateral aspect of the upper arm (D) Weakness extending the wrist and fingers (E) Weakness in grip strength

The answer is D: Weakness extending the wrist and fingers. The radial nerve is particularly vulnerable during midshaft humeral fractures because it is located directly against the bone in this region. The radial nerve innervates the extensor muscles of the arm and forearm and also carries sensory innervation from the posterior aspect of the arm, forearm, and hand and the lateral aspect of the arm (but not the forearm). Depending upon the exact placement of the fracture, the nerve fibers that inner- vate the triceps brachii may have already left the radial nerve. However, the fibers innervating the extensors of the wrist and fingers would still be bundled in the radial nerve and would be vulnerable to damage during a fracture of this type. Choice A (Numbness on the lateral [radial] aspect of the fore- arm) is incorrect. The radial nerve innervates the skin on the posterior aspect of the arm and forearm and the lateral aspect of the lower arm. The lateral (radial) aspect of the forearm is innervated primarily by the lateral cutaneous nerve of the forearm, the terminal branch of the musculocutaneous nerve. Choice B (Numbness of the medial aspect of the upper arm) is incorrect. The skin over the medial aspect of the upper arm is innervated by the medial cutaneous nerve of the arm (medial brachial cutaneous nerve) that originates from the medial cord of the brachial plexus. This nerve would be protected from the humeral fracture due to its location. Choice C (Numb- ness over the superolateral aspect of the upper arm) is incor- rect. The cutaneous innervation of the superolateral aspect of the upper arm is derived from the axillary nerve that shares a common origin with the radial nerve from the posterior cord of the brachial plexus. Though both nerves arise from the pos- terior cord, the axillary and radial nerves separate proximal to the site of the fracture, thus leaving the axillary nerve undam- aged by the fracture. Choice E (Weakness in grip strength) is incorrect. While the pain associated with the fracture would affect the integrity of all muscle activity in the limb, all of the muscles used for gripping (flexing the wrist and digits) are innervated by the median and ulnar nerves, which do not have close relations to the fracture site

51 A 3-year-old girl is brought to the emergency room holding her right arm with the elbow flexed and the forearm pronated. She refuses to move her arm and complains her elbow "hurts a lot." Her mother reports they were holding hands and running in the park when the child tripped. The mother pulled on the child's hand to prevent her from hitting the ground. Given the nature of this injury and the age of the patient, what structure is most likely damaged? (A) Interosseous membrane of forearm (B) Quadrate ligament (C) Radial collateral ligament of elbow (D) Ulnar collateral ligament of elbow (E) Anular ligament of radius

The answer is E: Anular ligament of radius. The anular ligament of the radius encircles and holds the head of the radius in the radial notch of the ulna (see diagram on next page). This ligament enables pronation and supination of the forearm. However, the head of the radius can be pulled distally out of this anular ligament resulting in a subluxation or dislocation of the radial head, which is frequently called "nursemaid's elbow." This injury is often seen in children, particularly girls, between the ages of 1 to 3 years old. It occurs when an extended arm is pulled, commonly during a fall, and the individual holding the hand does not let go, as reported in this case. Subluxation and dislocation of the radial head are also seen when the child is swinging while being held by the hands. Choice A (Interosseous membrane of the forearm) is incorrect. The interosseous membrane of the forearm connects the interosseous borders of the radius and ulna. This membrane would not be injured in this type of the injury, especially because the patient complains of pain in the elbow region. Choice B (Quadrate ligament) is incorrect. The quadrate ligament passes from the distal margin of the radial notch of the ulna to the neck of the radius. Damage to this ligament is rare in this type of injury when the extended arm is pulled. Choice C (Radial collateral ligament of elbow) is incorrect. The radial collateral ligament of the elbow extends from the lateral epicondyle of the humerus to the anular ligament of the radius. This ligament would be injured in forced adduction of the elbow joint, which was not seen in this patient. Choice D (Ulnar collateral ligament of elbow) is incorrect. The ulnar collateral ligament of the elbow extends from the medial epicondyle of the humerus to the coronoid process and olecranon of the ulna. This ligament would be injured in forced abduction of the elbow joint, which was not seen in this patient. This ligament is also frequently injured in athletes who use an overhead throwing motion. When the athlete releases the ball during their throwing motion, the ulna is pulled from the humerus, which can damage this ligament. American baseball pitchers often need surgery to repair the ulnar collateral ligament of the elbow, which is commonly termed "Tommy John surgery."

31 In both the upper and lower limbs, the superficial veins begin in a dorsal cutaneous arch that drains into medial and lateral cutaneous veins aligned mainly along the first and fifth digit sides of the limb. Which of the following veins in the upper limb is the equivalent of the great saphenous vein in the lower limb? (A) Radial vein (B) Ulnar vein (C) Brachial vein (D) Basilic vein (E) Cephalic vein

The answer is E: Cephalic vein. The great saphenous vein is a large cutaneous vessel that ascends the medial aspect of the lower limb to ultimately drain into the femoral vein at the top of the limb. The cephalic vein is a distinctive cutaneous vessel that ascends the lateral aspect of the arm to ultimately drain into the axillary vein. At fi rst glance, these two veins might seem to be traveling opposite venous routes. However, notice that each vein is aligned along the fi rst digit side (great toe and thumb) of its respective limb. Remember that the upper and lower limbs rotate in opposite directions during their development, resulting in the great toe placed on the medial side of the foot and the thumb on the lateral side of the hand. The alignment of the great saphenous and cephalic veins with the fi rst digits in the limbs denotes them as developmental equivalents. Choice A (Radial vein) is incorrect. The radial vein is a deep vena comitans, a vein that accompanies another structure; in this case, it travels with the radial artery. Like most deep veins of the limbs, this vein actually exists as two veins that travel as companions to the radial artery and are called venae commitantes. Being a deep vein, the radial vein is not an equivalent structure to the subcutaneous great saphenous vein. Choice B (Ulnar vein) is incorrect. The ulnar vein is a deep vena comitans, a vein that accompanies the ulnar artery. The ulnar vein also exists as two venae commitantes that travel collaterally to the ulnar artery within the same connective tissue sheath. The ulnar vein is not an equivalent structure to the great saphenous vein. Choice C (Brachial vein) is incorrect. The brachial vein is a deep, paired venae commitantes that accompanies the brachial artery. This vein is not an equivalent structure to the great saphenous vein due to its deep course within the arm. Choice D (Basilic vein) is incorrect. The basilic vein is a cutaneous vessel that ascends the medial aspect of the arm, in alignment with the fi fth digit in the hand. Thus, it is actually on the opposite side of the limb for equivalence with the great saphenous vein. Instead, the basilic vein is the counterpart of the small saphenous vein.

33 During an attempted suicide, a depressed young woman slashes the front of her wrist with a razor blade. However, she cuts only to the depth of the superficial aspect of the flexor retinaculum before passing out at the sight of her own blood. Which of the following muscle tendons may be severed? (A) Flexor digitorum superficialis (B) Brachioradialis (C) Flexor pollicis longus (D) Abductor pollicis longus (E) Flexor carpi radialis

The answer is E: Flexor carpi radialis. The fl exor retinaculum (transverse carpal ligament) is a thickening of investing deep fascia on the anterior (ventral) aspect of the wrist that forms the roof of the carpal tunnel. All issues related to the carpal tunnel revolve around an understanding of what structures are contained within the carpal tunnel versus the positions of structures outside the tunnel. The tendons of the fl exor carpi radialis and palmaris longus muscles, plus the ulnar nerve and ulnar artery, lie against the superfi cial aspect of the fl exor retinaculum. Any of these structures may be lesioned by a superfi cial cut across the front (anterior side) of the wrist. Choice A (Flexor digitorum superfi cialis) is incorrect. The tendons of this muscle are contained within the carpal tunnel, deep to the fl exor retinaculum. Choice B (Brachioradialis) is incorrect. This muscle inserts onto the base of the styloid process of the radius. Thus, its tendon is proximal and lateral to the fl exor retinaculum, and therefore unrelated to the carpal tunnel. Choice C (Flexor pollicis longus) is incorrect. The tendons of this muscle are contained within the carpal tunnel, deep to the fl exor retinaculum. Choice D (Abductor pollicis longus) is incorrect. This muscle is a member of the posterior compartment of the forearm. Its tendon loops out of that compartment to attach onto the lateral side of the base of the first metacarpal (the lateral base of the thumb). Thus, it is well removed from the flexor retinaculum and the carpal tunnel.

37 As part of a physical examination to evaluate intrinsic hand muscle function, a physician holds three fingers in the extended position, and instructs the patient to flex the proximal interphalangeal joint of the free finger, as shown. Which of the following muscles is the doctor specifically testing? (A) Flexor digitorum profundus (FDP) (B) Extensor digitorum (C) Second lumbrical (D) Dorsal interosseous (E) Flexor digitorum superficialis (FDS)

The answer is E: Flexor digitorum superfi cialis (FDS). The FDS muscle fl exes the proximal interphalangeal joints in digits 2 to 5. In this patient, this muscle is being tested in the third (middle) fi nger. By holding the other fi ngers in extension, the physician is eliminating the infl uences of the FDP muscles, which fl ex the distal interphalangeal joints in digits 2 to 5. Choice A (Flexor digitorum profundus [FDP]) is incorrect. Flexion of the distal interphalangeal joint in digits 2 to 5 is produced by the FDP; however, the physician is testing fl exion of the proximal interphalangeal joint in the middle fi nger, which is controlled by the FDS muscle. Choice B (Extensor digitorum) is incorrect. The extensor digitorum extends fi ngers #2 to #5. This muscle can easily be eliminated because the physician is testing a muscle on the fl exor side of the hand. Choice C (Second lumbrical) is incorrect. The second lumbrical muscle extends the interphalangeal joints of the middle (third) fi nger and fl exes the metacarpophalangeal joint of the same fi nger. The third lumbrical is an intrinsic hand muscle that arises off the tendon of the FDP and inserts into the extensor expansion of the middle fi nger. This muscle does not control fl exion of the proximal interphalangeal joint. Choice D (Dorsal interosseous) is incorrect. The primary action of the dorsal interosseous muscles is to abduct the fi ngers. However, because these muscles insert into the extensor expansion, they also extend the interphalangeal joints of the middle (third) fi nger and fl ex the metacarpophalangeal joint of the same fi nger. Although the dorsal interossei are not involved with fl exion of the distal interphalangeal joint, it is important to remember the middle (third) fi nger forms the axis of movement of the hand. Therefore, this fi nger has two dorsal interossei attached to it to enable abduction away from the midline in either direction.

34 A 17-year-old man has pain and moderate swelling over the dorsomedial aspect and in the hypothenar area of his right hand after punching a locker over a dispute with his girlfriend. What is the most likely finding on an X-ray of his hand? (A) Dislocation of the fifth metacarpophalangeal joint (B) Fracture of the triquetral bone (C) Fracture of the proximal phalanx of the ring finger (D) Fracture of the proximal phalanx of the little finger (E) Fracture of the fifth metacarpal bone

The answer is E: Fracture of the fi fth metacarpal bone. Fracture of the fi fth metacarpal bone is called a "boxer's fracture" because this injury is often seen after an individual improperly punches a solid object with a clenched fi st. The impact on the head of the fi fth metacarpal causes the distal shaft of this bone to fracture. Skilled pugilists are trained to direct the impact of the clenched fi st on the heads of the fi rst and second metacarpals to avoid this type of injury. Choice A (Dislocation of the fi fth metacarpophalangeal joint) is incorrect. This type of injury is rare, and a dislocation of the fi fth metacarpophalangeal joint would have been detected by the physician on examination. Choice B (Fracture of the triquetral bone) is incorrect. This carpal bone is rarely fractured in this type of impact. Choice C (Fracture of the proximal phalanx of the ring fi nger) is incorrect. The impact of the clenched fi st would have been localized to the head of the fi fth metacarpal. Damage to the proximal phalanx of the ring fi nger would have occurred only if the fi st was not clenched during impact. Choice D (Fracture of the proximal phalanx of the little fi nger) is incorrect. The impact of the clenched fi st would have been localized to the head of the fi fth metacarpal. Damage to the proximal phalanx of the little fi nger would have occurred only if the fi st was not clenched during impact.

5 During an attempted suicide, a depressed young woman slashes her wrist with a straight razor. She cuts just proximal to the pisiform bone to the depth of the superficial aspect of the flexor retinaculum before passing out at the sight of her own blood. As a result of this wound, she may suffer a neuromuscular deficit that results in which of the following? (A) Weakness in pronation (B) Inability to abduct the thumb (C) Weakness in flexion of the thumb (D) Weakness in opposition of the thumb (E) Inability to adduct the thumb

The answer is E: Inability to adduct the thumb. The location of the cut, superfi cial to the fl exor retinaculum, indicates injury to the ulnar nerve. This nerve controls most of the intrinsic muscles of the hand, including the adductor pollicis. Because this muscle is the sole adductor of the thumb, the result will be inability to adduct that digit. Choice A (Weakness in pronation) is incorrect. Pronation is controlled by branches of the median nerve to the pronator teres and pronator quadratus muscles. These are located proximal to the site of the injury. Choice B (Inability to abduct the thumb) is incorrect. Abduction of the thumb is governed by the radial nerve (to abductor pollicis longus) and the recurrent branch of the median nerve (to abductor pollicis brevis) at sites removed from the injury. Choice C (Weakness in fl exion of the thumb) is incorrect. Flexion of the thumb is controlled by branches of the median nerve at locations proximal (to fl exor pollicis longus) and distal (to fl exor pollicis brevis) to the cut fl exor retinaculum (transverse carpal ligament). However, the reported laceration was superfi cial to this ligament and proximal to the pisiform bone, so the median nerve would not have been damaged. Choice D (Weakness in opposition of the thumb) is incorrect. Opposition of the thumb is controlled by the recurrent branch of the median nerve (to opponens pollicis), distal to the injury site.

An obstetrician performs a median episiotomy to expand the birth canal during a childbirth. Which of the following structures is typically incised during this procedure? (A) Bulbospongiosus muscle (B) Urethra (C) Analcanal (D) Ischiocavernosus muscle (E) Perineal body

The answer is E: Perineal body. An episiotomy is a surgical incision from the posterior wall of the vagina into the perineum posterior to the vagina. This procedure is used to gain a con- trolled enlargement of the vaginal canal during childbirth. In a median episiotomy, the incision is made in the midline and extends through the perineal body (see diagram). The advan- tage of this procedure is that it may decrease excessive reduc- tion of the perineal body and minimize trauma to the pelvic diaphragm and perineal muscles. An alternate procedure is the posterolateral episiotomy. In this protocol, the incision begins in the median line and then is directed laterally and posteriorly toward the ischial tuberosity. The advantage here is that the perineal body is spared. Choice A (Bulbospongio- sus muscle) is incorrect. The bulbospongiosus is the skeletal muscle sheath surrounding the individual vestibular bulbs. The bulbs are located deep to the labia minora, lateral to the vaginal opening. Thus, the bulbs are anterior to the incision line in either type of episiotomy and are not cut. Choice B (Urethra) is incorrect. The urethra is anterior to the vagina. It is not in the line of incision in an episiotomy and is not cut. Choice C (Anal canal) is incorrect. The anal canal is directly in the line of incision in a median episiotomy. However, the incision normally stops short of the anal canal. It is undesir- able to have a communication between the anal and vaginal canals because of the danger of anal material contacting the fetus during delivery. Choice D (Ischiocavernosus muscle) is incorrect. The ischiocavernosus is the thin skeletal muscle layer surrounding the crus of the clitoris. Each crus is attached to the ischiopubic ramus, lateral to the vagina and anterior to the ischial tuberosity. Thus, the crus of the clitoris would not be incised because it is located off the median sagittal plane.

48 A 37-year-old factory worker fractures multiple bones distal to the elbow when his hand and forearm are crushed by equipment dropped by a faulty hydraulic lift. Which of the following bones, if fractured, would most likely develop avascular necrosis? (A) Distal radius (B) Midshaft ulna (C) Fifth metacarpal (D) Lunate (E) Scaphoid

The answer is E: Scaphoid. The proximal segment of the scaphoid bone has a poor supply of blood because the palmar carpal branch of the radial artery enters the distal part of the scaphoid and then supplies blood proximally. This small artery is often severed during fractures of the scaphoid bone leading to avascular necrosis, or death of the bone due to poor blood supply. Remember, the scaphoid is the most commonly fractured carpal bone, but a fracture within this bone is often not seen on initial radiographs. Radiographs taken several weeks later will show the fracture due to bone resorption at the fracture site. Choice A (Distal radius) is incorrect. The distal radius is often fractured in older individuals who fall on their outstretched hand. This fracture of the distal radius is called a Colles fracture ("dinner fork deformity") due to its appearance on a lateral radiograph. However, the distal radius has an adequate blood supply and is not prone to avascular necrosis. Choice B (Midshaft ulna) is incorrect. A fracture of the midshaft ulna is not prone to avascular necrosis due to its adequate blood supply. Choice C (Fifth metacarpal) is incorrect. The fi fth metacarpal is often fractured when an individual improperly punches a solid object with a clenched fi st, as in a "boxer's fracture." This bone is not prone to avascular necrosis due to its adequate blood supply. Choice D (Lunate) is incorrect. The lunate is the most commonly dislocated carpal bone, which leads to severe carpal instabilities. This dislocation often occurs in association with a trans-scaphoid fracture. If this bone were fractured in this work-related accident, it would not be prone to avascular necrosis due to its adequate blood supply.

A 16-year-old boy, who was riding his skateboard down a stair railing, falls and impacts his perineum on the railing, causing a straddle injury. He presents in the ER with discoloration and swelling in the scrotum, penis, and anterior abdominal wall, which is characteristic of extravasation of urine. Which of the following structures is most likely compromised? (A) Ureter (B) Urinary bladder (C) Prostatic urethra (D) Intermediate urethra (E) Spongy urethra

The answer is E: Spongy urethra. In men, a straddle injury may rupture the spongy urethra, which courses through the corpus spongiosum of the penis. Damage to the spongy urethra causes extravasation of urine, in which the urine leaves the spongy urethra and spills out into the surrounding tis- sue. This extravasation of urine would be confined by the superficial layer of perineal fascia, which is continuous with the superficial (dartos) fascia of the scrotum, the superficial fascia of the penis, and the membranous layer of subcutane- ous tissue of the lower abdomen (Scarpa fascia). Therefore, urine (and blood, due to the trauma and the discoloration noted in this presentation) will spread into the scrotum, penis, and lower anterior abdominal wall, as noted in this patient. Because the superficial layer of perineal fascia blends with the posterior edge of the perineal membrane and adheres onto the ischiopubic rami laterally, urine will not spread into the anal triangle of the perineum or the thighs, respectively. Damage to the spongy urethra can also occur following an incorrect insertion of a catheter to drain the urinary bladder. Choice A (Ureter) is incorrect. The ureters descend posterior to the peri- toneum of the abdomen to connect the kidneys to the urinary bladder. Due to their location, the ureters would not be at risk during a straddle injury to the perineum. Furthermore, rupture of the ureters would cause extravasation of urine in the abdomen or pelvis, but not the perineum, as noted in this patient. Choice B (Urinary bladder) is incorrect. The urinary bladder is located in the true pelvis (or pelvis minor), and due to its position above the perineal membrane and the pelvic diaphragm, it would not be at risk during a straddle injury to the perineum. Furthermore, rupture of the urinary bladder would cause extravasation of urine into extraperitoneal spaces or into the peritoneal cavity, which was not seen in this patient. Choice C (Prostatic urethra) is incorrect. The prostatic urethra is the portion of the male urethra that traverses the prostate gland. Because it is not located within the perineum, prostatic urethra damage would result in extravasation of urine in the true pelvis (or pelvis minor). Choice D (Intermediate urethra) is incorrect. The intermediate (membranous) urethra passes through the deep perineal compartment. This portion of the male urethra is the narrowest urethral segment. Due to its location within the deep perineal compartment, extravasation of urine from the intermediate urethra would reside between the inferior fascia of the pelvic diaphragm and the perineal membrane. So, damage to the intermediate urethra would not cause discoloration and swelling in the scrotum, penis, and anterior abdominal wall, which was noted in this patient.

15 A 36-year-old man broke a window with his fi st to rescue his child from a house fi re. The man sustained a laceration to the lateral aspect of his right forearm, but he only showed a sensory defi cit (numbness and paresthesia) to the dorsolateral aspect of his hand (as denoted by the shaded area within the given photo). What nerve was most likely damaged? (A) Dorsal cutaneous branch of the ulnar nerve (B) Lateral cutaneous nerve of the forearm (C) Posterior cutaneous nerve of the forearm (D) Deep branch of the radial nerve (E) Superfi cial branch of the radial nerve

The answer is E: Superfi cial branch of the radial nerve. The superfi cial branch of the radial nerve is entirely cutaneous, carrying sensation from the dorsolateral part of the hand from the anatomical snuffbox to the midline of the fourth fi nger. This nerve is vulnerable as it runs posteriorly between the brachioradialis and extensor carpi radialis longus tendons toward the dorsum of the hand. This nerve was damaged at this location, but the area of sensory loss is less than expected due to the overlap from cutaneous branches of the ulnar and median nerves. Choice A (Dorsal cutaneous branch of the ulnar nerve) is incorrect. The dorsal cutaneous branch of the ulnar nerve passes posterior between the ulna and fl exor carpi ulnaris to supply the subcutaneous tissue of the dorsal aspect of the posteromedial aspect of the hand, medial to the midline of the fourth fi nger. Its medial location and its sensory distribution make it an unlikely choice to be involved with this patient. Choice B (Lateral cutaneous nerve of the forearm) is incorrect. The lateral cutaneous nerve of the forearm is the continuation of the musculocutaneous nerve, which exits the arm between the biceps brachii and brachialis muscles. It supplies the skin on the lateral aspects of the forearm and wrist but would not be responsible for the numbness and paresthesia seen in this patient because the sensory defi cit is distal to this nerve's normal distribution. Choice C (Posterior cutaneous nerve of the forearm) is incorrect. The posterior cutaneous nerve of the forearm arises from the radial nerve and passes in close proximity to the lateral intermuscular septum of the arm near the origin of the brachioradialis. As its name implies, it supplies the posterior aspect of the forearm, but it does not extend past the wrist. Therefore, it would not be the nerve damaged in this patient. Choice D (Deep branch of the radial nerve) is incorrect. The deep branch of the radial nerve is entirely motor in its distribution to the posterior muscles of the forearm. Its lack of cutaneous innervation makes this choice easy to eliminate.

26 Lateral rotation of the arm is an important mechanical component of "bringing the arm back" when preparing to throw an object. What muscle acts to produce lateral rotation of the arm? (A) Supraspinatus (B) Teres major (C) Latissimus dorsi (D) Subscapularis (E) Teres minor

The answer is E: Teres minor. Throwing motions are complex mechanical events that involve multiple muscles interacting in moment-to-moment changing ways, with rotation of the humerus being one important outcome. The four rotator cuff muscles (supraspinatus, infrapsinatus, teres minor, subscapularis), plus several other muscles that cross the glenohumeral joint, contribute signifi cant forces to rotation of the arm. The teres minor and infraspinatus (i.e., half the rotator cuff group) lie completely across the posterior aspect of the glenohumeral joint and are primary lateral (external) rotators. These muscles are aided by the posterior fi bers of the deltoid muscle. Choice A (Supraspinatus) is incorrect. The supraspinatus is one of the rotator cuff muscles. However, it does not produce rotation. It lies across the superior aspect of the glenohumeral joint and initiates abduction of the arm from the rest position. Choice B (Teres major) is incorrect. The teres major arises posteriorly from the inferior angle of the scapula and crosses the glenohumeral joint to its anteriorly located insertion into the medial lip of the intertubercular groove of the humerus. Thus, the teres major muscle acts to adduct and medially (internally) rotate the arm rather than laterally rotate. It is not a member of the rotator cuff muscles. Choice C (Latissimus dorsi) is incorrect. The latissimus dorsi muscle arises posteriorly from the spinous processes of the inferior six thoracic vertebrae and the thoracolumbar fascia and crosses the glenohumeral joint anteriorly to insert into the fl oor of the intertubercular sulcus of the humerus. Thus, the latissimus dorsi muscle acts to extend, adduct, and medially rotate the humerus rather than laterally rotate. It is not a member of the rotator cuff muscles. Choice D (Subscapularis) is incorrect. The subscapularis muscle is part of the rotator cuff. In common with the teres major and latissimus dorsi muscles, it crosses the glenohumeral joint from posterior to anterior, thus producing medial rotation and adduction of the arm. It is not responsible for external (lateral) rotation of the arm.

25 The pectoralis minor muscle is an important landmark in identifying and describing neighboring structures in the chest and axillary regions. Which of the following relationships of the pectoralis minor is correct? (A) The lateral cord of the brachial plexus lies lateral to the muscle (B) The clavipectoral triangle lies lateral to the muscle (C) The anterior axillary lymph nodes lie along the medial border of the muscle (D) The lateral wall of the axillary fossa includes the muscle (E) The second part of the axillary artery lies deep to the muscle

The answer is E: The second part of the axillary artery lies deep to the muscle. The pectoralis minor muscle overlies the axillary artery in such a way as to divide it into three parts: fi rst (prepectoral; medial), second (subpectoral; deep), third (postpectoral; lateral). The fi rst part is medial to the pectoralis minor, running from the lateral border of the fi rst rib to the medial border of the pectoralis minor. The second part is deep to the muscle. The third part is lateral to the pectoralis minor, running from the lateral border of the muscle to the inferior border of the teres major muscle. Choice A (The lateral cord of the brachial plexus lies lateral to the muscle) is incorrect. The cords of the brachial plexus are so named by their positions relative to the second part of the axillary artery. Choice B (The clavipectoral triangle lies lateral to the muscle) is incorrect. The clavipectoral triangle lies medial to the pectoralis minor, between the muscle and the clavicle. This fascia-roofed space is noteworthy because it is pierced by the lateral pectoral nerve (on its way to the clavicular head of the pectoralis major), branches of the thoracoacromial artery, and the cephalic vein (on its way to empty into the axillary vein). Choice C (The anterior axillary lymph nodes lie along the medial border of the muscle) is incorrect. The anterior (pectoral; level 1) group of axillary lymph nodes lie along the lateral border of the pectoralis minor. Choice D (The lateral wall of the axillary fossa includes the muscle) is incorrect. The pectoralis minor forms the anterior wall of the axillary fossa, along with the pectoralis major.

Development of the primitive gonads into testes or ovaries is induced by invasion of the genital ridges by the primordial germ cells. These cells migrate from which of the following locations? (A) Paraxial mesoderm (B) Intermediate mesoderm (C) Lateral plate mesoderm (D) Neural crest (E) Yolk sac endoderm

The answer is E: Yolk sac endoderm. The primordial germ cells originate in the epiblast and migrate to the caudal endo- dermal wall of the yolk sac, near the allantois. From there, they migrate along the dorsal mesentery of the gut tube to the genital ridges. Once they invade the genital ridges, they induce the primitive gonads to develop into either testes or ovaries. Choice A (Paraxial mesoderm) is incorrect. The paraxial mesoderm is the thickened portion of mesoderm clos- est to the midline and neural tube. It becomes segmented and forms somites, which further differentiate into myotome, der- matome, and sclerotome units that form skeletal muscles, der- mis, and most of the vertebral column, respectively. Choice B (Intermediate mesoderm) is incorrect. The intermediate meso- derm is the small bridge of mesoderm that connects the parax- ial mesoderm with the lateral plate mesoderm. It differentiates into urogenital structures, including the kidney parenchyma and nephrons and the gonads. Thus, the intermediate meso- derm is the target of the migrating primordial germ cells rather than their source. Choice C (Lateral plate mesoderm) is incor- rect. The lateral plate mesoderm is a relatively flat, wide area, which divides into the parietal (somatic) mesoderm and vis- ceral (splanchnic) mesoderm. The parietal part becomes asso- ciated with the wall of the amniotic sac, ultimately forming (along with the ectoderm) the lateral body wall folds. The visceral part joins with the endodermal lining of the yolk sac and forms the wall of the gut tube. Choice D (Neural crest) is incorrect. The neural crest forms at the free lips of the neu- ral folds, dissociates from the neural tube, and spreads widely across the body. Crest cells form a diverse assortment of struc- tures, including melanocytes, neural ganglia, craniofacial skel- eton, and odontoblasts.

An 18-year-old man, in shock and bleeding profusely, is brought to the emergency department after being stabbed in the chest. An examination shows a knife wound to the right first intercostal space, immediately lateral to the manubrium. The knife entered parallel to the ground and penetrated to a depth of 6.5 cm. Which of the following structures is most likely injured? a. Ascending aorta b. Right ventricle c. Left common carotid artery d. Right subclavian artery e. Superior vena cava

e. Superior vena cava

A new born male infant is diagnosed with Hirschprung's disease affecting the sigmoid colon due to a defect in the migration of the neural crest cells. The attending physician should be aware of the possibility of defective migration of which of the other derivatives of the neural crest? Select one: a. Nucleus pulposus of the intervertebral disc b. Parathyroid gland c. Adrenal (suprarenal) cortex d. Annulus fibrosus of the intervertebral disc e. Melanocytes (pigment cells) in the epidermis of skin

e. melanocytes


संबंधित स्टडी सेट्स

chapter 23:goat breed identification and production mangement

View Set

Ch26:Respiratory Assessment, ATI- 17-25

View Set

Wk. 3 - measures of center/location, 5 # sum., boxplot

View Set

Intro to Network Chapter 7: Network Architecture

View Set